You are on page 1of 313

About Pearson

Pearson is the world’s learning company, with presence across 70 countries


worldwide. Our unique insights and world-class expertise comes from a long
history of working closely with renowned teachers, authors and thought leaders,
as a result of which, we have emerged as the preferred choice for millions of
teachers and learners across the world.
We believe learning opens up opportunities, creates fulfilling careers and hence
better lives. We hence collaborate with the best of minds to deliver you class-
leading products, spread across the Higher Education and K12 spectrum.
Superior learning experience and improved outcomes are at the heart of
everything we do. This product is the result of one such effort.
Your feedback plays a critical role in the evolution of our products and you can
contact us – reachus@pearson.com. We look forward to it.

F01_Eng-Maths (Aditya) Prelims Book 3.indd 1 7/24/2018 7:18:10 PM


This page is intentionally left blank

F01_Eng-Maths (Aditya) Prelims Book 3.indd 2 7/24/2018 7:18:10 PM


Engineering Mathematics-III

P. Sivaramakrishna Das
Professor of Mathematics
K.C.G. College of Technology
(a unit of Hindustan Group of Institutions
Karapakkam, Chennai)

C. Vijayakumari
Professor of Mathematics (Retired)
Queen Mary’s College (Autonomous)
Mylapore, Chennai

Contributors
G. Venkata Ramana
Senior Assistant Professor
Department of Humanities and Basic Sciences
Aditya Engineering College (A), Surampalem

V. Venkat Kumar
Assistant Professor
Department of Humanities and Basic Sciences
Aditya Engineering College (A), Surampalem

F01_Eng-Maths (Aditya) Prelims Book 3.indd 3 7/24/2018 7:18:11 PM


Copyright © 2017 Pearson India Education Services Pvt. Ltd

Published by Pearson India Education Services Pvt. Ltd, CIN: U72200TN2005PTC057128,


formerly known asTutorVista Global Pvt. Ltd, licensee of Pearson Education in South Asia.

No part of this eBook may be used or reproduced in any manner whatsoever without the publisher’s
prior written consent.

This eBook may or may not include all assets that were part of the print version. The publisher
reserves the right to remove any material in this eBook at any time.

ISBN 978-93-530-6489-1
eISBN: 9789353066253

Head Office: 15th Floor, Tower-B, World Trade Tower, Plot No. 1, Block-C, Sector-16,
Noida 201 301,Uttar Pradesh, India.
Registered Office: 4th Floor, Software Block, Elnet Software City, TS-140, Block 2 & 9,
Rajiv Gandhi Salai, Taramani, Chennai 600 113, Tamil Nadu, India.
Fax: 080-30461003, Phone: 080-30461060
www.pearson.co.in, Email: companysecretary.india@pearson.com

F01_Eng-Maths (Aditya) Prelims Book 3.indd 4 7/24/2018 7:18:11 PM


Contents

Prefacexi

1. Laplace Transforms 1.1


1.0 Introduction 1.1
1.1 Condition for Existence of Laplace Transform 1.1
1.2 Laplace Transform of Some Elementary Functions 1.2
1.3 Some Properties of Laplace Transform 1.4
Worked Examples1.5
Worked Examples1.8
Exercise 1.11.9
Answers to Exercise 1.11.10
1.4 Differentiation and Integration of Transforms 1.11
Worked Examples1.12
Exercise 1.21.20
Answers to Exercise 1.21.20
1.5 Laplace Transform of Derivatives and Integrals 1.21
Worked Examples1.23
1.5.1 Evaluation of Improper Integrals Using Laplace Transform 1.25
Worked Examples1.25
1.6 Laplace Transform of Periodic Functions and Other Special Type of Functions  1.27
Worked Examples1.29
Worked Examples1.33
1.6.1 Laplace Transform of Unit Step Function 1.36
1.6.2 Unit Impulse Function 1.37
1.6.3 Dirac-Delta Function 1.37
1.6.4 Laplace Transform of Delta Function 1.37
Worked Examples1.38
Exercise 1.31.39
Answers to Exercise 1.31.41

2. Inverse Laplace Transforms 2.1


2.1 Inverse Laplace Transforms 2.1
2.1.1 Type 1 – Direct and Shifting Methods 2.2
Worked Examples2.2
2.1.2 Type 2 – Partial Fraction Method 2.3
Worked Examples2.3

F01_Eng-Maths (Aditya) Prelims Book 3.indd 5 7/24/2018 7:18:11 PM


vi  n  Contents

2.1.3 Type 3 – Multiplication by s and Division by s2.8


Worked Examples2.8
2.1.4 Type 4 – Inverse Laplace Transform of Logarithmic
and Trigonometric Functions 2.10
Worked Examples2.10
Exercise 2.12.13
Answers to Exercise 2.12.14
2.1.5 Type 5 – Method of Convolution 2.15
Worked Examples2.17
Exercise 2.22.20
Answers to Exercise 2.22.20
2.7.6 Type 6:  Inverse Laplace Transform as Contour Integral 2.21
Worked Examples2.21
Exercise 2.32.24
Answers to Exercise 2.32.24
2.2 Application of Laplace Transform to the Solution
of Ordinary Differential Equations 2.25
2.2.1 First Order Linear Differential Equations
with Constant Coefficients 2.25
Worked Examples2.25
2.2.2 Ordinary Second and Higher Order Linear Differential
Equations with Constant Coefficients 2.28
Worked Examples2.28
2.2.3 Ordinary Second Order Differential Equations
with Variable Coefficients 2.32
Worked Examples2.32
Exercise 2.42.35
Answers to Exercise 2.42.36
2.2.4 Simultaneous Differential Equations 2.37
Worked Examples2.37
2.2.5 Integral–Differential Equation 2.43
Worked Examples2.43
Exercise 2.52.45
Answers to Exercise 2.52.46

3. Multiple Integrals and Beta Gamma Functions 3.1


3.1 Double Integration 3.1
3.1.1 Double Integrals in Cartesian Coordinates 3.1
3.1.2 Evaluation of Double Integrals 3.2
Worked Examples3.3
Exercise 3.13.6
Answers to Exercise 3.13.7
3.1.3 Change of Order of Integration 3.7
Worked Examples3.8
Exercise 3.23.15
Answers to Exercise 3.23.15

F01_Eng-Maths (Aditya) Prelims Book 3.indd 6 7/24/2018 7:18:11 PM


Contents  n  vii

3.1.4 Double Integral in Polar Coordinates 3.16


Worked Examples3.16
3.1.5 Change of Variables in Double Integral 3.19
Worked Examples3.19
Exercise 3.33.26
Answers to Exercise 3.33.27
3.1.6 Area as Double Integral 3.27
Worked Examples3.28
Exercise 3.43.31
Answers to Exercise 3.43.31
Worked Examples3.32
Exercise 3.53.37
Answers to Exercise 3.53.37
3.2 Area of a Curved Surface 3.37
3.2.1 Surface Area of a Curved Surface 3.38
3.2.2 Derivation of the Formula for Surface Area 3.38
3.2.3 Parametric Representation of a Surface 3.41
Worked Examples3.41
Exercise 3.63.49
Answers to Exercise 3.63.49
3.3 Triple Integral in Cartesian Coordinates 3.49
Worked Examples3.50
Exercise 3.73.55
Answers to Exercise 3.73.56
3.3.1 Volume as Triple Integral 3.56
Worked Examples3.56
Exercise 3.83.63
Answers to Exercise 3.83.64
3.4 Beta and Gamma Functions 3.64
3.4.1 Beta Function  3.64
3.4.2 Symmetric Property of Beta Function 3.64
3.4.3 Different Forms of Beta Function 3.64
3.5 The Gamma Function 3.66
3.5.1 Properties of Gamma Function 3.66
3.5.2 Relation between Beta and Gamma Functions 3.68
Worked Examples3.72
Exercise 3.93.86
Answers to Exercise 3.93.86

4. Vector Differentiation 4.1


4.0 Introduction 4.1
4.1 Scalar and Vector Point Functions  4.1
4.1.1 Geometrical Meaning of Derivative 4.2
4.2 Differentiation Formulae 4.3
4.3 Level Surfaces 4.4

F01_Eng-Maths (Aditya) Prelims Book 3.indd 7 7/24/2018 7:18:11 PM


viii  n  Contents

4.4 Gradient of a Scalar Point Function or Gradient of a Scalar Field 4.4


4.4.1 Vector Differential Operator 4.4
4.4.2 Geometrical Meaning of ∇ϕ4.4
4.4.3 Directional Derivative 4.5
4.4.4 Equation of Tangent Plane and Normal to the Surface 4.5
4.4.5 Angle between Two Surfaces at a Common Point 4.6
4.4.6 Properties of Gradients 4.6
Worked Examples4.8
Exercise 4.14.20
Answers to Exercise 4.14.21
4.5 Divergence of a Vector Point Function or Divergence of a Vector Field 4.22
4.5.1 Physical Interpretation of Divergence 4.22
4.6 Curl of a Vector Point Function or Curl of a Vector Field 4.23
4.6.1 Physical Meaning of Curl F4.23
Worked Examples4.24
Exercise 4.24.30
Answers to Exercise 4.24.31
4.7 Vector Identities 4.31
Worked Examples4.37

5. Vector Integration 5.1


5.1 Integration of Vector Functions 5.1
5.1.1 Line Integral 5.1
Worked Examples5.2
Worked Examples5.5
Exercise 5.15.7
Answers to Exercise 5.15.8
5.2 Green’s Theorem in a Plane 5.8
5.2.1 Vector Form of Green’s Theorem 5.11
Worked Examples5.11
5.3 Surface Integrals 5.17
5.3.1 Evaluation of Surface Integral 5.18
5.4 Volume Integral 5.19
Worked Examples5.19
5.5 Gauss Divergence Theorem 5.23
5.5.1 Results Derived from Gauss Divergence Theorem 5.25
Worked Examples5.29
5.6 Stoke’s Theorem 5.42
Worked Examples5.44
Exercise 5.25.58
Answers to Exercise 5.25.61

F01_Eng-Maths (Aditya) Prelims Book 3.indd 8 7/24/2018 7:18:12 PM


Contents  n  ix

Appendix A  Curve Tracing A.1


A.1 Curve TracingA.1
A.1.1 Procedure for Tracing the Curve Given by the Cartesian
Equation f (x, y) = 0 A.1
Worked ExamplesA.2
A.1.2 Procedure for Tracing of Curve Given by Parametric
Equations x = f (t), y = g(t)A.10
Worked ExamplesA.10
A.1.3 Procedure for Tracing of Curve Given by Equation in
Polar Coordinates f (r, θ) = 0 A.14
Worked ExamplesA.15
Exercise A.1A.19
Answers to Exercise A.1A.19

Index  I.1

F01_Eng-Maths (Aditya) Prelims Book 3.indd 9 7/24/2018 7:18:12 PM


This page is intentionally left blank

F01_Eng-Maths (Aditya) Prelims Book 3.indd 10 7/24/2018 7:18:12 PM


Preface

This book Engineering Mathematics-III  is written to cover the topics that are common to the syllabus
of Aditya Engineering College. Although this book is designed primarily for engineering courses,
it is also suitable for Mathematics courses and for various competitive examinations. The aim of the
book is to provide a sound understanding of Mathematics. The experiences of both the authors in
teaching undergraduate and postgraduate students from diverse backgrounds for over four decades
have helped to present the subject as simple as possible with clarity and rigour in a step-by-step
approach.
This book has many distinguishing features. The topics are well organized to create self-
confidence and interest among the readers to study and apply the mathematical tools in engineering
and science disciplines. The subject is presented with a lot of standard worked examples and exercises
that will help the readers to develop maturity in Mathematics.
At the end of each chapter, short answer questions and objective questions are given to enhance
the understanding of the topics.
Chapter 1 focuses on laplace transform.
Chapter 2 discusses inverse laplace transforms.
Chapter 3 deals multiple integrals and beta gamma functions.
Chapter 4 elaborates vector differentiation.
Chapter 5 deals with vector integration.

 P. Sivaramakrishna Das
 C. Vijayakumari

F01_Eng-Maths (Aditya) Prelims Book 3.indd 11 7/24/2018 7:18:12 PM


This page is intentionally left blank

F01_Eng-Maths (Aditya) Prelims Book 3.indd 12 7/24/2018 7:18:12 PM


Laplace Transforms 1
1.0 INTRODUCTION
Laplace transform is a powerful tool for solving linear differential equations. Laplace transform
converts a linear differential equation with initial conditions to an algebraic problem. This process of
changing from operations of calculus to algebraic operations on transforms is known as operational
calculus, which is an important area of applied mathematics. The advantage of Laplace transforms in
solving initial value problems lies in the fact that the initial conditions are taken care of at the outset
and the solution is directly obtained without resorting to finding the general solution and then the
arbitrary constants.
The name is due to the French mathematician Pierre Simon de Laplace who used this transforms
while developing the theory of probability.

Definition 1.1  Let f (t ) be defined for all t ≥ 0 , then the improper integral

∫e
2st
f ( t ) dt
0

is defined as the Laplace transform of f (t ), if the integral exists.


This integral is a function of the parameter s.
Symbolically, we write L[ f ( t )] 5 F ( s )

Thus, L[ f ( t )] 5 ∫ e2st f ( t ) dt 5 F ( s ) (1)


0

L is called the Laplace transform operator.


The operation is multiplication of f (t ) by e − st and integration between 0 and ∞.

Note  (1)  The parameter s is a real or complex number. We shall assume s is a real number.
Some times p is used instead of s.

1.1 Condition for existence of Laplace transform


Let f (t) be defined for all t ≥ 0 such that
(i) f (t) is piecewise continuous in the interval [0, ∞)
and (ii) f (t) is of exponential order a > 0, then the Laplace transform of f (t) exists for s > a.

Note
1. By piecewise continuity on [0, ∞), we mean that the function is continuous on every finite
sub interval 0 ≤ t ≤ a, except possibly at a finite number of points, where they are jump
discontinuities i.e., f ( x + ), f ( x −) exist, but not equal.
2. f (t) is of exponential order a > 0 if f (t ) ≤ Me at for all t ≥ 0 and some positive constant M.

Equivalently lim{e −at f (t )} is finite.


t →∞

M03_Eng-Maths (Aditya) CH01.indd 1 7/24/2018 1:04:10 PM


1.2  n  Engineering Mathematics-III

Geometrically, it means that the graph of f (t), t ≥ 0 does not grow faster than the graph of the
exponential function g (t ) = Me at , t ≥ 0
For example, t n is of exponential order as t → ∞ .

tn n!
{ lim(e − at t n ) = lim a t
= lim n at  [by L’Hopital’s rule]
t →∞ t →∞ e t →∞ a e

=0
\  t n is of exponential order a > 0.
Similarly, sin at, cos at, e at , e − at all satisfy this condition.
3. The above conditions are sufficient, but not necessary.
1
For example, Laplace transform of exists, but it is not continuous at t = 0and hence it is not
piece-wise continuous in [0, ∞). t
4. Generally, functions that represent physical quantities satisfy these conditions and hence we
assume they have Laplace transforms.
5. When Laplace transform for a given function exists, it is unique. Conversely, two continuous
functions having same Laplace transform must be equal and hence we say that inverse Laplace
transform is unique. This is of practical importance because Laplace transforms are used in
solving boundary value problems.

1.2 Laplace transform of some elementary functions


c
1.  L [c ] 5 , s > 0 and c is a constant.
s
Proof

L [c ] = ∫ e − st c dt
0
∞ ∞
 e − st  c −∞ c
= c ∫ e − st dt = c   = − e − e  =   if s > 0
0
({ t > 0)
0  − s 0 s s
1
In particular, L [1] = , s > 0 and L[0] = 0  n
s

1
2.  L e  5 if s > a
at

s 2a
Proof 
∞ ∞

 e − ( s − a )t 
L e  = ∫ e e dt = ∫ e
− ( s − a )t
at − st at
dt =  
0 0  −(s − a)  0
This limit exists if s − a > 0 as t > 0.
1 1
\ L e at  = e −∞ − e 0  =   if s > a  n
−(s − a) s −a
1
3. Similarly L e  5 if s > 2a
2at

s 1a

M03_Eng-Maths (Aditya) CH01.indd 2 7/24/2018 1:04:14 PM


Laplace Transforms  n  1.3

n!
4.  L  t n  5 n 1 1 if s > 0 and n = 0, 1, 2, 3, ....
s
Proof

L t n  = ∫ e − st t n dt ,  s > 0
0
du
Put st = u    \ dt = .
s

When t = 0, u = 0 and when t = ∞, u = ∞



u n du
\ L t n  = ∫ e −u n
0 s s

1 1 Γ ( n + 1)
∫e
−u
= n +1
u ( n +1) −1 du = n +1
Γ ( n + 1) =   if n + 1 > 0, s > 0
s 0 s s n +1
We know, if n is a positive integer, then Γ( n + 1) = n !
n!
\  L t n  = n
s n +1
1 1
L [1] 5 , s > 0  and  L [t ] 5 2 , s > 0
Corollary            
s s
a
5.  L [sin at ] 5 , s>0
s 1 a2
2

Proof 
∞ ∞
 e − st  d 
L [sin at ] = ∫ e − st sin at dt =  2  −s sin at − (sin at ) 
s + a
2
0 dt 0

 e − st 
= 2 ( −s sin at − a cos at ) 
s + a
2
0

e0 a
= 0− (0 − a cos 0) = 2   if s > 0 [{ e −∞ = 0 if s > 0]
s + a2
2
s + a2 n
s
6.  L [cos at ] 5 , s>0
s 2 1 a2
Proof
∞ ∞
 e − st  d 
L [cos at ] = ∫ e − st cos at dt =  2  −s cos at − dt (cos at )    
s + a
2
0  0

 e − st 
= 2 [ −s cos at + a sin at ]
s + a
2
0
e0 s
= 0− ( −s cos 0 + a sin 0) = 2 if s > 0
s +a
2 2
s + a2

M03_Eng-Maths (Aditya) CH01.indd 3 7/24/2018 1:04:18 PM


1.4  n  Engineering Mathematics-III

1 s
In particular, L [sin t ] =
  and L [cos t ] = 2
s +1 2
s +1
The Laplace transform of many functions can be obtained from the Laplace transform of
elementary functions and some general properties just as in differentiation of functions. One such general
property of Laplace transform is the linearity property.

1.3 Some properties of Laplace transform


Property 1  Linearity property
Let f (t ) and g (t ) be any two functions whose Laplace transform exist and a, b are any two constants
then
L [af (t ) + bg (t )] = aL [f (t )] + bL [g (t )]

Proof  By definition,

L [af (t ) + bg (t )] = ∫ e − st [af (t ) + bg (t )] dt
0

∞ ∞

= ∫ e − st af (t ) dt + ∫ e − st bg (t ) dt
0 0

∞ ∞  n
= a∫ e − st f (t ) dt + b ∫ e − st g (t ) dt = aL [f (t )] + bL [g (t )]
0 0

Note  Because of this property the Laplace transform operator L is a linear operator.
Using this we can find (1) L [sinh at ] and (2) L [cosh at ]

Proof
 e at − e − at 
1. L [sinh at ] = L  
 2 
1
= {L [e at ] − L [e − at ]},  [by linearity property]
2
1 1 1 
= − if s > a, s > −a

2  s − a s + a 
s + a − (s − a) 2a a
if s > a
= = = 2 
2(s − a)(s + a) 2(s − a ) s − a
2 2 2

a
\ L [sinh at ]5  if s > a
s 2 a2
2

 e at + e − at  1 − at
2. L [cosh at ] = L   = {L [e ] + L [e ]}
at

 2  2
1 1 1 
= + if s > a, s > −a

2  s − a s + a 

M03_Eng-Maths (Aditya) CH01.indd 4 7/24/2018 1:04:21 PM


Laplace Transforms  n  1.5

s +a+s −a = 2s s
= = 2 2
if s > a
2(s − a)(s + a) 2(s − a ) s − a
2 2

s
\ L [cosh at ]5 if s > a  n
s 2 a2
2

WORKED EXAMPLES
Example 1 
Find the Laplace transform of t 21 2.

Solution.

Let f (t ) = t −1 2   \  L t −1 2  = ∫ e − st t −1 2 dt
0

dx
Put st = x  \    s dt = dx   ⇒  dt =
s
When t = 0, x = 0 and when t = ∞, x = ∞
∞ −1 2
x dx
\ L t −1 2  = ∫ e − x   ⋅
0
s s

1

1
∞ 
∫e ∫e
−x
= x −1 2 dx = −x
x1 2 −1 dx = 1 Γ(1/2) = p { Γ(1/2) = p 
s s  
0 0
s s

Example 2
Find the Laplace transforms of (i)  sin 2t sin3t.

Solution.
(i) Let f (t) = sin 2t sin3t
1 1
=
2
[cos(3t − 2t ) − cos(3t + 2t )] = 2 [cos t − cos 5t ]
1
\ L [f (t )] =
2
[ L (cos t ) − L (cos 5t )]
s  s s + 25 − s − 1 s 
2 2
1 s 24 
=  2 2− 2 = =  2
2  s + 1 s + 5  2 (s + 1)(s + 25) 2  (ss + 1)(s + 25) 
2 2 2 2

12s
⇒ L [sin 2t sin 3t ] =
(s + 1)(s 2 + 25)
2

M03_Eng-Maths (Aditya) CH01.indd 5 7/24/2018 1:04:24 PM


1.6  n  Engineering Mathematics-III

We shall now list some important Laplace transform pairs f (t) and F (s) for ready reference to do
problems.
Function f (t) Laplace transform
F (s) = L[f (t)]
n!
(1) t n , n = 0, 1, 2, 3, … , s > 0, n = 0, 1, 2, 3, …
s n +1

Γ(a + 1)
(2) t a, a > 0 , a > 0, s > 0
s a +1

1
(3) e at ,s >a
s −a

a
(4) sin at ,s >0
s 2 + a2
s
(5) cos at ,s >0
s 2 + a2

a
(6) sinh at ,s > a
s 2 − a2

s
(7) cosh at ,s > a
s 2 − a2

−1 p
(8) t 2
,s >0
s

Example 3
Find the Laplace transforms of (i)  cos22t (ii)  sin32t.
Solution.
1 + cos 4t
(i) Let f (t ) = cos 2 2t =
2
\
1
L[ f (t )] =
2
[ L(1) + L (cos 4t )]
1 1 s  1  s 2 + 16 + s 2  1  2s 2 + 16  s2 + 8
=  + 2 2
=  =  2 =
2  s s + 4  2  s( s + 16)  2  s( s + 16)  s( s 2 + 16)
2

1
(ii) We know sin 3u = 3 sin u − 4 sin 3 u   ⇒  sin 3 u =
4
[3 sin u − sin 3u]
\ 1 
L [sin 3 2t ] = L  (3 sin 2t − sin 6t ) 
4 

M03_Eng-Maths (Aditya) CH01.indd 6 7/24/2018 1:04:27 PM


Laplace Transforms  n  1.7

3 1
= L [sin 2t ] − L [sin 6t ]
4 4
3 2 1 6
= ⋅ 2 − ⋅ 2
4 s +2 2
4 s + 62
3 1 1 
=  2 − 2
2  s + 4 s + 36 
3  s 2 + 36 − s 2 − 4  3  32  48
=  2 = =
2  (s + 4) (s 2 + 36)  2  (s 2 + 4) (s 2 + 36)  (s 2 + 4) (s 2 + 36)

Property 2  First shifting property (or the s-shifting)
If L [ f (t )] 5 F ( s ) then (i)  L e at f (t )  5 F ( s 2 a ) if s 2 a > a
L e − at f (t )  5 F ( s 1 a ) if s 1 a > a 
(ii) 

Proof  Given L [f (t )] = F (s )

\ L e at f (t )  = ∫ e − st e at f (t ) dt
0

= ∫ e − ( s − a )t f (t ) dt = F (s − a)  (if s − a > a)


0

Similarly, L e f (t )  = F (s + a)  if s + a > a


− at
n

Note  The above properties are called shifting properties because the multiplication of f (t) by
e at and e − at shifts the argument s to s − a, s + a respectively.

The results can be rewritten as L e at f (t )  = [F (s )]s →s − a = L[ f (t )]s → s − a

and L e − at f (t )  = [F (s )]s →s + a = L[ f (t )]s → s − a


‘s → s + a ’ means s is replaced by s + a or s changed to s + a

Property 3  Change of scale property


1 s
If L [ f ( t )] 5 F ( s ), then L [ f ( at )] 5 F  , a > 0
a  a
Proof  Given L [f (t )] = F (s ) ( where s > a)

Now L [f (at )] = ∫ e − st f (at ) dt


0

1
Put    at = u \ adt = du   ⇒  dt = du , a > 0
a
When t = 0, u = 0 and when t = ∞, u = ∞

M03_Eng-Maths (Aditya) CH01.indd 7 7/24/2018 1:04:33 PM


1.8  n  Engineering Mathematics-III

∞ − su ∞
du 1 − ( s a )u 1  s
\ L [f (at )] = ∫ e a
⋅ f (u ) = ∫e f (u ) du = F    n
0
a a0 a  a
1  s − a 1  s + a
Note L e at f (bt )  = F  ;  L e f (bt )  = F 
− at

b  b  b b 

WORKED EXAMPLES
Example 4
Find the Laplace transform of the following functions:
(i) e at t n , n e N , (ii) e at sin bt , (iii) e at sinh bt .  

Solution.
(i) L e at t n  =  L (t n )  s →s − a , by shifting formula

 n!  n!
=  n +1  =
 s  s→ s − a ( s − a ) n +1

(ii) L e at sin bt  = [ L (sin bt ) ]s →s − a       =  2


b  b
2
=
 s + b  s →s − a (s − a) + b
2 2

(iii) L e at sinh bt  = [ L (sinh bt ) ]s →s − a =  2


b  b
2
=
 s − b  s →s − a (s − a) − b
2 2

Example 5
 cos t
Find L  .
 t 
Solution.
( t) +( t) −( t)
2 4 6

cos t = 1− +…
2! 4! 6!
52
cos t 1  t t 2 t 3 … = t −1 2 − 1 t 1 2 + 1 t 3 2 − t + …
\ = 1 − + − + 
t  2! 4 ! 6 ! 2! 4! 6!
t 
 cos t  1 1 1
 = L [t ] − L [t ] + L [t ] − L [t ] + …
−1 2 12 32 52
L
 t  2 ! 4 ! 6 !

p 1 Γ (3 2) 1 Γ (5 2) 1 Γ (7 2) …
= − + − +
s 2! s 3 2 4! s 5 2 6! s 7 2

p 1 2 Γ (1 2) 1 3 2 ⋅1 2 Γ (1 2) 1 5 2 ⋅ 3 2 ⋅1 2 ⋅ Γ (1 2)
= − + −
s 2!s 3 2 4! s5 2 6! s7 2
p  1 1 1 1 1 1  p − 41s
= 1 − 1! 4s + 2 ! 2
− 3
+ … = e
s  ( 4s ) 3! ( 4s )  s

M03_Eng-Maths (Aditya) CH01.indd 8 7/24/2018 1:04:36 PM


Laplace Transforms  n  1.9

Example 6
Find the Laplace transform of cosh at cos at.

Solution.
 e at + e − at 
L [cosh at cos at ] = L  ⋅ cos at 
 2 
1 1
= L [e at cos at ] + L [e − at cos at ]
2 2
1 1
= [ L [cos at ]]s →s − a + [ L [cos at ]]s →s + a
2 2
1 s  1 s 
=  2 +
2  s + a2  s →s − a 2  s 2 + a2  s →s + a
1 s −a s+a 
=  + 2
2  (s − a) + a (s + a) + a 
2 2 2

(s − a)[(s + a) 2 + a2 ] + (s + a)[(s − a) 2 + a2 ]
=
2[(s − a) 2 + a2 ][(s + a) 2 + a2 ]
(s − a)[(s + a)[s + a + s − a] + a2 [s + a + s − a]]
=
2(s 2 − 2as + a2 + a2 )(s 2 + 2as + a2 + a2 )
22ss((ss 22 −− aa22 )) ++ 22sa
sa2
2
==
22((ss ++ 22aa −− 22as
22 22
as)()(ss ++ 22aa2 ++ 22as
22 2
as))

2s − 2a s + 2a s
33 22 22
== 2s2 − 2a2 s2 + 2a 2s 2
[(ss 2 ++ 22aa2 )) 2 −− 44aa2ss 2 ]]
22[(
22ss33 ss33
== =
=
22[[ss 44 ++ 44aa22ss 22 ++ 44aa44 −− 44aa22ss 22 ]] ss 44 ++ 44aa44

EXERCISE 1.1
Find the Laplace transform of the following functions:
1. sin 6t + 5e-3t + cos 3t + 2  2. cos33t   3.  sin 2t cos 3t
4. sin 5t ⋅ sin 3t   5. sinh 2t  6. (t2 + 1)2
2

7. (sin t - cos t) 2  
8.  cosh (5t + 2)   9.  sin 3t cos2t
-2t
10. e sin 4t 11.  t3 e-3t 12.  e-t sin2t
13. e (3 sinh 2t - 5 cosh 2t) 14. (t + 2) e 15. 
-t 2 t
e-3t cos2t
16.  (1 + te-t)3 17. e4tsin 2t cos t 18.  e3tsin 2t.sin t
19. (2e + e ) ⋅ t 20. 
t -2t 2 2 12
t 21.  e −5t t 7
e t t −1 2 
e at cosh bt 24. 
22.  e at cos bt 23. 
(1 + te −t )3
e −3t ( 2 cos 5t − 3 sin 5t ) 27. 
25. cosh at sin bt 26. 

M03_Eng-Maths (Aditya) CH01.indd 9 7/24/2018 1:04:38 PM


1.10  n  Engineering Mathematics-III

t 3 ( t −1) 2
28.  e sin 2t 29.  30. sin t
t
e at
31.  .
t

ANSWERS TO EXERCISE 1.1

6 5 s 2 s 1 3  1 5 1 
1. + + 2 + 2.   2 + 2  3.   2 − 2 
s + 36 s + 3 s + 9 s
2
4  s + 81 s + 9  2  s + 25 s + 1

4.  2
s 1 1  1 s 1 24 4 1
−  5.   −  6.  + +
2  s + 4 s 2 + 64  2  s 2 − 16 s  s5 s3 s

1 2 1  e2 e −2  3 1 s 2 − 13 
7. − 2 8.   +  9.   2 + 4 
s s +4 2 s − 5 s + 5 2  s − 9 s − 10s 2 + 169 

4 6 2
10. 11.  12. 
s + 4s + 20
2
(s + 3) 4
(s + 1)(s + 2s + 5)
2

1 − 5s 2( 2s 2 − 2s + 1) 1 1 s+3 
13. 2 14.  15.  +
s + 2s − 3 (s − 1)3 2  s + 3 (s + 3) 2 + 4 

1 3 6 6 1 3 1 
16. + + + 17.   + 
s (s + 1) (s + 2) (s + 3)
2 3 4
2  (s − 4 ) + 9 (s − 4 ) + 1 
2 2

1  s −3 s −3  8 2 8
18.  −  19.  + +
2  (s − 3) + 1 (s − 3) + 9 
2 2
(s − 2) (s + 4) (s + 1)3
3 3

p 7! s −a
20. 32
21.  22. 
2s (s + 5) 8
(s − a) 2 + b 2
s −a p   if s − 1 > 0
23.  24. 
(s − a) 2 − b 2 s −1

 1 1  2s − 9
25.  b  + 2
26. 
2  (s − a) + b
2 2
(s + a) + b 
2
( s + 3) 2 + 25

1 3 6 6 48
27. + + + 28. 
s (s + 1) (s + 2) (s + 3)
2 3 4
[(s − 1) + 4][(s − 1) 2 + 36]
2

p(1 + 2s ) − 4 s p p
29. 30.  3 2 e −1 4 s 31.  ,s-a>0
2s 32
2s s −a

M03_Eng-Maths (Aditya) CH01.indd 10 7/24/2018 1:04:45 PM


Laplace Transforms  n  1.11

1.4  Differentiation and Integration of Transforms


Theorem 1.1  Differentiation of transform (or multiplication by tn)
dn
If L[f (t)] 5 F (s) then L[t n f ( t )] 5 (21) n ( F ( s )), n 5 1, 2, 3,⊃
ds n

Proof  By definition, F (s ) = ∫ e − st f (t ) dt
0

Differentiating w.r.to s under the integral sign by Leibnitz’s theorem,



dF (s )
we get = ∫ −te − st f (t ) dt (1)
ds 0

= − ∫ e − st tf (t ) dt = − L [tf (t )]
0

dF ( s )
\ L[tf ( t )] 52
ds
Differentiating (1) w.r.to s, we get

d 2 ( F ( s))
= − ∫ −te − st (tf (t )) dt
ds 2 0
∞ ∞

= ∫ e − st t 2 f (t ) dt = ( −1) 2 ∫ e − st t 2 f (t ) dt = ( −1) 2 L[t 2 f (t )]


0 0

d 2 F ( s)
\ L[t 2 f ( t )] 5 (21)2
ds 2
Proceeding in this way, we get

d n F (s )
L [t n f (t )] = ( −1) n , n = 1, 2, 3, … n
ds n

Theorem 1.2  Integration of transform (or Division by t)



 f (t ) 
If L[f (t)] 5 F (s), then L   5 ∫ F ( s ) ds
 t  s
Proof 

By definition, F (s ) = ∫ e − st f (t )dt
0
Integrating both sides w.r.to s from s to ∞, we get
∞ ∞∞

∫ F( s)ds = ∫ ∫ e f (t )dt
− st

s s 0

M03_Eng-Maths (Aditya) CH01.indd 11 7/24/2018 1:04:51 PM


1.12  n  Engineering Mathematics-III

Since t is independent of s, changing the order of integration, we have


∞ ∞
∞ ∞
 ∞ − st   e − st 
∫s F ( s )ds = ∫0 f (t )  ∫s e ds  dt = ∫ f (t ) 
 −t  s
 dt
  0

f (t ) −∞
=∫ [e − e − st ] dt
0
−t

 f (t ) 
dt = L 
f (t ) 
= ∫ e − st    [{ t > 0; s > 0; e −∞ = 0]
0  t   t 

 f (t ) 
\ L  5 ∫ F ( s ) ds  n
 t  s
Note
1. In theorem 1.1 multiplication of f (t) by t results in differentiation of L[f (t)] = F (s) w.r. to s.
Multiplication by t2 results in differentiation of L[f (t)] twice and so on, with proper sign.
2. In theorem 1.2 division of f (t) by t results in integrating F (s) from s to ∞.
Division by t 2 will result in integration w.r.to s twice
∞∞
 f (t ) 
\ L  2  = ∫ ∫ F (s ) ds ds
 t  ss
Proceeding like this, we get
∞∞ ∞
 f (t ) 
L  n  = ∫ ∫ … ∫ F (s )(ds ) n , where n is a positive integer.
 t  ss s

WORKED EXAMPLES
Example 1 
Find L[t cos3t].

Solution.
Let f (t) = cos3t
\  t f (t) is the multiplication of f (t) by t.
1
We know cos 3t = 4 cos3 t − 3 cos t   ⇒  cos3 t = [cos 3t + 3 cos t ]
4
1 1 s s 
\ F (s ) = L [f (t )] = L [cos3 t ] =
[L [cos 3t ] + 3L [cos t ]] =  2 +3 2 
4 4 s + 9 s + 1
d d  1  s 3 s  
\ L [t cos3 t ] = − F (s ) = −   2 + 
ds ds  4  s + 9 s 2 + 1 

=−  +
2
{
1  (s 2 + 9) ⋅1 − s ⋅ 2s 3 (s + 1) ⋅1 − s ⋅ 2s 

}
4 ( s 2 + 9) 2 (s 2 + 1) 2 


1  (9 − s 2 ) 3(1 − s 2 )  1  (s 2 − 9) 3(s 2 − 1) 
=−  2 + =  + 
4  (s + 9) 2 (s 2 + 1) 2  4  (s 2 + 9) 2 (s 2 + 1) 2 

M03_Eng-Maths (Aditya) CH01.indd 12 7/24/2018 1:04:54 PM


Laplace Transforms  n  1.13

Example 2
Find L[te2t sin t ].

Solution.
Let f (t ) = e −t sin t

 1  1
\ F (s ) = L [f (t )] = L e −t sin t  = L [sin t ]s →s +1 =  2  =
 s + 1 s →s +1 (s + 1) + 1
2

Now te −t sin t = tf (t ) is the multiplication of f (t) by t.

dF (s ) d  1 
\ L [tf (t )] = − =−  
ds ds  (s + 1) + 1
2

 
−1  d  1  ( −1) ⋅ 2 x 
= −  2(s + 1)  { dx  2 2
= 2 
{
 (s + 1) 2 + 1 }  x + a  ( x + a2 )2 
2

 
2(s + 1)
=
{(s + 1) }
2
2
+1

Example 3
Find L[t2e23t sin 2t].

Solution.
Let f (t) = sin 2t
d 2 F (s )
\ L [t 2 f (t )] = L [t 2 sin 2t ] = ( −1) 2 ,
ds 2
where
2
F (s ) = L [sin 2t ] =
s +4
2

d2  2  d  ( −1) ⋅ 2s 
\ L [t 2 f (t )] =   =2  2
ds 2  s 2 + 4  ds  (s + 4) 2 
d  s 
= −4
ds  ( s 2 + 4) 2 

[( s 2 + 4) 2 ⋅1 − s ⋅ 2( s 2 + 4) ⋅ 2 s] [ss 2 + 4 − 4 s 2 ] [3s 2 − 4]
= −4 = −4 =+4 2
( s + 4)
2 4
( s + 4)
2 3
( s + 4)3
4[3s 2 − 4]
\ L [t 2 sin 2t ] =
(s 2 + 4 ) 3
\ L [t 2e −3t sin 2t ] = L [t 2 sin 2t ]s →s + 3

 4(3s 2 − 4)  4 ⋅ [3(s + 3) 2 − 4] 4[3s 2 + 18s + 23]


= 2  = =
 (s + 4 )  s → s + 3 [(s + 3) 2 + 4]3 (s 2 + 6s + 13)3
3

M03_Eng-Maths (Aditya) CH01.indd 13 7/24/2018 1:04:57 PM


1.14  n  Engineering Mathematics-III

Example 4

Find L 
1 2cos 2t 
.
 t 
Solution.
Let f (t) = 1 - cos 2t
1 s
\  F (s) = L[f (t)] = L [1 − cos 2t ] = −
s s2 + 4

\ 1 − cos 2t   f (t ) 
L  = L  t  [division of f (t) by t]
 t   

= ∫ F ( s) ds  [By theorem 1.2, page 1.11]


s


1 s 
= ∫ − 2 ds
s  s s + 4 
∞ ∞
1 s ds
= ∫ ds − ∫ 2
s
s s s +4

1 2s ds
= [ loge s ]s −

2 ∫s s 2 + 4

 1 
= loge s − loge (s 2 + 4) 
 2 s
1
= [2 loge s − loge (s 2 + 4)]s∞
2
1
= [loge s 2 − loge (s 2 + 4)]s∞
2

1  s 2 
= loge  2 
2  s + 4  s

1 1 
=  loge
2 4
 1+ 2 
 s s 
1 1   4 
= loge 1 − loge
2 4  since s 2 → 0 as s → ∞ 
 1+ 2   
 s 

1 s2  1 s2 + 4
= 0 − loge 2  = loge  [{ loge 1 = 0]
2 s + 4 2 s2

M03_Eng-Maths (Aditya) CH01.indd 14 7/24/2018 1:04:59 PM


Laplace Transforms  n  1.15

Example 5
12e t
Find the Laplace transform of .
t
Solution.
Let f (t) = 1- et
1 1
\ F (s ) = L [f (t )] = L [1 − e t ] = L [1] − L [e t ] = −
s s −1
1− e t   f (t )  ∞
\
 t  ∫s
L =L  = F (s )ds  [division of f (t ) by t ]
 t 

1 1 
= ∫ −  ds
s
 s s − 1
= [log s − log(s − 1)]s∞

 s 
= log
 s − 1 s
1 1 s  s − 1
= log 1 − log = 0 − log = − log = log 
1 1 s −1  s 
1− 1−
s s
Example 6
sin at
Find the Laplace transform of .
t
Solution.
sin at a sin at sin at
Since lim = lim = a, is continuous  ∀ t ≥ 0
t →0 t t → 0 at t
sin at
\ Laplace transform of exists.
t
Let f (t ) = sin at

a
\ F (s ) = L [f (t )] = L [sin at ] =
s 2 + a2

 sin at   f (t ) 
\  L   = L  t  = ∫ F ( s) ds  [division of f (t ) by t ]
 t    s

a ds
=∫ 2
s s + a2

1  s  s p  s  s
= a ⋅  tan −1    = tan −1 ∞ − tan −1   = − tan −1   = cot −1  
a  a s  a 2  a  a

Note  L 
cos at  cos at
 does not exist because the function t is discontinuous at t = 0.
 t 

M03_Eng-Maths (Aditya) CH01.indd 15 7/24/2018 1:05:03 PM


1.16  n  Engineering Mathematics-III

Example 7
e 2at 2e 2bt
Find the Laplace transform of .
t
Solution.
Let f(t) = e-at - e-bt

1 1
\ F (s ) = L [f (t )] = L [e − at − e −bt ] = −
s +a s +b

 e − at − e −bt   f (t ) 
\ L  = L  t  ∫ F (s ) ds 
= [division by t]
 t    s

 1 1 
= ∫ − ds
s  s + a s + b 

= [ log e ( s + a) − log e ( s + b) ]s


 s + a
= log e
 s + b  s

  a   a
 1 + s   1 + s  s+a s+b
= log e    = log e 1 − log e   = − log e = log e
 b
1 +   1+ b s+b s+a
  s   s  s
Example 8
 e 23t sin 2t 
Find L   .
 t 
Solution.
Let f (t) = e-3t sin 2t
2
\ F (s ) = L [f (t )] = L [e −3t sin 2t ] = [L [sin 2t ]]s →s + 3 =
(s + 3) 2 + 4

 e −3t sin t   f (t ) 
\ L  = L  ∫ F ( s) ds 
= [division of f (t ) by t ]
 t   t  s

2
=∫ ds
s ( s + 3) + 4
2


1  s + 3 
= 2  tan −1 
2  2   s
 s + 3 p  s + 3  s + 3
= tan −1 ∞ − tan −1   = − tan −1   = cot −1 
 2  2  2   2 

M03_Eng-Maths (Aditya) CH01.indd 16 7/24/2018 1:05:05 PM


Laplace Transforms  n  1.17

Example 9

Find L 
1 2cos t 
2  .
 t 
Solution.
Let f (t) = 1- cos t
1 s
\ F (s ) = L [1 − cos t ] = L [1] − L [cos t ] = − 2
s s +1
\ 1 − cos t   f (t ) 
L 2  = L  2  [dividing f (t) by t2]
 t   t 
∞∞

= ∫ ∫ F (s )ds ⋅ ds  [by theorem 1.2]


s s

∞ ∞ ∞
1 s  ds 1 2s ds
Now ∫ F (s )ds = ∫  − 2  ds = ∫ − ∫ 2
s s 
s s + 1  s
s 2 s +1

 1 
= loge s − loge (s 2 + 1) 
 2 s
1 ∞
=  2 loge s − logge (s 2 + 1) 
2 s


1  s2  
= loge  2  
2  s + 1  s

1  1 
=  log e
2   1  
 1 + 2   
   s    s
1  1  1  s2  1 s2 + 1
= log e 1 − log e  = − log e  2  = log e 2
2  1  2  s + 1 2 s
  1 + 2  
s 

1 − cos t  1  s 2 + 1
\ L 2  ∫ 2 e  s 2  ds
= log
 t  s

1
2 ∫s
= [loge (s 2 + 1) − loge s 2 ] ds


1
2 ∫s
= [loge (s 2 + 1) − 2 loge s ] ds

∞ ∞
1 1
2 ∫s
= loge (s 2 + 1) ds − ∫ loge s ds = I 1 − I 2
s
2
∞ ∞
∞ 1
where I 1 = ∫ loge (s 2 + 1) ds = s loge (s 2 + 1)  s − ∫ 2 2s ⋅ s ds
s s s +1

M03_Eng-Maths (Aditya) CH01.indd 17 7/24/2018 1:05:08 PM


1.18  n  Engineering Mathematics-III


∞ s2 +1−1
= s loge (s 2 + 1)  s − 2∫ ds
s s2 +1

∞  1 
= s loge (s 2 + 1)  s − 2∫ 1 − 2  ds
s  ( s + 1) 
∞ ∞
= s loge (s 2 + 1)  s − 2 s − tan −1 s  s
∞ ∞
and I 2 = loge s ds = [s loge s ]s∞ − s ⋅ 1 ds =[s loge s ]s∞ − [s ]s∞
∫s ∫s s
1 − cos t  1
\ L
 t 2

{ 2 ∞ −1 ∞ ∞ ∞
 = 2 [s loge (s + 1)]s − 2[s − tan s ]s − [s loge s ]s − [s ]s } { }

s 
=  loge (s 2 + 1) − s + tan −1 s − s loge s + s 
 2 s

= s loge s 2 + 1 + tan −1 s − s loge s 
 s

 s2 +1 
= s loge 2
+ tan −1 s 
 s  s

ss + + 11
2 2
=
2
+ 11 + tan −−11 ∞ − s log ss 2 + − −1
= lim
lim ss log
logee + tan ∞ − s logee − tan
tan −1 ss
s →∞
s →∞ ss 22 ss 22
 s 22 + 1 

= lim ss log 1+ + 11  + p
p
− s log s + 1 − tan −−11 s
= lim→∞ 2
logee 1++ 22  + − s logee  2  − tan s
ss →∞ 2  s  2 s 2  ss 
2

 s 22 + 1 
= ss  11 − 11 + 11 − …  p
… +
p
− −1 s − s log
−1 s +1
= lim
lim 
→∞ 2  s 2 2 − 4 + 6 −  + − tan
tan s − s log ee  
ss →∞ 2  s 2s
2s 4 3s
3s 6
 22  ss 

11 1 1   s2 + 1
= lim  − 3 + 5 − … + cot −1 s − s loge  
s →∞ 2  s 
2s 3s  s 

 s2 + 1 s2 +1
= 0 + cot −1 s − s loge  −1
 = cot s − s loge
 s  s

Example 10
t , 0 < t < 4
Find the Laplace transform of f ( t ) 5  .
 5, t > 4
Solution.
t , 0 < t < 4
Given f (t ) = 
5, t > 4

M03_Eng-Maths (Aditya) CH01.indd 18 7/24/2018 1:05:10 PM


Laplace Transforms  n  1.19

∞ 4 ∞
L [f (t )] = ∫ e − st f (t ) dt = e − st f (t ) dt + e − st f (t ) dt
\
0
∫ 0
∫ 4
4 ∞

= ∫ e − st t dt + ∫ e − st 5 dt
0 4
4 ∞
 e − st e − st   e − st 
= t ⋅ − 1. 2
+ 5 
 −s ( −s )  0  −s  4
 4e −4 s e −4 s  e0   5
= − + 2 −  0 + 2   − [e −∞ − e −4 s ]
 s s  s  s
−4 s −4 s −4 s
e e 1 e 1
= − 2 + 2 = + 2 (1 − e −4 s )
s s s s s

Example 11
Find the Laplace transform of f (t ) 5 t 21 1 t 11 , t ≥ 0 .

Solution.
Given f (t ) = t − 1 + t + 1 , t ≥ 0 0 1 ∞

We know x =x if x ≥ 0 and x = − x if x < 0


If 0 ≤ t < 1, then t + 1 > 0 and t - 1 < 0   \  t + 1 = t + 1 and t - 1 = - (t - 1)

\ f (t ) = −(t − 1) + t + 1 = 2   if 0 ≤ t < 1
If t ≥ 1, t + 1 > 0, t − 1 ≥ 0   \  t + 1 = t + 1, t − 1 = t − 1

\ f (t ) = t + 1 + t − 1 = 2t   if t ≥ 1
2 if 0 ≤ t < 1
Thus f (t ) =     
2t if t ≥ 1
   ∞ 1 ∞

\ L [f (t )] = ∫ e − st f (t ) dt = ∫ e − st f (t ) dt + ∫ e − st f (t ) dt
0 0 1
1 ∞

= ∫ e − st 2 dt + ∫ e − st 2t dt
0 1
11 ∞
e 
−−st
 te −−stst e −−stst  ∞
== 22  e  ++ 22  te −−11⋅⋅ e 2 
st

 −−ss 00  −−ss ((−−ss)) 2 1


1
2   e −−ss
e −−ss  
== −− 2 [[ee−−ss −− ee00 ]] −− 22 00 −−  e ++ e 2  
ss   ss ss 2  

22 −−ss −−ss  1 1  2 2e −−ss 2
== −− ((ee −−11)) ++ 22ee  ++ 2  == ++ 2e2 == 22 [[ss ++ ee−−ss ]]
1 1 2
ss  ss ss 2  ss ss 2 ss 2

M03_Eng-Maths (Aditya) CH01.indd 19 7/24/2018 1:05:16 PM


1.20  n  Engineering Mathematics-III

EXERCISE 1.2
Find the Laplace Transform of the following functions.
1. te −t sin 2t      2. t cosh 3t          3.  te at sin at       4. t2 cos22t

5. (t + 2) cos 3t    6. te −2t sinh t       


 7. te −t cosh t       8.  t 2e 3t sinh t
cos 2t − cos 3t e −3t − e −4t 1− e −t cos 4t sin 2t
9.      10.       
11.        12. 
t t t t
sin 2 t 1− e − at sin t , 0 < t < p
13.       14.  15. f (t ) = 
       
t t 0, t ≥p
e t if 0 < t < 1 cos t if 0 < t < p
16. f (t ) =           17. f (t ) = 
 0 if t ≥ 1 sin t if t ≥ p
t
18. f (t ) =  c if 0 < t < c     19.  t cos 3t.  20. 
te −4t sin 3t .
1, if t ≥ c

21. t 2 e − t cos t .        22. t2 sin at.   23.  t sin 3t cos 2t.

e at − cos bt
24. cos at − cos bt .     25.  .
t t

ANSWERS TO EXERCISE 1.2

4(s + 1) s2 + 9 2a(s − a) 1 s (s 2 − 48)


1.  2. 2 3.      4.  3 + 2
(s + 2s + 5)
2 2
( s − 9) 2 (s − 2as + a )
2 2 2
s (s + 16)3
s2 − 9 2s 2(s + 2) s 2 + 2s + 2 1 1
5. + 2  6. 2   7.      8.  +
( s + 9) s + 9
2 2
(s + 4s + 3) 2 s + 4s + 4s
4 3 2
( s − 2) ( s − 4 ) 3
3

 s2 + 9 s + 4  s + 1    12.  1  −1 s s
 10.  log 
1
9. log  2   11. 
log    tan − tan −1 
2  s + 4  s + 3  s  2 2 6

1 s2 + 4 (1 + e − ps )
 s + a  15.  1 − e ⋅ e −s
13. log  14.  log      16. 
2 s2  s  s2 +1 s −1
1 1 − sc s2 − 9
17. [s + (s − 1)e − ps ]  18.  − (e − 1)    19.  2
s +1
2 2
sc ( s + 9) 2
6(s + 4) 2(s + 1)[s 2 + 2s − 2] 2a[3s 2 − a2 ] 5s s
20. 2  21. 22.         23.  2 + 2
[(s + 4) + 9]
2
[(s + 1) + 1]
2 3 (s + a )
2 2 3
(s + 25) (s + 1) 2
2

1 s2 + b2  1 s2 + b2
24. loge  2 2 
25. log
s + a 
e
2 2 (s − a) 2

M03_Eng-Maths (Aditya) CH01.indd 20 7/24/2018 1:05:31 PM


Laplace Transforms  n  1.21

1.5 Laplace transform of derivatives and integrals


Theorem 1.3  Laplace transform of derivative 
Let f (t) be continuous for all t ≥ 0 and f ′(t) be piecewise continuous on every finite interval 0 ≤ t ≤ a
in [0, ∞). If f (t) and f ′(t) are of exponential order as t → ∞, then
L[ f ′( t )] 5 s L[ f ( t )] 2 f (0) .

Proof  By definition

L [f ′(t )] = ∫ e − s t f ′(t ) dt
0

= [e − s t f (t )]0∞ − ∫ −se − s t f (t ) dt 
0 [using integration by parts]

= lim e − s t f (t ) − e 0 f (0) + s ∫ e − s t f (t ) dt
t →∞
0

−s t
Since f (t) is of exponential order,      lim e f (t ) = 0, if s > a
t →∞

\ L [f ′(t )] = s ∫ e − s t f (t ) dt − f (0) = s L [f (t )] − f (0)


0
\ L[ f ′( t )] 5 s L[ f ( t )] 2 f (0)  n

Note  Laplace transform of f ′(t) results in multiplication of L [f (t)] by s

Corollary L[f ′′(t )] = s 2 L [f (t )] − sf (0) − f ′(0)

We have, L [f ′(t )] = s L [f (t )] − f (0)

\ L [f ′′(t )] = s L [f ′(t )] − f ′(0)

= s {s L[ f (t )] − f (0)} − f ′(0) = s 2 L[ f (t )] − s f (0) − f ′(0)

\ L[ f ′′( t )] 5 s 2 L[ f ( t )] 2 s f (0) 2 f ′(0)


Similarly, we can prove
L[ f ′′′( t )] 5 s 3 L[ f ( t )] 2 s 2 f (0) 2 sf ′(0) 2 f ′′(0)
and in general

L[ f ( n ) ( t )] 5 s n L[ f ( t )] 2 s n −1 f (0) 2 s n −2 f ′(0) 2 …2 s f ( n− 2 ) (0) 2 f ( n −1) (0)

Theorem 1.4  Laplace transform of integral 


If f (t) is piecewise continuous in every finite interval 0 ≤ t ≤ a in [0, ∞) and f (t) is of exponential order
a > 0 and if L [f (t )] = F (s ), then

t  F ( s)
L  ∫ f ( u) du 5      for s . a
0  s

M03_Eng-Maths (Aditya) CH01.indd 21 7/24/2018 1:05:36 PM


1.22  n  Engineering Mathematics-III

t  L [f (t )]
i.e., L  ∫ f (u )du  =    for s > a
0  s
t

Proof  Let g (t ) = ∫ f (u ) du
0

Then g ′(t ) = f (t ) and g (0) = ∫ f (u ) du = 0


0

\ L [f (t )] = L [g ′(t )]
t 
= s L [g (t )] − g (0) = s L [g (t )] = s L  ∫ f (u ) du 
0 
1  t
  t
 L[ f ( t )]
\ L [f (t )] = L  ∫ f (u )du    ⇒  L  ∫ f ( u)du 5  n
s 0  0  s

Note
1. The Laplace transform of integral of f (t) results in division of L[f (t)] by s.
t  L [f (t )]
2. Replacing the dummy variable u by t, the above result is usually written as L  ∫ f (t ) dt  =
0  s
 t t
 L [f (t )]
Similarly L  ∫ ∫ f (t ) dt dt  =
0 0  s2
t a t

3. ∫ f (t ) dt = ∫ f (t ) dt + ∫ f (t ) dt
0 0 a

t t a

\ ∫ f (t ) dt = ∫ f (t ) dt − ∫ f (t ) dt
a 0 0

t  t  a 
\ L  ∫ f (t ) dt  = L  ∫ f (t ) dt  − L  ∫ f (t ) dt 
a  0  0 
a

But ∫ f (t ) dt is a constant, say K.


0

a  K
\ L  ∫ f (t ) dt  = L [K ] = K L [1] =
0  s
a  1a
\ L  ∫ f (t ) dt  = ∫ f (t ) dt
0  s0
t  L [f (t )] 1 a
\ L  ∫ f (t ) dt  = − ∫ f (t ) dt
a  s s0

M03_Eng-Maths (Aditya) CH01.indd 22 7/24/2018 1:05:40 PM


Laplace Transforms  n  1.23

WORKED EXAMPLES
Example 1  t

Find the Laplace transform of ∫ te 2t sin t dt .


0

Solution.
t  L (te −t sin t )
L  ∫ te −t sin t dt  =                  [by theorem 1.4] (1)
0  s

Let f (t ) = e −t sin t
1
\ F ( s) = L[ f (t )] = L[e − t sin t ] = L[sin t ]s → s +1 =
( s + 1) 2 + 1
dF (s ) d  1 
\ L [te −t sin t ] = L [tf (t )] = − =−  
ds ds  (s + 1) + 1
2

 ( −1) ⋅ 2(s + 1)  2(s + 1) 2(s + 1)


= − 2
= = 2
 [( s + 1) 2
+ 1]  [( s + 1) 2
+ 1]2
[s + 2s + 2]2
Substituting in (1) we get,

t  1 2(s + 1)
L  ∫ te −t sin t dt  = ⋅ 2
 s (s + 2s + 2)
2
0

Example 2
 t e 2t sin t 
Find L  ∫ dt  .
0 t 
Solution.
 t e −t sin t  1  e −t sin t 
L ∫ dt  = L   [by theorem 1.4]
0 t  s  t 

 e −t sin t  ∞
 = ∫ L [e sin t ] ds 
−t
But L [by theorem 1.2]
 t  s

1
=∫ ds
s ( s + 1) + 1
2

p
= [tan −1 ( s + 1)]∞s = tan −1 ∞ − tan −1 ( s + 1) = − tan −1 ( s + 1) = cot −1 ( s + 1)
2

 t e −t sin t  cot −1 (s + 1)
\ L ∫ dt  =
0 t  s

M03_Eng-Maths (Aditya) CH01.indd 23 7/24/2018 1:05:47 PM


1.24  n  Engineering Mathematics-III

Example 3
 t sin x 
Find L  ∫ dx 1 te 2t cos 2 2t .
0 x 
Solution.

 t sin x   t sin x 
L ∫ dx + te −t cos 2 2t  = L  ∫ dx  + L [te −t cos 2 2tt ](1)
0 x  0 x 
 [By linearity property]

 t sin x   t sin t 
Now L ∫ dx  = L  ∫ dt 
0 x  0 t 
1  sin t  cot −1 s
= L =  [example 6, page 1.15]
s  t  s

L [te −t cos 2 2t ] = L [e −t t cos 2 2t ]


 d 
= L [t cos 2 2t ]s →s +1 =  − L (cos 2 2t ) 
 ds  s → s +1
1 + cos 4t
Let f (t ) = cos 2 2t =
2

\ 1 + cos 4t  1 1 1 s 
L [f (t )] = L   = [L [1] + L [cos 4t ]] =  + 2
 2  2 2  s s + 16 

\ L [t cos 2 2t ] = L [tf (t )]

d
\ L [t cos 2 2t ] = − L [f (t )]
ds
d  1 1 s 
=−  + 2 
ds  2  s s + 16  

1  1 (s 2 + 16) ⋅1 − s ⋅ 2s  1  1 s 2 − 16 
= − − 2 +  =  + 
2 s (s 2 + 16) 2  2 s
2
(s 2 + 16) 2 
11 s 2 − 16 
\ L [te −t cos 2 2t ] =  2+ 2 
2 s (s + 16) 2  s →s +1

1 1 ( s + 1) 2 − 16  1  1 s 2 + 2s − 15 
=  + 2
=  + 2 
2  ( s + 1) [( s + 1) + 16]  2  ( s + 1) ( s + 2 s + 17) 2 
2 2 2

Substituting in (1), we get


 t sin x  cot −1 s 1  1 s 2 + 2s − 15 
L ∫ dx + te −t cos 2 2t  = +  + .
0 x  s 2  (s + 1) 2 (s 2 + 2s + 17) 2 

M03_Eng-Maths (Aditya) CH01.indd 24 7/24/2018 1:05:57 PM


Laplace Transforms  n  1.25

1.5.1 Evaluation of improper integrals using Laplace transform

WORKED EXAMPLES
Example 4

Evaluate ∫ e 22 t t sin 3t dt .
0

Solution.

Let I = ∫ e −2t t sin 3t dt .


0

If f (t ) = t sin 3t ,

then I = ∫ e −2t f (t )dt = [F (s )]s = 2 = F ( 2), [by definition of Laplace transform]


0

and F (s ) = L [f (t )] = L [t sin 3t ]

d d  3   3( −1)  6s
=− L[sin 3t ] = −  2  = − 2 ⋅ 2s = 2
ds ds  s + 9   ( s + 9 ) 2
 ( s + 9) 2

6×2 12 12
\ F( 2) = =   \  I = F ( 2) =
( 2 + 9)
2 2
169 169

Example 5

sin 2 t
Evaluate ∫ e 2t dt .
0
t
Solution. ∞
sin 2 t
Let I = ∫ e −t dt
0
t
2 ∞
sin t
If f (t ) = I = ∫ e −t f (t ) dt = [F (s )]s =1 = F (1), 
,  then  [by definition of L. T]
t 0

But  sin 2 t  ∞
F (s ) = L   = ∫ L [sin t ] ds
2

 t  s


1 − cos 2t 
= ∫ L  ds
s  2 
∞ ∞
1 1 s  1 1 
= ∫  − 2
2 s s s + 4  ds = log e s − log e ( s 2 + 4) 
2 2 s
1 ∞
=  2 log e s − log e ( s 2 + 4)  s
4
1 ∞
= log e s 2 − log e ( s 2 + 4)  s
4

M03_Eng-Maths (Aditya) CH01.indd 25 1  s2   7/24/2018 1:06:01 PM

1 − cos 2t 
= ∫ L  ds
s  2 
1.26  n  Engineering ∞
1 Mathematics-III
1 s  1 1 

= ∫ − 2 ds = log e s − log e ( s 2 + 4) 
2 s  s s + 4  2 2 s
1 ∞
=  2 log e s − log e ( s 2 + 4)  s
4
1 ∞
= log e s 2 − log e ( s 2 + 4)  s
4

1  s2  
= log e  2 
4  s + 4   s

 1   1  2
1 log 1 log 1 − log e  = − 1 log e s
=  e 4 =  e 4
4 1 + 2   4 1+ 2  4 s2 + 4
 s  s  s 

1 s2 + 4 1 1
⇒       
F(s) = loge   and F (1) = loge (1 + 4) = loge 5
4 s2 4 4
loge 5
\       I   = F (1) =
4
Example 6

 1 2 cos t 
Evaluate ∫ e 2t   dt . 
0
 t
Solution.

 1 − cos t 
Let I = ∫ e −t   dt
0
 t

1 − cos t
,   then I = ∫ e f (t ) dt = [ F (s ) ]s =1 = F (1)
−t
If f (t ) =
t 0

1 − cos t 
But F (s ) = L [f (t )] = L  
 t 

= ∫ L [1 − cos t ] ds
s

1 s 
= ∫  − 2  ds
s 
s s + 1


 1 
= loge s − loge (s 2 + 1) 
 2 s

1  s2  
= loge  2  
2  s + 1  s

  1    1 
1 1 1 s2
= loge  1   = log 1 − log  1   = − loge 2
2  1 + 2   2  1 + 2  
e e
2 s +1
 s s  s 

M03_Eng-Maths (Aditya) CH01.indd 26 7/24/2018 1:06:05 PM


Laplace Transforms  n  1.27

1 s2 +1 1  1 + 1 1
⇒ F (s ) = loge 2   and   F (1) = loge  = log 2
2 s 2  1  2 e
1
\ I = F (1) = loge 2
2
Example 7
pe 21/ 4 s  cos t  p 21/ 4 s
Given L [sin t ] 5 3/ 2
, show that L  5 e .
2s  t  s
Solution.
pe −1/ 4 s
Given L [sin t ] =
2s 3/ 2
If f (t ) = sin t
1 1 cos t
then f (0) = 0   and f ′(t ) = cos t ⋅ =
2 t 2 t

We know L [f ′(t )] = s L [f (t )] − f (0)

 1 cos t 
⇒ L  = s L [sin t ] − 0
2 t 

1  cos t  pe −1/ 4 s  cos t  p −1/ 4 s p −1/ 4 s


⇒ L  = s⋅ 3/ 2
  ⇒  L   = 1/ 2 e = e
2  t  2s  t  s s

1.6 Laplace transform of periodic functions and other special type of


functions
Mathematical representation of physical quantities whose values repeat periodically give rise to
periodic functions.

Definition 1.2  A real function f (t) is said to be periodic if there exists a positive constant T such that
f (t + T) = f (t) for all values of t. The smallest such T is called the period of the function or fundamental
period.
For example: sin (t + 2p) = sin t,   sin(t + 4p) = sin t , sin(t + 6p) = sin t ∀ t ∈ R
\ 2p is the smallest number which satisfies sin(t + T ) = sin t ∀t .

So, the period of sin t is 2p.

Note  It is clear that f (t + T ) = f (t ) ∀ t

⇒ f (t + 2T ) = f (t + T + T ) = f (t + T ) = f (t ) and so on.

M03_Eng-Maths (Aditya) CH01.indd 27 7/24/2018 1:06:10 PM


1.28  n  Engineering Mathematics-III

In general, f (t + nT ) = f (t ) ∀ t , where n is an integer positive or negative.


The advantage of periodic function is that it is enough we study it in an interval of length T,
because copies of it only will be available in all other intervals of length T. So, the Laplace transform
of f (t) will be expressed as an integral of e-st f (t) over the interval (0, T) of length T.

Theorem 1.5  Laplace transform of periodic function


If f (t) is a periodic function with period T, then
T

∫e
−s t
f (t ) dt
L [f (t ) ] = 0

1 − e − sT

Proof  Since f (t) is periodic with period T, we have

f (t ) = f (t + T ) = f (t + 2T ) = … = f (t + nT ) for all t

By definition,

L [f (t ) ] = ∫ e − s t f (t ) dt
0

T 2T 3T

= ∫ e − s t f (t ) dt + ∫ e − s t f (t ) dt + ∫e
−s t
f (t ) dt + …
0 T 2T

Put t = u + T in the second integral


t = u + 2T in the third integral and so on.
Then dt = du
In the second integral,   when t = T, u = 0   and when t = 2T, u=T
In the third integral,     when t = 2T,  u = 0    and when t = 3T,  u = T    and so on
T T T
\ L [f (t )] = e − s t f (t ) dt + e − s (u +T ) f (u + T ) du + e − s (u + 2T ) f (u + 2T ) du + …
∫ 0
∫ 0
∫ 0

T T T T
= ∫ e − s t f (t ) dt + e − sT ∫ e − s u f (u ) du + e −2sT ∫ e − s u f (u ) du + e −3sT ∫ e − s u f (u ) du + …
0 0 0 0
T

= [1 + e − sT + e −2sT + e −3sT + …] ∫ e − s t f (t ) dt
0

∫e
−s t
f (t ) dt
1
T
[1 + e − sT + e −2sT + … is a infinite
1 − e − sT ∫0
e − s t f (t ) dt =
0
= 
1 − e − sT geometric series with C.R e − s T ]
 n
T
Note  If f (x) is periodic with period T, then f (ax + b) is periodic with period if a > 0.
a

M03_Eng-Maths (Aditya) CH01.indd 28 7/24/2018 1:06:13 PM


Laplace Transforms  n  1.29

WORKED EXAMPLES
Example 1
Find the Laplace transform of the function
 p
sin vt , 0 < t ≤ v 2p
f (t ) 5  with period .
0 p 2p v
, <t <
 v v
Solution.
 2p  2p
Here f (t) is given in the interval  0,  and f (t) is periodic with period T = because sin t is
 v v
periodic with period 2p.
T

∫e
−s t
f (t ) dt
\ L [f (t )] = 0
 [by theorem 1.5]
1 − e − sT
2p
v

∫e
−s t
f (t ) dt
= 0
−2ps
1− e v

vp 2p
v

1  −s t 
 ∫ e sin vt dt + ∫ e ⋅ 0 dt 
−s t
= −2ps
1− e v 0 p 
 v 
p
v

∫e
−s t
sin vt dt
= 0
−2ps
1− e v
p
 e −s t v
 2 [ −s sin vt − v cos vt ]
s + v
2
0
= −2ps
1− e v


 −vps
1  e   p  p  e0 
2 {
=  − s sin  v  − v cos  v −
  2 −s sin 0 − v cos 0}

2ps  s 2 + v2  v v  s +v 
1− e v 

 −
sp

1  v e v v 
= +

2ps  s 2 + v2 s 2 + v2 
1− e v  

 −
sp

v 1 + e v 
=   =
v v
sp = − sp
 −
sp
  −
sp
 −  
( s 2 + v2 ) 1 − e v  1 + e v  ( s + v )[1 − e v ] ( s 2 + v2 ) 1 − e v 
2 2

    
M03_Eng-Maths (Aditya) CH01.indd 29 7/24/2018 1:06:16 PM
 −
sp

1.30  n  1 Mathematics-III
Engineering  ve
v v 
= +

2ps  s 2 + v2 s 2 + v2 
1− e v  
 −
sp

v 1 + e v 
=   =
v v
sp = − sp
 −
sp
  −
sp
 −  
( s 2 + v2 ) 1 − e v  1 + e v  ( s + v )[1 − e v ] ( s 2 + v2 ) 1 − e v 
2 2

    

 −2sp
 − ps 
2

{ e v =  e v  and a2 − b 2 = (a − b )(a + b ) 
    

2p
Note  This function is called the half-sine wave rectifier function of period .
v
Example 2
Find the Laplace transform of the periodic function

t , 0≤t ≤a
f (t ) 5  and f ( t 1 2a ) 5 f ( t ).
2a − t , a < t ≤ 2a

Solution.
t , 0≤t ≤a
Given f (t ) =  and f (t ) is of period 2a.
 2 a − t , a < t ≤ 2a
\  by theorem 1.5,
T 2a

∫ e f (t ) dt ∫e
−s t −s t
f (t ) dt
L [f (t )] = 0
= 0

1 − e −s T 1 − e −2as
a 2a

∫ e f (t ) dt + ∫ e f (t ) dt
−s t −s t

= 0 a

1 − e −2as
a 2a

∫ e t dt + ∫ e (2a − t ) dt
−s t −s t

= 0 a

1 − e −2as
1   te − s t e −s t  
a
e − s t ( −1) ⋅ e − s t  
2a

=  − 1⋅  + ( 2a − t ) −  
1 − e −2as   −s ( −s ) 2  0  −s ( −s ) 2  a 

   a ⋅ e e − as   e0  
− as
1
=  −  + 2  + 0 + 2 
1 − e −2as    s s   s 
 e −2as   ae − as e − as   
+  0 + 2  −  − + 2  
 s   s s   

M03_Eng-Maths (Aditya) CH01.indd 30 7/24/2018 1:06:19 PM


Laplace Transforms  n  1.31

1  1 e −2 as 2e − sa 
=  2+ 2 − 2 
1 − e −2 as s s s 
− as 2
1 (1 − e ) (1 − e − as ) 2 1 − e − as 1 as
= = = = tan h .
1 − e −2 as s2 s 2 (1 − e − as )(1 + e − as ) s 2 (1 + e − as ) s 2 2
Note  This function is called the triangular wave function of period 2a.

Example 3 
Find the Laplace transform of the square-wave function (or Meoander function) of period a
 a
1 when 0 < t ≤ 2
defined as f ( t ) 5  .
21 when a < t < a f(t)
 2
Solution.
1
 a
 1, if 0 < t <
Given 2
f (t ) = 
a
 −1, if < t < a t
a a
 2 3a
2
−1 2
\  f (t) is of period T = a

By theorem 1.5
a
2 a

∫ e f (t ) dt + ∫ e f (t ) dt
T −s t −s t

∫e
−s t
f (t ) dt 0 a

L [f (t )] = 0
= 2

1 − e − sT 1− e − sa
a
2 a

∫ e ⋅1dt + ∫ e ( −1) dt
−s t −s t

0 a

= 2
− sa
1− e
  e − s t  2  e − s t  a 
a

1
=   −  
[1 − e − sa ]   −s  0  −s  a 
 2 

 −2sa − sa
− sa

1  − e e 0
e e 2

= + + −
[1 − e − sa ]  s s s s 
 
− sa
1
= [1 − 2e 2 + e − sa ]
s (1 − e − sa )
2
 − sa

1 − e 
2 sa

  1− e 2 1 sa
= = = tan h
 − sa
 − sa
 −
sa
s 4
s 1 − e 2  1 + e 2  s[1 + e 2 ]
  

M03_Eng-Maths (Aditya) CH01.indd 31 7/24/2018 1:06:22 PM


1.32  n  Engineering Mathematics-III

Theorem 1.6  Initial value theorem


If the Laplace transforms of f (t) and f ′(t) exist and L [f (t )] = F (s ),
then lim f ( t ) 5 lim s F ( s ).
t → 01 s→

Proof  We know that L [f ′(t )] = s L [f (t )] − f (0)


∫e
−s t
⇒ f ′(t ) dt = s F (s ) − f (0)
0

\ lim ∫ e − s t f ′(t ) dt = lim s F (s ) − f (0)


s →∞ s →∞
0

∫ lim e
− st
f ′(t ) dt = lim s F (s ) − f (0)
s →∞ s →∞
0

∫ lim e
− st
Since t is independent of s, lim e − st = 0   \  f ′(t ) dt = 0 .
s →∞ s →∞
0

\ f (0) = lim s F (s )
s →∞

⇒ lim f (t ) = lim s F (s ) [Since f (t) is continuous on the right at 0,


t →0 s →∞
 lim f (t ) = f (0)] n
t→ 0 +

Theorem 1.7  Final value theorem 


If the Laplace transform of f (t) and f ′(t) exist and F (s ) = L [f (t )], then
lim f ( t ) 5 lim s F ( s ).
t →∞ s→0

Proof  We know that L [f ′(t )] = s L [f (t )] − f (0)

∫e
− st
⇒ f ′(t )dt = s F (s ) − f (0)
0

\ lim ∫ e − st f ′(t ) dt = lim (s F (s ) − f (0))


s →0 s →0
0

∫ lim e
− st
⇒ f ′(t ) dt = lim s F (s ) − f (0)
s →0 s →0
0

⇒ ∫ f ′(t ) dt = lim s F (s ) − f (0)


0
s →0

⇒ [f (t )]0∞ = lim s F (s ) − f (0)


s →0

M03_Eng-Maths (Aditya) CH01.indd 32 7/24/2018 1:06:27 PM


Laplace Transforms  n  1.33

⇒ lim f (t ) − f (0) = lim s F (s ) − f (0)


t →∞ s →0

⇒ lim f ( t ) 5 lim s F ( s )  n
t →∞ s→0

Note  Since f (t) is defined for all t ∈ [0, ∞), f (0) is the first value or initial value of f (t) and lim f (t )
t→∞
is the final value of f (t).

WORKED EXAMPLES
Example 4 
Verify initial value theorem for the function f (t ) 5 e 2t sin t .

Solution.
Initial value theorem is lim f (t ) = lim s F (s )
t →0 s →∞

Here f (t) = e-t sin t


1
\ F (s ) = L [f (t )] = L [e −t sin t ] =
(s + 1) 2 + 1
Now L.H.S. = lim f (t ) = lim e −t sin t = e 0 ⋅ 0 = 0
t →0 t →0

s ∞ 
and R.H.S. = lim sF (s ) = lim   ∞ form 
s →∞ s →∞ (s + 1) 2 + 1
1
= lim = 0 [by L’Hopital’s rule]
s →∞ 2(s + 1)
\ L.H.S = R.H.S.
Hence the theorem is verified.

Example 5 

Verify final value theorem for f (t ) 5 1 1 e 2t (sin t 1 cos t ) .

Solution.
Final value theorem is lim f (t ) = lim s F (s )
t →∞ s →0

Given f (t ) = 1 + e −t (sin t + cos t )


\ lim f (t ) = lim[1 + e −t (sin t + cos t )] = 1 + 0 = 1
t →∞ t →∞

F (s ) = L [f (t )]
= L [1 + e −t (sin t + cos t )]
= L [1] + L [e −t sin t ] + L [e −t cos t ]
1 1 s +1
= + +
s (s + 1) 2 + 1 (s + 1) 2 + 1

M03_Eng-Maths (Aditya) CH01.indd 33 7/24/2018 1:06:32 PM


1.34  n  Engineering Mathematics-III

1 1 s +1 
\ lim s F (s ) = lim s  + + 
s →0 s →0
 s ( s + 1) 2
+ 1 ( s + 1) 2 + 1
 s s2 + s 
= lim 1 + +  =1
 (s + 1) + 1 (s + 1) + 1
s →0 2 2

\ lim f (t ) = lim s F (s )
t →∞ s →0

Hence the theorem is verified.

Example 6
Verify the initial and final value theorems for f (t ) 5 e 2t (t 1 2 ) 2 .

Solution.
(1)  Initial value theorem is lim f (t ) = lim s F (s )
t →0 s →∞

Here f (t ) = e −t (t + 2) 2

\ F (s ) = L [f (t )] = L [e −t (t 2 + 4t + 4)]
= L [e −t t 2 ] + 4 L [e −t t ] + 4 L [e −t ]
2 1 4
= + 4⋅ +
(s + 1) 3
(s + 1) s + 1
2

Now L.H.S. = lim f (t ) = lim e −t (t + 2) 2 = e 0 (0 + 2) 2 = 4


t →0 t →0

 2s 4s 4s 
and R.H.S. = lim s F (s ) = lim  + +
s →∞ s →∞ (s + 1)3
 ( s + 1) 2
s + 1

2s 2
But lim = lim = 0 [By L’Hopital’s rule]
s →∞ (s + 1)3 s →∞ 3(s + 1) 2

4s 4
lim = lim =0 [By L’Hopital’s rule]
(s + 1) s →∞ 2(s + 1)
s →∞ 2

4s 4
lim = lim =4
s →∞ s +1 s →∞ 1
1+
s
Hence, R.H.S. = lim s F (s ) = 0 + 0 + 4 =4
s →∞

\ L.H.S. = R.H.S.
Hence, the initial value theorem is verified.

M03_Eng-Maths (Aditya) CH01.indd 34 7/24/2018 1:06:36 PM


Laplace Transforms  n  1.35

(2)  Final value theorem is lim f (t ) = lim s F (s ).


t →∞ s →0

(t + 2) 2 ∞ 
Now L.H.S. = lim e −t (t + 2) 2 = lim   ∞ form 
t →∞ t →∞ et
2(t + 2)
= lim  [By L’Hopital’s rule]
t →∞ et
2
= lim = 0 [By L’Hopital’s rule]
t →∞ et

 2s 4s 4s 
and R.H.S. = lim s F (s ) = lim  + + =0
s →0 s → 0 (s + 1)

3
(s + 1) (s + 1) 
2

\ L.H.S. = R.H.S.
Hence, final value theorem is verified.

Definition 1.3  Unit step function or Heaviside function 

0, if t < 0
The unit step function u is defined by u (t ) = 
1, if t ≥ 0
This function has jump discontinuity at t = 0
More generally, if a is any positive number, then u(t - a) is the unit step function shifted a units to
the right and is defined as

0, if t < a
u (t − a) = 
1, if t ≥ a
u(t) u(t)

(0, 1) (a, 1)
(0, 1)

O t O (a, 0) t

u(t - a) is also denoted by ua(t)

Note  The unit step function may be thought of as a flat signal of magnitude 1 or a flat switching
function. It is also called Heaviside function in honour of the British electrical engineer Oliver
Heaviside.

M03_Eng-Maths (Aditya) CH01.indd 35 7/24/2018 1:06:39 PM


1.36  n  Engineering Mathematics-III

1.6.1 Laplace transform of unit step function


By definition, L [u (t − a)] = ∫ e − st u (t − a) dt
0
a ∞

= ∫ e − st u(t − a) dt + ∫ e − st u(t − a) dt
0 a

∞ ∞
a
 e − st  1 −∞ e − as
= ∫ e − st ⋅ 0 dt + ∫ e − st dt = 0 +   = − [ e − e − as
] = if s > 0
0 a  −s a s s

e2as
\ L[u( t 2 a )] 5 if s > 0
s

Theorem 1.8  Second shifting property (or t-shifting) 


If L [f (t )] = F (s ), then L [f (t − a)u (t − a)] = e − as F (s )

Proof  The unit step function is


0 if t < a
u (t − a) = 
1 if t ≥ a
0, if t < a
\ f (t − a)u (t − a) = 
f (t − a ), if t ≥a

\ L [f (t − a)u (t − a)] = ∫ e − st f (t − a)u (t − a) dt


0

a ∞

= ∫ e − st f (t − a)u (t − a) dt + ∫ e − st f (t − a)u (t − a) dt
0 a

∞ ∞
a

= ∫ e − st ⋅0 dt + ∫ e − st f (t − a) dt = ∫ e − st f (t − a) dt
0 a a

Put t − a = x   \  dt = dx

When t = a, x = 0 and when t = ∞, x = ∞


∞ ∞

L [f (t − a)u (t − a)] = ∫ e f ( x ) dx = e ∫ e f ( x ) dx = e F (s )
−s ( x + a) − sa − sx − sa
\
0 0

\ L [f (t − a)u (t − a)] = e − sa L [f (t )]  n

Note  The above property can also be stated without using the name unit step function as below.

M03_Eng-Maths (Aditya) CH01.indd 36 7/24/2018 1:06:43 PM


Laplace Transforms  n  1.37

0 if t < a
If L [f (t )] = F (s ) and G (t ) = 
f (t − a) if t ≥ a
then L [G (t )] = e − as F (s ) = e − as L [f (t ) ]

1.6.2  Unit impulse function


Many engineering applications involve the concept of an impulse, which may be considered as a very
large force acting for a very short time.
For example: the force due to a hammer blow. u(t)
We can model an impulse as below in terms of unit step
function. (0, 1/∈) (∈, 1/∈)
For any positive number ∈, the impulse function d∈ is
defined by
1 O ∈ t
 [u (t ) − u (t − ∈)], 0 ≤ t <∈
d∈(t ) =  ∈
0, otherwise
1
This is a pulse of magnitude and duration ∈.

1
Note that the area of the graph representing the pulse is . ∈= 1, which remains a constant 1 as ∈ → 0.

1.6.3  Dirac-delta function
The function d∈ (t) obtained by taking ∈ → 0 is called the u(t)
Dirac delta function and denoted by d(t).
Thus d(t ) = lim d∈(t )
∈→ 0 +
(0, 1/∈)
0 if t ≠ 0
In other words d(t ) = 
 ∞ if t = 0
 1 
 ∈→ 0 ⇒ ∈ → ∞  a+∈
  O a t
0 if t ≠ a
The shifted delta function is d(t − a) = 
∞ if t = a

1.6.4 Laplace transform of delta function


1
We have d∈ (t − a ) =[u(t − a) − u(t − a − ∈)]   if a ≤ t ≤ a+ ∈

1
Then L [d∈(t − a)] = L [u (t − a) − u (t − a − ∈)]

1
= [L [u (t − a)] − L [t − (a+ ∈)]]


1  e − as e − ( a +∈)s  e − as (1 − e − ∈s )
=  − =
∈ s s  ∈s

M03_Eng-Maths (Aditya) CH01.indd 37 7/24/2018 1:06:47 PM


1.38  n  Engineering Mathematics-III

1 − e −∈s  1 − e −∈s 
e − as0lim
As ∈→ 0, L [d(t − a)] =∈→ ,∈→ 0 +L[d(t − a)] = e − as lim  
 ∈s  ∈→ 0 +
 ∈s 
 1 − e −x 
i.e. L [d(t − a)] = e − as  lim
 x →0
{ = 1
 x 
In particular if a = 0, then we get L [d(t )] = e = 1
0

An important property of Dirac-delta function is ∫ f ( t ) d( t 2 a ) dt 5 f ( a )


0

Note  Paul Dirac (1902–84) was a British physicist who was awarded the noble prize in 1933 for his
work in quantum mechanics.
In mechanical problems delta function is used to represent an impulse. In electrical systems it is
used to represent the application of a very large voltage for a short time or the sudden discharge of
energy contained in a capacitor. This function is applied to study the behaviour of circuits subjected to
transients like high input voltage. Sometimes the system may break down. So, before a circuit is built,
transients are modeled by the delta function and their effects on the circuit are studied to set safety
standards for the system to be fabricated.

WORKED EXAMPLES
Example 7
Find L [(t 21) 2 u(t 21)].

Solution.
Here f (t) = t2
\ f (t − 1) = (t − 1) 2 and a = 1
2 2e − s
\ L [(t − 1) u (t − 1)] = e
2 − as
L [t ] = e − s ⋅ 3 = 3 
2
[By theorem 1.8]
s s
Example 8
Find L [e 24 t u(t 21)] .

Solution.
−4t −4 ( t −1+1)
L [e u (t − 1)] = L [e u (t − 1)] = e −4 L [e −4 (t −1)u (t − 1)]
Here f (t ) = e −4t and a = 1
1
L [e −4 (t −1)u (t − 1)] = e − s L [e −4t ] = e − s
\ s+4  [By theorem 1.8]
1 e −( 4+ s)
L[e −4 t u(t − 1)] = e −4 ⋅ e − s ⋅ =
\ s+4 s+4
Example 9
  p 
Find L (sin t )u  t 2   .
  2

M03_Eng-Maths (Aditya) CH01.indd 38 7/24/2018 1:06:51 PM


Laplace Transforms  n  1.39

Solution.
  p    p p    p   
L (sin t )u  t −   = L sin t − +  u  t −   
  2   2 2    2   

  p    p  
= L cos  t −  u  t −   
  2   2  
p
− s  p
=e L [cos t ] 
\ a= [Using theorem 1.8]
2 
2

ps

ps −
− s se 2
=e 2
⋅ 2 = 2
s +1 s +1
Example 10
t 21, 1 < t < 2
Find the Laplace transform of f ( t ) 5  .
32 t , 2 < t < 3
Solution.
We shall rewrite f (t) in terms of unit-step function.
f (t ) = (t − 1)[u (t − 1) − u (t − 2)] + (3 − t )[u (t − 2) − u (t − 3)]
[{ u (t − 1) = 1, u (t − 2) = 0 in 1 < t < 2; u (t − 2) = 1,

u (t − 3) = 0 in 2 < t < 3]
\ f (t ) = (t − 1)u (t − 1) + [3 − t − t + 1]u (t − 2) + (t − 3)u (t − 3)
= (t − 1)u (t − 1) − 2(t − 2)u (t − 2) + (t − 3)u (t − 3)
\ L [f (t )] = L [(t − 1)u (t − 1)] − 2L [(t − 2)u (t − 2)] + L [(t − 3)u (t − 3)]

= e − s L [t ] − 2e −2s L [t ] + e −3s L [t ]
1 e −2s e −3s e − s e − s (1 − e − s ) 2
= e −s ⋅ 2
− 2 2 + 2 = 2 [1 − 2e − s + e −2s ] =
s s s s s2

EXERCISE 1.3
Evaluate the following integrals using Laplace transform.
∞ ∞ ∞
e −t − e −3t dt cos 6t − cos 4t
1. ∫ ∫
2.  dt  3. ∫ e 3t t 3 cos t dt
0
t 0
t 0

∞ ∞ ∞ ∞

∫ e t sin 3t ⋅ dt  6. ∫ te sin t dt   7. ∫ e


4. ∫ t 3e −t sin t dt 5.  −2t −t −3t
t sin t dt
0 0 0 0

∞ ∞ ∞
e −3t − e −6t cos at − cos bt sin 3t
8. ∫ ∫
dt 9.  ∫
dt 10.  e −t dt
0
t 0
t 0
t

Verify initial and final value theorems for the following functions.
11. f (t ) = 1 − e − at 12. 
f (t ) = t 2e −3t f (t ) = a e − bt
13.  f (t ) = ( 2t − 3) 2 14. 

M03_Eng-Maths (Aditya) CH01.indd 39 7/24/2018 1:06:58 PM


1.40  n  Engineering Mathematics-III

K , 0 ≤ t ≤ a
15. Find the Laplace transform of the square-wave function f (t) given by f (t ) = 
 − K , a ≤ t ≤ 2a
and f (t + 2a) = f (t ).
16. Find the Laplace transform of the full-sine wave rectifier function f (t ) = sin vt , t ≥ 0 .
at
17. Find the Laplace transform of the periodic saw-tooth wave function given by f (t ) = ,0 < t < v
v
and f (t + v) = f (t ). 
18. Find the Laplace transform of the periodic function defined by the triangular wave
t
 a , 0≤t ≤a
f (t ) =  and f (t + 2a) = f (t ). 
 2a − t
, a ≤ t ≤ 2a
 a
19. Find the Laplace transform of the periodic function
cos t , 0 < t < p
f (t ) =  , f (t + 2p) = f (t ) .
0, p < t < 2p
t for 0 < t < p
20. Find the Laplace transform of f (t ) =  and f (t + 2p) = f (t ).
p − t for p < t < 2p
 t
 1 − cos u  
t

21. Find L e −2t ∫   du  . 22. Find L[e − t ∫ t 2 cos t dt ].


 0
 u   0

 e − t sin t 
t t

23. Find L  ∫ dt  . 24. Find L [e −t ∫ t cos t dt ]] .


0 t  0

t
sin t
25. Find the Laplace transform of e −t ∫ dt .
0
t
p
2
 sin at  sin t p
26. Find L 
 t 
 and hence prove that ∫
0
t
dt = . 
2
s 2 − a2
27. Using Laplace transform of derivative formula, prove that L [t cos at ] = .
(s 2 + a 2 ) 2
2vs
28. If L [t sin vt ] = evaluate L [vt cos vt + sin vt ] .
(s + v 2 ) 2
2


29. Find the Laplace transform of the saw-toothed wave function of period T given by
t
f (t ) = , 0 < t < T .
T
3s 2 + 5s + 2
30. If F (s ) = , then find f (0) and f (∞).
s 3 + 4s 2 + 2s

M03_Eng-Maths (Aditya) CH01.indd 40 7/24/2018 1:07:04 PM


Laplace Transforms  n  1.41

ANSWERS TO EXERCISE 1.3


2 168 12
1. log e 3  2. log e 3.  4. 0 5. 
3 10 4 169
1 3 b p
6. 7.  8.  loge 10. 
loge 2 9. 
2 50 a 3

k  as  v  ps  17. a a e − vs 1  as 
15.  tanh   16.  coth   − 18.  tanh  
s  2 s 2 + v2 2v  vs 2 s (1 − e − vs ) as 2  2

s 1 sp pe − sp 1  s 2 + 4s + 5 
19.  20.  tanh − 21.  log  s 2 + 4s + 4 
(s 2 + 1)(1 − e −ps ) s2 2 s (1 + e − sp ) 2(s + 2)

2(s 2 + 2s − 2) 1 −1 s ( s + 2) cot −1 (s + 1)
2
2.  2 23.  cot ( s + 1) 24.  25. 
(s + 2s + 2)3 s (s + 1)(s 2 + 2s + 2) 2 s +1

(s 2 − a 2 ) 2vs 2 1 e − sT
27.  28.  29.  − 30.  3 and 1
(s + a )
2 2 2
(s + v )
2 2 2
Ts 2
s (1 − e − sT )

M03_Eng-Maths (Aditya) CH01.indd 41 7/24/2018 1:07:09 PM


This page is intentionally left blank

M03_Eng-Maths (Aditya) CH01.indd 42 7/24/2018 1:07:09 PM


Inverse Laplace Transforms 2
2.1  INVERSE LAPLACE TRANSFORMS
Definition 2.1  If the Laplace transform of a function f (t) is F (s), then f (t) is called the inverse Laplace
transform of F (s) and is written as L21 [ F ( s )] 5 f ( t )

i.e., if L [f (t )] = F (s ), then L−1 [F (s )] = f (t )

L21 is called the inverse Laplace transform operator.

For example: L−1 


1  1
 = e , since L [e ] = s − a
at at

 s − a 

Linearity Property
If L [f (t )] = F (s ) and L [G (t )] = G (s )

then L−1 [aF (s ) + bG (s )] = a L−1 [F (s )] + b L−1 [G (s )]

First shifting property


L [f (t )] = F (s ), L [e − at f (t )] = F (s + a) and  L [e at f (t )] = F (s − a)
If

L−1 [ F ( s − a)] = e at L−1 [ F ( s)]


then (i)  L−1 [F (s + a)] = e − at L−1 [F (s )] (ii) 

 s+2  s + 4 − 2 −4t −1  s − 2  1 2 
For example: L−1  2
= L−1  2  = e L  2  = e −4t L−1  − 2  = e −4t [1 − 2t ]
 (s + 4)   (s + 4 )   s  s s 
We have seen while finding Laplace transforms of most of the functions F (s) are rational algebraic
functions. Hence, to find the inverse Laplace transforms, the most important technique is to split F (s)
into partial fractions and using the table given below, we get L-1[F (s)].
Many problems given under the other types can also be done by partial fraction method.
The following table is important in finding the inverse Laplace transform.

S.No. L [f (t )] = F (s ) L−1[F (s )] = f (t )

1 1
1. L [1] = L−1   = 1
s s 

1 1
2. L [t ] = , s>0 L−1  2  = t
s2 s 

M03_Eng-Maths (Aditya) CH02.indd 1 7/24/2018 1:06:02 PM


2.2  n  Engineering Mathematics-III

 1  t
n
n!
3. L [t n ] = , n = 1, 2, 3, … L−1  n +1  = , n = 1, 2, 3, …
s n +1  s  n!

Γ (a + 1)  1  ta
4. L [t a ] = , L−1  a +1  = ,
s a +1  s  Γ( a + 1)
a is a real number > −1

1  1 
5. L [e at ] = , s>a L−1  =e
at

s −a s − a

1  1 
6. L [e − at ] = , s > −a L−1  =e
− at

s+a s + a

a  1  sin at
7. L [sin at ] = , s>0 L−1  2 2
=
s 2 + a2 s + a  a

s  s 
8. L [cos at ] = , s>0 L−1  2 2
= cos at
s 2 + a2 s + a 

a  1  sinh at
9. L [sinh at ] = , s> a L−1  2 2
=
s 2 − a2 s − a  a

s  s 
10. L [cosh at ] = , s> a L−1  2 2
= cosh at
s − a2
2
s − a 

2.1.1  Type 1 – Direct and shifting methods


WORKED EXAMPLES
Example 1 
 3s 1 5 
Find L21  2 .
s 19 
Solution.
 3s + 5  −1  s  −1  1 
L−1  2  = 3L  s 2 + 9  + 5L  s 2 + 9 
 s + 9     
sin 3t 9 cos 3t + 5 sin 3t
= 3 cos 3t + 5 =
3 3
Example 2
 s 2 2 3s 1 4 
Find L21  .
 s3 

M03_Eng-Maths (Aditya) CH02.indd 2 7/24/2018 1:06:07 PM


Inverse Laplace Transforms  n  2.3

Solution.
 s 2 − 3s + 4  −1  1  −1  1  −1  1 
 = L   − 3L  2  + 4 L  3  = 1 − 3t + 2t
2
L−1  3
 s  s  s  s 
Example 3
 s 13 
Find L21  2 .
 s 2 4 s 113 
Solution.
 s+3   s+3 
L−1  2  = L−1  
 s − 4s + 13   (s − 2) − 4 + 13 
2

 s −2+5 
= L−1  
 ( s − 2) + 9 
2

 s + 5 
= e 2t L−1  2  [by shifting property]
s + 9
  s   1 
= e 2t L−1  2  + 5L−1  2 
 s + 9  s + 9 
 sin 3t  e 2t
= e 2t cos 3t + 5. = [3 cos 3t + 5 sin 3t ]
 3  3
Example 4
 s 
Find L21  3 .
 (s 1 6) 
Solution.
 s  s + 6 − 6
L−1  3
= L−1  3 
 (s + 6 )   (s + 6 ) 

s − 6  −1  1  −1  1  
= e −6 t L−1  3  = e −6 t  L  2  − 6 L  3 
 s   s   s 
 t2 
= e −6 t t − 6  = e −6 t [t − 3t 2 ] = te −6 t (1 − 3t )
 2 !

2.1.2  Type 2 – Partial fraction method


worked examples
Example 5
 s12 
Find L21  .
 s ( s 1 4 )( s 1 9 ) 

M03_Eng-Maths (Aditya) CH02.indd 3 7/24/2018 1:06:10 PM


2.4  n  Engineering Mathematics-III

Solution.
s+2
Given F (s ) =
s (s + 4)(s + 9)

Splitting into partial fractions, we get


s+2 A B C
= + +
s (s + 4)(s + 9) s s + 4 s + 9

⇒ s + 2 = A (s + 4)(s + 9) + Bs (s + 9) + Cs (s + 4)
1
Put s = 0.   \         A (0 + 4)(0 + 9) = 2    ⇒     36 A = 2   ⇒  A =
18
1
Put s = - 4.  \       B( −4)( −4 + 9) = −4 + 2  ⇒  −20 B = −2   ⇒  B =
10
7
Put s = - 9.  \ C( −9)( −9 + 4) = −9 + 2  ⇒    45C = −7   ⇒  C = −
45
1 1 1 1 7 1
\ F (s ) = . + . − .
18 s 10 s + 4 45 s + 9

\ 1 −1  1  1 −1  1  7 −1  1 
L−1 [F (s )] = L  + L   − L s + 9
18  s  10  s + 4  45  
1 e −4t 7e −9t
= + −
18 10 45

Example 6

 5s 1 3 
Find L21  2 .
 ( s 2 1)( s 1 2 s 1 5 ) 
Solution.
5s + 3
Given F (s ) =
(s − 1)(s 2 + 2s + 5)

For    s 2 + 2 s + 5, b 2 − 4 ac = 4 − 20 < 0    So, it cannot be factorised into real factors.

Splitting into partial fractions, we get


5s + 3 A Bs + C
= + 2
(s − 1)(s + 2s + 5) s − 1 s + 2s + 5
2

⇒ 5s + 3 = A (s 2 + 2s + 5) + ( Bs + C )(s − 1)
Put s = 1. \ A (1 + 2 + 5) = 5 + 3  ⇒  8A = 8       
⇒  A = 1
Put s = 0. \ 5A − C = 3 ⇒ C = −3 + 5A = −3 + 5 = 2.

M03_Eng-Maths (Aditya) CH02.indd 4 7/24/2018 1:06:15 PM


Inverse Laplace Transforms  n  2.5

Equating, the coefficients of s2 on both sides, we get A + B = 0   ⇒  B = − A = −1


1 −s + 2 1 s−2
\ F (s ) = + = −
s − 1 s 2 + 2s + 5 s − 1 s 2 + 2s + 5
1 s−2 1 s +1− 3
= − = −
s − 1 (s + 1) + 4 s − 1 (s + 1) 2 + 4
2

 1   s +1− 3 
\ L−1 [F (s )] = L−1   − L−1  
 s − 1  (s + 1) + 4 
2

 s −3 
= e t − e −t L−1  2 
s + 4 
  s  −1  1 
= e t − e −t L−1  2  − 3L  s 2 + 4  
  s + 4   
 3 sin 2t  e −t
= e t − e −t cos 2t − = e t
− [2 cos 2t − 3 sin 2t ]
 2  2

Example 7
 4s 1 5 
Find L21  2 . 
 ( s 2 1) ( s 1 2 ) 
Solution.
4s + 5
Given F (s ) =
(s − 1) 2 (s + 2)
Splitting into partial fractions, we get
4s + 5 A B C
= + +
(s − 1) (s + 2) s − 1 (s − 1) s + 2
2 2

⇒ 4s + 5 = A (s − 1)(s + 2) + B (s + 2) + C (s − 1) 2
9
Put s = 1. \ B(1 + 2) = 4 + 5 ⇒ B= =3
3
1
Put s = -2.  \ C( −2 − 1) 2 = −8 + 5  ⇒  9C = −3   ⇒  C = −
3
1
Equating coefficients of s2, we get A + C = 0 ⇒ A = −C =
3
1 1 1 1 1
\ F (s ) = ⋅ + 3⋅ − ⋅
3 (s − 1) (s − 1) 2 3 (s + 2)
1 −1  1  −1  1  1 −1  1 
L−1 [F (s )] = L   + 3L  (s − 1) 2  − 3 L  s + 2 
3  s − 1     
1 1 1
= e t + 3e t L−1  2  − e −2t
3 s  3
1 1 et e −2t
= e t + 3e t ⋅ t − e −2t = (1 + 9t ) −
3 3 3 3

M03_Eng-Maths (Aditya) CH02.indd 5 7/24/2018 1:06:20 PM


2.6  n  Engineering Mathematics-III

Example 8
 s 2 1 16 
Find L21  2 2 .
 ( s 1 1 )( s 1 4 ) 

Solution.
s 2 + 16
Given F (s ) =
(s + 1)(s 2 + 4)
2

Since there is no odd powers of s, we can regard F (s) as a function of s2 and write the special partial
fraction treating s2 as x.
x + 16 A B
\ = +
( x + 1)( x + 4) x + 1 x + 4
⇒ x + 16 = A ( x + 4) + B ( x + 1)
Put x = -1.     \ A ( −1 + 4) = −1 + 16   ⇒    3A = 15  ⇒  A = 5

Put x = -4.     \ B( −4 + 1) = −4 + 16  ⇒    −3B = 12   ⇒  B = −4


x + 16 5 −4
\ = +
( x + 1)( x + 4) x + 1 x + 4
s 2 + 16 5 4
⇒ = −
(s + 1)(s 2 + 4) s 2 + 1 s 2 + 4
2

5 4
⇒ F (s ) = − 2
s +1 s + 4
2
\
 1   1 
L−1 [ F ( s)] = 5 L−1  2  − 4 L−1  2 
 s + 1 s + 4
1
= 5 sin t − 4 ⋅ sin 2t = 5 sin t − 2 sin 2t
Example 9 2

 s 
Find L21  4 4 
.
 s 1 4a 

Solution.
s s
Given =  [{ a2 + b 2 = (a + b ) 2 − 2ab ]
s 4 + 4a4 (s 2 + 2a2 ) 2 − 2s 2 .2a2

s
=
(s 2 + 2a2 ) 2 − ( 2sa) 2
s
= 2
(s + 2a + 2sa)(s 2 + 2a2 − 2sa)
2

s
= 2
(s + 2sa + 2a )(s 2 − 2sa + 2a2 )
2

M03_Eng-Maths (Aditya) CH02.indd 6 7/24/2018 1:06:24 PM


Inverse Laplace Transforms  n  2.7

Splitting into partial fractions, we get

s As + B Cs + D
= +
(s 2 + 2as + 2a2 )(s 2 − 2as + 2a2 ) s 2 + 2as + 2a2 s 2 − 2as + 2a2

\ s = ( A s + B )[s 2 − 2as + 2a2 ] + (Cs + D )[s 2 + 2as + 2a2 ]

Equating coefficients of s3, s2, s and constant terms on both sides, we get
A + C = 0   
⇒  C = - A(1)

−2aA + B + 2aC + D = 0    
⇒    2a(C − A ) + B + D = 0 (2)

2a2 A − 2aB + 2a2C + 2aD = 1

⇒ 2a2 ( A + C ) + 2a( D − B ) = 1    
⇒  2a( D − B ) = 1 [using (1)]  (3)

and 2a2 B + 2a2 D = 0   ⇒  B + D = 0  ⇒  D = - B (4)

1
\ ⇒  −4aB = 1  ⇒  B = −
2a[ − B − B ] = 1     [{ D = − B ]
4a
1
\ D=
4a
Substituting C = -A and B + D = 0 in (2), we get

2a( − A − A ) = 0    
⇒  −4 A = 0   ⇒  A = 0  \  C = 0

\ s 1  1  1  1 
=−  2 2
+  2
(s + 2as + 2a )(s − 2as + 2a )
2 22 2
4a  s + 2as + 2a  4a  s − 2as + 2a 
2

1  1  1  1 
⇒ F (s ) = − +
4a  s + 2as + 2a  4a  s 2 − 2as + 2a2 
 2 2

1 −1  1  1 −1  1 
\ L−1 [F (s )] = − L  2 2
+ L  2 2
4a  s + 2as + 2a  4a  s − 2as + 2a 
1 −1  1  1 −1  1 
=− L  2
+ L  2
 (s + a) + a  4a  (s − a) + a 
2 2
4a
e − at −1  1  e at −1  1 
=− L  2 2
+ L  2 2
4a  s + a  4a s + a 
e − at sin at e at sin at sin at at sin at sinh at
=− . + . = [e − e − at ] =
4a a 4a a 4a 2 2a2

M03_Eng-Maths (Aditya) CH02.indd 7 7/24/2018 1:06:31 PM


2.8  n  Engineering Mathematics-III

2.1.3  Type 3 – Multiplication by and Division by


1. If L-1[F (s)] = f (t) and f (0) = 0, then

         L-1 [s F (s)] = f ′(t) =


d −1
dt
{
L [F (s )] }
In general,         L [s F (s )] = f (t ), if f (0) = 0, f ′(0) = 0, …, f ( n −1) (0)
−1 n (n)

t
 F ( s) 
2. If L−1 [F (s )] = f (t ), then L−1   = ∫ f (t ) dt
 s  0
t t
 F (s ) 
Similarly, L−1  2  = ∫ ∫ f (t ) dt dt
 s  00
d
3. We know that L [tf (t )] = − [F (s )] = F ′(s )
ds
L−1 [F ′(s )] = −tf (t ) = −tL−1 [F (s )]
\  

WORKED EXAMPLES
Example 10

 s 
Find L21  2 2 2 
.
 ( s 1 a ) 
Solution.
s
Let F ′ (s ) = , then L−1 [F ′(s )] = −t L−1 [F (s )]
(s 2 + a 2 ) 2
Integrating (1) w.r.to s, we get
s 1
F (s ) = ∫ ds = ∫ (s 2 + a2 ) −2 ⋅ 2s ds
(s + a )
2 2 2
2
1 (s 2 + a2 ) −2 +1 1  [f ( x ) n +1 ] 
= ⋅ =−  { ∫ [f ( x )]n
f ′ ( x )dx = 
2 ( −2 + 1) 2(s + a2 )
2
 n +1 
 1 
\ L−1 [F ′(s )] = −tL−1  − 2 
 2(s + a ) 
2

t −1  1  t sin at t sin at
= L  2 2
= ⋅ =
2 s + a  2 a 2a

Example 11

 1 
Find L21  3 
.
 s (s 1 2) 

M03_Eng-Maths (Aditya) CH02.indd 8 7/24/2018 7:11:22 PM


Inverse Laplace Transforms  n  2.9

Solution.

 1   F (s )  ,    where F (s ) = 1
L−1  3
= L−1  
 s (s + 2)   s  ( s + 2) 3

 1  t −1
\ L−1  3
= ∫ L [ F (s ) ] dt  by formula (2)
 s (s + 2)  0
t
 1 
= ∫ L−1  3
dt
0  ( s + 2) 
t
1
= ∫ e −2t L−1  3  dt
0 s 
t
t2
= ∫ e −2t . dt
0
2!
t
1
= ∫ t 2e −2t dt
20
t
1  e −2t e −2t e −2t 
= t 2 − 2t . + 2. 
2  −2 ( −2) 2
( −2)3  0
1  t 2e −2t t −2t 1 −2t  e 0  
= − − e − e −− 
2 2 2 4  4

1 1
=  −e −2t ( 2t 2 + 2t + 1) + 1 = 1 − e −2t ( 2t 2 + 2t + 1) 
8 8

Note  This problem can also be done by partial fraction method.

Example 12 
 s12 
Find L21  2 2 
.
 ( s 1 4 s 1 5) 
Solution.
s+2 s+2
Let F ′ (s ) = . Then F (s ) = ∫ 2 ds
(s 2 + 4s + 5) 2 (s + 4s + 5) 2
1
Put x = s 2 + 4s + 5   \  dx = 2s + 4 = 2(s + 2) ds ⇒ dx = (s + 2) ds
2
dx 1
\ F (s ) = ∫ 2
= ∫ x −2dx
2x 2
1  x −2 +1  1 1
=  =− =−
2  −2 + 1 2x 2(s 2 + 4s + 5)

But L−1 [F ′(s )] = −t L−1 [F (s )] [Formula 3, page 2.8]

M03_Eng-Maths (Aditya) CH02.indd 9 7/24/2018 1:06:39 PM


2.10  n  Engineering Mathematics-III

 1 
= −t L−1  − 
 2 ( s 2
+ 4 s + 5) 
t  1  t −2t −1  1  t −2t
= L−1   = 2 e L  2  = 2 e sin t
2  ( s + 2 ) 2
+ 1   s + 1

 s+2  t −2t
\ L−1  2 2
= e sin t
 (s + 4s + 5)  2

2.1.4 Type 4 – Inverse Laplace transform of logarithmic and trigonometric functions


WORKED EXAMPLES
Example 13
  s 1 1 
Find L21 log e  .
  s 2 1  
Solution.
s +1
Let F (s ) = loge = loge (s + 1) − loge (s − 1)
s −1
1 1
\ F ′ (s ) = −
s +1 s −1
1 1 1 1
But we know that L [t f (t )] = − F ′(s ) = − + = −
s +1 s −1 s −1 s +1

\  1  −1  1 
t f (t ) = L−1   − L  s + 1 = e − e
t −t

 s − 1   
e t − e −t 2 sinh t
\ f (t ) = =
t t
  s + 1   2
⇒ L−1 loge  = sinh t .
  s − 1  t

Example 14
 s ( s 1 1) 
Find L21 log e 2 .
 s 11 

Solution.
s (s + 1)
Let F (s ) = loge = loge s + loge (s + 1) − loge (s 2 + 1)
s2 +1
1 1 1
\ F ′ (s ) = + − 2 2s
s s +1 s +1

M03_Eng-Maths (Aditya) CH02.indd 10 7/24/2018 1:06:42 PM


Inverse Laplace Transforms  n  2.11

1 1 2s 2s 1 1
But L [t f (t )] = − F ′(s ) = − − + 2 = 2 − −
s s +1 s +1 s +1 s s +1

\  s  1  1  = 2 ⋅ cost − 1 − e −t
t f (t ) = 2L−1  2  − L−1   − L−1  
 s + 1 s   s + 1
2 cos t − (1 + e −t )
\ f (t ) =
t
 s (s + 1)  2 cos t − (1 + e −t )
\ L−1 loge 2 =
 s + 1  t
Example 15

Find L21  tan21


2
.
 s 

Solution.
2 1  2 2
Let F (s ) = tan −1   \  F ′(s ) =  − 2  = − 2
s  2
2
s s +4
1+  
 s
2
But L [tf (t )] = − F ′(s ) =
s +4
2

\  2  1
tf (t ) = L−1  2  = sin 2t   ⇒  f (t ) = sin 2t
s + 4 t

\  2  sin 2t
L−1  tan −1  =
 s t

Example 16
 s 
Find L s log e 1 cot21 s  .
21

 s2 11 

Solution.

s  1 
Let F (s ) = s loge + cot −1 s = s loge s − loge (s 2 + 1)  + cot −1 s
s +1
2
 2 

\ 1 1   1  −1
F ′ (s ) = s  − ⋅ 2s  + loge s − loge (s 2 + 1)  ⋅1 +
 s 2(s + 1)
2
  2  1+ s2
s2 1 1
= 1− 2 − + loge s − loge (s 2 + 1)
s +1 s2 +1 2

M03_Eng-Maths (Aditya) CH02.indd 11 7/24/2018 1:06:47 PM


2.12  n  Engineering Mathematics-III

s2 + 1 1 1
= 1− + log e s − log e ( s 2 + 1) = log e s − log e ( s 2 + 1)
s +1
2
2 2
1
But L [t f (t )] = − F ′(s ) = loge (s 2 + 1) − loge s
2
1 
t f (t ) = L−1  loge (s 2 + 1) − loge s 
 2 
1 1 
\ f (t ) = L−1  loge (s 2 + 1) − loge s  
t  2 
1 1 2s 1 s 1
Let loge (s 2 + 1) − loge s   \  G ′(s ) = . 2
G (s) = − = −
2 2 s +1 s s2 +1 s
1 s
L [t g (t )] = −G ′(s ) = − 2
But
s s +1

\ 1  s 
t g (t ) = L−1   − L−1  2 
s   s + 1
t 2 t
⇒   ⇒  g (t ) = sin 2
t g (t ) = 1 − cos t = 2 sin 2
2 t 2
1 2 2t 2 t
\ f (t ) = ⋅ sin = 2 sin 2
t t 2 t 2

\ L−1 s loge  2


s t
+ cot −1 s  = 2 sin 2
 s +1
2
 t 2

Example 17
 2 
Find the inverse Laplace transform of cot21   .
 s 1 1
Solution.
 2 
Let F (s ) = cot −1 
 s + 1
1  2  2
\ =
F ′ (s ) = −
4  − (s + 1) 2  (s + 1) 2 + 4
1+  
(s + 1) 2
2
But L [t f (t )] = − F ′(s ) = −
(s + 1) 2 + 4

 1 
\ t f (t ) = −2L−1  
 (s + 1) + 4 
2

 1  − t sin 2t
= −2e −t L−1  2 −t
 = −2e . 2 = −e sin 2t
 s + 4 

M03_Eng-Maths (Aditya) CH02.indd 12 7/24/2018 1:06:51 PM


Inverse Laplace Transforms  n  2.13

e −t sin 2t
\ f (t ) = −
t
  2  e −t sin 2t
\ L−1 cot −1  = −
  s + 1  t

EXERCISE 2.1
Type I  Find the inverse Laplace transform of the following:
3s − 2 s 3s 2 − 4s + 6
1. 2 2. 3.
s +1 ( s + 2) 2
s4
2s + 1 s 5s + 3
4. 5. 6.
s +s
2
s + 4s + 8
2
s + 2s + 5
2

s 3s + 7 3s − 2
7. 2 8. 9.
s + 4s + 5 s 2 − 2s − 3 s 2 − 4s + 20
2s 2 + 5s + 2 4s + 15 s 3 − 3s 2 + 8s − 6
10. 11. 12.
( s − 2) 4
16s − 25
2
(s 2 − 2s + 2) 2
3s + 2 s
13. 14. 2s − 3
s −4
2 a 2 2
s + b 2 15.
s + 4s + 13
2

s 1
16. 17.
s 2 − 4s + 5 (s − 3)5

Type II Find the inverse Laplace transform of the following functions by partial fraction method.
s −1 s+2 1− s
18. 19. 20.
s 2 + 3s + 2 s (s − 1)(s − 4) (s + 1)(s 2 + 4s + 13)
s+5 1 s2
21. 22. 23.
(s + 1)(s 2 + 1) s 2 (s 2 + 4 ) (s 2 + a2 )(s 2 + b 2 )

s s2 + s − 2 7s − 11
24. 25. 26.
(s + 1)(s + 4)
2 2
s (s + 3)(s − 2) ( s + 1)(s − 2) 2
s
27. [Hint: s 4 + s 2 + 1 = (s 2 + 1) 2 − s 2 = [(s 2 + 1) + s ][(s 2 + 1) − s ]]
s + s2 +1
4

s s 1
28. 29. 30.
(s + 1)(s − 2) 2
(s + 1) (s + 1)
2 2
s (s 2 + 8)
2

1 s 2s 2 − 6s + 5
31. 32. 33.
(s + a)(s + b ) (s − 4)(s + 5) s − 6s 2 + 11s − 6
3

s+9 2s + 1 s
34. 35. 36.
(s + 2)(s 2 + 3) (s + 2) 2 (s − 1) 2 (s 2 + a2 )(s 2 + b 2 )

M03_Eng-Maths (Aditya) CH02.indd 13 7/24/2018 1:07:00 PM


2.14  n  Engineering Mathematics-III

Type III  Find the inverse Laplace transform by multiplication by s and division by s types.

s2 s2 s
37. 38. 39.
( s − 2) 2
(s − 1) 4 ( s + 2) 4
1 1 s2
40. 41. 42.
s ( s + 2) 3
s (s − 2s + 5)
2
(s + a 2 ) 2
2

1
43.
(s + a 2 ) 2
2

Type IV  Find the inverse Laplace transform of the following Logarithmic and trigonometric
functions.
 v2   s + 2  46.  s 2 + a2 
44. log 1 + 2  45.
log   log  2 
 s   s + 4  s − b 2 
s −a   s + a  
47. log  2 2
48.  2  49.
tan −1   cot −1 
s +a   s + 1  b 
1+ s  s +1  a  s
50. loge  2  51. s loge tan −1   + cot −1  
+ 2 52.
 s  s −1  s  b

ANSWERS TO EXERCISE 2.1


e −2t (1 − 2t )
1. 3 cos t − 2 sin t 2. 3. 3t − 2t 2 + t 3
4. 1+ e −t 5. e −2t (cos 2t − sin 2t ) 6.
e −t (5 cos 2t − sin 2t )
7. e −2t (cos t − 2 sin t ) 8. 4e 3t − e −t 9.
e 2t (3 cos 4t + sin 4t )
e 2t 1 5t 3 5t
10. (12t + 39t 2 + 20t 3 ) 11. cos + sinh 12. e −t (cos t + 2t sin t )
16 4 4 4 4
1  bt  e −2t
13. 3 cosh 2t + sinh 2t 14. cos   15. [6 cos 3t − 7 sin 3t ]
a2 a 3
t 4e 3t
16. e 2t {cos t + 2 sin t } 17. 18. −2e −t + 3e −2t
24
1 t 1 4t 1 −t
19. −e + e (e − e −2t cos 3t − 2e −2t sin 3t )
20.
2 2 5
1 1
21. 2e t + 3 sin t − 2 cos t 22. ( 2t − sin 2t ) 23. [a sin at − b sin bt ]
8 a − b2
2

1 1 4 −3t 2 2t
24. (cos t − cos 2t ) 25. −2e −t + 2e 2t + te 2t
+ e + e 26.
3 3 15 5
2 3 t 1 t 1
27. sin t .sinh 28. [e − e −t − 2te t ] 29. (sin t − te −t )
3 2 2 4 2

M03_Eng-Maths (Aditya) CH02.indd 14 7/24/2018 1:07:10 PM


Inverse Laplace Transforms  n  2.15

1  sin 9t  1 1
30.  t −  31. [e − at − e −bt ] 32. [4e 4t + 5e −5t ]
81 9 b −a 9

et 3 sin 3t t
33. [1 − 2e t + 5e 2t ] 34.
e −2t − cos 3t + 35. [e t − e −2t ]
2 3 3
1 t  t3 
36. [cos at − cos bt ] 4
37. e 2t
(1 + 2t ) 38.
e  t + t 2
+
b 2 − a2 6 
1 −2t 2 1 1 t
39. e (3t − 2t 3 ) 40. [1 − e −2t ( 2t 2 + 2t + 1)] 41. [e sin 2t − 2e t cos 2t + 2]
6 8 10
1 1 2
42. [t ⋅ a cos at + sin at ] 43. 3 [sin at − at cos at ] 44. (1 − cos v t )
2a 2a t
1 −4t 2 1
45. (e − e −2t ) 46. (cosh bt − cos at ) 47. ( 2 cos at − e at )
t t t
e −t sin 2t 1 − at 1
48. 49. e sin bt 50. ( 2 − e −t )
t t t
2 1
51. [t cosh t − sinh t ] 52. (sin at + sin bt )
t2 t

2.1.5  Type 5 – Method of convolution


Definition 2.2  Let f (t) and g(t) be two functions defined for all t ≥ 0. The convolution of f (t) and g(t)
is defined as the integral
t

∫ f (u) g ( t 2 u)du .
0

It is denoted by f (t) * g(t) or (f * g)(t)


t

\ f ( t ) ∗ g ( t ) 5 ∫ f ( u) g ( t 2 u) du
0

Note
t
 a a

f (t ) ∗ g (t ) = ∫ f (t − u )g (t − (t − u )) du  { ∫ f ( x ) dx = ∫ f (a − x ) dx 
0  0 0 
t t

= ∫ f (t − u )g (u ) du = ∫ g (u )f (t − u ) du = g (t ) ∗ f (t )
0 0

\  the operation of the convolution or convolution product is commutative.

M03_Eng-Maths (Aditya) CH02.indd 15 7/24/2018 1:07:17 PM


2.16  n  Engineering Mathematics-III

Theorem 2.1 Convolution theorem

If L [f (t )] = F (s ) and L [g (t )] = G (s ),
L [f (t ) ∗ g (t )] = L [f (t )]L [g (t )] = F (s ).G (s )
then

Equivalently, L−1 [F (s ).G (s )] = f (t ) ∗ g (t ) = L−1 [F (s )] ∗ L−1 [G (s )]


t

Proof  We have    f (t ) ∗ g (t ) = ∫ f (u )g (t − u ) du


0
∞ ∞
t 
L [f (t ) ∗ g (t )] = ∫ e − st [f (t ) ∗ g (t )] dt = ∫ e  ∫ f (u )g (t − u )du  dt
− st
\
0 0 0 
∞ t

⇒ L [f (t ) ∗ g (t )] = ∫ ∫ e − st f (u )g (t − u )du dt (1)
0 0

The region of integration of this double integral is bounded by the lines u = 0, u = t, t = 0 and t = ∞
as in figure.
Changing the order of integration, we take a strip parallel to t-axis.
t varies from u to ∞ and u varies from 0 to ∞

∞ 
\ L [f (t ) ∗ g (t )] = ∫ f (u )  ∫ e − st g (t − u ) dt  du
0 u  u
Put v = t − u in the inner integral  \ dv = dt u=t
t=0
When t = u , v = 0 and when  t = ∞, v = ∞
∞ ∞

∫ e g (t − u ) dt = ∫ e
− st − s (v + u )
\ g (v ) dv
u 0 u=0 t

= e − su ∫ e − sv g (v ) dv
0


 ∞

\ L [f (t ) ∗ g (t )] = ∫ f (u ) e − su ∫ e − sv g (u ) dv  du
0  0 
∞ ∞

= ∫ e − su f (u ) du ∫ e − sv g (v )dv
0 0

= L [f (t )] ⋅ L [g (t )] = F (s ) ⋅ G (s )

\ L21 [ F ( s )G ( s )] 5 f ( t ) ∗ g ( t ) 5 L21 [ F ( s )] ∗ L21 [G ( s )]


t

It can be written as L−1 [F (s )G (s )] = ∫ f (u )g (t − u )du  n


0

M03_Eng-Maths (Aditya) CH02.indd 16 7/24/2018 1:07:22 PM


Inverse Laplace Transforms  n  2.17

WORKED EXAMPLES
Example 1 
 s 
Find L21  2 2 2 
using convolution theorem.
 ( s 1 a ) 
Solution.
s s 1
We can write = 2 . 2
(s + a )
2 2 2
(s + a ) (s + a 2 )
2

s 1
Here F (s ) = and G (s ) =
s 2 + a2 s 2 + a2
 s   s 1 
\ L−1  2 2 2
= L−1  2 . 2 2
 (s + a )  s + a s + a 
2

 s  −1  1  [by convolution
= L−1  2 2 
∗L  2 2 
 theorem]
 (s + a )   (s + a ) 
1
= cos at ∗ sin at
a
t
t 1
== ∫∫ cos au..1 sin
cosau sinaa((tt −−uu))du
du
0
0 aa
    1 tt
== 2a ∫∫ 22sin(
1 sin(atat −− au
au))cos
cosauaudu
du
2a 0 0
t
11 sin(at − au + au ) + sin(at − au − au )} du
== 2a ∫∫{{sin(
t

at − au + au ) + sin(at − au − au )} du
2a 0 0
t
11 sin at + sin(at − 2au )} du
== 2a ∫∫{{sin
t

at + sin(at − 2au )} du
2a 0 0
t
11 sin at ⋅ u − cos( at −− 22au
cos(at au))t
== 2a sin at ⋅ u −  0
2a  −−22aa 0

11  sin at + 11 cos at −  0 + co at  11


cossat
== 2a tt sin at + 2a cos at − 0 + 2a  == 2att sin
sinat
at
   2a  2a  2a   2a

Example 2 
 1 
Apply the convolution theorem to find L21  2 2 
.
 s ( s 2a ) 
Solution.
1 1 1
We can write = . 2
s (s − a ) s (s − a 2 )
2 2

M03_Eng-Maths (Aditya) CH02.indd 17 7/24/2018 1:07:25 PM


2.18  n  Engineering Mathematics-III

1 1
\ F (s ) = and G (s ) = 2
s s − a2
 1   1  1  [by convolution
\ L−1  2 = L−1   ∗ L−1  2
 s ( s − a 2 
)   s   s − a2  theorem]
t
1 1
= 1 ∗ sinh at = ∫ sinh au ⋅1du
a a0

Here f (t) = sinh at, g(t) = 1   \  f (u) g(t-u) = f (u) ⋅1 = sinh au
t
 1  1  cosh au  1 1
\ L−1  2 2 
=   = 2 [cosh at − cosh 0] = 2 [cosh at − 1] [{ cosh 0 = 1]
 s (s − a )  a  a  0 a a

Example 3
 s2 
Find L21  2 2 2 2 
using convolution theorem.
 ( s 1 a )( s 1 b ) 
Solution.
 s2   s s 
L−1  2 2 
= L−1  2 . 2
 ( s + a 2
)( s 2
+ b )   s + a 2
s + b 2 

 s   s 
== LL−−11  2 s 2 ∗∗LL−−11  2 s 2 
ss 2 ++aa2  ss 2 ++bb 2 
== cos
cosat at ∗∗cos
cosbt
bt
t
t
== ∫ cos au.cos b (t − u )du
∫ cos au.cos b(t − u )du
0
0
t
1 t
== 1 ∫ 22cos au cos(bt − bu )du
22 0∫ cos au cos(bt − bu )du
0
t
11 t
== ∫ [ cos{( a − b )u + bt } + cos((a + b )u − bt ) ] du
22 0∫ [ cos{(a − b )u + bt } + cos((a + b )u − bt ) ] du
0
t
11  sin[( a − b )u + bt ] sin[(a + b )u − bt ]  t
==  sin[(a − b )u + bt ] ++ sin[(a + b )u − bt ]
22  aa−−bb aa++bb 0
0

1
=
2(a − b )
[sin{(a − b )t + bt} − sin{(a − b ).0 + bt}]
1
+
2(a + b )
[sin{(a + b )t − bt} − sin(0 − bt )]
1 1
= (sin at − sin bt ) + (sin at + sin bt )
2(a − b ) 2(a + b )

M03_Eng-Maths (Aditya) CH02.indd 18 7/24/2018 1:07:29 PM


Inverse Laplace Transforms  n  2.19

1  a+b + a −b a − b − (a + b ) 
= 
sin at + sin bt 
2  (a + b )(a − b ) (a + b )(a − b ) 
1  2a 2b  a sin at − b sin bt
=  2 sin at − 2 sin bt  =
2 a − b2 a − b2  a2 − b 2

Example 4 
4
Using convolution theorem find the inverse Laplace transform of .
( s 2 1 2s 1 5) 2
Solution.
 4   2 2 
L−1  2 2
= L−1  2 . 2 
 (s + 2s + 5)   (s + 2s + 5) (s + 2s + 5) 
 2  −1  2 
= L−1  2 ∗L  2 
 (s + 2s + 5)   (s + 2s + 5) 
 2  −1  2 
= L−1  ∗L  
 (s + 1) + 4   (s + 1) + 4 
2 2

 2  −t −1  2 
= e −t L−1  2  ∗e L s2 + 4 
s + 4  
= e −t sin 2t ∗ e −t sin 2t
t

= ∫ e −u sin 2u e − (t −u ) sin 2(t − u )du


0
t

= ∫ e −t sin 2u sin( 2t − 2u )du


0

e −t
t

2 ∫0
= [cos( 4u − 2t ) − cos 2t ]du

t
e −t  sin( 4u − 2t ) 
=  − cos 2t ⋅ u 
2  4 0
e −t 1 
=  4 [sin 2t − sin(0 − 2t )] − coos 2t [t − 0]
2
e −t  sin 2t + sin 2t 
=  − t cos 2t 
2  4 

e −t  2 sin 2t  e
−t
= − t cos 2t = [sin 2t − 2t cos 2t ]
2  4 
 4

Example 5
 1 
Find L21  2  using convolution theorem.
 ( s 1 1)( s 1 2 s 1 2 ) 

M03_Eng-Maths (Aditya) CH02.indd 19 7/24/2018 1:07:31 PM


2.20  n  Engineering Mathematics-III

Solution.
 1  −1  1 1 
L−1   = L  (s + 1) . 2
 (s + 1)(s + 2s + 2) 
2
 s + 2s + 2 

 1   1 
= L−−11  1  ∗ L−−11  2 1 
= L  s + 1 ∗ L  s 2 + 2s + 2 
 s + 1  s + 2s + 2 
 1   1 
= L−−11  1  ∗ L−−11  1 
= L  s + 1 ∗ L  (s + 1) 22 + 1
 s + 1  (s + 1) + 1
−t − t −1  1 
= e −t ∗ e −t L−1  2 1 
= e ∗ e L  s 2 + 1
 s + 1
= e −−tt ∗ e −−tt sin t
= e ∗ e sin t
t

= ∫ e −−uu sin u e −− ((tt −−uu ))du


t

= ∫0 e sin u e du
[Here f (t ) = e − t sin t , g (t ) = e − t ]
0 
t
= e − t ∫ sin u du = e − t [ − cos u ]t0 = −e − t [cos t − cos 0] = e − t [1 − cos t ]
0

EXERCISE 2.2
Using convolution theorem evaluate the inverse Laplace transform of the following functions.
1 1 s2
1. 2. 3.
(s + a)(s + b ) s (s + 4 )
2
(s 2 + 4 ) 2
1 s2 + s s
4. 5. 6.  
s 2 (s 2 + 9) (s 2 + 1)(s 2 + 2s + 2) (s 2 + 4 ) 3
1 2 1
  7.  8.   9.
s (s 2 + 4 ) 2 (s + 1)(s 2 + 4) s 2 (s + 1) 2
s 1 1
10.  11. 12.  
s +4
4
( s + 1)( s + 2 ) ( s + 1)( s 2 + 1)

ANSWERS TO EXERCISE 2.2


1 1 1
1. [e − at − e −bt ] 2. [1 − cos 2t ] 3. [sin t + 2t cos 2t ]
b −a 4 4
1 1
4. [3t − sin 3t ] 5. [e −t (sin t − 3 cos t ) + sin t + 3 cos t ]
27 5
t 1 1
6. [sin 2t − 2t cos 2t ] 7. [1 − cos 2t − t sin 2t ] 8. [2e −t + sin 2t − 2 cos 2t ]
64 16 5

M03_Eng-Maths (Aditya) CH02.indd 20 7/24/2018 1:07:36 PM


Inverse Laplace Transforms  n  2.21

sin t sinh t
9. t − 2 + e −t (t + 2) 10. e −t (1 − e −t )
11.
2
1
12. [sin t − cos t + e −t ]
2

2.7.6  Type 6:  Inverse Laplace Transform as Contour Integral


Let f (t) be a piecewise continuous function for all t ≥ 0 and is of exponential order a > 0. Suppose
L [f (t )] = F (s ), where s is complex and Re s > a. then F(s) is analytic in the domain Re s > a.
If F(s ) → 0 as s → ∞, then
c+i ∞
1
2p i c −∫i ∞
f (t ) = e st F (s ) ds , c > a (1)

where a is large enough so that all the finite number of singularities of F(s) lie in the part of the
Re s < a.
The integral (1) is called the complex inversion formula or Bromwhich’s integral formula,
which gives the inverse Laplace transform of the given function F(s).
The complex line integral (1) is evaluated by using the residue theorem, choosing a contour C
consisting of the line L R from c − iR to c + iR and the semicircle of radius R and centre at s = c
lying on the left of the line L R as in figure. The radius R is taken so large such that C encloses all the
singularities of the function e st F (s ). y
As R → ∞, we get c + iR
c+i ∞
1 Re s = c
f (t ) = L−1 [F (s )] = ∫
2pi c − i ∞
e st F (s ) ds
c x
n O α
= ∑ Res [e F (s )]s = sk , t > 0
st
|s-c| = R LR
k =1
c − iR

Working Rule
Given F(s), where L [f (t )] = F (s ) and F (s ) → 0 as s → ∞, then
c+i ∞
1
2pi c −∫i ∞
f (t ) = L−1 [F (s )] = e st F (s ) ds

= Sum of the residues of e st F (s ) at the poles of F (s ) .

WORKED EXAMPLES
EXAMPLE 1
 1 
Evaluate L21  2  by method of residues.
 ( s 21)( s 11) 

M03_Eng-Maths (Aditya) CH02.indd 21 7/24/2018 1:07:42 PM


2.22  n  Engineering Mathematics-III

Solution.
1
Given F(s ) = Clearly as s → ∞, F (s ) → 0
(s − 1)(s 2 + 1)

By contour integral, L−1 [F (s )] = Sum of the residues of e st F(s ) at the poles of F(s ).
The poles of F(s ) are given by (s − 1)(s 2 + 1) = 0   ⇒  s = 1 and s = ±i ,
which are simple poles.
1
R (1) = lim( s − 1)e st F ( s) = lim( s − 1)e st
s →1 s →1 ( s − 1)( s 2 + 1)
e st et et
= lim = =
s →1 s +1 1+1 2
2

1
R (i ) = lim(s − i ) e st F (s ) = lim(s − i )e st
s→i s→i (s − 1)(s + i )(s − i )
e st e it
= lim =
s→i (s − 1)(s + i ) (i − 1)(i + i )
e it e it e it (1 − i )
= =− =−
2( −1 − i ) 2(1 + i ) 4
e − it (1 + i )
Changing i to –i, R ( −i ) = −
4
 1 
\ L−1   = Sum of the residues of e F(s ) at the poles of F(s )
st

 (s − 1)(s + 1) 
2

e t e it (1 − i ) e − it (1 + i )
= − −
2 4 4
t
e 1 i
= − [e it + e − it ] + [e it − e − it ]
2 4 4
e t 2 cos t i
= − + 2i sin t
2 4 4
t
e cos t sin t 1 t
= − − = [e − cos t − sin t ]
2 2 2 2

EXAMPLE 2
1
Evaluate the inverse Laplace transform of by the method of residues.
s (s 2 a2 )
2 2

Solution.
1
Given F (s ) = Clearly as s → ∞, F (s ) → 0
s 2 (s 2 − a 2 )
By contour integral,
L−1 [F (s )] = Sum of the residues of e st F(s ) at the poles of F(s )
The poles of F(s) are s = 0, s = ±a, where s = 0 is a pole of order 2 and s = a, −a are simple poles.

M03_Eng-Maths (Aditya) CH02.indd 22 7/24/2018 1:07:46 PM


Inverse Laplace Transforms  n  2.23

1 d
\ R ( 0) = lim [s 2e st F (s )] = lim d s 2 e st 
( 2 − 1)! s → 0 ds  2 
 s (s − a ) 
s → 0 ds 2 2

d  e st 
= lim  2 
s → 0 ds (s − a 2 )
 
(s 2 − a2 )e st ⋅ t − e st ⋅ 2s
= lim
s→0 (s 2 − a 2 ) 2
( −a2 )e 0 ⋅ t − 0 × e 0 −a2t t
= = 4 =− 2
(0 − a2 ) 2 a a
R (a) = lim[(s − a)e st F (s )]
s→a
e st e st e at e at
= lim(s − a) = 2 = 2 = 3
s→a s (s − a ) s (s + a) a ⋅ 2a 2a
2 2 2

e − at e − at
Changing a to –a, R( −a) = =− 3 .
2( −a) 3
2a

 1 
\ L−1  2 2 2 
= Sum of the residues of e st F(s ) at the poles of F(s )
 s ( s − a ) 
t e at e − at
=− 2+ 3− 3
a 2a 2a
t 1 t sinh at 1
= − 2 + 3 (e ⋅ −e − at ) = − 2 +
at
3
= 3 [sinh at − at ]
a 2a a a a

EXAMPLE 3
1
Find the inverse Laplace transform of , by the method of residues.
( s 2 1 1) 2
Solution.
1
Given F (s ) = Clearly as s → ∞, F (s ) → 0
(s 2 + 1) 2

By contour integral L−1 [F (s )] = Sum of the residues of e st F(s ) at the poles of F(s ).

The poles of F(s) are s = ±i , which are poles of order 2.


1 d
\ R (i ) = lim [(s − i ) 2 e st F (s )]
( 2 − 1)! s → i ds

d  (s − i ) 2 e st 
= lim  
s→i ds  (s + i ) 2 (s − i ) 2 
d  e st 
= lim  
s→i ds  (s + i ) 2 

(s + i ) 2 ⋅ e st ⋅ t − e st 2(s + i ) ⋅1
= lim
s→i (s + i ) 4
(s + i )t e st − 2e st
= lim
M03_Eng-Maths (Aditya) CH02.indd 23
s→i (s + i ) 3 7/24/2018 1:07:49 PM
d  (s − i ) 2 e st 
= lim  
s→i ds  (s + i ) 2 (s − i ) 2 
2.24  n  Engineering Mathematics-III
d  e st 
= lim  
s→i ds  (s + i ) 2 
(s + i ) 2 ⋅ e st ⋅ t − e st 2(s + i ) ⋅1
= lim
s→i (s + i ) 4
(s + i )t e st − 2e st
= lim
s→i (s + i ) 3
(i + i )t e it − 2e it 2i t e it − 2e it 1
= = = − [te it + ie it ]
(i + i ) 3 −8i 4
1
Changing i to –i,  R ( −i ) = − [te − it − ie − it ]
4
 1  1 1
\ L−1  2 2
= − [te it + ie it ] − [te − it − ie − it ]
 (s + 1)  4 4

1
= − {t (e it + e − it ) + i[e it − e − it ]}
4
1 1 1
= − [t 2 cos t + i 2i sin t ] = − [t cos t − sin t ] = [sin t − t cos t ]
4 2 2

EXERCISE 2.3
I.  Evaluate the Laplace transform of the following functions by using the method of residues:
1 1 2s + 3
1. 2. 3.
(s − 2)(s + 1)
2
(s − 1) (s + 1)
2 2
(s − 2)(s + 1) 2
1 s2 2
4. 5. 6.
(s + 1) (s − 2) 2
3
(s + 4 )
2 2
(s + 1)(s 2 + 1)
5 s 2s
7. 8. 9.
s 2 (s + 5) 2 (s 2 + 1)(s 2 + 4) 2s 2 + 1
1 2s − 2
10. 11.
(s + 1)(s − 2) 2
(s + 1)(s 2 + 2s + 5)

ANSWERS to EXERCISE 2.3


1 1 t 7 2t 1
1. [e −2 t − 2 sin t − cos t ] 2. [e (t − 1) + cos t ] 3. e − (7 + 3t )e − t
5 2 9 9
e 2t e −t 2 1
4. (t − 1) + [3t + 4t + 2] 5. [sin 2t + 2 sin t ] 6. e − t + cos t + 3 sin t
27 54 4
t 2  t 
7. [e −5 t + 1] + [e −5 t − 1] 8. cos t − cos 2t 9. cos 
5 25  2 
1
10. [e − t + e 2 t (3t − 1)] 11. e − t [cos 2t + sin 2t − 1]
9

M03_Eng-Maths (Aditya) CH02.indd 24 7/24/2018 1:07:54 PM


Inverse Laplace Transforms  n  2.25

2.2 Application of Laplace transform to the solution of ordinary


differential equations
Given the linear differential equation with constant coefficients, we apply Laplace transform L on both
sides and get the solution of the differential equation.
For this we apply the formulae
 dy 
L   = L [ y ′ ] = s L [ y ] − y ( 0)
 dt 
d 2y 
L  2  = L [ y ′′ ] = s 2 L [ y ] − sy (0) − y ′(0)
 dt 

d 3y 
and L  3  = L [ y ′′′ ] = s 3 L [ y ] − s 2 y (0) − sy ′(0) − y ′′(0)
 dt 
Then the equation is reduced to an algebraic equation in L[y] and s, incorporating the initial conditions.
We group the terms and obtain L[y] = F (s)

\ y = L−1 [F (s )] = f (t )

which is the required solution.

2.2.1  First Order Linear Differential Equations with Constant Coefficients

WORKED EXAMPLES
EXAMPLE 1
dx
Solve the equation 1 x 5 sin vt , x ( 0 ) 5 2 by using Laplace transforms.
dt
Solution.
dx
The given equation is + x = sin vt (1)
dt
and when t = 0, x = 2.
Taking Laplace transform on both sides of equation (1), we get

 dx 
L   + L [x ] = L [sin vt ]
 dt 
v
⇒ sL [x ] − x (0) + L ( x ) =
s 2 + v2
v
⇒ (s + 1)L [x ] − 2 = 2
s + v2
v
⇒ (s + 1)L [x ] = 2 +2
s + v2

M03_Eng-Maths (Aditya) CH02.indd 25 7/24/2018 1:07:56 PM


2.26  n  Engineering Mathematics-III

v 2
⇒ L [x ] = +
(s + v )(s + 1) (s + 1)
2 2

 v  −1  2  −1  v 
\ x = L−1  2  + L  (s + 1)  = L  2  + 2e
−t

 (s + v )(s + 1)   (s + v )(s + 1) 
2 2
 
Using partial fractions,
1 A Bs + C
Let = +
(s 2 + v2 )(s + 1) s + 1 s 2 + v2
1 = A (s 2 + v2 ) + ( Bs + C )(s + 1)
1
Putting s = −1, 1 = A (1 + v2 ) ⇒ A =
1 + v2
Equating coefficients of s 2 and s on both sides, we get

1
A +B =0 ⇒ B = −A = −
1 + v2
1
and B +C = 0 ⇒ C = −B =
1 + v2
1 1
− s+
1 1 1
+ 1 + v2 1 + v2
2
\ =
(s + 1)(s + v ) (1 + v ) (s + 1)
2 2 2
s + v2

1 1 1  s 1 
= ⋅ +  − s 2 + v2 + s 2 + v2 
(1 + v ) ( s + 1) 1 + v2
2
 
\  v  v  1  v  s  1  v 
L−1  2 
= L−1   − L−1  2 2
+ L−1  2 2
 (s + 1)(s + v )  (1 + v )  s + 1 1 + v s + v  1+ v s + v 
2 2 2 2

v v 1
= e −t − cos vt + sin vt
1+ v 2
1+ v 2
1 + v2
v 1
= e −t + [sin vt − v cos vt ]
1+ v 2
1 + v2
\ the solution is
v 1
x = 2e −t + e −t + [sin vt − v cos vt ]
1+ v 2
1 + v2

 v  −t 1
⇒ x = 2 + 2
e + [sin vt − v cos vt ]
 1+ v  1 + v2

EXAMPLE 2
dy
Using Laplace transform, solve 2 y 5 1 2 2t , given that y 5 21 when t 5 0.
dt

M03_Eng-Maths (Aditya) CH02.indd 26 7/24/2018 1:08:00 PM


Inverse Laplace Transforms  n  2.27

Solution.
dy
The given equation is − y = 1 − 2t (1)
dt
and when t = 0, y = −1.
Taking Laplace transform on both sides of equation (1), we get
 dy 
L   − L [ y ] = L [1 − 2t ]
 dt 
⇒  
sL [ y ] − y [0] − L [ y ] = L [1] − 2L [t ]
1 2
⇒ (s − 1)L [ y ] + 1 = −
s s2
1 2
(s − 1)L [ y ] = − −1
s s2
⇒ 1 2 1 s−2 1
L[y ] = − − = −
s (s − 1) s 2 (s − 1) s − 1 s 2 (s − 1) s − 1
\
 s−2  −1  1  −1  s − 2 
y = L−1  2  − L  s − 1 = L  2  −e
t

 s (s − 1)     s (s − 1) 
Using partial fractions,
s−2 A B C
Let = + + 2
(s − 1)s 2
s −1 s s

⇒ s − 2 = A s 2 + Bs (s − 1) + C (s − 1)
Putting  s = 0, we get − 2 = −C ⇒ C=2
Putting  s = 1, we get 1− 2 = A ⇒ A = −1
2
Equating coefficients of s on both sides, we get
A + B = 0 ⇒ B = −A = 1

s−2 1 1 2
\ =− + + 2
(s − 1)s 2
s −1 s s

\  s−2   1  −1  1  1
L−1  2
= − L−1   + L   + 2 L−1  2  = −e t + 1 + 2t
 ( s − 1) s   s − 1 s s 
\ the solution is
y = − e t + 1 + 2t − e t = 1 + 2t − 2e t

M03_Eng-Maths (Aditya) CH02.indd 27 7/24/2018 1:08:05 PM


2.28  n  Engineering Mathematics-III

2.2.2 Ordinary Second and Higher Order Linear Differential Equations with Constant
Coefficients
WORKED EXAMPLES
Example 1
d 2y dy
Solve, using Laplace transform 2
24 1 3 y 5 e 2t , given y(0) 5 1 and y′(0) 5 0.
dt dt
Solution.
The given equations is y ′′ − 4 y ′ + 3y = e −t (1)
and y (0) = 1, y ′(0) = 0

Taking Laplace transform on both sides of equation (1), we get


L [ y ′′ ] − 4 L [ y ′ ] + 3L [ y ] = L [e −t ]
1
⇒ s 2 L [ y ] − sy (0) − y ′(0) − 4{sL [ y ] − y (0)} + 3L [ y ] =
s +1
1
⇒ [s 2 − 4s + 3]L [ y ] = s + − 4 [Since y (0) = 1 y ′(0) = 0]
s +1

s (s + 1) + 1 − 4(s + 1)
=
s +1
s + s + 1 − 4s − 4 s 2 − 3s − 3
2
= =
s +1 s +1
s 2 − 3s − 3 s 2 − 3s − 3
\ L [y ] = =
(s 2 − 4s + 3)(s + 1) (s − 3)(s − 1)(s + 1)

 s 2 − 3s − 3 
\ y = L−1  
 (s − 3)(s − 1)(s + 1) 
s 2 − 3s − 3 A B C
Let = + +
(s − 3)(s − 1)(s + 1) s − 3 s − 1 s + 1

⇒ s 2 − 3s − 3 = A (s − 1)(s + 1) + B (s + 1)(s − 3) + C (s − 3)(s − 1)

3
Put s = 3, then   A (3 − 1)(3 + 1) = 9 − 9 − 3   ⇒  8A = −3  ⇒  A = −
8
5
Put s = 1, then  B(1 + 1)(1 − 3) = 1 − 3 − 3     ⇒   −4 B = −5   ⇒  B =
4
1
Put s = −1, then  C( −1 − 3)( −1 − 1) = 1 + 3 − 3  ⇒  8C = 1  ⇒  C =
8

M03_Eng-Maths (Aditya) CH02.indd 28 7/24/2018 1:08:10 PM


Inverse Laplace Transforms  n  2.29

s 2 − 3s − 3 3 1 5 1 1 1
\ =− + ⋅ +
(s − 3)(s − 1)(s + 1) 8 (s − 3) 4 (s − 1) 8 (s + 1)

\  s 2 − 3s − 3  −1  3 1 5 1 1 1 
L−1   = L − ⋅ + ⋅ + 
 (s − 3)(s − 1)(s + 1)   8 (s − 3) 4 (s − 1) 8 (s + 1) 
3  1  5 −1  1  1 −1  1 
⇒ y = − L−1   + L  s − 1 + 8 L  s + 1
8 s − 3 4    
3 3t 5 t 1 −t 1 −t
= − e + e + e = [e − 3e 3t + 10e t ]
8 4 8 8
Example 2
d 2y dy
Using Laplace transform, solve 2
1 5 t 2 1 2t given that y 5 4 and y′ 5 22 when t 5 0.
dt dt
Solution.
The given equation is y ′′ + y ′ = t 2 + 2t

and y = 4, y′ = -2 when t = 0.
Taking Laplace transform on both sides of equation (1), we get

L [ y ′′ ] + L [ y ′ ] = L [t 2 ] + 2L [t ]

2! 1
⇒ s 2 L [ y ] − sy (0) − y ′(0) + sL [ y ] − y (0) = 3
+ 2⋅ 2
s s
2(s + 1)
⇒ (s 2 + s ) L [ y ] − 4s + 2 − 4 =
s3
2(s + 1) 2(s + 1)
⇒ s (s + 1) L [ y ] = 4s + 2 + 3
= 2(s + 1) + 2s +
s s3
2 2 2
\ L[y ] = + +
s s +1 s4

\ 1  1  1
y = 2L−1   + 2L−1   + 2L−1  4 
s   s + 1 s 
t3 t 3 6(1 + e −t ) + t 3
= 2 ⋅1 + 2 ⋅ e −t + 2 = 2(1 + e −t ) + =
3! 3 3

Example 3
d 2y dy dy
Solve 1 2 2 y 5 3 cos 3t 2 11 sin 3t with y(0) 5 0 and 5 6 at t 5 0 using Laplace
dt 2 dt dt
transforms.

M03_Eng-Maths (Aditya) CH02.indd 29 7/24/2018 1:08:13 PM


2.30  n  Engineering Mathematics-III

Solution.
The given equation is y ′′ + y ′ − 2 y = 3 cos 3t − 11sin 3t  (1)
and y = 0, y′ = 6 at t = 0
Taking Laplace transform on both sides, we get

L [ y ′′ ] + L [ y ′ ] − 2L [ y ] = 3L [cos 3t ] − 11L [sin 3t ]

s 3
⇒ s 2 L [ y ] − sy (0) − y ′(0) + sL [ y ] − y (0) − 2L [ y ] = 3 − 11⋅ 2
s +9
2
s +9
3(s − 11)
⇒ (s 2 + s − 2) L [ y ] − s ⋅ 0 − 6 − 0 =
s2 + 9
3(s − 11)
⇒ (s + 2)(s − 1) L [ y ] = 2 +6
( s + 9)
3(s − 11) + 6(s 2 + 9) 6s 2 + 3s + 21
⇒ L[y ] = =
(s − 1)(s + 2)(s + 9) (s − 1)(s + 2)(s 2 + 9)
2

 6s 2 + 3s + 21 
\ y = L−1  
 (s − 1)(s + 2)(s + 9) 
2

6s 2 + 3s + 21 A B Cs + D
Let = + + 2
(s − 1)(s + 2)(s + 9) s − 1 s + 2 s + 9
2

⇒ 6s 2 + 3s + 21 = A (s + 2)(s 2 + 9) + B (s − 1)(s 2 + 9)
+ (Cs + D )(s − 1)(s + 2)
Put s = 1. \                            A (1 + 2)(1 + 9) = 6 + 3 + 21 ⇒  30 A = 30       ⇒  A = 1
Put s = -2 \                         B( −2 − 1)( 4 + 9) = 24 − 6 + 21 ⇒ −39B = 39 ⇒   B = −1

Equating coefficients of s3,                     A + B + C = 0 ⇒      1 - 1 + C = 0     ⇒       C = 0


Equating the constant terms, 18A - 9B - 2D = 21   ⇒   18 + 9 - 2D = 21  ⇒ -2D = -6 ⇒ D = 3

\  1 1 3 
y = L−1  − + 2 
s −1 s + 2 s + 9
 1  −1  1  −1  1 
= L−1   − L  s + 2  + 3L  s 2 + 9 
 s − 1     
sin 3t
= e t − e −2t + 3 = e t − e −2t + sin 3t
3

EXAMPLE 4
d 3y d 2y dy dy d 2y
Solve the differential equation 1 2 2 2 2 y 5 0 , where y 5 1, 5 2 , 5 2 at t 5 0
dt 3 dt 2 dt dt dt 2
t 5 0.

M03_Eng-Maths (Aditya) CH02.indd 30 7/24/2018 1:08:18 PM


Inverse Laplace Transforms  n  2.31

Solution.
The given equation is
d 3y d 2 y dy
3
+2 2 − − 2 y = 0 (1)
dt dt dt
dy d 2y
and when t = 0, y = 1, = 2 and 2 = 2
dt dt
That is, when t = 0, y = 1, y ′(0) = 2 and y ″(0) = 2.
Taking Laplace transform on both sides of equation (1), we get

d 3y  d 2y   dy 
L  3  + 2  2  − L   − 2L [ y ] = 0
 dt   dt   dt 
⇒  s 3 L [ y ] − s 2 y (0) − sy ′(0) − y ″(0) + 2[s 2 L [ y ] − sy (0) − y ′(0)] − [sL [ y ] − y (0)] − 2L [ y ] = 0

⇒ [s 3 + 2s 2 − s − 2]L [ y ] − s 2 − 2s − 2 − 2s − 4 + 1 = 0

⇒ [s 2 (s + 2) − (s + 2)]L [ y ] − (s 2 + 4s + 5) = 0

⇒ [(s 2 − 1) (s + 2)]L [ y ] = s 2 + 4s + 5
s 2 + 4s + 5 s 2 + 4s + 5
⇒ L[y ] = =
(s − 1) (s + 2) (s − 1) (s + 1) (s + 2)
2

 s 2 + 4s + 5 
\ y = L−1  
 (s − 1)(s + 1)(s + 2) 
Using partial fractions,
s 2 + 4s + 5 A B C
let = + +
(s − 1)(s + 1)(s + 2) s − 1 s + 1 s + 2
⇒ s 2 + 4s + 5 = A (s + 1)(s + 2) + B (s − 1)(s + 2) + C (s − 1)(s + 1)
5
Putting s = 1, we get 1 + 4 + 5 = A(1 + 1)(1 + 2) ⇒ 6 A = 10 ⇒ A =
3
Putting s = -1, we get 1 − 4 + 5 = B( −1 − 1)( −1 + 2) ⇒ − 2 B = 2 ⇒ B = −1

1
Putting s = -2, we get  4 − 8 + 5 = C ( −2 − 1)( −2 + 1) ⇒ 3C = 1 ⇒C=
3
s 2 + 4s + 5 5 1 1 1 1
\ = − +
(s − 1)(s + 1)(s + 2) 3 s − 1 s + 1 3 s + 2

 s 2 + 4s + 5  5 −1  1   1  1 −1  1 
\ L−1  = L   − L−1   + L s + 2
 (s − 1)(s + 1)(s + 2)  3  s − 1  s + 1 3  
5 1
⇒ y = e t − e −t + e −2t
3 3

M03_Eng-Maths (Aditya) CH02.indd 31 7/24/2018 1:08:22 PM


2.32  n  Engineering Mathematics-III

5 1 1
\ the solution is y = e t + e −2t − e −t = [5e t + e −2t ] − e −t
3 3 3

2.2.3  Ordinary Second Order Differential Equations with Variable Coefficients


We know that d
L [t f (t )] = − {L [f (t )]}
ds
d
L [t f ′(t )] = − {L [f ′(t )]}
ds
d
L [t f ″ (t )] = − {L [f ″(t )]}
ds
Using these formulae, we can solve second order differential equations with variable coefficients.

WORKED EXAMPLES
EXAMPLE 1
d 2y dy
Solve the differential equation t 2
12 1 ty 5 cos t , given that y ( 0 ) 5 1.
dt dt
Solution.
d 2y dy
The given solution is t 2
+2 + ty = cos t (1)
dt dt
and when t = 0, y = 1
Taking Laplace transform on both sides of equation (1), we get

 d 2y   dy 
L t 2  + 2L   + L [ty ] = L [cos t ]
 dt   dt 
d d s
⇒ {L [ y ″ ]} + 2L [ y ′ ] − {L [ y ]} = 2
  −
ds ds s +1
d d s
⇒    − [s 2 L [ y ] − sy (0) − y ′(0)] + 2[sL [ y ] − y (0)] − {L [ y ]} = 2
ds ds s +1
d d s
⇒ −  s 2 L[ y ] − s − y ′(0)  + 2 sL[ y ] − 2 − { L[ y ]} = 2
ds ds s +1
∴ y ′(00) is a constant 
 d  d s d 
⇒ −  s 2 { L[ y ]} + 2 sL[ y ] + 1 + 2 s { L[ y ]} − 2 − { L[ y ]} = 2
 ds  ds s +1  ( y ′(0)) = 0 
 ds 
d s
⇒ −(s 2 + 1) {L [ y ]} = 2 + 1
ds s +1
d s 1
⇒ {L [ y ]} = − 2 2 − 2
ds (s + 1) s + 1

M03_Eng-Maths (Aditya) CH02.indd 32 7/24/2018 1:08:25 PM


Inverse Laplace Transforms  n  2.33

 s 
{L [ y ]} = − L−1  2 2  − L−1  2 
d 1
⇒ L−1
ds  ( s + 1)   + 1
s

We know L−1 [F ′(s )] = −t f (t ), where L [f (t )] = F (s ) . Here y = f(t)

 s   s 
\ −ty = − L−1  2 2
− sin t ⇒ ty = L−1  2 2
+ sin t
 ( s + 1)   ( s + 1) 
 s 
To find L−1  2 2
 ( s + 1) 
1 1
We know L [t ] =   \  L [t e i t ] =
s2 (s − i ) 2
(s + i ) 2
⇒ L [t (cos t + i sin t )] =
(s − i ) 2 (s + i ) 2
s 2 − 1 + 2i s s2 −1 2s
⇒ L [t cos t ] + iL [t sin t ] = = 2 +i 2
(s + 1)
2 2
(s + 1) 2 (s + 1) 2

Equating imaginary parts, we get

2s  s 
L [t sin t ] =   ⇒  t sin t = 2L−1  2 2
(s 2 + 1) 2  ( s + 1) 
 s  1
\ L−1  2 2
= t sin t
 ( s + 1)  2
1
\ ty= t sin t + sin t
2
1 sin t  1 1
⇒ y = sin t + =  +  sin t
2 t 2 t 
EXAMPLE 2
Solve the differential equation y ″ 1 2t y ′ 2 4 y 5 1, y ( 0 ) 5 y ′( 0 ) 5 0 .

Solution.
The given equation is
y ″ + 2ty ′ − 4 y = 1(1)

and when t = 0, y = 0 and y ′ = 0

Taking Laplace transforms on both sides of equation (1), we get

L[ y ″ ] + 2 L[t y ′ ] − 4 L[ y ] = L[1]

M03_Eng-Maths (Aditya) CH02.indd 33 7/24/2018 1:08:30 PM


2.34  n  Engineering Mathematics-III

d d
We know L[t y ′ ] = − {L[ y ′ ]} = − {sL[ y ] − y(0)}
ds ds
d 1
\ s 2 L[ y ] − s y(0) − y ′(0) − 2
ds
[ ( sL[ y ] − y(0)) ] − 4 L[ y ] =
s
 d  1
⇒ s 2 L[ y ] − 2  s ( L[ y ]) + L[ y ] − 4 L[ y ] =
 ds  s
d 1
⇒ −2 s ( L[ y]) − (6 − s 2 ) L[ y] =
ds s
d 6 − s2 1
⇒ ( L[ y]) + L[ y ] = − 2
ds 2s 2s

This is linear equation in L [ y ], where

6 − s2 3 s 1
P= = − , Q=− .
2s s 2 2s 2
\ the solution is

L[ y ]e ∫ = ∫ Q e∫
P ds P ds
ds + c

3 s s2 s2
Now, ∫ P ds = ∫  s − 2  ds = 3 loge s −
4
= loge s 3 −
4
s2 s2
e∫ = e
3
P ds loge s − −
\ 4
=se 3 4

2 2
s s
∫ P ds ds = − 1 s3 e − 4 ds = − 1 e − 4 s ds
and ∫ Q e ∫ 2s 2 2∫

s2 2s
Put = t ⇒ ds = dt   ⇒    s ds = 2 dt
4 4
2
1 −t  e −t  s

∫ Q e∫ ds = − 2 ∫ e 2 dt            = −  −1  = e = e 4
P ds −
−t
\

s2 s2 s2
− −
\ L [ y ]s e3 4
=e 4
+ c   ⇒  L[ y ]s = 1 + c e 3 4

2
1 c s
⇒ L[ y ] = 3 + 3 e 4
s s

All the initial conditions are used.


\ there is no initial condition to determine c.

M03_Eng-Maths (Aditya) CH02.indd 34 7/24/2018 1:08:35 PM


Inverse Laplace Transforms  n  2.35

Since we must have

lim L [ y ] = lim F (s ) = 0, we choose c = 0.


s →∞ s →∞

1 1 1
\  L[y ] = 3
  ⇒  y = L−1  3  = t 2 , t ≥ 0
s s  2
1 2
\ the solution is y = t , t ≥0
2

Exercise 2.4
I. Solve the following first order differential equations
dy dx
1. + 2 y = e 2t , given y = 0, when t = 0. 2. + x = e t , given x(0) = 1.
dt dt
dy
3. − y = e x , y (0) = −1. y ′(t ) − 4 y (t ) = t , y (0) = −1.
4.
dx
dy
5. + 2 y = sin t , y (0) = −1.
dt

II. Solve the following second and higher order differential equations:

1. y ′′ + 2 y ′ − 3y = 3 given y(0) = 4, y ′(0) = −7.

2. y ′′ + 4 y ′ + 3y = e −t , y(0) = y ′(0) = 1.

3. y ′′ − 3y ′ + 2 y = 4t + e 3t , when y(0) = 1 and y ′(0) = −1.

d 2y dy dy
4. +2 − 3y = sin t , y = = 0 when t = 0.
dt 2 dt dt

d 2x dx
5. + 2 + 5x = e −t sin t , when x(0) = 1, x ′(0) = 1.
dt 2 dt
6. y ′′ + y = t , y (0) = 1, y ′(0) = −2.

d 2y dy dy
7. 2
+4 + 4 y = 10 sin t , given y = 0 = when t = 0.
dt dt dt
d 2x
8. + x = t cos 2t , x (0) = 0, x ′(0) = 0.
dt 2
d 2y dy
9. 2
+6 + 9 y = 2e −3t , y (0) = 1, y ′(0) = −2.
dt dt
10. y ′′ − 3y ′ + 2 y = e −t , given y (0) = 1, y ′(0) = 0.

11. y ′′ − 2 y ′ + y = e , given y (0) = 2, y ′(0) = 1.


t

M03_Eng-Maths (Aditya) CH02.indd 35 7/24/2018 1:08:40 PM


2.36  n  Engineering Mathematics-III

12. 2 y ′′ − 5y ′ + 2 y = 3 sin t , given y (0) = 1, y ′(0) = 0.


d 2y dy
13. 2
−2 − 8y = 0 , given y(0) = 3, y′(0) = 6.
dt dt
14. (D2 + 4D + 4)y = e-t given that y(0) = 0, y′(0) = 0.

15. (D2 + 9)y = cos2t given that y(0) = 1, y   = −1.


p
 2
d 2x dx dx
16. − 2 + x = e t with x = 2, = −1 at t = 0.
dt 2 dt dt
17. y ′′ + 5y ′ + 6 y = 2 gives y ′(0) = 0 and y(0) = 0.

18. y ′′ − 3y ′ + 2 y = 4 given that y(0) = 2, y′(0) = 3.

d 2y dy
19. 2
− 3 + 2 y = e −t with y(0) = 1 and y′(0) = 0.
dt dt
d 3y d 2 y dy
20. +2 2 − − 2 y = 0 , given y (0) = 1, y ′(0) = 2 and y ″(0) = 2. at t = 0
dt 3 dt dt
d 3y d 2 y dy
21. + 2 − − 2 y = 0 , given y (0) = y ′(0) = 0 and y ″(0) = 6.
dt 3 dt 2 dt
d 3 y dy
22. − = 2 cos t , y (0) = 3, y ′(0) = 2, y ″(0) = 1.
dt 3 dt
d 3y d 2y dy
23. 3
− 3 2
+ 3 − y = 16e 3t , given y (0) = 0, y ′(0) = 4, y ″(0) = 6.
dt dt dt
d4y d 2y
24. + 2 2 + y = sin t , given y (0) = y ′(0) = y ″(0) = y ″′(0) = 0.
dt 4 dt
III. Solve the following second order differential equations with variable coefficients:
25. ty ″ + (1 − 2t ) y ′ − 2 y = 0 , when y (0) = 1, y ′(0) = 2.
26. y ″ + 2ty ′ − y = t , when y (0) = 0, y ′(0) = 1.
27. ty ″ + 2 y ′ + ty = cos t , given y (0) = 1.

ANSWERS TO EXERCISE 2.4


I.
1 e t + e −t
1. y = [e 2t − e −t ] 2.
y = y = e x ( x − 1)
or cosh t 3.
3 2
1 5 1 1 3
y = e −t − cos t + sin t
4. y = − t − e 4t 5.
4 4 2 2 2

M03_Eng-Maths (Aditya) CH02.indd 36 7/24/2018 1:08:49 PM


Inverse Laplace Transforms  n  2.37

II.
1 1
1. y = (15e t + 5e −t − 4) 2. y = (7e −t − 3e −3t − 2te −t )
4 4
1 1 1 1
3. y = 2t + 3 + (e 3t − e t ) − 2e 2t 4. y = e t − e −3t − ( 2 sin t + cos t )
2 8 40 10
1 −t
5. x = ⋅ e (sin t + sin 2t )   6. y = t − sin t + cos t
3
1 1
7. y = (5e −t − e −3t ) + sin t − 2 cos t 8. x = ( 4 sin 2t − 5 sin t − 3t cos 2t )
2 9
1
9. y = e −3t (1 + t + t 2 ) 10.
y = (e −t + 9e t − 4e 2t )
6
1 t 1
11. y = ⋅ e (t − 2) 2 12. y = (3 sin t + e 2t − 4e t / 2 )
2 5
e −t − (1 + t )e −2t
13. 2e 4t + e −2t 14.

1 et 2
15. [4(cos 3t + sin 3t ) + cos 2t ] 16. [t − 6t + 4]
5 2
1
17. [1 − 3e −2t + 2e −3t ] 18. 2 − 3(e t − e 2t )
3
3 2 1 1
y = [5e t + 2e −2t ] − e −t
19. ⋅ e t − ⋅ e 2t + ⋅ e −t . 20.
2 3 6 3
7 e −t
21. y = e t − 3e −t + 2e −2t 22.
y = et + − cos t
2 2
1 2t 1
23. y = [e + e −2t + 2t ] 24.
y = [(3 − t 2 ) sin t − 3t cos t ]
4 8

III.
1  2
25. y = e 2t 26.
y = t 27.
y = 1 +  sin t
2 t 

2.2.4  Simultaneous Differential Equations

WORKED EXAMPLES
EXAMPLE 1
dx dy
Solve by using Laplace transform 1 y 5 sin t , 1 x 5 cos t given that x 5 2 and y 5 0,
dt dt
when t 5 0.

M03_Eng-Maths (Aditya) CH02.indd 37 7/24/2018 1:08:54 PM


2.38  n  Engineering Mathematics-III

Solution.
The given equations are
dx dy
+ y = sin t     (1)    and + x = cos t (2)
dt dt

When t = 0, x = 2 and y = 0
Taking Laplace transform on both sides of equations (1) and (2), we get
 dx 
L   + L [ y ] = L [sin t ]
 dt 
1
⇒ sL[ x ] − x(0) + L[ y ] = 2
s +1
1
⇒ sL [x ] − 2 + L [ y ] = 2
s +1
1
⇒ sL [x ] + L [ y ] = 2 + 2 (3)
s +1
 dy 
and L   + L [x ] = L [cos t ]
 dt 
s
⇒ sL[ y ] − y(0) + L[ x ] =
s +1
2

s
⇒ L [x ] + sL [ y ] = 2 (4)
s +1
s
(3) × s ⇒ s 2 L [x ] + sL [ y ] = 2s + (5)
s2 +1
s s
(5) - (4) (s 2 − 1)L [x ] = 2s + 2 − = 2s
s +1 s2 +1
2s
⇒ L[ x ] =
s2 − 1
 2s 
\ x = L−1  2  = 2 cosht = e t + e −t
 s − 1
dx
(1) ⇒ (1) → y = sin t −
dt
d t
= sin t − (e + e −t ) = sin t − [e t − e −t ] = sin t − e t + e −t
dt
\ the solution is
x = e t + e −t    and   y = sin t − e t + e −t

M03_Eng-Maths (Aditya) CH02.indd 38 7/24/2018 1:08:59 PM


Inverse Laplace Transforms  n  2.39

EXAMPLE 2
The coordinates (x, y) of a particle moving along a plane curve at any time t are given by
dy dx
1 2 x 5 sin 2t and 2 2 y 5 cos 2t , (t > 0 ).
dt dt
If at t 5 0, x 5 1 and y 5 0, then show that by Laplace transforms, that the particle moves along
the curve 4 x 2 1 4 xy 1 5 y 2 5 4.

Solution.
The given equations are
dx dy
− 2 y = cos 2t    (1)  and  + 2x = sin 2t  (2)
dt dt
Given, when t = 0, x = 1 and y = 0.
Taking Laplace transform on both sides of equations (1) and (2), we get

 dx 
L   − 2L [ y ] = L [cos 2t ]
 dt 
s
⇒ sL [x ] − x (0) − 2L [ y ] =
s +4
2

s
⇒ sL [x ] − 1 − 2L [ y ] = 2
s +4
s
⇒ sL [x ] − 2L [ y ] = + 1(3)
s2 + 4
 dy 
and L   + 2L [x ] = L [sin 2t ]
 dt 
2
⇒ sL[ y ] − y(0) + 2 L[ x ] =
s +4 2

2
⇒ 2L [x ] + sL [ y ] = 2 (4)
s +4
2s
(3) × 2 ⇒ (3) × ( 2) ⇒ 2sL [x ] − 4 L [ y ] = 2 + 2(5)
s +4
2s
(4) × s ⇒ ( 4) × s ⇒ 2sL [x ] + s 2 L [ y ] = 2 (6)
s +4
(6) - (5) ⇒ (6) − (5) ⇒ (s 2 + 4)L [ y ] = −2

2 −1  2 
⇒ L[y ] = −   ⇒  y = − L  2    ⇒  y = − sin 2t
s +42
s + 4

dy d
From equation (2), 2x = sin 2t − = sin 2t − ( − sin 2t ) = sin 2t + 2 cos 2t
dt dt

M03_Eng-Maths (Aditya) CH02.indd 39 7/24/2018 1:09:03 PM


2.40  n  Engineering Mathematics-III

1
⇒ x= [sin 2t + 2 cos 2t ]
2
\ the general solution is
1
x=
(sin 2t + 2 cos 2t )
2
and y = − sin 2t
This is the parametric equations of the curve.
Eliminating t, we get the Cartesian equation.
      y = − sin 2t ⇒ sin 2t = − y
1 1 2x + y
\          x = [ − y − 2 cos 2t ]  ⇒  x + y = − cos 2t ⇒ cos 2t = −
2 2 2
and        sin 2t = -y
( 2x + y ) 2
But    cos 2 2t + sin 2 2t = 1   ⇒   +y2 =1
4
⇒      4 x 2 + 4 xy + y 2 + 4 y 2 = 4   
⇒   4 x 2 + 4 xy + 5y 2 = 4(7)

\ the particle moves along the curve given by equation (7).

EXAMPLE 3
dx dy d 2x dx
Solve the differential equations 1 5 t and 2 y 5 e 2t , given x 5 3, 5 22 , y 5 0 when t 5 0
dt dt dt 2 dt
when t 5 0.

Solution.
The given equations are
dx dy d 2x
+ = t    (1)   and 2 − y = e −t (2)
dt dt dt

Given when t = 0, x (0) = 3, x ′(0) = −2 and y (0) = 0


Taking Laplace transform on both sides of equations (1) and (2), we get
 dx   dy 
L   + L   = L [t ]
 
dt  dt 
1
⇒ sL [x ] − x (0) + L [ y ] − y (0) =
s2
1
⇒ sL [x ] − 3 + sL [ y ] − 0 =
s2
1 1 3
⇒ s ( L [x ] + L [ y ]) = + 3   ⇒  L [x ] + L [ y ] = 3 + (3)
s2 s s

M03_Eng-Maths (Aditya) CH02.indd 40 7/24/2018 1:09:08 PM


Inverse Laplace Transforms  n  2.41

d 2x 
and L  2  − L [ y ] = L [e −t ]
 dt 
1
⇒ s 2 L [x ] − sx (0) − x ′(0) − L [ y ] =
s +1
1 1
⇒ s 2 L [x ] − 3s + 2 − L [ y ] =   ⇒  s 2 L [x ] − L [ y ] = + 3s − 2(4)
s +1 s +1
1 3 1
(3) + ( 4) ⇒ (s 2 + 1)L [x ] =
(3) + (4) ⇒ + + + 3s − 2
s3 s s + 1
1 3 1 3s 2
⇒ L [x ] = 3 2 + + + −
s (s + 1) s (s 2 + 1) (s + 1)(s 2 + 1) s 2 + 1 s 2 + 1

 1  −1  1  −1  1 
⇒ x = L−1  3 2  + 3L  2 +L  
 s (s + 1)   s (s + 1)   (s + 1)(s + 1) 
2

 s   1 
+ 3L−1  2  − 2L−1  2 
 s + 1  s + 1
 1  t t t −1  1 
Now L−1  3 2  = ∫ ∫ ∫ L  2  dt dt dt
 s (s + 1)  0 0 0  s + 1
t t t

= ∫ ∫ ∫ sin t dt dt dt
0 0 0
t t

= ∫ ∫ [ − cos t ]t0dt dt
0 0
t t

= ∫ ∫ −[cos t − cos 0] dt dt
0 0
t t

= ∫ ∫ (1 − cos t ) dt dt
0 0
t

= ∫ [t − sin t ]t0 dt
0
t

= ∫ [t − sin t ] dt
0
t
t 2  t2 t2
=  − ( − cos t )  = − 0 + [cos t − cos 0] = + cos t − 1
2 0 2 2

 1  t −1  1 
and L−1  2  = ∫ L  2  dt
 s (s + 1)  0  s + 1
t

= ∫ sin t dt = [ − cos t ]t0 = −(cos t − cos 0) = 1 − cos t



0

M03_Eng-Maths (Aditya) CH02.indd 41 7/24/2018 1:09:12 PM


2.42  n  Engineering Mathematics-III

 1 
To find L−1  
 ( s + 1)( s 2
+ 1) 
Using partial functions,
1 A Bs + C
Let = +
(s + 1)(s 2 + 1) s + 1 s 2 + 1

1 = A (s 2 + 1) + ( Bs + C )(s + 1)

1
Putting s = -1, 1 = 2A ⇒ A =
2
Equating the coefficients of s 2 and constant terms on both sides, we get
1
A+ B = 0 ⇒ B = −A = −
2
1 1
and and A + C = 1 ⇒ C = 1− A = 1− =
2 2
1 1
− s+
1 1 1
\ = + 22 2
(s + 1)(s 2 + 1) 2 (s + 1) s +1

 1  1 −1  1  1 −1  s  1 −1  1 
\ L−1   = 2 L  s + 1 − 2 L  2  + 2 L  2 
 ( s + 1)( s 2
+ 1)     s + 1  s + 1
1 −t 1 1
= e − cos t + sin t
2 2 2

 s   1 
L−1  2  = cos t and L−1  2  = sin t
 s + 1  s + 1
t2 1 1 1
\ x= + cos t − 1 + 3(1 − cos t ) + e −t − cos t + sin t + 3 cos t − 2 sin t
2 2 2 2
t 2 1 −t 1 3
⇒ x= + e + cos t − sin t + 2  (5)
2 2 2 2
d 2x
From equation (2), y = − e −t
dt 2

Differentiating Eq. (5) with respect to t, we get


dx 1 1 3
= t − e −t − sin t − cos t
dt 2 2 2
2
d x 1 1 3
= 1 + e −t − cos t + sin t
dt 2 2 2 2

M03_Eng-Maths (Aditya) CH02.indd 42 7/24/2018 1:09:15 PM


Inverse Laplace Transforms  n  2.43

1 1 3
\ y = 1 + e −t − cos t + sin t − e −t
2 2 2
1 1 3
= 1 − e −t − cos t + sin t
2 2 2
\ the solution is
t 2 1 −t 1 3
x= + e + cos t − sin t + 2
2 2 2 2
1 1 3
and y = 1 − e −t − cos t + sin t
2 2 2

2.2.5  Integral–Differential Equation


Differential equations involving integrals is known as integral–differential equation.

Examples:
t t
dy dx
1. + 3y + 2∫ y dt = t 2.  = t + ∫ x (t − u ) cos u du
dt 0
dt 0

WORKED EXAMPLES
EXAMPLE 1 t
dx
Solve the integral–differential equation 5 t 1 ∫ x (t 2 u ) cos u du , x (0) 5 2.
dt 0
Solution.
The given equation is
t t
dx
= t + ∫ x (t − u ) cos u du  ⇒   x ′(t ) = t + ∫ x (t − u ) cos u du  (1)
dt 0 0

and x (0) = 2.
In this equation, the integral is a convolution integral.
t

We know, f (t ) ∗ g (t ) = ∫ f (u ) g (t − u ) du
0

\ ∫ x (t − u ) ⋅ cos u du = cos t ∗ x (t )
0

\ the equation (1) becomes


x ′(t ) = t + cos t ∗ x (t ) (2)
Applying Laplace transform on both sides of equation (2), we get
L [x ′(t )] = L [t ] + L [cos t ] ⋅ L [x (t )]
1 s
⇒ sL [x (t )] − x (0) = + 2 L [x (t )]
s 2
s +1

M03_Eng-Maths (Aditya) CH02.indd 43 7/24/2018 1:09:19 PM


2.44  n  Engineering Mathematics-III

 s  1
⇒ s − s 2 + 1 L [x (t )] − 2 = s 2
 
 s (s 2 + 1) − s  1
⇒   L [x (t )] = 2 + 2
 s 2
+ 1  s
s3 1
⇒ L [x (t )] = 2 + 2
s +1
2
s
s2 +1  1 
⇒ L [x (t )] = 3  2
+ 2
s s 

s2 +1
= [1 + 2s 2 ]
s5
s 2 + 2s 4 + 1 + 2s 2 3s 2 + 2s 4 + 1 3 2 1
= = = 3+ + 5
s5 s5 s s s
3 2 1
\ x (t ) = L−1  3  + L−1   + L−1  5 
s  s  s 
1 1
= 3 t 2 + 2 ⋅1 + t 4
2 4!
3 2 t4
⇒ x (t ) = t + +2
2 24

which is the solution.

EXAMPLE 2 t

Solve the integral–differential equation y ′ 1 3 y 1 2 ∫ y dt 5 t , y ( 0 ) 5 0.


0

Solution.
The given equation is
t

y ′ + 3y + 2∫ y dt = t  (1)
0

and when t = 0, y = 0

Applying Laplace transform on both sides, we get


t 
L [ y ′ ] + 3L [ y ] + 2L  ∫ y dt  = L [t ]
0 
1 1
⇒ sL [ y ] − y ′(0) + 3L [ y ] + 2 ⋅ L [ y ] = 2
s s
2 1
⇒ sL [ y ] − 0 + 3L [ y ] + L [ y ] = 2
s s

M03_Eng-Maths (Aditya) CH02.indd 44 7/24/2018 1:09:23 PM


Inverse Laplace Transforms  n  2.45

 2 1
⇒ s + 3 + s  L [ y ] = s 2
 
 s 2 + 3s + 2  1
⇒   L[y ] = 2
 s  s

1 1
⇒ L[y ] = =
s (s + 3s + 2) s (s + 1)(s + 2)
2

 1 
\ y = L−1  
 s (s + 1)(s + 2) 
Using partial fractions,
1 A B C
Let = + +
s (s + 1)(s + 2) s s + 1 s + 2

⇒ 1 = A (s + 1)(s + 2) + Bs (s + 2) + C s (s + 1)
1 1
s = 0, then 1 ⇒
Putting s = 0, then 1 = 2A = 2AA = ⇒ A=
2 2
Putting s = −1, then 1 = Bs(=−1−)(11,)then 1 ⇒
= B (− =1−) 1
B1)( ⇒ B = −1
1 1
Putting s = −2, then 1 = Cs (=−2−)(
2,−then
1) 1 ⇒= C ( −C2)(
= −1) ⇒ C=
2 2
 1 1 1 1 1 
\ y = L−1  ⋅ − + ⋅ 
2 s s +1 2 s + 2
1 −1  1   1  1 −1  1 
= L   − L−1   + L s + 2
2 s   s + 1 2  
1 1
= ⋅1 − e −t + e −2t
2 2
1 1
= [1 − 2e −t + e −2t ] = (1 − e −t ) 2
2 2

Exercise 2.5
I. Solve the following simultaneous equations:
dx dy
1. + 2 y = 5e t , − 2x = 5e t , given x = −1, y = 3 when t = 0
dt dt
dx dy
2. + y = sin t + 1, + x = cos t , given that x = 1 and y = 2 at t = 0
dt dt
dx dy
3. + y = sin t , + x = cos t , given x = 2, y = 0 at t = 0
dt dt
dx dy
4. + 5x − 2 y = t , + 2x + y = 0, given x = y = 0 when t = 0
dt dt

M03_Eng-Maths (Aditya) CH02.indd 45 7/24/2018 1:09:28 PM


2.46  n  Engineering Mathematics-III

II. Solve the following integral–differential equations:

t t

1. y ′(t ) − 4 y (t ) + 3∫ y (t ) dt = t , y (0) = 1 2.
y ′(t ) − y (t ) = 6∫ y (t ) dt + sin t , y (0) = 2
0 0

t t

3. y (t ) = 1 + ∫ sin(t − u ) y (u ) du 4.
y (t ) = e −t + ∫ sin(t − u ) y (u ) du
0 0

ANSWERS to Exercise 2.5


I.

1. x = −e t , y = 3e t 2. x = e −t , y = 1 + sin t + e −t
1 2
3. x = e −t + e t , y = e −t − e t + sin t 4. x= [1 + 3t − (1 + 6t )e −3t ], y = [2 − 3t − e −3t (3t + 2)]
27 27
II.
1 −t 5 3t 7 1 63 22
1. y (t ) = y (t ) = − cot − sin t + e 3t + e −2t
− e + e 2.
3 3 50 50 50 50
t2
3. y (t ) = 1 + y (t ) = −1 + t + 2e −t
4.
2

M03_Eng-Maths (Aditya) CH02.indd 46 7/24/2018 1:09:31 PM


Multiple Integrals and Beta
Gamma Functions 3
3.1  Double Integration
Double integrals occur in many practical problems in science and engineering. It is used in problems
involving area, volume, mass, centre of mass. In probability theory it is used to evaluate probabilities
of two dimensional continuous random variables.

3.1.1  Double Integrals in Cartesian Coordinates


A double integral is defined as the limit of a sum. Let f(x, y) be a continuous function of two inde-
pendent variables x and y defined in a simple closed region R. Sub-divide R into element areas
∆A1 , ∆A 2 , …, ∆A n by drawing lines parallel to the coordinate axes.

y ∆Ai

(xi , yi )
R
O x

Fig. 3.1
Let (xi, yi) be any point in ∆A i .

Find the sum f ( x1, y1 )DA 11 f ( x2 , y2 )DA 2 1…1 f ( xn , yn )DA n 5∑ f ( xi , yi ) DA i


i =1

Increase the number of sub-divisions indefinitely large.  i.e., n → ∞ so that each ∆A i → 0.


n

In this limit, if the sum exists, i.e.,nlim


→∞
∑ f (x , y ) ∆A
i i i exists, then it is called the double integral of
∆A i → 0 i =1

f(x, y) over the region R and it is denoted by



∫∫ f ( x, y) dxdy.
R

M03_Eng-Maths (Aditya) CH03.indd 1 7/24/2018 2:00:09 PM


3.2  n  Engineering Mathematics-III

Note  The continuity of f(x, y) is a sufficient condition for the existence of double integral, but not
necessary. The double integral exists even if finite number of discontinuous points are there in R, but
f should be bounded.

3.1.2 Evaluation of Double Integrals


In practice, a double integral is computed by repeated single variable integration, integrate with
respect to one variable treating the other variable as constant.
Case 1:  If the region R is a rectangle given by R = {( x, y ) a ≤ x ≤ b , c ≤ y ≤ d}
where a, b, c, d are constants, then
y
b
 d
 d
 b

∫∫ f ( x, y) dx dy = ∫ ∫ f ( x, y) dy  dx = ∫ ∫ f ( x, y) dx  dy
R a c c a y=d
R
If the limits are constants the order of integration is immaterial, y=c
provided proper limits are taken and f(x,  y) is bounded in R
O x=a x=b x
Case 2:  If the region R is given by
Fig. 3.2
R = {( x, y ) a ≤ x ≤ b , g ( x ) ≤ y ≤ h ( x )}
y
where a and b are constants, then


b h(x )
 h(x )
∫∫R f ( x , y ) dx dy = ∫a  g ∫( x ) f (x, y ) dy  dx R
  g(x )

Here the limits of x are constants and the limits of y are functions
of x, so we integrate first with respect to y and then with respect O x=a x=b x
to x.
Case 3:  If the region R is given by Fig. 3.3

R = {( x , y ) g ( y ) ≤ x ≤ h ( y ) , c ≤ y ≤ d}
h(y)
y
where c and d are constants then
y=d
d
 h(y )

∫∫ f (x , y ) dx dy = ∫  ∫
R c g (y )
f ( x , y ) dx  dy

R

y=c
Since the limits of x are functions of y, we integrate first w.r.to g(y)
x and then w.r.to y. O x

Note  Fig. 3.4


(1) When variable limits are involved we have to integrate first
w.r.to the variable having variable limits and then w.r.to the
variable having constant limits.
(2) When all the limits are constants, the order of dx, dy determine the limits of the variable.

M03_Eng-Maths (Aditya) CH03.indd 2 7/24/2018 2:00:11 PM


Multiple Integrals and Beta Gamma Functions  n  3.3

WORKED EXAMPLES
Example 1
1 2

Evaluate ∫ ∫ x ( x 1 y ) dydx . 
0 1

Solution.
1 2

1 2

Let I = ∫ ∫ x( x + y ) dy dx = ∫  ∫ x( x + y ) dy  dx
0 1 0 1 
2
 y2 
1

= ∫ x  xy +  dx
0
 2 1
1
 22   1 
= ∫ x   x ⋅ 2 +  −  x ⋅1 +   dx
0  2  2 
1
1
 3
1
 3x   x 3 3x 2  1 3 13
= ∫ x  x +  dx = ∫  x 2 +  dx =  +  = + =
 2   2   3 2 ⋅ 2  0 3 4 12
0 0

Example 2
1 1
dxdy
Evaluate
∫∫
0 0 12 x 2 12 y 2
.

Solution.
1 1 1 1
dxdy dx dy
Let I = ∫ ∫ =∫ ⋅∫
0 0 1− x 2
1− y 2
0 1− x 2
0 1 − y2
p p p2
= [sin −1 x ]10 [sin −1 y ]10 = (sin −1 1 − sin −1 0) (sin −1 1 − sin −1 0) = ⋅ =
2 2 4

Note  We could write the integral in Example 2 as a product of two integrals because the limits are
constants and the functions could be factorised as x terms and y terms. This is not possible in Example 1,
even though the limits are constants.

Example 3
a a 2 2x 2

Evaluate ∫ ∫
0 0
x 2 y dxdy .

Solution.
a a2 − x 2 a a2 − x 2
Let I = ∫ ∫ x y dxdy = ∫
2
∫ x 2 y dydx
0 0 0 0

a2 − x 2
 y2 
a

=∫x   2
dx
0
 2 0
a
1 2 2
2 ∫0
= x ( a − x 2 )dx

a
1  x3 x5  1 a 3 a 5  1 2a 5 a 5
a
1
M03_Eng-Maths (Aditya) CH03.indd 3
=
2 ∫ ( a 2 x 2 − x 4 )dx =  a 2
2 3
−  = a2 ⋅ −  = ⋅
5 0 2 
=
3 5  2 15 15 7/24/2018 2:00:12 PM
3.4  n  Engineeringa Mathematics-III
a −x 2 2

y 2
=∫x   2
dx
0
 2 0
a
1 2 2
2 ∫0
= x ( a − x 2 )dx

a
1  x3 x5  1 a 3 a 5  1 2a 5 a 5
a
1
= ∫ ( a 2 x 2 − x 4 )dx =  a 2 −  = a2 ⋅ −  = ⋅ =
20 2 3 5 0 2  3 5  2 15 15

Example 4

∫∫ xy dxdy over the positive quadrant of the circle x


2
Evaluate 1y 2 5 a2 .
R

Solution.
Given that the region R is bounded by the coordinate axes y = 0, x = 0 and the circle x2 + y2 = a2.
So, the region of integration is the shaded region OAB as in Fig. 3.5.
To find the limits for x, consider a strip PQ parallel to x-axis, x varies from x = 0 to x = a2 − y 2 .
When we move the strip to cover the region it moves from y = 0 to y = a.
\ limits for y are y = 0 and y = a

a a2 − y 2 y
\
∫∫ xy dxdy = ∫ ∫ xy dxdy
R 0 0
(0, a) B x 2 + y 2 = a2

a2 − y 2
a
 x2 
= ∫ y⋅  dy P Q
0
 2 0
a A
1
2 ∫0
= y( a 2 − y 2 ) dy O (a, 0) x

a
1
a
1  y2 y4  Fig. 3.5
= ∫ ( a 2 y − y 3 ) dy =  a 2 − 
20 2 2 4 0
1  2 a2 a4  1 a4 a4
= a ⋅ −  = ⋅ =
2 2 4 2 4 8

Example 5

Evaluate ∫∫ xy dxdy , where A is the region bounded by x 5 2a and the curve x2 5 4ay.
A

Solution.
Given that the shaded region OAB is the region of integration bounded by y = 0, x = 2a and the parab-
ola x2 = 4ay as in Fig 3.6.
We first integrate w.r.to y and then w.r.to x.
To find the limits for y, we take a strip PQ parallel to the y-axis, its lower end P lies on y = 0 and
x2
upper end Q lies on x 2 = 4ay ⇒ y =
4a

M03_Eng-Maths (Aditya) CH03.indd 4 7/24/2018 2:00:14 PM


Multiple Integrals and Beta Gamma Functions n 3.5

x2 y y
x 2 = 4ay
\ the limits for y are y = 0 and y = .
4a
B
When the strip is moved to cover the area,
x varies from x = 0 to x = 2a.
Q x = 2a
2
x x2
2a 4a 2a
 y2  4a O
\ ∫∫ xy dxdy = ∫ ∫ xy dydx = ∫
R 0 0 0
x   dx
 2 0
P A(2a, 0) x

Fig. 3.6
2a
1  x6  1  26 a 6  a 4
2a 2a
1 x4 1
= ∫ x⋅ dx = ∫ x dx = 32a = =
5
2  
 6 0 
2 0 16 a 2 32a 2 0 32a 2  6  3

ExaMplE 6

Evaluate ∫∫ xy 2 y 2 dxdy, where R is the triangle with vertex (0, 0), (10, 1), (1, 1).
R

Solution.
Given that the region of integration is the triangle OAB as shown as Fig. 3.7.
y −0 x −0 y
Equation of OA is = ⇒ y =x
0 −1 0 −1
y −0 x −0 x A(1, 1) B
Equation of OB is = ⇒ y = (0, 1)
(10, 1)
0 − 1 0 − 10 10
We first integrate w.r.to x and then w.r. to y.
P Q
To find the limits for x, take a strip PQ parallel
to the x-axis. Its left end P is on x = y and right end
Q is on x = 10y.
\ the limits for x are x = y and x = 10y. O(0, 0) x
When the strip is moved to cover the region,
y varies from 0 to 1. Fig. 3.7

1 10 y 1 10 y 1 1
\ ∫∫
R
xy − y 2 dxdy = ∫
0 y
∫ xy − y 2 dxdy = ∫
0 y
∫ y 2 ( x − y ) 2 dxdy

1 1 10 y 1 
= ∫ y 2  ∫ ( x − y ) 2 dx  dy
0  y 
10 y

1  ( x − y)
1
3
2 
=∫y  2
 dy  ( x − a ) n +1 
{ ∫ ( x − a) dx =
n
3 
 2
0
 y
 n +1 

M03_Eng-Maths (Aditya) CH03.indd 5 7/24/2018 2:00:17 PM


3.6  n  Engineering Mathematics-III

2 12  
1 3 3
= ∫ y (10 y − y ) 2 − ( y − y ) 2  dy
30  
1
1
2 12 3 1
2 3 2  y3 
3 ∫0 3 ∫0
= y ( 9 y ) 2
dy = 3 y d y = 18   dy = 6[1 − 0] = 6.
 3 0

Example 7
Evaluate ∫∫ x dxdy
R
over the region R bounded by y2 5 x and the lines x 1 y 5 2, x 5 0, x 5 1.

Solution.
Given that the region of integration is the shaded y
region OAB as in Fig. 3.8. B (0, 2) Q y2 = x
To find A, solve x + y = 2 and y2 = x A(1, 1)
x+y=2
⇒ y2 = 2−y P x=1

⇒ y2 +y −2= 0 O (1, 0) x

⇒ (y + 2)(y -1) = 0  ⇒  y = −2, 1


\ x = 4, 1
\ A is (1, 1) and B is (0, 2) which is the point of Fig. 3.8
intersection of x = 0 and x + y = 2.
It is convenient to integrate with respect to y first
and hence find y limits.
Take a strip PQ parallel to y-axis. P lies on y2 = x and Q lies on x + y = 2.
\ the limits for y are y = x and y = 2 − x .
When the strip is moved to cover the region, x varies from 0 to 1.
1 2− x 1

\ ∫∫ x dxdy = ∫ ∫
R 0
x dydx = ∫ x ⋅ [ y ]2 −x x dx
0
x

= ∫ x  2 − x − x  dx
0
1
1
 x 2 x 3 x 5/ 2  1 2 15 − 5 − 6 4
= ∫ ( 2 x − x 2 − x 3/ 2 ) dx =  2 − −  = 1− − = =
0  2 3 5 /2 0 3 5 15 15

EXERCISE 3.1

∫∫ xy dxdy over the first quadrant of the circle x + y = a .


2 2 2
1. Evaluate

2. Evaluate x dxdy over the region bounded by the hyperbola xy = 6 and the lines y = 0, x = 1, x = 3.
∫∫
2

3. Evaluate
∫∫ xy − y dxdy , where R is a triangle with vertices (0, 0), (5, 1) and (1, 1).
2

R
x2 y 2
4. Evaluate ∫∫ ( x + y ) 2 dxdy over the area bounded the ellipse + = 1.
a2 b 2

M03_Eng-Maths (Aditya) CH03.indd 6 7/24/2018 2:00:20 PM


Multiple Integrals and Beta Gamma Functions  n  3.7

5. Evaluate ∫∫ ( x 2 + y 2 ) , where R is the region bounded by x = 0, y = 0 and x + y = 1.


R

6. Evaluate ∫∫ e 2 x + 3 y dxdy over the triangle bounded by x = 0, y = 0 and x + y = 1.


1 1 1 1
x −y x −y
7. Show that ∫ ∫ dydx ≠ ∫ ∫ dxdy .
0 0 (x + y ) 0 0 (x + y )
3 3

8. Compute the value of ∫∫ y dxdy , where R is the region in the first quadrant bounded by the ellipse
R
x2 y 2
+ = 1.
a2 b 2
9. Evaluate ∫∫ xy dxdy , where A is the domain bounded by x-axis, ordinate x = 2a and curve x2 = 4ay.
A
a b b a

10. Show that ∫ ∫ ( x + y ) dxdy = ∫ ∫ ( x + y ) dydx .


0 0 0 0
4 2 1+ x 2
dxdy 1
dydx
11. Evaluate ∫ ∫ . 12. 
Evaluate
2 1
xy ∫ 0

0 1+ x 2 + y 2
.

13. Evaluate ∫∫ xy ( x + y ) dxdy over the area between y = x and y = x.


2

14. Evaluate ∫∫ xy dxdy , where R is the region bounded by the parabola y2 = x, the x-axis and the line
R
x + y = 2, lying on the first quadrant.
15. Evaluate ∫∫ y dxdy over the region R bounded by y = x and y = 4x - x2.
R

ANSWERS TO EXERCISE 3.1


4
a 16 p ab 2 1
1. 2. 24 3.  4.  (a + b 2 ) 5. 
4 9 4 6
1 ab 2
a 4
ab
6. (e − 1) 2 ( 2e + 1) 8.  9.  10.  (a + b )
6 3 3 2
p
loge (1 + 2 ) 3 3 54
11. (log2) 2 12. 
4 13.  14.  15. 
56 8 5

3.1.3  Change of Order of Integration


a h(x )

The double integral with variable limits for y and constant limits for x is ∫ ∫ f ( x , y ) dydx . To evaluate
b g (x )
this integral, we integrate first w.r.to y and then w.r.to x. This may sometimes be difficult to evalu-
ate. But change in the order of integration will change the limits of y from c to d where c and d are
d h1 ( y )
constants and the limits of x from g1(y) to h1(y). The double integral becomes ∫ ∫ f ( x , y ) dxdy and
c g1 ( y )
hence the evaluation may be easy. To evaluate this integral, we integrate first w.r.to x and then w.r.to y.

M03_Eng-Maths (Aditya) CH03.indd 7 7/24/2018 2:00:24 PM


3.8  n  Engineering Mathematics-III

This process of changing a given double integral into an equal double integral with order of integra-
tion changed is called Change of order of integration.
For doing this we have to identify the region R of integration from the limits of the given double
integral. Sometimes this region R may split into two regions R1 and R2 when we change the order of
integration and hence the given double integral ∫∫ f ( x , y ) dxdy will be the sum of two double integrals.
R

i.e., ∫∫ f (x , y ) dxdy = ∫∫ f (x , y ) dxdy + ∫∫ f (x , y ) dxdy


R R1 R2

WORKED EXAMPLES
Example 1 
∞∞
e 2y
Evaluate ∫ ∫ dydx by changing the order of integration.
0 x
y
Solution.
∞∞
e −y
Let I = ∫ ∫ dydx
0 x
y
The region of integration is bounded by y = x, y = ∞, x = 0, x = ∞.
\ the region is unbounded as in Fig. 3.9.
In the given integral, integration is first with respect to y and then w.r.to x.
After changing the order of integration, first integrate w.r.to x and then w.r.to y. To find the limits of x,
take a strip PQ parallel to x-axis (see Fig. 3.10) with P on the line x = 0 and Q on the line x = y respectively.
y
y=x y y=x

P Q

O x O x

Fig. 3.9 Fig. 3.10


Given order of integration After the change of order of integration

\ the limits of x are x = 0 and x = y and the limits of y are y = 0 and y = ∞

∞ y −y ∞ ∞ −y ∞ ∞

\ I = ∫∫ e e− y e  e− y 
dxdy = ∫ ⋅ [ x ]0y dy = ∫ ⋅ ydy = ∫ e − y dy =  −∞
 = −(e − e ) = −(0 − 1) = 1
0

0 0
y 0
y 0
y 0
 − 1  0

Example 2 4 a 2 ax

Evaluate by changing the order of integration ∫ ∫ dydx .


0 x2
4a

M03_Eng-Maths (Aditya) CH03.indd 8 7/24/2018 2:00:26 PM


Multiple Integrals and Beta Gamma Functions  n  3.9

Solution.
4 a 2 ax

Let I = ∫ ∫ dydx
0 x2
4a
x2
The region of integration is bounded by y = , y = 2 ax and x = 0, x = 4a.
4a
x2
⇒ x 2 = 4ay is a parabola and y = 2 ax ⇒ y = 4ax is a parabola.
2
y =
4a
In the given integral, integration is first w.r.to y and then w.r.to x. After changing the order of
integration, we have to integrate first w.r.to x and then w.r. to y.

x 2 = 4ay x 2 = 4ay
y y
y 2 = 4ax y 2 = 4ax

A(4a, 4a) A(4a, 4a)


P
Q
x
O x O y =0

Fig. 3.11 Fig. 3.12


Given order of integration After the change of order of integration

To find the points of intersection of the curves x2 = 4ay and y2 = 4ax, solve the two equations.
x 4 = 16a2 y 2 = 16a2 ⋅ 4ax = 64a3 x
⇒ x ( x 3 − 64a3 ) = 0   ⇒  x = 0 and x 3 − 64a3 = 0
Now x 3 − 64 a3 = 0 ⇒ x 3 = 64 a3 = ( 4 a)3 ⇒ x = 4a

x 2 16a2
When x = 0, y = 0 and when x = 4a, y = = = 4a
4a 4a
Points of intersection are O(0, 0) and A is (4a, 4a)
Now to find the x limits, take a strip PQ parallel to the x-axis (see Fig. 3.11) where P lies on y2 = 4ax
and Q lies on x2 = 4ay.
y2
\ the limits of x are x = and x = 2 ay
4a
When the strip is moved to cover the region, y varies from 0 to 4a.
4a 2 a y 4a

\  I = ∫ ∫ dxdy = ∫ [ x]
2 a y
y2 / 4 a
dy
a y2 0
4a
4a
 y2 
= ∫
0


2 a y −  dy
4a 
4a
 1/ 2 1/ 2 y 2 
= ∫
0
 2a y −  dy
 4a 
4a
 y 3/ 2 1 y 3  4 a1/ 2 1 ( 4a)3 32a 2 16a 2 16 a 2
=  2a1/ 2 −  = ( 4 a)3 / 2 − = − =
M03_Eng-Maths (Aditya) CH03.indd 9
 3/2 4a 3  0 3 4a 3 3 3 3 7/24/2018 2:00:29 PM
3.10  n  Engineering Mathematics-III
4a
2
y 
= ∫ 2
0
a y−  dy
4a 
4a
 y2 
= ∫ 2a y1/ 2 −
1/ 2
 dy
0
4a 
4a
 y 3/ 2 1 y 3  4 a1/ 2 1 ( 4a)3 32a 2 16a 2 16 a 2
=  2a1/ 2 −  = ( 4 a ) 3/ 2
− = − =
 3/2 4a 3  0 3 4a 3 3 3 3

Example 3
a a1 a 2 2 y 2

Change the order of integration in ∫ ∫


0 a2 a 2 2 y 2
xy dxdy and then evaluate it.

Solution.
a a + a2 − y 2

Let I = ∫ ∫ xy dxdy
0 a − a2 − y 2

The region of integration is bounded by the lines y = 0, y = a and the curves x = a − a 2 − y 2 , x = a + a 2 − y 2

x = a ± a2 − y 2   ⇒  x − a = ± a2 − y 2 ⇒ ( x − a) + y = a
2 2 2
i.e.,
which is a circle with (a, 0) as centre and radius a.
The region of integration is the upper semi-circle OAB as in Fig. 3.14.
The original order is first integration w.r.to x and then w.r.to y. After changing the order of
integration, first integrate w.r.to y and then w.r.to x. To find the limits of y, take a strip PQ parallel to y-axis
(see Fig. 3.14), where P lies on y = 0 and Q lies on the circle ( x − a) 2 + y 2 = a2 .

y y

A Q A y=a
y=a (x − a)2 + y 2 = a2 (x − a)2 + y 2 = a2

O x O x
(a, 0) B(2a, 0) P (a, 0) B(2a, 0)

Fig. 3.13 Fig. 3.14


Given order of integration After the change of order of integration

\ the limits of y are y = 0 and y = a 2 − ( x − a) 2 = 2ax − x 2


When the strip is moved to cover the region, x varies from 0 to 2a.
2 a 2 ax − x 2 2 ax − x 2
2a  y2 
\ I = ∫ ∫
0 0
xydydx = ∫
0
x 
 2 0
dx.

M03_Eng-Maths (Aditya) CH03.indd 10 7/24/2018 2:00:34 PM


Multiple Integrals and Beta Gamma Functions  n  3.11

2a
1
2 ∫0
= x [2ax − x 2 ]dx

2a
1
2 ∫0
= ( 2ax 2 − x 3 )dx

2a
1  ( 2a)3 ( 2a) 4  1 16 a 16 a  1 16a
4 4 4
1  x3 x 4  2
=  2 a −  =  2 a − = 
 2 3 −  = = a4
2 3 4 0 2 3 4  4  2 12 3

Example 4 1 22 x

Change the order of integration in ∫ ∫ xy dydx and hence evaluate.


0 x2
Solution.
1 2− x

Let I = ∫ ∫ xy dydx
0 x2

The region of integration is bounded by x = 0, x = 1, y = x2, y = 2 – x.


In the given integral, first integrate with respect to y and then w.r.to x. After changing the order we
have to first integrate w.r.to x, then w.r.to y.

y y
y = x2 y = x2
B (0, 2) B

P′ Q′
A(1, 1) C A(1, 1)
P Q
O x=1 x O x

y=2−x y=2−x

Fig. 3.15 Fig. 3.16


Given order of integration After the change of order of integration

To find A, solve y = x 2 , y = 2−x

⇒ x 2 = 2 − x   ⇒  x 2 + x − 2 = 0   ⇒  ( x + 2)( x − 1) = 0 ⇒ x = −2, 1

Since the region of integration is OAB, x = 1 ⇒ y = 1

\ A is (1, 1) and B is (0, 2), which is the point of intersection of y-axis x = 0 and y = 2 – x

Now to find the x limits, take a strip parallel to the x-axis. We see there are two types of strips PQ and
P′ Q′ after the change of order of integration (see Fig. 3.16) with right end points Q and Q′ are respectively
on the parabola y = x2 and the line y = 2 – x. So, the region OAB splits into two regions OAC and CAB
as in Fig. 3.16.

M03_Eng-Maths (Aditya) CH03.indd 11 7/24/2018 2:00:37 PM


3.12  n  Engineering Mathematics-III

Hence, the given integral I is written as the sum of two integrals


In the region OAC, x varies from 0 to y and y varies from 0 to 1
In the region CAB, x varies from 0 to 2 – y and y varies from 1 to 2
\ I= ∫∫ xy dxdy = ∫∫ xy dxdy + ∫∫ xy dxdy
OAB OAC CAB

1 y 2 2− y

=∫ ∫ x y dx dy + ∫ ∫ xy dx dy
0 0 1 0
y 2− y
1
 x2  2
 x2 
= ∫ y ⋅   dy + ∫ y   dy
0
 2 0 1
 2 0
1 2
1 1
2 ∫0
= y y dy + ∫ y ⋅ ( 2 − y ) 2 dy
21
1 2
1 2 1
2 ∫0
= y dy + ∫ y( 4 − 4 y + y 2 )dy
21
1
1  y3  1
2

=   + ∫ ( 4 y − 4 y 2 + y 3 )dy
2  3 0 2 1
2
1 1  y2 y3 y 4 
= + 4 − 4 + 
6 2 2 3 4 1
1 1 2 2 4 3 3 1 4 4 
= + 2( 2 − 1 ) − ( 2 − 1 ) + ( 2 − 1 ) 
6 2  3 4 

1 1 4 1  1 1 [72 − 112 + 45] 1 5 9 3


= + 6 − × 7 + × 15 = + . = + = =
6 2 3 4  6 2 12 6 24 24 8
Example 5
1 2−x2
x
Evaluate ∫ ∫
0 x x +y2
2
dydx by changing the order of integration.

Solution.
1 2−x 2
x
Let I=∫ ∫ dydx
0 x x2 + y 2

The region of integration is bounded by x = 0, x = 1 and y = x, y = 2 − x 2 .

Now y = 2 − x2 ⇒ y2 = 2 − x2
x2 + y2 = 2, which is a circle, with centre (0, 0) and radius 2.
The region of integration is OAB as in Fig. 3.18.
To find A, solve
y = x and x 2 + y 2 = 2

M03_Eng-Maths (Aditya) CH03.indd 12 7/24/2018 2:00:40 PM


Multiple Integrals and Beta Gamma Functions  n  3.13

\   x 2 + x 2 = 2  ⇒  2x 2 = 2   ⇒  x = ±1
y=x
B
Since A is in the first quadrant, x = 1  \  y = 1
A (1, 1)
\ A is (1, 1) and B is (0, 2 ) , which is the point x=0
x 2 + y2 = 2
of intersection of x = 0 and x2 + y2 = 2
In the given integral, integration is w.r.to y x=1
O
first and then w.r.to x. After changing the order of
integration, first integrate w.r.to x and then w.r.to y. Fig. 3.17
To find the x limits, take a strip parallel to the x-axis. Given order of integration
We see there are two strips PQ and P′Q′ with ends
Q, Q′ on the line y = x and circle x2 + y2 = 2 respectively.
y=x
So, the region splits into 2 regions OAC and CAB. B
P′ Q′
In the region OAC, (0, 1)C A (1, 1)
x varies from 0 to y and y varies from 0 to 1 P Q x 2 + y2 = 2
In the region CAB,
O
x varies from 0 to 2 − y 2
and y varies from 1 to 2 Fig. 3.18
After the change of order of
x x
\ I= ∫∫
OAC x +y
2 2
dxdy +
CAB
∫∫ x +y2
2
dxdy integration

1 y 2 2 − y2

= ∫ ∫ (x + y )2 2 −1/ 2
x dxdy + ∫ ∫ ( x 2 + y 2 ) −1/ 2 x dxdy
0 0 1 0

1 y 2 2 − y2
1 1
= ∫ ∫ ( x 2 + y 2 ) −1/ 2 2 x dxdy + ∫ ∫ ( x 2 + y 2 ) −1/ 2 2 x dxdy
200 2 1 0

y 2 − y2
1  ( x 2 + y 2 )1 2   ( x 2 + y 2 )1 2 
1 2
1
= ∫  dy + ∫   dy
20 1 2 1
 1  
 2 0  2 0
1 2

= ∫ [( y 2 + y 2 )1/ 2 − ( y 2 )1/ 2 ]dy + ∫ [(2 − y


2
+ y 2 )1/ 2 − ( y 2 )1/ 2 ]dy
0 1

1 2
= ∫ ( 2 y − y )dy + ∫( 2 − y )dy
0 1
1 2
 y2   y2 
= ( 2 − 1)  +  2 y − 
 2 0  2 1
1 2  1
= ( 2 − 1) + 2 ⋅ 2 − −  2 ⋅1 − 
2 2  2
2 −1 ( 2 2 − 1) 2 −1+ 2 − 2 2 +1 2 − 2
= + 2 −1− = =
2 2 2 2

M03_Eng-Maths (Aditya) CH03.indd 13 7/24/2018 2:00:42 PM


3.14  n  Engineering Mathematics-III

Example 6
a ay 2a 2a2y a 2 a 2x

Show that ∫∫ xy dxdy 1 ∫ ∫ xy dxdy 5 ∫ ∫ xy dydx and hence evaluate.


0 0 a 0 0 x2
a
Solution.
a ay 2a 2a − y

Let I = ∫ ∫ xy dxdy + ∫ ∫ xy dxdy.


0 0 a 0

The given integral I has same integrand defined over two region R1 and R2 given by the two double
integrals.
Region R1 is bounded by y = 0, y = a and x = a, y
x 2 = ay
x = ay ⇒ x 2 = ay B(0, 2a)
y = 2a
x = a and x = ay intersect at A (a, a)
2 Q
R2
Region R2 is given by y = a and y = 2a and x = 0, C A(a, a)
x = 0 y=a
x = 2a − y. R1
(2a, 0)
The regions R1 and R2 are as shown in Fig. 3.19. P
O x x
R1 is the shaded region OAC
x + y = 2a
R2 is the shaded region CAB
x=a
The line x + y = 2a also passes through A and B.
Combining the two regions R1 and R2, we get the Fig. 3.19
shaded region OAB. In the given integral, we have
to integrate first with respect to x and then w.r.to y. Changing the order integration, we first integrate
w.r.to y, then w.r.to x. To find the y limits, take a strip PQ parallel to the y-axis with P on x2 = ay  ⇒ 
x2
y= and Q on x + y = 2a  ⇒  y = 2a - x.
a
x2
\ the limits for y are y = and y = 2a − x and the limits for x are x = 0 and x = a
a
a ay 2a 2a − y a 2a − x
\ I=∫ ∫ xy dxdy + ∫ ∫ xy dxdy = ∫ ∫ xy dydx
0 0 a 0 0 x2
a
2a− x
a
 y2 
= ∫ x   2 dx
0
 2 x
a

1
a
 x4 
= ∫
20 
x ⋅ ( 2a − x ) 2 − 2  dx
a 

1  2 x4 
a

2 ∫0 
= x  4 a − 4 ax + x 2
−  dx
a2 
1  2 x5 
a

2 ∫0 
=  4 a x − 4 ax 2
+ x 3
−  dx
a2 
a
1 x2 x3 x 4 x6 
=  4a 2 − 4a + − 2 
2 2 3 4 a ⋅ 6 0
1  2 2 4 a 3 a 4 a6 
=  2a ⋅ a − a + − 
2 3 4 6a2 

M03_Eng-Maths (Aditya) CH03.indd 14 7/24/2018 2:00:45 PM


1  2 x4 
a

= ∫ x  4 a − 4 ax + x 2 − 2  dx
20  a 
1  2 x5 
a

= ∫  4 a x − 4 ax + x − 2  dx
2 3

20 a
Multiple Integrals and Beta
a Gamma Functions  n  3.15
1 x2 x3 x 4 x6 
=  4a 2 − 4a + − 2 
2 2 3 4 a ⋅ 6 0
1  2 2 4 a 3 a 4 a6 
=  2a ⋅ a − a + − 
2 3 4 6a2 

1  4 4 a 4 a 4 a 4  a 4 [24 − 16 + 3 − 2] a 4 9 3a 4
=  2a − + − = = ⋅ =
2 3 4 6 2 12 2 12 8

EXERCISE 3.2
Change the order of integration in the following integrals and evaluate.
a 2a − x a a
x+ y
1. ∫ ∫0 ∫y x 2 + y 2 dxdy
∫ xy dydx 2. 
2
0 x /a
2 2
a/ 2 a − y ∞ y 1 2−x

∫ ∫ ∫ ∫ ye ∫ ∫ xy dydx
− y2 / x
  3.  log( x 2 + y 2 ) dxdy , a > 0 4.  dxdy 5. 
0 y 0 0 0 x2

6. ∫∫ xy dxdy , where R is region bounded by the line x + 2 y = 2 and axes in the first quadrant.
R

2 4 − y2 a a 2 a 3a − x
x
7. ∫ ∫ ∫0 ∫y x 2 + y 2 dxdy 9. 
∫0 2∫ ( x + y ) dydx
2 2
y dxdy 8. 
0 2− y x / 4a
3
16 − x 2
4 4 4 2 x 1 2 − x2
x
10.  ∫ ∫ ∫0 2∫ dydx 12. 
x dydx 11.  ∫0 ∫x x 2 + y 2 dxdy
0 0 x /4

a 2 ax 1 1+ y
2
3 4− y
dxdy
∫ ∫ ∫0 ∫1 ( x + y) dxdy
∫0 ∫0 1 + x 2 + y 2 15. 
2
13.  x dydx 14. 
0 0
a
2 a2 − x 2 1 2− y 3 6/ x

∫ ∫ ∫ ∫ x dydx.
∫ ∫ xy dxdy 18. 
2
16.  y 2 dydx 17. 
0 x 0 y 1 y=0

ANSWERS TO EXERCISE 3.2


3a4 pa a p a2  1 1 3
1. 2.  + log e 2 3.   log e a −  4.  5. 
8 4 2 2 2 2 8
1 4 pa 314 4
6. 7.  8.  9.  a 10. 10
6 3 4 35

64 1 4a4 p
11. 12. 1 − 13.  14.  log(1 + 2 )
3 2 7 4
241 a4 1
15. 16.  ( 2 + p) 24
17.  18. 
60 32 3

M03_Eng-Maths (Aditya) CH03.indd 15 7/24/2018 2:00:50 PM


3.16  n  Engineering Mathematics-III

3.1.4  Double Integral in Polar Coordinates


To evaluate the double integral of f(r, u) over a region R in polar coordinates, generally we integrate
u2 r = f 2 ( u )

first w.r.to r and then w.r.to u. So, the double integral is ∫ ∫


u1 r = f1 ( u )
f ( r , u) drd u.

However, whenever necessary, the order of integration may be changed with suitable changes in the
limits. As in Cartesian, when we integrate w.r.to r, treat u as constant.

WORKED EXAMPLES
Example 1
p
2 2 cos u
Evaluate ∫ ∫
p 0
r 2 drd u.
2
2

Solution.

p p p
2 cos u
2 2 cos u 2
r 
3
1 2

∫ ∫ ∫  3  du = ∫ 8 cos
3
Let I = r 2 drd u = ud u
p 0 p 0 3 p
− − −
2 2 2

p p [{ cos3 u is an even
2
8 8 2
= ∫p =
3 ∫0
⋅ 2 cos3 ud u  function ]
3
cos u d u
3

2

16 2 32
= ⋅ ⋅1 = [Using formula ]
3 3 9 

Important Formulae
p/2 p/2
n −1 n − 3 2
∫ cos n xdx = ∫ sin xdx = ⋅ … ⋅1 if n is odd and n ≥ 3
n

0 0
n n−2 3
p/2 p/2
n −1 n − 3 1 p
and ∫ cos n xdx = ∫ sin xdx = ⋅ … ⋅ if n is even
n

0 0
n n−2 2 2

n [ f ( x )]n +1
∫ [ f ( x )] f ′ ( x ) dx =
n +1
if n ≠ −1

Example 2
Evaluate ∫∫ rsin u drd u over the area of the cardioid r 5 a(1 1 cosu) above the initial line.
A

Solution.
Let I = ∫∫ r sin udrd u
A

M03_Eng-Maths (Aditya) CH03.indd 16 7/24/2018 2:00:53 PM


Multiple Integrals and Beta Gamma Functions  n  3.17

First integrate w.r.to r.


Take a radial strip OP, its ends are on r = 0 and
r = a(1 + cosu). When it is moved to cover the area, P
u varies from 0 to p
a (1+ cos u )
θ
p a (1+ cos u )
 r2 
p
θ=π O θ=0
∴ I=∫ ∫ r sin u drd u = ∫   sin ud u x
0 0 0
 2 0
p
1 2 Fig. 3.20
2 ∫0
= a (1 + cos u) 2 sin ud u

p
a2
2 ∫0
=− (1 + cos u) 2 ( − sin u)d u

a 2  (1 + cos u)3  
p
 d 
=−   { du (1+ cos u) = − sin u
2  3 0  

a2 a2 8a 2 4 a 2
=− (1 + cos p)3 − (1 + cos 0)3  = − (1 − 1)3 − (1 + 1)3  = =
6 6 6 3
Example 3

∫∫ r drd u, over the area bounded between the circles r 5 2 cos u and r 5 4 cos u .
3
Evaluate

Solution.
Let I = ∫∫ r 3drd u,
y π
A θ=
4
where the region A is the area between the circles
Q
r = 2 cos u and r = 4 cos u
P
The area A is the shaded area in the Fig. 3.21
We first integrate w.r.to r. So, take a radius vector OPQ, O θ
where r varies from P to Q. x
Area
\ r varies from 2 cos u to 4 cos u
When PQ is varied to cover the area A between
p p
r = 2 cos u and r = 4 cos u, u varies from − to
2 2
p p π
4 cos u θ=−
2 4 cos u
 r4  2 2
\ I = ∫ ∫ r 3 drd u = ∫   du
− p 2 cos u − p  4  2 cos u Fig. 3.21
2 2
p

1 2
= ∫ ( 4 4 cos 4 u − 24 cos 4 u)d u
4 −p
2
p
2
1
4 −∫p
= ( 256 − 16) cos 4 u d u
2

M03_Eng-Maths (Aditya) CH03.indd 17 7/24/2018 2:00:56 PM


3.18  n  Engineering Mathematics-III

p p

240 2 2
3 1 p 45p 4 u is even ]
4 −∫p ∫0 cos u d u = 120 × 4 ⋅ 2 2 =[{ cos
= cos 4
u d u = 60 × 2 4

2
2
p
2
3 1 p 45p
= 60 × 2 ∫ cos 4 u d u = 120 × ⋅ =  [Using formula ]
0
4 22 2
Example 4
rdrd u
∫∫
2 2
Evaluate , where R is the area of one loop of the leminiscate r 5 a cos 2u .
2 2
R r 1a
Solution.
π
rdrd u θ=
Let I = ∫∫
R r 2 + a2
First integrate with respect to r
P
Take a radial strip OP, its ends are r = 0 and
θ r =a 2θ
r = a cos 2 u
O x
When the strip covers the region, u varies
p p
from − to
4 4
π
θ=−
p p
  4 a cos 2 u 4 1 a cos 2 u 
\ r Fig. 3.22
I=
−p
∫ ∫ 0 r +a
2 2
drd u = ∫p  2 ∫0
( r 2 + a 2 ) −1/ 2 2rdr  d u


4 4
a cos 2 u
 2 +1 
p −1
1 4
 ( r + a 2 2
) 
=
2 ∫p  −1 
du
−  +1 
4  2 0
p
4
a cos 2 u
= ∫ (r + a 2 )1/ 2 
2
du
0
p

4
p
4
= ∫ (a cos 2 u + a 2 )1/ 2 − ( a 2 )1/ 2  d u
2

p

4
p
4
= ∫ {a[cos 2 u + 1] − a}d u
1/ 2

p

4
p

4
= ∫ [a(2 cos u)1/ 2 − a]d u
2

p

4

M03_Eng-Maths (Aditya) CH03.indd 18 7/24/2018 2:00:59 PM


Multiple Integrals and Beta Gamma Functions  n  3.19

p
4
= 2 ∫ a( 2 cos u − 1)d u  [{ 2 cos u − 1 is even function ]
0

p/4
= 2a  2 sin u − u
0

 p p   1 p   p
= 2a   2 sin −  − ( 2 sin 0 − 0) = 2a  2 ⋅ −  − 0  = 2a 1 − 
  4 4     2 4    4

3.1.5  Change of Variables in Double Integral


The evaluation of a double integral, sometimes become simpler if the variables of integration are
transformed suitably into new variables.
For example, from cartesian coordinates to polar coordinates or to some variables u and v.
1.  Change of variables from x, y to the variables u and v.
Let
∫∫
f ( x, y ) dxdy be the given double integral.
R

∂( x, y )
Suppose x = g(u, v), y = h(u, v) be the transformations. Then dxdy = J dudv , where J = is
the Jacobian of the transformation. ∂(u, v )

\
∫∫ f ( x, y) dxdy = ∫∫ F (u, v)
R R
J dudv

2.  Change of variable from Cartesian to polar coordinates

Let ∫∫ f ( x, y ) dxdy be the double integral.


R

Let x = rcosu, y = rsinu be the transformation from Cartesian to polar coordinates.


Then dxdy = J drd u
∂( x, y )
where J = is the Jacobian of transformation.
∂( r, u)
∂x ∂x
∂r ∂ u cos u − r sin u
and J= = = r cos 2 u + r sin 2 u = r (cos 2 u + sin 2 u) = r
∂y ∂y sin u r cos u
∂r ∂u
\ dxdy = rdrd u  and \
∫∫ f ( x, y) dxdy = ∫∫ F (r, u) r drdu
R R

WORKED EXAMPLES
Example 1
∞∞ ∞

Evaluate ∫ ∫ e 2( x dxdy by changing to polar coordinates and hence evaluate ∫ e


2
1y 2 ) 2x 2
dx .
0 0 0

M03_Eng-Maths (Aditya) CH03.indd 19 7/24/2018 2:01:02 PM


3.20  n  Engineering Mathematics-III

Solution.
∞∞
y
Let I = ∫∫e − ( x2 + y2 )
dxdy
0 0
P
Since x varies from 0 to ∞ and y varies from 0 to ∞, it
is clear that the region of integration is the first quadrant r
as in Fig. 3.23 θ
To change to polar coordinates, put x = rcosu, y = rsinu
O x
\   dxdy = rdrdu
and   x + y2 = r2cos2u + r2sin2u = r2(cos2 u + sin2 u) = r2
2
Fig. 3.23
p
\  r varies from 0 to ∞ and u varies from 0 to
p
2
2 ∞

∫∫e
− r2
\ I= rdrd u
0 0

dt
Put r2 = t  ⇒  2rdr = dt   ⇒  rdr =
2
When r = 0, t = 0 and when r = ∞, t = ∞
p p p

1 ∞ 2
 1 2  e−t  12
\  I = ∫  ∫ e − t dt d u = ∫   d u = − ∫ (e −∞ − e )d u
0

0 
2 0  2 0
 −1  0 2 0
p p
2 2
1 1 1 p2 p
2 ∫0 2 ∫0
=− ( 0 − 1) d u = d u = [u]0 =
2 4
∞∞
p
∫∫e
− ( x2 + y2 )
\ dxdy =
0 0
4

To find ∫ e 2x dx
2

2
∞∞ ∞ ∞
p  − x2 

 ∞ ∞

=  ∫ e dx   ∫0 ∫0 e dy 
− x2 − y2
∫ ∫e dxdy = ∫ e dx ⋅ ∫ e dy   ⇒  =
−( x 2 + y 2 ) −x2 −y 2
Now, { e dx
0 0 0 0
4 0  

p p
∫e
− x2
\ dx = =
0
4 2
Example 2
2 2 x 2x 2
x
Evaluate ∫ ∫
0 0 x 1y 2
2
dydx by changing into polar coordinates.

Solution.
2 2x − x 2
x
Let I=∫ ∫ dydx
0 0 x2 + y 2
The limits for y are y = 0 and y = 2x − x 2

M03_Eng-Maths (Aditya) CH03.indd 20 7/24/2018 2:01:06 PM


Multiple Integrals and Beta Gamma Functions  n  3.21

Now, y = 2x − x 2   ⇒  y 2 = 2x − x 2   ⇒  x 2 + y 2 − 2x = 0   ⇒  ( x − 1) + y = 1,
2 2

y
x 2 + y 2 − 2x = 0
which is a circle with centre (1, 0) and radius r = 1
or r = 2cosθ
and x varies from 0 to 2.
\ the region of integration is the upper semi-circle P x=2
as in Fig. 3.24
To change to polar coordinates, r
put x = rcosu, y = rsinu θ
\ dx dy = r dr du O (1, 0) (2, 0) x
\ x2 + y2 − 2x = 0 Fig. 3.24

⇒ r 2 cos 2 u + r 2 sin 2 u − 2r cos u = 0

⇒ r 2 − 2r cos u = 0   ⇒  r ( r − 2 cos u) = 0   ⇒  r = 0, 2 cos u


p
Limits of r are r = 0 and r = 2cosu and limits of u are u = 0 and u =
p p
2
2 2 cos u 2 2 cos u
r cos u
∴ I= ∫ ∫
0 0
r
rdrd u = ∫ ∫
0 0
r cos u drd u

p
2
 2 cos u 
= ∫ cos u  ∫ rdr  d u
0  0 
p
2 cos u
2
 r2 
= ∫ cos u   du
0
 2 0
p p

12 2
3 −1 4
= ∫ cos u 4 cos 2 ud u = 2 ∫ cos3 d u = 2 ⋅ ⋅1 =
20 0
3 3
Example 3
2 a 2 ax 2x 2
By changing into polar coordinates, evaluate the integral ∫ ∫
0 0
( x 2 1 y 2 )dydx .

Solution.
2 a 2 ax − x 2

Let  I = ∫ ∫
0 0
( x 2 + y 2 ) dydx y
x 2 + y 2 − 2ax = 0
or r = 2acosθ
The limits for y are y = 0 and y = 2ax − x 2
x = 2a
Now,  y = 2ax − x 2     ⇒  y 2 = 2ax − x 2 P
r
⇒ x 2 + y 2 − 2ax = 0    ⇒  ( x − a) + y = a
2 2 2
θ
which is a circle with centre (a, 0) and radius r = a. O (2a, 0)x
∴ x varies from 0 to 2a
Fig. 3.25

M03_Eng-Maths (Aditya) CH03.indd 21 7/24/2018 2:01:10 PM


3.22  n  Engineering Mathematics-III

\ the region of integration is the upper semi circle as in Fig. 3.25.


To change to polar coordinates, put x = rcos u and y = rsin u.
\ dxdy = rdrdu and x 2 + y 2 = r 2 cos 2 u + r 2 sin 2 u = r 2

and x 2 + y 2 − 2ax = 0 ⇒ r 2 − 2ar cos u = 0   ⇒  r ( r − 2a cos u) = 0   ⇒  r = 0, r = 2a cos u


p
\ r varies from 0 to 2a cos u  and  u varies from 0 to
2
p p
2 2 a cos u 2
 2 a cos u 
\ I= ∫ ∫
0 0
r 2 ⋅ r drd u = ∫  ∫ r 3 dr  d u
0  0 
p
2 a cos u
2
 r4 
= ∫  du
0
 4 0
p p
2
cos 4 u 16 a 4 2 4 3 1 p 3a 4 p
= ∫ ( 2a) 4
4 ∫0
du = cos 4
u d u = 4 a ⋅ ⋅ ⋅ =
0
4 4 2 2 4

Example 4
4a y
x 2 2y 2
Evaluate ∫∫
0 y2 x 2 1y 2
dxdy by changing to polar coordinates.
4a

Solution.
4a y
x2 − y 2
Let I= ∫∫
0 y2 x2 + y 2
dxdy
y y=x

4a y 2 = 4ax
P
y2 r
Given, the limits for x are x = and x = y θ
4a x
⇒ y 2 = 4 ax and y = x

And the limits for y are y = 0 and y = 4a


To find the point of intersection of y 2 = 4ax and y = x , Fig. 3.26
solve the two equations.
Now y2 = 4ax  ⇒  y2 = 4ay  ⇒  y(y - 4a) = 0  ⇒  y = 0,   y = 4a
\ x = 0,   x = 4a
\ the points are (0, 0), (4a, 4a)
\ the region of integration is the shaded region as in Fig. 3.26 which is bounded by y2 = 4ax and y = x.
To change to polar coordinates, put x = rcosu, y = rsinu
\ dxdy = rdrdu and x2 + y2 = r2
\ x2 - y2 = r2cos2u - r2sin2u = r2(cos2u - sin2u) = r2cos2u
and y2 = 4ax becomes  r2sin2u = 4a ⋅ rcosu  ⇒  r(rsin2u - 4acosu) = 0

M03_Eng-Maths (Aditya) CH03.indd 22 7/24/2018 2:01:14 PM


Multiple Integrals and Beta Gamma Functions  n  3.23

4a cos u
⇒  r = 0  and  rsin2u - 4acosu = 0  ⇒  r =
sin 2 u
4a cos u p p p
\ limits for r are 0, 2
and u varies from to . [{ slope of the line is tan u = 1 ⇒ u = ]
sin u 4 2 4
p 4 a cos u p
 4 a cos u

2 sin 2 u sin 2 u
\ r 2 cos 2 u 2
 
I= ∫ ∫
p 0 r2
rdrd u = ∫ cos 2u  ∫ rdr  d u
p  0 
4 4
p 4 a cos u
2
 r 2  sin2 u
= ∫ cos 2u   du
p  2 0
4

12 16 a 2 × cos 2 u
= ∫ cos 2 u du
2p sin 4 u
4
p

16a2 2 cos 2 u
=
2 p∫ (cos 2 u − sin 2 u) 4 d u
sin u
4
p

 cos 2 u  sin 2 u cos 2 u


2

= 8a2 ∫  2 − 1 du

p sin u
 sin 4 u
4
p
2
cos 2 u
= 8a2 ∫ (cot 2 u − 1) du
p sin 2 u
4

p
2
= 8a2 ∫ (cosec 2 u − 1 − 1) cot 2 ud u
p
4

p
2
= 8a ∫ (cosec u − 2) cot
2 2 2
u du
p
4

p
2
= 8a2 ∫ (cosec 2u cot 2 u − 2 cot 2 u) d u
p
4

 p2 p
2

 
= 8a  ∫ cosec u cot ud u − 2 ∫ cot ud u
2 2 2 2

 p4 p

4

M03_Eng-Maths (Aditya) CH03.indd 23 7/24/2018 2:01:16 PM


3.24  n  Engineering Mathematics-III

 p2 p
2

2  
= 8a  ∫ cot u cosec ud u − 2 ∫ (cosec u − 1) d u
2 2 2

 p4 p
4

 p2 
2  
p

= 8a  ∫ − cot 2 u ( −cosec 2 u)d u − 2[ − cot u − u]p2 


 p4 4 

 1 p
 p p  p p  
= 8a 2  − [cot 3 u]p2 + 2 cot + −  cot +   
 4 4  
 3 4  2 2
 1 p p  p p 
= 8a 2  −  cot 3 − cot 3  + 2  0 + − 1 −  
 3  2 4   2 4
 1  p p  1 p  8a 2 4a 2
= 8a 2  − ( −1) − 2 + 2  −   = 8a 2  − 2 +  = (3p − 10) = (3p − 10)
 3  2 4 3 2 6 3

Example 5
a a 2 2y 2

Evaluate ∫ ∫ ( x 2 1 y 2 ) dxdy by changing into polar coordinates.


0 0

Solution.
a a2 − y 2

Let I=∫ ∫ ( x 2 + y 2 ) dxdy


0 0

Limits for x are x = 0 and x = a2 − y 2

Now x = a2 − y 2   ⇒  x 2 = a 2 − y 2  ⇒  x 2 + y 2 = a2

which is circle with centre (0, 0) and radius a


Limits for y are y = 0 and y = a y=a (0, a)
\ the region of integration is as in Fig. 3.27
x 2 + y 2 = a2
bounded by y = 0, y = a and x = 0, x = a2 − y 2 r P or r = a
θ
To change to polar coordinates, O (a, 0) x
put x = rcosu, y = rsinu

\ dxdy = rdrdu and x2 + y2 = r2
\ x2 + y2 = a2  ⇒  r2 = a2  ⇒  r = ± a Fig. 3.27
\ in the given region, r varies from 0 to a and
p
u varies from 0 to
2
p p p p
a
2 a
a 3  2 4
 r4  4
a4 a4 p
pa 4
\ I = ∫ ∫ r ⋅ rdrd u = ∫  ∫ r dr  d u = ∫   d u = ∫0 4 = =
2
d u [u ] 2
.
 4 0
0
0 0 0 0  0
4 8

M03_Eng-Maths (Aditya) CH03.indd 24 7/24/2018 2:01:19 PM


Multiple Integrals and Beta Gamma Functions  n  3.25

Example 6

Evaluate ∫∫ ydxdy, where R is the region bounded by the semi-circle x2 1 y2 5 2ax and the x-axis
R
and the lines y 5 0 and y 5 a.

Solution.
Let I = ∫∫ ydxdy
R

The region R is as in Fig. 3.28


y
We have x2 + y2 = 2ax
(2, 2)
y= a
⇒ x 2 − 2ax + y 2 = 0
x 2 + y 2 = 2ax
r or r = 2acosθ
⇒ ( x − a) 2 + y 2 = a2 θ P
O x
(2a, 0)
which is a circle with centre (a, 0) and
radius a
To change to polar coordinates,
put x = rcosu, y = rsinu
Fig. 3.28
\  dxdy = rdrd u and x 2 + y 2 = r 2

Now  x 2 + y 2 = 2ax   ⇒  r 2 = 2ar cos u

⇒ r 2 − 2ar cos u = 0   ⇒  r ( r − 2a cos u) = 0   ⇒  r = 0, r = 2a cos u

p
\ r varies from 0 to 2a cos u and u varies from 0 to
2
p p
2 2 a cos u 2
r sin u ⋅ rdrd u = ∫ sin u  r 2 ∫ dr  d u
2 a cos u
\ I= ∫ ∫
0 0 0
 0 
p
2 a cos u

2
 r3 
= ∫ sin u   du
0 3
p
2
1
3 ∫0
= sin u ( 2a)3 cos3 ud u

8a3 2
3 ∫0
= cos3 u sin ud u

p
8a3  − cos 4 u  2 2a 3  4 p  2a 3 2a 3
=   =−  cos − cos 0  = − (0 − 1) =
3  4 0 3  2  3 3

M03_Eng-Maths (Aditya) CH03.indd 25 7/24/2018 2:01:22 PM


3.26  n  Engineering Mathematics-III

EXERCISE 3.3
Polar Coordinates
p p
2 a 2 2 2 a cos u
1. Evaluate ∫∫
0 0
rdrd u.  2.  Evaluate ∫ ∫ rdrd u.
p 0
4

3. Find the area of a loop of the curve r = a sin3u. 4.  Find the area of a loop of the curve r = a cos3u.
5. Find the area common to the circles r = a 2 and r = 2acosu.
p
2 ∞
rdrd u
6. Find the area of the cardioid r = a(1 - cosu).    7.  Evaluate ∫ ∫ (r
0 0
2
+ a2 )2
.

8. Evaluate ∫∫ r 3 drd u, where A is the area between the circles r = 2 sin u and r = 4 sin u.
A

p
2 a cos u p a (1+ cos u )

9. Evaluate ∫ ∫ r a − r drd u.
2 2
10.  Evaluate ∫ ∫ r 2 cos u dr d u.
0 0 0 0

Change of Variables
a a
xdxdy
11. Change into polar coordinates and evaluate ∫ ∫ .
0 y ( x 2
+ y 2 )3 / 2

∫∫ (x + y 2 )7 / 2dxdy by changing into polar coordinates where R is the region bounded


2
12. Evaluate
R

by the circle x2 + y2 = 1.
2 a 2 ax − x 2
13. Change into polar coordinates and evaluate ∫ ∫
0 0
( x 2 + y 2 )dxdy .
xydxdy by changing into polar coordinates, where R is the region in the positive
14. Evaluate
∫∫
R x2 + y 2
quadrant.
dxdy
15. Evaluate
∫∫
R x + y 2 + a2
2
by changing into polar coordinates, where R is the I quadrant.

x 2y 2
16. Evaluate
∫∫R x 2 + y 2 dxdy by changing into polar coordinates, where R is the annular region
between the circles x2 + y2 = 4 and x2 + y2 = 16.
2p 4

[Hint  I = ∫ ∫r
2
cos 2 u sin 2 udrd u ]
0 2

17. Evaluate
∫∫ a 2 − x 2 − y 2 dxdy where R is the semi-circle x + y = ax in the I quadrant, changing
2 2

R
to polar coordinates.

M03_Eng-Maths (Aditya) CH03.indd 26 7/24/2018 2:01:25 PM


Multiple Integrals and Beta Gamma Functions  n  3.27

a a
x 2dxdy by changing to polar coordinates.
18. Evaluate
∫∫
0 y x2 +y2

n
19. Evaluate
∫ ∫ xy( x + y ) 2 dxdy over the positive quadrant of x + y = 4, supposing n + 3 > 0.
2 2 2 2

2 4 − x2

20. Transforming to polar coordinates evaluate the integral ∫ ∫


0 0
( x 2 y + y 3 ) dxdy .

ANSWERS TO EXERCISE 3.3

pa2 a2 pa2 pa2


1. 2.  (p − 2) 3.  4.  a2 (p - 1)
5. 
2 4 12 12

3pa2 p 45p a3 5pa3


6. 7.  2
8.  9.  (3p − 4) 10. 
2 4a 2 18 8

pa 2p 3pa4 a3 p
11. 12.  13.  14.  15. 
4 9 4 6 4a2

a3 a3 2n + 3 32
16. 15 p 17.  (3p − 4) 18. log e ( 2 + 1) 19.  20. 
18 3 n+4 5

3.1.6  Area as Double Integral


(a) Area as double integral in cartesian coordinates
Double integrals are used to compute area of bounded plane regions. The area A of a plane
bounded region R in cartesian coordinates is
A5 ∫∫ dxdy.
R

(i) If the region R is bounded by curves y = f1(x), y = f2(x) and lines x = a, x = b where a and b are
constants, then

b f2 (x)

A 5 ∫  ∫ dy dx
a 
 f1 (x) 
(ii) If the region R is bounded by curves x = g1(y), x = g2(y) and line y = c, y = d where c and d are
constants, then

d g 2 (y)

A 5 ∫  ∫ dx  dy
c 
 g1 (y) 

M03_Eng-Maths (Aditya) CH03.indd 27 7/24/2018 2:01:29 PM


3.28  n  Engineering Mathematics-III

WORKED EXAMPLES
Example 1
x2 y 2
Find the area bounded by the ellipse 1 51, using double integration.
a2 b 2
Solution. y
x2 y 2
Equation of the ellipse is 2 + 2 = 1
a b
x2
y = b 1−
By the symmetry of the curve, the area of the (0, b) a2
ellipse is A = 4 × Area in the first quadrant
a b 1− x 2 / a 2

= 4∫ ∫ dydx (a, 0) x
0 0
a

= 4∫ [ y ]0b 1− x 2 / a 2
dx
0
Fig. 3.29
a a
x2 4b
= 4∫ b 1 −
a ∫0
dx = a 2 − x 2 dx
0 a2
a
4b  x a 2 − x 2 a 2 x 4b  a2 −1  p
=  + sin −1  = 0 + sin 1 = 2ab ⋅ = pab
a  2 2 a 0 a  2  2

Example 2
Using double integral find the area enclosed by the curves y 5 2x2 and y2 5 4x.

Solution.
The region of integration is the shaded region y = 2x 2
y
(as in Fig. 3.30) bounded by y2 = 4x and y = 2x2 y 2 = 4x
To find A, solve the equations A
Q (1, 1)
  y2 = 4x and y = 2x2
x=1
⇒  y2 = 4x4 P
O x
⇒ 4x = 4x ⇒  x(x - 1) = 0  ⇒  x = 0, 1
4  3

\ A is (1, 1)

Required area = ∫∫ dxdy Fig. 3.30


R
Take a strip PQ parallel to y axis with P lies on
y = 2x2, Q lies on y2 = 4x ⇒ y = 2 x
\  the limits of y are y = 2x2 to y = 2 x and the limits of x are x = 0 to x = 1
1 2 x 1
1 1
 x 3/ 2 x3  4 2 2
\  area = ∫0 ∫2 dydx = ∫ [y ]22x2x dx = ∫ [2 x − 2x 2 ]dx = 2 3/ 2 − 2 3  0 = 3 − 3 = 3
2x 0 0

M03_Eng-Maths (Aditya) CH03.indd 28 7/24/2018 2:01:32 PM


Multiple Integrals and Beta Gamma Functions  n  3.29

Example 3
Find the smaller of the areas bounded by y 5 2 2 x and x2 1 y2 5 4 using double integral.

Solution.
The region R is the shaded part in Fig. 3.31.

Required area A = ∫∫ dxdy


y
R

To find limits for y, take a strip PQ parallel to the B(0, 2)


Q
y-axis, P lies on y = 2 – x and Q lies on the circle x 2 + y2 = 4
x2 + y2 = 4
P
A(2, 0)
\ y limits are y = 2 – x to y = 4 − x 2 and the
O x
x limits are x = 0 to x = 2 y=2−x

2 4−x 2

\  A = 
∫0  ∫
2−x
dy dx


2 Fig. 3.31
= 0∫ [ y ]2 −4x− x dx
2

= ∫  4 − x 2 − ( 2 − x )  dx
0
2
x 4 x x2  4 p
= 4 − x 2 + sin −1 − 2 x +  = 0 + 2(siin −1 1 − sin −1 0) − 2 ⋅ 2 + = 2 ⋅ − 4 + 2 = p − 2
2 2 2 2 0 2 2

Example 4
Find the area bounded by the parabola y2 5 4 2 x and y2 5 4 2 4x as a double integral and
evaluate it.
Solution.
Given y2 = 4 - x = - (x - 4) is a parabola with vertex (4, 0) and towards
the negative x-axis, axis of symmetry the x-axis.
and y2 = 4 - 4x = - 4 (x - 1) is a parabola with vertex (1, 0) and towards
the negative x-axis, axis of symmetry the x-axis.
To find the points of intersections, solve y2 = 4 - x and y2 = 4 - 4x,
\ 4 - x = 4 - 4x ⇒ 3x = 0 ⇒   x = 0
and y2 = 4 - x     ⇒  y2 = 4 ⇒   y = ±4 and the points are (0, 2), (0, -2)
Draw the graph and determine the region. The region is the shaded region as in Fig. 3.32.
Both curves are symmetric about x-axis.
\ required area A = 2 Area above the x-axis = 2∫∫ dxdy
R

M03_Eng-Maths (Aditya) CH03.indd 29 7/24/2018 2:01:34 PM


3.30  n  Engineering Mathematics-III

It is convenient to take strip PQ parallel to the y


y 2 = 4 − 4x
x-axis. P lies on y2 = 4 - 4x
(0, 2) y 2= 4 − x
and Q lies on   y = 4 - x.
2

P Q
y2 B
Now y2 = 4 - 4x ⇒  x = 1 −
4 A (1, 0) x
(4, 0)
and y = 4 - x  ⇒ x = 4 - y2
2

and the limits of y are y = 0, y = 2 (0, −2)

2 4−y 
2
2
Fig. 3.32
∴ area A = 2∫  ∫ dx  dy = 2∫ [ x ]4 −yy2 dy
2

0  y2  1 −
0 4
1− 4 
2
  y2  
= 2∫  4 − y 2 − 1 −   dy
 4 
0 
2
2
 3   3 y3   8
= 2∫  3 − y 2  dy = 2 3 y − ⋅  = 2 3 × 2 −  = 2[6 − 2] = 8
 4   4 3 0  4
0

Example 5
Using double integration find the area of the parallelogram whose vertices are A(1, 0), B(3, 1),
C(2, 2), D(0,1)

Solution.
The given points A(1, 0), B(3, 1), C(2, 2) and D(0,1) are the vertices of a parallelogram ABCD.
Required area is the area of the parallelogram
ABCD as in Fig 3.33. y
Area of the parallelogram ABCD
= 2 (area of the triangle ABD) C(2, 2)
We shall find the equations of AB and AD.
We know the equation of the line joining the D B
(0, 1) (3, 1)
the points (x1, y1) and (x2, y2) is Q
P
y − y1 x − x1 O A(1, 0) x
=
y 1 − y 2 x1 − x 2
Fig. 3.33
\ equation of AB, the line joining (1, 0) and (3, 1)
y − 0 x −1 1
=   ⇒  y = ( x − 1)(1)
0 −1 1− 3 2
Equation of AD, the line joining (1, 0), (0, 1) is
y − 0 x −1
= ⇒ y = − x + 1 (2)
0 −1 1− 0

M03_Eng-Maths (Aditya) CH03.indd 30 7/24/2018 2:01:36 PM


Multiple Integrals and Beta Gamma Functions  n  3.31

Area of D ABD =
ABD
∫∫ dx dy
Take a strip PQ parallel to the x-axis with P is on (2) and Q is on (1).
\ x = -y + 1 and x = 2y + 1 and y varies from 0 to 1.

1 2 y +1
 1

area of ∆ABD = ∫  ∫ dx  dy = ∫ [ x ]− y +1 dy
2 y +1
\ [ Note that BD is parallel to the x-axis]
0 
 − y +1  0
1
1 1
 y2  3
= ∫ [2 y + 1 − ( − y + 1)]dy = ∫ 3 y dy = 3   =
0 0
 2  0 2
3
\ area of the parallelogram ABCD is = 2 × = 3.
2

EXERCISE 3.4
1. Find the area bounded by the parabola x2 = 4y and the straight line x – 2y + 4 = 0.
2. Evaluate the area bounded by y = x and y = x2.
3. Evaluate the area bounded by y2 = 4ax and x2 = 4ay.
4. Evaluate the area bounded by y = 4x – x2 and y = x.
x2 y 2 x y
5. Evaluate the smaller area bounded by + = 1 and the line + = 1.
9 4 3 2
6. Evaluate the smaller area bounded by x + y = 4 and x + y = 2.
2 2

7. Evaluate the area bounded by y2 = 4x, x + y = 3 and the X-axis.


x2
8. Evaluate the area bound by y = , y = ax , x = 0 and x = 4a.
4a
9. Find the area common to y2 = x and x2 + y2 = 4.
10. Find the area bounded by y2 = 4 - x, y2 = x.
11. Find the area of the curve a2y2 = x2(2a - x).
12. Find the area of a circle of radius a by double integration.
13. Find the area between the parabola y = 4x – x2 and the line y = x by double integration.

ANSWERS TO EXERCISE 3.4


1 16a2 9 3
1. 9 2.  3.  4.  5.  ( p − 2)
6 3 2 2
10 16a2 p 16 2
6. p - 2 7.  3 3 + 4 10. 
9. 
8. 
3 3 3 3
9
11.  4a 12.  pa2 13. 
2

(b) Area as double integral in polar coordinates


As double integral, area in polar coordinates is ∫∫ rdrd u
R
where R is the region for which the area is required.

M03_Eng-Maths (Aditya) CH03.indd 31 7/24/2018 2:01:39 PM


3.32  n  Engineering Mathematics-III

WORKED EXAMPLES
Example 1
Find the area bounded between r 5 2cosu and r 5 4cosu.

Solution.

Area A = ∫∫ rdrdu
R
where the region R is the region between the circles
r = 2cosu and r = 4cosu r = 2cosθ
Y
The area is the shaded region as in Fig. 3.34. π r = 4cosθ
θ=
We first integrate w.r.to r and so, we take the radius 2
Q
vector OPQ. When PQ is moved to cover the area A,
r varies from r = 2cosu to r = 4cosu,
p p O θ P
and u varies from u = − to u = X
2 2
p p
4 cos u
2 4 cos u 2
 r2 
\ Area A =
∫p 2 cos∫ u rdrdu = ∫p  2  2 cos u du θ= −
π
− − 2
2 2

p
Fig. 3.34
2
1
= ∫ (4 cos 2 u − 22 cos 2 u) d u
2

2 p

2
p
2 p/2
1 p
= 6 ∫ cos 2 u d u = 6 × 2 ∫ cos 2 u du
2
u = 2 × 6 ⋅ ⋅ = 3p [{ cos u is even ]
p 0
2 2

2


Example 2
Find the area of one loop of the leminiscate r2 5 a2cos2u.

Solution.
Given r2 = a2cos2u
Area of the loop = ∫∫ rdrdu , where R is the region as in Fig. 3.35.
R
Since the loop is symmetric about the initial line, required area is twice the area above the
initial line.
First we integrate w.r.to r
In this region, take a radial strip OP, its ends are

r = 0 and r = a cos 2u

M03_Eng-Maths (Aditya) CH03.indd 32 7/24/2018 2:01:42 PM


Multiple Integrals and Beta Gamma Functions  n  3.33

When the strip is moved to cover the region R, π


p θ=
u varies from 0 to (above ox)
4
p
4 a cos 2 u
P
Required Area A = 2 ∫ ∫ rdrd u
0 0 θ
O x
p
a cos 2 u
 r  
4 2
= 2 ∫    du
 2  0
0 
p π
4 θ=−
= ∫ a 2 cos 2u d u
0 Fig. 3.35
p p
4 2 2
 sin 2 u  4 a  p  a
= a ∫ cos 2u d u = a 
2
= 2
 sin − sin 0  =
0
 2  0 2  2  2

Example 3
Find the area of a loop of the curve r 5 a sin3u.

Solution
Given r = asin3u
π
The area of the loop = ∫∫ rdrd u θ=
3
R

But the loop is formed by two consecutive values of u


when r = 0.
X
When r = 0, asin3u = 0 O
p
⇒ 3u = 0 or p ⇒ u = 0 or
3 Fig. 3.36
and r varies from r = 0 to r = asin3u
p p
3 a sin 3 u a sin 3u
3
 r2 
\ area of the loop = ∫ ∫
0 0
rdrd u = ∫  
0
 2 0
du

p
3
1 2 2
2 ∫0
= a sin 3 ud u

a 2 3 1 − cos 6 u
2 ∫0
= du
2
p
a2  sin 6u  3 a 2  p sin 2p − sin 0  pa 2
=  u − = −  = 12
4 6  0 4  3 6

M03_Eng-Maths (Aditya) CH03.indd 33 7/24/2018 2:01:45 PM


3.34  n  Engineering Mathematics-III

Example 4
Find the area which is inside the circle r 5 3acosu and outside the cardioid r 5 a(1 1 cosu).

Solution.
Given r = 3acosu   (1) and r = a(1 + cosu)(2)
r = 3acosθ
Required area A = ∫ ∫ rdrdu
Eliminating r from (1) and (2), we get P′

3a cos u = a(1 + cos u) θ


P
O x
⇒ 2 cos u = 1   ⇒  cos u = 1
2
p p
\ u=− or r = a(1 + cosθ)
3 3
Required area is the shaded region as in Fig. 3.37. Fig. 3.37
Since both the curves are symmetrical about the initial line, required area is twice the area above
the initial line.
In this region take a radial strip OPP′ where P lies on (2) and P′ lies on (1).
When it moves, it will cover the required area.
p
\ r varies from a(1 + cosu) to 3acosu and u varies from 0 to .
3
p p
3 r = 3 a cos u 3a cos u
 r2 
3
equired area = 2 ∫
R ∫ rdrd u = 2 ∫0  2 a(1+ cos u)d u
0 r = a (1+ cos u )
p
3
= ∫ [9a2 cos 2 u − a2 (1 + cos u) 2 ] d u
0

p
p
3

∫∫ [9 cos
=a 23
[9 cos 2
u − (1 + 2 cos u + cos 2 u)] d u
=a 2
0
2
u − (1 + 2 cos u + cos 2 u)] d u
0
p
p
3

= a 2 ∫∫0 [8 cos 2 u − 1 − 2 cos u] d u


= a 2 3 [8 cos 2 u − 1 − 2 cos u] d u
0

{{ }}
p

 1 + cos 2 u 
p
3

∫∫ 8
=a 23
8 1 + cos 2 u − 1 − 2 cos u d u
=a 2
2 − 1 − 2 cos u d u
0
0
2  p
  sin 2 u   p3
= a22  4  u + sin 2 u  − u − 2 sin u 3
= a 4  u + 2  − u − 2 sin u 0
  2  0

M03_Eng-Maths (Aditya) CH03.indd 34 7/24/2018 2:01:47 PM


Multiple Integrals and Beta Gamma Functions  n  3.35

  2p  
  p sin 3  p p 
= a 4  +
2
 − − 2 sin − 0 
 3 2  3 3 
 4p 3 p 3  4p p 
= a2  +2 − − 2  = a2  −  = pa 2
 3 2 3 2   3 3

Example 5
Find the area common to r5a 2 and r = 2acosu.

Solution.

Given r = a 2    (1) and r = 2acosu (2)


y
(1) is a circle with centre (0, 0) and radius a 2
(2) is a circle with centre (a, 0) and radius a r =a 2

Solve (1) and (2) to find the point of intersection. Q B r = 2a cosθ


C
\ a 2 = 2a cos u P
π/4 x
1 p O A
⇒ cos u = ⇒ u=
2 4

Since the circles are symmetrical about the initial


line OX,
required area = 2 [area OABC] = 2 [area OAB
+ area OBC]
Fig. 3.38
In OAB, take a strip OP.
When OP moves it covers the area OAB. Ends of OP are, r = 0 and r = a 2
p
\ r varies from 0 to a 2 and u varies from 0 to
4
In the area OBC, take a strip OQ. Ends of OQ are, r = 0 and r = 2acosu
When OQ moves it covers the area OBC.
p p
\ r varies from 0 to 2acosu and u varies from to
4 2
 p4 a 2 p
2 2 a cos u

 
\ Required area = 2  ∫ ∫ rdrd u + ∫ ∫ rdrd u
 0 0 p
4
0 
p p
a 2 2 a cos u
 r2 
4 2
 r2 
= 2∫   d u + 2∫   du
0
 2 0 p  2 0
4
p p
4 2
= ∫ 2a2d u + ∫ 4a2 cos 2 ud u
0 p
4
p

 1 + cos 2u 
p 2
= 2a2 [u] + 4a2 ∫ 
4
 d u
M03_Eng-Maths (Aditya) CH03.indd 35 0  7/24/2018 2:01:50 PM
p p
a 2 2 a cos u
3.36  n  Engineering Mathematics-III
4
 r2  2
 r2 
=2 ∫  2 
0 0
d u + 2∫  
p  2 0
du
4
p p
4 2
= ∫ 2a2d u + ∫ 4a2 cos 2 ud u
0 p
4
p

 1 + cos 2u 
p 2
= 2a2 [u] + 4a2 ∫ 
4
 d u
0 2
p
4

p
p  sin 2 u  2
= 2a ⋅ + 2a 2 u +
2

4  2  p
4

pa 2
p p 1  p 
= + 2a 2  − +  sin p − sin  
2  2 4 2  2
pa 2  p 1  pa
2
pa 2
= + 2a 2  −  = + − a 2 = a 2 (p − 1)
2  4 2 2 2
Example 6
Find the area inside the circle r 5 asinu but lying outside the cardiod r 5 a(1 2 cosu).

Solution.
Given r = a sinu   (1) and  r = a(1 -
cosu)(2) Y π
r = a(1 − cosθ) θ=
2
Area = ∫ ∫ rdrdu
Q
Eliminating r from (1) and (2), we get P
r = a sinθ
X
asinu = a(1 - cosu) O

⇒ sinu + cosu = 1

Squaring, sin2u + cos2u + 2sinu cosu = 1 Fig. 3.39


⇒ 1 + 2sin2u = 1

p
⇒  sin 2u = 0  ⇒  2u = 0, p ⇒ u = 0 or
2
p p
a sin u a sin u
2
 r2  2
\ Area = ∫ ∫ rdrd u = ∫   du
0 a (1− cos u ) 0
 2  a (1− cos u )
p

12
= ∫ [a 2 sin 2 u − a 2 (1 − cos u) 2 ] d u
20
p

a2 2
2 ∫0
= [sin 2 u − (1 − 2 cos u + cos 2 u)] d u

a2 2
2 ∫0
= {−1 + 2 cos u − (cos 2 u − sin 2 u)} d u
M03_Eng-Maths (Aditya) CH03.indd 36 7/24/2018 2:01:52 PM
2
 r2 
= ∫  du
0
 2  a (1− cos u )
p

12
= ∫ [a 2 sin 2 u − a 2 (1 − cos u) 2 ] d u
20
p
2 2 Multiple Integrals and Beta Gamma Functions  n  3.37
a
= ∫ [sin u − (1 − 2 cos u + cos u)] du
2 2

2 0
p
2 2
a
= ∫ {−1 + 2 cos u − (cos u − sin 2 u)} d u
2

2 0

p p

a2  2 2

2  ∫0
= [ −1 + 2 cos u}d u − ∫ {cos u − sin u] d u
2 2

0 
p p p
2 2 2 2
a
= ∫ ( −1 + 2 cos u) du  [since ∫ cos 2 d u = ∫ sin 2 ud u]
2 0 0 0

2 2 2 2
a a  pp
p a  p  a
= [ −u + 2 sin u] = − + 2 sin  =
2
− + 2 = [4 − p]
2  2 2  2  2
0
2  4

EXERCISE 3.5
1. Find the area bounded between r = 2sinu and r = 4sinu.
2. Find the area of one loop of r = acos3u.
3. Find the area that lies inside the cardioid r = a(1 + cosu) and outside the circle r = a.
4. Find the area of the cardioid
(i) r = a(1 + cosu), (ii) r = 4(1 + cosu)
5. Find by double integration, the area lying inside the cardioid r = 1 + cosu and out the parabola
r(1 + cosu) = 1.
6. Calculate the area included between the curve r = a(secu + cosu) and its asymptote.
7. Find the area of the cardioid r = a(1 - cosu).

ANSWERS TO EXERCISE 3.5


pa2 a2 3pa2
1. 3p 2.  3.  (p + 8) 4. (i)   (ii) 24p
12 4 2
9p + 16 5pa2 3pa2
5. 6.  7. 
12 4 2

3.2 Area of a Curved Surface


Introduction
Let D ⊂ R, say D = [a, b]. If f : D → R is a function, then the graph of the function f is the set of points
{(x, y) : y = f(x) ∀ x ∈ D} which is a subset of R2.
This subset of R2 is called a curve in R2 whose equation is

y = f(x) ∀ x [a, b]

In implicit form the equation of the curve is F(x, y) = 0


For example y = x2 is the equation of the parabola in explicit form, where as x2 - y = 0 is the
implicit form of the equation of the parabola.
Let D ⊂ R2

M03_Eng-Maths (Aditya) CH03.indd 37 7/24/2018 2:01:55 PM


3.38  n  Engineering Mathematics-III

If f : D → R is a function, then the graph of the function f is the set of points

{(x, y, z) : z = f(x, y) ∀ (x, y) ∈ D}


which is a subset of R3.
This subset of R3 is called a surface in R3, whose equation is

z = f(x, y) ∀ (x, y) ∈ D

This explicit form is called Monge’s form of the equation of the surface.
The general form of the surface is the implicit form

F(x, y, z) = 0

Sphere, cone, cylinder are surfaces in R3.


The equation  x2 + y2 + z2 = a2 is a sphere in R3 or 3-dimensional space.
The equation   x2 + y2 = a2 is a cylinder in R3 or 3-dimensional space.
The equation  x2 + y2 = 4z2 is a cone in R3 or 3-dimensional space.

Smooth surface
Definition 3.1  A surface S is said to be smooth if at each point unique normal exists and it varies
continuously as the point moves on S.
Piece-wise Smooth surface
Definition 3.2  A surface S is said to be piece-wise smooth if it can be divided into a finite number of
smooth surfaces.
For example: the surface of a cube is a piece-wise smooth surface.

3.2.1  Surface Area of a Curved Surface


In earlier classes you have seen the area of surface of revolution. That is a surface obtained by
revolving an arc of a curve about an axis.
For example the surface of a sphere is obtained by revolving the semi-circle about its bounding diameter.
This surface area is expressed as an integral of a function of a single independent variable.
b 2
b
ds  dy 
We know that surface area = ∫ 2py dx = 2p∫ y 1 +   dx
a
dx a
dx
where y = f(x).
But the general problem of finding the area of a curved surface S is found as a double integral over
the orthogonal projection D of S on one of the coordinate planes.
This is possible if any line perpendicular to the chosen coordinate plane meets the surface S in not
more than one point.

3.2.2  Derivation of the Formula for Surface Area


Let S be a surface of finite area represented by the equation F(x, y, z) = 0.
Let D be the orthogonal projection of S on the xy-plane as in Fig. 3.40

M03_Eng-Maths (Aditya) CH03.indd 38 7/24/2018 2:01:56 PM


Multiple Integrals and Beta Gamma Functions  n  3.39

S P(x, y, z)
dS

dx dxdy
dy

x
D

Fig. 3.40

Divide the region D into element rectangular areas by drawing lines parallel to x-axis and y-axis.
Let dS be the element area of the surface whose projection is shaded, which is a rectangle of sides dx, dy
\  element area = dxdy
Let P(x, y, z) be any point on dS and n be the outward unit normal at P.
∇F ∂F ∂F ∂F
Then n = , where ∇F = i +j +k
∇F ∂x ∂y ∂z = i Fx + j Fy + k Fz

\ ∇F = Fx 2 + Fy 2 + Fz 2

Let g be the angle between the plane of dS and the plane of dxdy.
We know that the angle between two planes is the angle between their normals.
The normal to the plane of dS is n and the normal to the plane of dxdy is k.

n⋅k
\ cos g = = n ⋅ k , Since n and k are unit vectors.
n⋅k

We always take the acute angle, which is given by cos g = n ⋅ k


Since dxdy is the projection of dS, we have

dxdy = cos g × dS  ⇒  dS = dxdy


cos g
dxdy dxdy (1)
S = ∫∫
cos g ∫∫
Integrating, =
D D n⋅k

M03_Eng-Maths (Aditya) CH03.indd 39 7/24/2018 2:01:59 PM


3.40  n  Engineering Mathematics-III

Similarly, projecting on yz plane, we get


dydz (2)
S = ∫∫
D1 n ⋅ i

where, D1 is the orthogonal projection of S on the yz plane.


Projecting on the zx-plane, we get

dzdx (3)
S = ∫∫
D2 n⋅ j
Cartesian form of surface area
∇F i Fx + j Fy + k Fz
Since n=
∇F =
Fx 2 + Fy 2 + Fz 2

Fz   \  n ⋅ k = Fz
n⋅k =
Fx + Fy + Fz
2 2 2
Fx + Fy 2 + Fz 2
2

\ dxdy Fx 2 + Fy 2 + Fz 2
S = ∫∫ = ∫∫ dx dy (4)
D n⋅k D
Fz
Similarly,
dydz = Fx 2 + Fy 2 + Fz 2
S = ∫∫ ∫∫D Fx
dy dz (5)
D1 n ⋅i 1

dzdxS = Fx 2 + Fy 2 + Fz 2
and S = ∫∫ ∫∫ F
dz dx (6)
D2 n ⋅ j D 2 y

Corollary  If the equation of the surface is given explicitly or rewritten as z = f(x, y).
Then f(x, y) - z = 0
Here F(x, y, z) = f(x, y) - z
∂F ∂f ∂z ∂F ∂f ∂z ∂F
\ = = ,  = =  and  = −1
∂x ∂x ∂x ∂y ∂y ∂y ∂z
2 2
 ∂z   ∂z 
\ (4) ⇒ S = ∫∫   +   + 1 dxdy
 ∂x   ∂y 
D

If the equation of the surface is given by x = f(y, z), then as above


2 2
(5) ⇒  ∂x   ∂x 
S = ∫∫   +   + 1 dydz
D1
 ∂y   ∂z 

If the equation of the surface is given by y = f2(x, z), then


2 2
(6) ⇒  ∂y   ∂y 
S = ∫∫   +   + 1 dz dx
 ∂x   ∂z 
D2

M03_Eng-Maths (Aditya) CH03.indd 40 7/24/2018 2:02:04 PM


Multiple Integrals and Beta Gamma Functions  n  3.41

3.2.3  Parametric representation of a surface


The parametric equations of a surface F(x, y, z) = 0 are written interms of two parameters as
x = x(u, v), y = y(u, v), z = (u, v) where (u, v) ∈ D ⊂ R2 in the u, v-plane. u and v are the parameters.
For example: The parametric equation of the equation of the sphere x2 + y2 + z2 = a2 in spherical
polar coordinates are x = a sin u cos f, y = a sin u sin f, and z = a cos u; where u and f are the
parameters.

Worked examples
Example 1
Find the surface area of the sphere of radius a.

Solution.
Let x2 + y2 + z2 = a2(1)
be the equation of the sphere.
Since the sphere is symmetric about all the coordinate axes y
the surface area S = 8 × Surface area of the sphere in the
positive octant.
The projection of the surface in the first octant is a
­quadrant of the circle x2 + y2 = a2 as in Fig 3.42.
dxdy
\  surface area S = 8∫∫ O y
D n⋅k
∇F
But n=
∇F
x
where F = x2 + y2 + z2 - a2

\ Fx = 2x,  Fy = 2y,  Fz = 2z Fig. 3.41

\ ∇F = 2 x i + 2 y j + 2 z k
y
∇F = 4 x + 4 y + 4 z = 4a = 2a
2 2 2 2

2x i + 2 y j + 2z k = x i + y j + z k
\ n=
2a a a a = = −

x y z 
\ n ⋅ k =  i + j + k  ⋅ k = z = 1 a2 − x 2 − y 2
a a a  a
a
[ using (1)] + =

a a2 − y 2
dxdy dxdy
\ S = 8∫∫ = 8a∫ ∫
1 2 a − x2 − y 2
2
D a − x2 − y 2 0 0
a Fig. 3.42

M03_Eng-Maths (Aditya) CH03.indd 41 7/24/2018 2:02:07 PM


3.42  n  Engineering Mathematics-III

a  a2 − y 2 
dx
= 8a∫  ∫  dy

0  0 ( a 2 − y 2 ) − x 2 
a a2 − y 2
 x 
= 8a∫ sin −1 dy
2 
0  a − y 0
2

a  a2 − y 2  

= 8a∫ sin −1   − sin −1 0 dy
 a −y 
2 2
0
 
a a
p
= 8a∫ (sin −1 1 − 0) dy = 8a∫ dy = 4ap[ y ]a0 = 4pa[a − 0] = 4pa2
2
0 0

Aliter:  Since the sphere is symmetric in all the 8 octants. Consider the sphere in the I octant and
project it on the xy-plane. We get the quadrant of the circle x2 + y2 = a2.
2 2
 ∂z   ∂z 
\  Surface area S = 8∫∫   +   + 1 dxdy
 ∂x   ∂y 
D

where D is the region of the circle in the first quadrant as in Fig.3.42


The equation of the sphere is
x2 + y2 + z2 = a2
Treating z as a function of x and y and differentiating partially w. r. to x and y respectively, we get
∂z ∂z x
2x + 2z =0 ⇒ =−
∂x ∂x z
and ∂z ∂z y
2 y + 2z =0 ⇒ =−
∂y ∂y z
2 2

\  ∂z   ∂z  x2 y 2 x 2 + y 2 + z 2 a2
+
    + 1 = + + 1 = = 2
∂x  ∂y  z2 z2 z2 z
2 2

\  ∂z   ∂z  a
  +   + 1 =
∂x  ∂y  z
a a2 − y 2
a 1
\  Surface area S = 8∫∫ dxdy = 8a∫ ∫ dxdy = 4pa2  [as above]
D
z 0 0 a −x −y
2 2

Example 2
Find the surface area of the cone x 2 1 y 2 5 4 z 2 lying above the xy-plane and inside the cylinder
x 2 1 y 2 5 3ay .

Solution.
The equation of the cone is x 2 + y 2 = 4z 2  (1)

M03_Eng-Maths (Aditya) CH03.indd 42 7/24/2018 2:02:10 PM


Multiple Integrals and Beta Gamma Functions  n  3.43

z
The equation of the cylinder is x 2 + y 2 = 3ay  (2)

The surface area of (1) lying inside (2) is required.


Project this surface on the xy-plane.
x 2 + y 2 = 4z 2
2 2
∂z  ∂z 
\  surface area S = ∫∫   +   + 1 dxdy
D
 ∂x   ∂y  y

Differentiating (1) partially w. r. to x, y treating z as a x + y = 3ay


2 2

function of x and y, we get


∂z ∂z x
8z = 2x ⇒ = x
∂x ∂x 4 z Fig. 3.43
and ∂z ∂z y
8z = 2y ⇒ =
∂y ∂y 4 z
2 2
∂z  ∂z  x2 y2 x 2 + y 2 + 16 z 2 20 z 2 5
\   +   + 1 = + +1 = = =
 ∂x   ∂y  16 z 2
16 z 2
16 z 2 16 z 2 4
2 2
∂z  ∂z 
\   +   + 1 =
5
 ∂z   ∂y  2

\ 5
Surface area S =
∫∫D 2 dxdy
5 5 5 2
2 ∫∫
= dxdy = (area of the circle x 2 + y 2 = 3ay ) = pr
D 2 2
where r is the radius of the circle.
2

⇒ x 2 + y 2 − 3ay = 0   \  radius r =   =
3a 3a
x 2 + y 2 = 3ay
 2 2
2
5  3a  9 5 2
\ S= p  = pa
2  2 8
Example 3
Find the area cut from the sphere x 2 1 y 2 1 z 2 5a 2 by the cylinder x 2 1 y 2 5ax .

Solution.
We interchange the x, y axis for convenience as in Fig 3.44.
The equation of the sphere is
x 2 + y 2 + z 2 = a2 (1)

The equation of the cylinder is


x 2 + y 2 = ax (2)

M03_Eng-Maths (Aditya) CH03.indd 43 7/24/2018 2:02:15 PM


3.44  n  Engineering Mathematics-III

Both the surfaces are symmetric about the axes. z


\  surfaces above and below the xy-plane are the
same. Since the cylinder lies on the side of the
positive x-axis, the required surface area
= 2 (surface area above the xy-plane)
x 2 + y 2 + z 2 = a2
Project the surface of the sphere cut off by the
cylinder onto the xy plane. O x
This is the circle x 2 + y 2 = ax .
x 2 + y 2 = ax
2 2
∂z  ∂z 
\ S = 2∫∫   +   + 1 dxdy y
D
 ∂x   ∂y 
Fig. 3.44
Differentiating (1) partially w. r. to x, y, treating z
as a function of x and y, we get
∂z ∂z x
2x + 2z =0 ⇒ =−
∂x ∂x z

and ∂z ∂z y
2 y + 2z =0 ⇒ =−
∂y ∂y z
2 2

\  ∂z   ∂z  x2 y 2 x 2 + y 2 + z 2 a2
  +   + 1 = 2 + 2 + 1 = = 2
∂x  ∂y  z z z2 z
2 2
\  ∂z   ∂z  a a
  +   + 1 = =
∂x  ∂y  z a − x2 − y 2
2

\ a
S = 2∫∫ dxdy
D a − x2 − y 2
2

where D is the circle in the xy plane.


The equation of this circle is
x 2 + y 2 = ax   ⇒  y 2 = ax − x 2   ⇒  y = ± ax − x 2
a ax − x 2
1
\ S = 2a∫ ∫ dydx
0 − ax − x 2 (a − x 2 ) − y 2
2

a ax − x 2
1
= 4a∫ ∫ dydx  { (a2 − x 2 ) − y 2 is even function of y 
0 0 (a − x ) − y
2 2 
2 

ax − x 2
a
 y 
= 4 a∫ sin −1   dx
2 
0 
  a − x   0
2

a  ax − x 2  a
 x( a − x )  a
x
= 4 a∫ sin −1   dx = 4 a ∫ sin −1
  dx = 4 a ∫ sin −1 dx
0  a −x 
2 2
0  a −x 
2 2
0
a+ x

M03_Eng-Maths (Aditya) CH03.indd 44 7/24/2018 2:02:18 PM


Multiple Integrals and Beta Gamma Functions  n  3.45

Put x   ⇒  x   ⇒  x
t = sin −1 sint = sin 2 t =
a+x a+x a+x
\ x a+ x− x a
cos 2 t = 1 − sin 2 t = 1 − = =
a+ x a+ x a+ x
a
⇒ a+x = 2
= a sec 2 t   ⇒  x = a sec t − a = a(sec t − 1) = a tan t
2 2 2

cos t
\ dx = 2a tan t sec 2 t dt
1 p
When x = 0, t = 0  and when x = a, t = sin −1 =
2 4
p p
a 4 4
x
∫ sin dx = ∫ t ⋅ 2a tan t sec 2 t dt = 2a ∫ t (tan t ⋅ sec 2 t ) dt
−1
\
0
a+x 0 0

We integrate using integration by parts.


So, take
u=t and dv = tan t sec 2 t dt

tan 2 t
\ du = dt and ∫ dv = ∫ tan t sec t dt ⇒v=
2

2
 p p

a
x 
 t ⋅ tan t
2
 4 4
tan 2 t 
∫ sin  − ∫ 1⋅
−1
\ dx = 2a   dt 
0
a+x  2 0 0 2 
 
 p
4

 p 2 p  
= a  ⋅ tan − 0 − ∫ (sec t − 1) dt 
2

 4  0
4 
 
p p

= a  − [tan t − t ]04 
4 
p 
 4 
p p 
= a  −  tan −   = a
4 4  
p
4
−1+
p
4{ p  a
2  2 }
= a  − 1 = (p − 2)

a
\ S = 4a ⋅ (p − 2) = 2a2 (p − 2)
2
Note  This problem can also be stated as below.
Find the surface area of the portion of the sphere x2 + y2 + z2 = a2 lying inside the cylinder x2 + y2 = ax.
Example 4
Find the surface area of the part of the plane x 1 y 1 z 5 2a which lies in the first octant and is
bounded by the cylinder x2 1 y2 5 a2.

Solution.
The required surface area is the part of plane
x + y + z = 2a

M03_Eng-Maths (Aditya) CH03.indd 45 7/24/2018 2:02:23 PM


3.46  n  Engineering Mathematics-III

bounded by x = 0, y = 0, z = 0 z
and the cylinder x + y = a 2 2 2

The projection of the surface on the xy plane


is the quadrant of the circle
(0, 0, 2a)
x2 + y2 = a2.
2 2

\  ∂z   ∂z 
S = ∫∫   +   + 1 dxdy
 ∂x   ∂y 
D

The surface is z = 2a - x - y O
(0, 2a, 0) y
\      ∂z = −1, ∂z
= −1
∂x ∂y x2 + y2 = a2

2 2 (2a, 0, 0)
∂z ∂z
\    +   + 1 = 1 + 1 + 1 = 3
 ∂x   ∂y 

∂z ∂z
2 2 x
\   +   + 1 = 3
 ∂x   ∂y  Fig. 3.45
1
∫∫ 3 dxdy = 3 ∫∫ dxdy = 3 × 4 area of the circle x + y = a
2 2 2
\S =
D D

3 2 [{ radius of the circle = a]


= pa 
4

Example 5
Find the surface area of the cylinder x 2 1 y 2 5 a 2 cut out by the cylinder x 2 1 z 2 5 a 2 .

Solution.
Given two right circular cylinders
x 2 + y 2 = a2(1)
with z-axis as the axis of the cylinder
and x 2 + z 2 = a2 (2)
with y-axis as the axis of the cylinder.
Both the cylinders are symmetric about the three axis.
\  the surface area are the same in all octants.
Projecting the surface x 2 + y 2 = a2 on the xz-plane, we get the required surface area S.
2 2
 ∂y   ∂y 
\ S = 8∫∫   +   + 1 dzdx
D1
 ∂x   ∂z 

where D1 is the circle x 2 + z 2 = a2 in the xz-plane.


The surface is  x 2 + y 2 = a2

M03_Eng-Maths (Aditya) CH03.indd 46 7/24/2018 2:02:26 PM


Multiple Integrals and Beta Gamma Functions  n  3.47

Differentiating, partially w. r. to x z
and z, treating y as a function of x and z,
we get

∂y ∂y x
2x + 2 y =0 ⇒ =−
∂x ∂x y
x2 + z2 = a2
∂y ∂y
and 2y =0 ⇒ =0
∂z ∂z
2 2
∂y ∂y x2
\    +   + 1 = 2 + 1 O
 ∂x   ∂z  y y

x 2 + y 2 a2
= = 2
y2 y x2 + y2 = a2
2 2
\  ∂y  +  ∂y  + 1 = a = a
x
 ∂x   ∂z  y a − x2
2

2 2 Fig. 3.46
\ Surface area  ∂y   ∂y 
S = 8∫∫   +   + 1 dxdz
D1
 ∂x   ∂z 
a a2 − x 2
a
= 8∫ ∫ dzdx
0 0 a − x2
2

a  a2 − x 2 
1
= 8a∫  ∫ dz  dx

0 
 0 a2 − x 2 
a
1
= 8a∫
2
−x 2
[z ]0 a dx
0 a −x
2 2

a a
1  a2 − x 2 − 0  dx = 8a dx = 8a[x ]a = 8a2
= 8a∫ ∫ 0
0 a2 − x 2 0

Example 6
Find the surface area of the cylinder x 2 1 z 2 5 4 lying inside the cylinder x 2 1 y 2 5 4 .

Solution.
In the above example 5, putting a = 2, we get the surface area.
\  surface area S = 8 ⋅ 22 = 32.

Example 7
Find the surface area cut off from the cylinder x2 1 y2 5 ax by the sphere x2 1 y2 1 z2 5 a2.

Solution
The equation of the sphere is x 2 + y 2 + z 2 = a2 (1)

M03_Eng-Maths (Aditya) CH03.indd 47 7/24/2018 2:02:29 PM


3.48  n  Engineering Mathematics-III

z
The equation of the cylinder is
    x 2 + y 2 = ax (2)
The surface area of the cylinder cut off
by the sphere is required.
Projecting the surface on the xz-plane,
we get the required surface area S.
2 2
x 2 + y 2 + z 2 = a2
 ∂y   ∂y 
\ S = 2∫∫   +   + 1 dx dz x
D1
∂x ∂z O

where D1 is the region obtained by


eliminating y2 from (1) and (2) x 2 + y 2 = ax
\      z + ax = a (3)
2 2
y
The surface is x 2 + y 2 = ax
Differentiating partially w.r.to x and z, Fig. 3.47
treating y as function of x and z, we get
∂y ∂y a − 2 x
2x + 2 y =a ⇒ =
∂x ∂x 2y
∂y ∂y
and 2y =0 ⇒ =0
∂z ∂z
2 2
∂y ∂y (a − 2 x ) 2
\    +   + 1 = +1
 ∂x   ∂z  4y 2

(a − 2 x ) 2 + 4 y 2 a2 − 4ax + 4 x 2 + 4(ax − x 2 ) a2
= = =  [ using ( 2)]
4y 2 4y 2 4y 2
2 2
\  ∂y  +  ∂y  + 1 = a = a  [ using ( 2)]
 ∂x   ∂z  2y 2 ax − x 2
[x and y axes are interchanged for convenience of the figure]
We have
z 2 + ax = a 2 ⇒ z 2 = a 2 − ax ⇒ z = ± a 2 − ax
a a 2 − ax
a 1
\ S = 2∫∫ dx dz = a∫ ∫ dz dx
D1 2 ax − x 2
0 − a 2 − ax ax − x 2
a
 1 
= a∫ 
2
[ z ] a −2 ax  dx
− a − ax
0  ax − x
2

a
 1  a 2 − ax + a 2 − ax   dx
= a∫ 
0  ax − x
2  
  
a
a 2 − ax
= 2a ∫ dx
0 ax − x 2
a
a( a − x )
= 2a ∫ dx
M03_Eng-Maths (Aditya) CH03.indd 48
x( a − x ) 7/24/2018 2:02:34 PM
a
 1 
= a∫ 
2
[ z ] a −2 ax  dx
− a − ax
0  ax − x
2

a Multiple Integrals and Beta Gamma Functions  n  3.49
 1  a 2 − ax + a 2 − ax   dx
= a∫ 
0  ax − x
2  

a
a 2 − ax
= 2a ∫ dx
0 ax − x 2
a
a( a − x )
= 2a ∫ dx
0
x( a − x )
a
a a 1  12  1
a x
dx = 2a a   = 4a a (a 2 − 0) = 4a2

= 2a∫ dx = 2a a ∫ x 2
x  1 
0 0
 
 2 0

Exercise 3.6

1. Find the surface area of that part of the plane x + y + z = a intercepted by the coordinate planes.

x y z
2. Find the surface area of that part of the plane + + = 1 intercepted by coordinate planes.
a b c

3. Find the surface area of the portion of the cylinder x 2 + y 2 = 4 y lying inside the sphere
x 2 + y 2 + z 2 = 16.

4. Find the surface area of the portion of the cone x 2 + y 2 = 3z 2 lying above the xy-plane inside the
cylinder x 2 + y 2 = 4 y .

5. Find the area of the surface of the sphere x 2 + y 2 + z 2 = 9a2 cut off by the cylinder x 2 + y 2 = 3ax .

ANSWERS TO EXERCISE 3.6


3 2 1 2 2 8p
1. a 2.  b c + c 2 a 2 + a 2b 2 3. 64 4.  9a2 (p − 2)
5. 
2 2 3

3.3  TRIPLE INTEGRAL IN CARTESIAN COORDINATES


Let f(x, y, z) be a continuous function at every point in a closed and bounded region D in space.
Subdivide the region into a number of element volumes by drawing planes parallel to the
coordinate planes. Let DV1, DV2, …, DVn be the number of element volumes formed. Let (xi, yi, zi) be
n
any point in DVi, where DVi = Dxi Dyi Dzi. Form the sum ∑ f ( x i , y i , z i ) ∆Vi . The limit of the sum as
i =1
n → ∞ and DVi. → 0, if it exists, is called the triple integral of f(x, y, z) over D and is denoted by

∫ ∫ ∫ f ( x , y , z ) dV
D
or ∫ ∫ ∫ f ( x , y , z ) dxdydz (1)
D

As in the case of double integrals, the triple integral is evaluated by three successive integration of
single variable.

M03_Eng-Maths (Aditya) CH03.indd 49 7/24/2018 2:02:37 PM


3.50  n  Engineering Mathematics-III

Consider the triple integral


z 1 y1 x1

∫ ∫ ∫ f ( x , y , z ) dxdydz
z0 y0 x0

(1) If all the limits are constants, then the integration can be performed in any order with proper
limits,
z1 y1 x1 x1 y1 z1 x1 z1 y1

i.e.,  ∫ ∫ ∫ f (x , y , z ) dxdydz = ∫ ∫ ∫ f (x , y , z ) dzdydx = ∫ ∫ ∫ f (x , y , z ) dydzdx


z0 y0 x0 x0 y0 z0 x0 z0 y0

(2) If x0 = f0(y, z),  x1 = f1(y, z),  y0 = g0(z),  y1 = g1(z),  z0 = a,  z1 = b,


z1 y1 x1 b y1 = g1 ( z ) x1 = f1 ( y, z )

then ∫∫
z0 y0 x0
∫ f (x , y , z ) dxdydz = ∫ ∫ ∫ f ( x , y , z ) dx dy dz
a y 0 = g0 ( z ) x 0 = f 0 ( y, z )

First we integrate w.r.to x, treating y and z as constants and substitute limits of x. Next integrate
the resulting function of y and z w.r.to y, treating z as constant and substitute the limits of y.
Finally we integrate the resulting function of z w.r.to z and substitute the limits of z.

WORKED EXAMPLES
Example 1
1 2 2

∫∫∫x
2
Evaluate yz dxdydz .
0 0 1

Solution.
1 2 2
Let I = ∫ ∫ ∫ x 2 yz dxdydz
0 0 1

1 2 2 1 2 2
 2  2  3
⇒  I = ∫ zdz ∫ ydy ∫ x dx =  z   y   x  = 1 ⋅ 4  8 − 1  = 7 
2
[{ limits are constants]
0 0 1  2  0  2  0  3  1 2 2  3 3 3

Example 2
a b c

Evaluate ∫ ∫ ∫ ( x 2 1 y 2 1 z 2 ) dxdydz .
0 0 0

Solution.
a b c c
a b
x3 
Let I = ∫ ∫ ∫ ( x 2 + y 2 + z 2 ) dxdydz = ∫ ∫  + ( y 2 + z 2 )x  dydz
0 0 0 0 0  3 0

a b
 c3 
= ∫ ∫  + ( y 2 + z 2 ) c  dydz
0 0  3 
a b
 c2 
= c ∫ ∫  + y 2 + z 2  dydz
 3 
0 0

M03_Eng-Maths (Aditya) CH03.indd 50 7/24/2018 2:02:40 PM


Multiple Integrals and Beta Gamma Functions  n  3.51

b
c2 a
y3 
= c∫  y + + z 2 y  dz
0  0
3 3

a
 c 2b b 3 
= c∫  + + z 2b  dz
0  
3 3
a
 c2 b2 
= bc ∫  + + z 2  dz
0  
3 3
a
 c 2 b 2  z3   b 2 + c 2  a3   a2 + b2 + c2 
= bc  +  z +  = bc   a +  = abc  
 3 3  3 0  3  3  3 

Example 3
log 2 x x1y

Evaluate ∫∫∫0 0 0
e ( x1y1z ) dxdydz.

Solution.
log 2 x x + y log 2 log 2

∫∫∫ ∫ ∫
x+ y
Let I = e ( x + y + z ) dzdydx = e x + y .[e z ] dydz = e x + y .(e x + y − 1)dydz
0 0 0 0 0

 2 x  e 2 y  
log 2 x log 2 x
y x
∫ ∫ (e ∫
(2x+2 y)
= − e x + y )dydz = e .   − e .[e ]0  dz
x

0 0 0   2 0 
log 2 log 2
1 1
=
2 ∫
0
e 2 x (e 2 x − 1) − 2e x (e x − 1)  dz =
2 ∫
0
e 4 x − e 2 x − 2e 2 x + 2e x  dx

log 2
1
=
2 ∫
0
(e 4 x − 3e 2 x + 2e x )dx

loge 2
1  e4x e2x 
=  − 3 + 2e x 
2 4 2 0
1  e 4 loge 2 3 2 loge 2   1 3 
=  − e + 2e loge 2  −  − + 2 
2 4 2   4 2 
1  e loge 16 3 loge 4 3  1 16 3 ⋅ 4 3 3 3 5
=  − e + 2e loge 2 −  =  − + 2⋅ 2 −  = 2 − 3 + 2 − = 1− =
2 4 2 4 2  4 2 4 8 8 8
 [{ e loge x = x ]

Example 4
42 z 4 z2 x 2

Evaluate ∫∫ ∫
0 0 0
dydxdz.

M03_Eng-Maths (Aditya) CH03.indd 51 7/24/2018 2:02:44 PM


3.52  n  Engineering Mathematics-III

Solution.
42 z 4z −x 2 42 z
Let I=∫ ∫ ∫ dydxdz = ∫ ∫ [y ]
4z −x 2
dxdz [Treating x , z constants]
0 0
0

0 0 0
42 z

=∫ ∫ 4 z − x 2 dxdz
0 0

4 2 z
x 4z x 
= ∫ 4z − x 2 + sin −1  dz  [Treating z constant ]
0 
2 2 2 z 0
4

= ∫ [ z 4 z − 4 z + 2 z sin −1 1 − 0] dz
0
4
4
p
4
 z2  p
= ∫ 2z dz = p∫ z dz = p   = (16) = 8p
0
2 0
 2  0 2
Example 5
log 2 x x1 log 2

Evaluate ∫∫ ∫
0 0 0
e x1y1z dzdydx .

Solution.
log 2 x x + loge y log 2 x

Let I = ∫∫ ∫ e x + y + z dzdydx = ∫ ∫e ⋅ e y [e z ]0x + log y dydx


x

0 0 0 0 0

log 2 x

= ∫ ∫e ⋅ e y [e x + log y − e 0 ] dydx
x

0 0

log 2 x

= ∫ ∫e e (e x ⋅ e log y − 1)dydx
x y

0 0
log 2 x

= ∫ ∫e e (e x ⋅ y − 1) dydx  [{ e loge y = y ]
x y

0 0
log 2 x

= ∫ ∫ (e ⋅ ye y − e x ⋅ e y ) dydx
2x

0 0

log 2
 2x x y x

= ∫ e ∫ ye dy − e ∫ e dy  dx
x y

0  0 0 
log 2

= ∫ {e [ y ⋅ e y − 1⋅ e y ]0x − e x [e y ]0x }dx 


2x
[Using Bernoulli’s formula ]
0

log 2

= ∫ {e [ xe x − e x − (0 − 1)] − e x (e x − 1)}dx
2x

log 2

= ∫ {( x − 1)e + e 2 x − e 2 x + e x }dx
3x

0

M03_Eng-Maths (Aditya) CH03.indd 52 7/24/2018 2:02:48 PM


Multiple Integrals and Beta Gamma Functions  n  3.53

log 2

= ∫ {( x − 1)e + e x }dx
3x

loge 2
 e3x e3x 
= ( x − 1) − 1⋅ + ex 
 3 9 0
1 1  1 1 
=  (log e 2 − 1)e 3 loge 2 − e 3 loge 2 + e loge 2 −  − − + 1 
 3 9  3 9 
1 8 5 3
= (log e 2 − 1) ⋅ 8 − + 2 − [{ e 3 loge 2 = e loge 2 = 23 = 8 and e loge 2 = 2]
3 9 9
8 8 8 5 8 19 1
= log e 2 − − + 2 − = log e 2 − = ( 24 log 2 − 19)
3 3 9 9 3 9 9
Example 6
a a 22x2 a 2 2 x 2 2 y2
dzdydx
Evaluate
∫ ∫
0 0

0 a 2 x2 2 y2 2 z2
2
.

Solution.
a a 2 − x2 a2 − x 2 − y 2 a a2 − x 2 a2 − x 2 − y 2
dzdydx  −1 z 
Let I=∫ ∫ ∫ =∫ ∫ sin dydx
2 
0 0 0 (a2 − x 2 − y 2 ) − z 2 0 0  a − x − y 0
2 2

a2 − x 2 − y 2 a2 − x 2 a2 − x 2 − y 2
dzdydx
a
 −1 z   dx x
∫ =∫ ∫ sin 2 
dydx  { ∫ a −x
2
= sin −1 
a2
0 (a − x − y ) − z
2 2 2 2
0 0  a − x − y 0
2 2


a a − x2
2

=∫ ∫ [sin −1 1 − sin −1 0] dydx


0 0

a a2 − x 2
p
=∫ ∫ dydx
0 0
2
a
p

2 2
= [ y ]0 a − x dx
20
a
p
2 ∫0
= a 2 − x 2 dx

a
p x 2 a2 x p a2  p a2 p p2 a2
=  a − x 2 + sin −1  = 0 + sin −1 1 − 0  = ⋅ ⋅ =
2 2 2 a 0 2  2  2 2 2 8

Example 6(a)

1 12 x 2 12 x 2 2 y2
dxdydz
Evaluate
∫ ∫
0 0
∫0 12 x 2 2 y 2 2 z 2
.

M03_Eng-Maths (Aditya) CH03.indd 53 7/24/2018 2:02:51 PM


3.54  n  Engineering Mathematics-III

Solution.
1 1− x 2 1− x 2 − y 2
In example 6, putting a = 1, we get dxdydz p 2 ⋅1 p 2
∫ ∫
0 0

0 1− x 2 − y 2 − z 2
=
8
=
8
Example 7
Evaluate ∫ ∫ ∫ xyz dxdydz
V
over the volume V enclosed by the three coordinate planes and the

x y z
plane 1 1 51.
a b c
Solution.
x y z
Let V be the volume enclosed by the plane + + = 1 and it meets the coordinate axes in
a b c
A(a, 0, 0), B(0, b, 0), C(0, 0, c) respectively.
z
The projection of V on the xy-plane is the DOAB
x y (0, 0, c)
bounded by x = 0, y = 0, + = 1 C  x y
a b z = C 1− − 
 a b

 x y
z varies from 0 to z = c 1 − −  (a, b, 0)
 a b O y
B
y varies from 0 to b 1− 
x
 a A x
+
y
= 1
and x varies from 0 to a. x
a b

Fig. 3.48
 x  x y
b  1−  c  1− − 
a  a  a b

I=∫ ∫ ∫ xyz dzdydx


0 0 0
 x  x y
b  1−  c  1− − 
a  a
 z2   a b
=∫ ∫ xy   dydx
0 0
 2 0
 x
b  1− 
a  a 2
1  x y
2 ∫0 ∫
= xyc 2 1 − −  dydx
 a b
0
 x
b  1− 
c 2 a  a
 x  y 
2

=
2 ∫x
0

0
y 1 −  −  dydx
 a  b 
 x
b  1− 
  x  y  3  x  y  
4  a

 y  1 −  −  1 −  −  
c2
a
 a b 
= ∫x  − 1⋅ 
a b 
dx  [Using Bernouli’s formula ]
2 0  1 −3  −1  
 − ⋅3 ⋅  ⋅ 4 
 b b  b  0

M03_Eng-Maths (Aditya) CH03.indd 54 7/24/2018 2:02:54 PM


Multiple Integrals and Beta Gamma Functions  n  3.55

c2
a b  x  b2   x 
4

2 ∫0  3  a 
= x   1 −  ( 0 ) − 0 −  0 −  1 −   dx
12   a   

a 4
c2 ⋅ b2  x
24 ∫0  a 
= x 1 −  dx

a
  x5  x 
6

2 2   1−   1−  
b c   a  a 2 2
 =bc
 a2  b2c2 a2 a2b2c2
= x⋅ − 1⋅  0 − ( 0 − 1) = ⋅ =
24  1 5  −6   24  30  24 30 720
 − ⋅5 − ⋅  
 a a  a  0

EXERCISE 3.7
Evaluate the following integrals
dxdydz
1. ∫ ∫ ∫ , where D is the region bounded by x + y + z = 1 and the coordinate planes.
D ( x + y + z + 1)3
1 1− x x + y 1 1− x 2 1− x 2 − y 2
dxdydz
2. ∫ ∫ ∫ ∫0 ∫0 ∫0 1 − x 2 − y 2 − z 2 .
xdzdydx. 3.
0 0 0

1 z x+z p a2 − r 2

4. ∫ ∫ ∫ ( x + y + z ) dydxdz. 5.
2 a cos u a

−1 0 x − z ∫ ∫ ∫ rdzdrd u. 0 0 0

∫ ∫ ∫ xyz dxdydz , where D is the region interior to the sphere x2 + y2 + z2 = a2 in the I octant.
6. 
D

∫ ∫ ∫ xyz dxdydz taken over the volume for which x, y, z ≥ 0 and x2 + y2 + z2 = 9.


7. 
 x  x y
b  1−  c  1− − 
a  a  a b
2 2
z = 5 x = 6 y = 36 − x 1 1− x ( x + y )

8. ∫ ∫ ∫ dydxdz . 9.  ∫ ∫ ∫
∫ ∫ ∫ xdzdydx. 10.  x 2 zdzdydx.
z = 0 x = −6 y = − 36 − x 2 0 0 0 0 0 0

c b a a x x+y 1 1− x 1− x − y

11. ∫ ∫ ∫ (x
2 2 2
∫∫
+ y + z ) dxdydz. 12. 
0 0

0
∫ ∫ ∫ xyz dxdydz.
e x + y + z dzdydx. 13. 
−c −b −a 0 0 0

2
1 1− x x + y 3 1 xy 1 1− x ( x + y )

14.  ∫∫ ∫
z
e dzdydx. 15.  ∫ ∫ ∫ x dzdydx.
∫ ∫ ∫ xy dzdydx. 16. 
0 0 0 1 1 0 0 0 0
x

1 1− x 2 1− x 2 − y 2

17.  ∫ ∫
0 0

0
xyz dxdydz.

M03_Eng-Maths (Aditya) CH03.indd 55 7/24/2018 2:03:00 PM


3.56  n  Engineering Mathematics-III

ANSWERS TO EXERCISE 3.7


1 5 1 p2 5pa3 a6 243
1. log e 2 − 2.  3.  4. 0 5.  6.  7. 
2 16 4 8 64 48 19

1 a3bc 2 8abc 2
10.  11.  ( a + b + c )
2 2
8. 180p 9. 
10 360 3

1 4a 1 1 2 2 
12. [e − 6e 2a + 8e a − 3] 13.  14.  15.   (9 3 − 1) − log e 3
8 720 2 5  5 

16. 1 17. 
1
10 48

3.3.1  Volume as triple integral


Triple integrals can be used to evaluate volume V of a finite bounded region D in space.
The volume      V = dxdydz . ∫∫∫ D
[Taking f(x, y, z) = 1 in (1) of 3.3, page 3.49, we get the volume]

WORKED EXAMPLES
Example 1
x y z
Find the volume of the tetrahedron bounded by the plane 1 1 51 and the coordinate
a b c
planes.

Solution.
The region of integration is the region bounded by

x y z z
+ + = 1, x = 0, y = 0, z = 0.
a b c
C  x y
Its projection in the xy-plane is the DOAB z = C 1− − 
 a b
x y
bounded by x = 0, y = 0 and + = 1
a b
O
∴  volume V = ∫ ∫ ∫ dxdydz
D
B y
A
 x  x y x y
   b  1−  c  1− − 
a  a  a b + = 1
x a b
=∫ ∫ ∫ dzdydx
0 0 0 Fig. 3.49
 x
b  1− 
a  a  x y
c  1− − 
=∫ ∫ [z ]0 a b
dydx
0 0
 x
b  1− 
a  a
 x y
=∫ ∫ c 1 − −  dydx
 a b
0 0

M03_Eng-Maths (Aditya) CH03.indd 56  x 7/24/2018 2:03:04 PM


a b 1− 
2  
 x  x y
b  1−  c  1− − 
a  a  a b

=∫ ∫ ∫ dzdydx
0 0 0
 x
b  1− 
a  a  x y Multiple Integrals and Beta Gamma Functions  n  3.57
c  1− − 
=∫ ∫ [z ]0 a b
dydx
0 0
 x
b  1− 
a  a
 x y
=∫ ∫ c 1 − −  dydx
 a b
0 0
 x
b  1− 
 x a
y2  a
= c ∫ 1 −  y −  dx
 a
0 
2b  0

a
 x   x  1  x 
2

= c ∫ 1 −  b 1 −  − b 2 1 −   dx
 a   a  2b  a  
0 
a
 x3 
1−
bc   a  
a 2
bc  x  − abc abc
= ∫
2 0


1 −
a


dx =
2  1 
=
6
[0 − 1] =
6
 − a ⋅ 3  0

Example 2
Find the volume bounded by the cylinder x2 1 y2 5 4 and the planes y 1 z 5 4, z 5 0.

Solution.
Required volume of the cylinder x2 + y2 = 4, cut off between the planes z = 0 and y + z = 4 is
V = ∫ ∫ ∫ dxdydz
D
\  z varies from z = 0 to z = 4 - y
The projection of the region in the xy plane is z

x + y2 = 4  ⇒  y = ± 4 − x 2
2

\ y varies from − 4 − x 2 to + 4 − x 2 and


x-varies from – 2 to 2 y + z= 4
2 4 − x2 4 − y

\ Volume V = ∫ ∫ ∫ dzdydx
−2 − 4 − x 2 0
O
x

−2 4 − x2 x2 + y2 = 4
= ∫ ∫
−2 − 4 − x 2
[z ]04 − y dydx x

Fig. 3.50
2 4 − x2

= ∫ ∫
−2 − 4 − x 2
( 4 − y ) dydx

4 − x2

2
y2
= ∫ 4y −  dx
−2
 2 − 4 − x2

{ }
2
1
= ∫ 4  4 − x 2 − ( − 4 − x 2 )  − [4 − x 2 − ( 4 − x 2 )] dx
−2
  2
2

= 8 ∫ 4 − x 2 dx
−2

M03_Eng-Maths (Aditya) CH03.indd 57 7/24/2018 2:03:09 PM


3.58  n  Engineering Mathematics-III

= 8 ⋅ 2∫ 4 − x 2 dx  [{ 4 − x 2 is even ]
0
2
 x 4 − x 2 4 −1 x  p
= 16  + sin  = 16[0 + 2 sin −1 1 − (0 + 0)] = 16 ⋅ 2 = 16p
 2 2 2 0 2

Example 3
dxdydz
Change to spherical polar coordinates and hence evaluate ∫∫∫ x
V
2
1 y2 1 z2
where V is the
volume of the sphere x2 1 y2 1 z2 5 a2.

Solution.
1
I = ∫∫∫ dxdydz
V x +y2 +z2
2

Using spherical polar coordinates (r, u, f), x = rsinucosf, y = rsinusinf, z = rcosu


Then the Jacobian of transformation is
∂( x , y , z )
J= = r 2 sin u 
∂( r , u, f)

\ dxdydz = J drd u df = r 2 sin u drd u df
x 2 + y 2 + z 2 = r 2 sin 2 cos 2 f + r sin 2 u sin 2 f + r 2 cos 2 u

= r 2 sin 2 u [cos 2 f + sin 2 f] + r 2 cos 2 u = r 2 [sin 2 u + cos 2 u] = r 2
p
2p 2 a
r 2 sin u drd u d f
\ I = 2∫ ∫∫
0 0 0 r2
p
2p 2 a

= 2∫ ∫ ∫ sin u drd udf


0 0 0
p
2p 2 a p
= 2 ∫ df∫ sin u∫ dr = 2[f]02p [ − cos u]02 [r ]a0 = 2 ⋅ 2p[ −0 + 1][a − 0] = 4pa
0 0 0

Example 4
Find the volume of the region of the sphere x2 1 y2 1 z2 5 a2 lying inside the cylinder x2 1 y2 5 ay.

Solution.
x2 + y2 = ay
2
 a a2
⇒ x 2 + y 2 − ay = 0   ⇒  x 2 +  y −  =
 2 4

M03_Eng-Maths (Aditya) CH03.indd 58 7/24/2018 2:03:12 PM


Multiple Integrals and Beta Gamma Functions  n  3.59

z
a
which is a circle with centre (0, a/2) radius r = in
the xy-plane, z = 0 2
So, the cylinder has this circle as guiding curve
and generators parallel to the z-axis.
x2 + y2 + z2 = a2 is a sphere with centre (0, 0, 0)
and radius = a. y
The volume inside the cylinder bounded by the
sphere is symmetric about the xy-plane. So, the
required volume = 2 (volume inside the cylinder) x
above the xy-plane.
Its projection in the xy-plane is the circle Fig. 3.51
x2 + y2 = ay.
z
The circle is symmetric about the y-axis.

\ volume V = 4∫ ∫ ∫ dxdydz
P(r, θ, z)
D

where D is the common region in the first octant. z


O
Changing to cylindrical polar coordinates (r, u, z), θ y
r
we have x = rcosu, y = rsinu, z = z
M
\ dx dy dz = r dr du dz and x + y = r 2 2 2 x

\ z varies from 0 to a2 − x 2 − y 2 = a2 − r 2
x2 + y2 = ay  ⇒  r2 = arsinu  ⇒  r = 0 and r = asinu

p
\ r varies from 0 to asinu and u varies from 0 to
2
p p
2
2 a sin u a − r
2 2 a sin u

volume V = 4 ∫ ∫ ∫ rdzdrd u = 4 ∫ ∫
2
− r2
\ r[z ]0 a drd u
0 0 0 0 0
p
2 a sin u
= 4∫ ∫ r a2 − r 2 drd u
0 0

p
a sin u
= −2∫ 2 ∫ a 2 − r 2 ( −2r ) drd u
0 0
a si n u
p  ( a 2 − r 2 )3 / 2 
= −2∫ 2  3  du
0  
 2 0
4 p2 2
3 ∫0
=− [( a − a 2 sin 2 u)3 / 2 − a3 ]d u

4 p 3

= − ∫ 2  a3 (1 − sin 2 u) 2 − a3 d u
3 0

4 a3 p
=−
3 ∫ 0
2
(cos3 u − 1)d u

M03_Eng-Maths (Aditya) CH03.indd 59 4 3


 p
2 3
p
2
 7/24/2018 2:03:16 PM
p
a sin u
= −2∫ 2 ∫ a 2 − r 2 ( −2r ) drd u
0 0
a si n u
p  ( a 2 − r 2 )3 / 2 
3.60  n   Mathematics-III 
= −2∫
Engineering
2
3 du
0  
 2 0
4 p2 2
3 ∫0
=− [( a − a 2 sin 2 u)3 / 2 − a3 ]d u

4 p 3

= − ∫ 2  a3 (1 − sin 2 u) 2 − a3 d u
3 0

4 a3 p
=−
3 ∫0
2
(cos3 u − 1)d u

4  p p

= − a3  ∫ 2 cos3 ud u − ∫ 2 d u
3  0 0

4a  2
3 p
 4 a  2 p  2a 3
3
=−  ⋅1 − [u]0  = −
2
− = [3p − 4]
3 3  3  3 2  9

Example 5
Find the volume of the cylinder x2 1 y2 5 4 bounded by the plane z 5 0 and the surface
z 5 x2 1 y2 1 2.

Solution.
The region is bounded by the cylinder x2 + y2 = 4 above the xy-plane
and the surface z = x2 + y2 + 2.
Changing to cylindrical polar coordinates, we get z = x2 + z2 + 2
x = r cos u, y = r sin u, z = z

\ dxdydz = rdrdudz

and x2 + y2 = r2

z = x2 + y2 + 2 = r2 + 2 O
r x
θ
\ z varies from 0 to r2 + 2
x2 + y2 = 4
r varies from 0 to 2 and u varies from 0 to 2p y

Fig. 3.52
\ volume V = ∫ ∫ ∫ dxdydz = ∫ ∫ ∫ rdrd udz
D D

2p 2 r2 + 2

= ∫ ∫ ∫
u=0 r =0 z =0
rdzdrd u

2p 2

∫ ∫ r[ z ]
r2 + 2
= 0 drd u
0 0
2
2p 2
 r4 r2  16 
= ∫ d u∫ ( r + 2r ) dr = [u]  + 2 ⋅  = 2p  + 4  = 16p
3 2p
0
0 0
 4 2  0  4 

M03_Eng-Maths (Aditya) CH03.indd 60 7/24/2018 2:03:18 PM


Multiple Integrals and Beta Gamma Functions  n  3.61

Example 6
x2 y2 z2
Find the volume of the ellipsoid 1 1 5 1.
a2 b2 c2
Solution.
x2 y 2 z 2
Since the ellipsoid + + = 1 is symmetric about the coordinate planes, the volume of the
a2 b 2 c 2
ellipsoid = 8 × volume in the first octant
Volume of ellipsoid in the first octant is bounded by the planes x = 0, y = 0, z = 0 and the ellipsoid
x2 y 2 z 2
+ + =1
a2 b 2 c 2 = − −

z2 x2 y 2
⇒ = 1 − −
c2 a2 b 2
 x2 y 2 
⇒ z 2 = c 2 1 − 2 − 2  =
 a b 

x2 y 2
⇒ z = ±c 1 − − . ≠ =
a2 b 2
x2 y 2 Fig. 3.53
In the first octant z varies from z = 0 to z = c 1 − − .
a2 b 2
The section of the ellipsoid by the xy plane z = 0 is the ellipse
x2 y2  x2  x2
+ =1 ⇒ y 2 = b 2 1 − 2  ⇒ y = ±b 1 −
a2 b2  a  a2
x2
\ y varies from 0 to b 1 − and x varies from 0 to a
a2
x 22 x2 y 2 x2
bb 11−− x2 c 1− 2 − 2 b 1−
aa aa 2 a bx 2 y 2 a a2 x2 y2
c 1− 2 − 2 c 1− −
\ Volume v == 88∫∫ ∫ 0 [ z] ∫
a b dz dy dx = 8∫
dy dx ∫ [ z ]0 a2 b2
dy dx
00 00 0 0 0
x2 x2
b 1− b 1−
a a2  x2 y2 
a a2  x2 y2 
= 8∫ ∫  a b 
∫0
= 8dx
c 1 − 2 − 2  dy ∫
0
c 1 − 2 − 2  dy dx
 a b 
0 0
x2 x2
b 1− b 1−
c  2  x2   c a2
 x2  
2 a
a a
= 8∫ ∫  b 1 − 2  −=y82 ∫ dy∫ dx  b 2 1 − 2  − y 2  dy dx
b   a   0 b 
0
 a  
0 0
x2 x2
b 1− b 1−
   x2   2 a2 x 2   a2
b 2 a 1 − 2   b 1−
a 
8c y 2  x  2
8c  y a 2 −1 x  y 2   a 2 
2
y 
= ∫ b 1 − 2  − y 2 =+ ∫  1 − 2  − y 2 + 
−1
b sin dx sin dx
b 0 2  a  b 0 22 a x  2 x2 
  b 1 −  b 1 − 
  a2  0 a2 0
4c 
a
2  x2  −1 4c
a
−1  2  x 2  
∫ ∫
−1 −1


= 0 + b 1 − 2  {sin =1 − sin 0 0+}b dx
1 − 2  {sin 1 − sin 0} dx
b 0 a b 0  a 

M03_Eng-Maths (Aditya) CH03.indd 61 7/24/2018 2:03:23 PM


3.62  n  Engineering Mathematics-III

4c 2  x 2  p
a

b ∫0  a 2  2
= b 1 −  ⋅ dx

a
 x2 
= 2p bc ∫ 1 − 2  dx
0
 a 
a
 1 x3   1  2  4
= 2p bc  x − 2 ⋅  = 2p bc  a − 2 ⋅ a3 − 0  = 2p bc  a  = p abc
 a 3 0  3a  3  3

Note  If a = b = c, the ellipsoid becomes the sphere x2 + y2 + z2 = a2.


4 4p 3
The volume of the sphere = ⋅ p ⋅ a ⋅ a ⋅ a = a
3 3
Example 7
A Circular hole of radius b is made centrally through a sphere of radius a. Find the volume of
the remaining sphere.

Solution
Both the sphere and circular hole are symmetric about the xy plane.
So, volume of the hole = 2 × volume of the hole above the xy-plane

= 2∫∫∫ dxdydz .
V

V is the volume above the xy-plane

 a2 − x 2 − y 2 
= 2∫∫  ∫ dz  dy dx

R  0


where the region R is the circle x2 + y2 = b2,


b is the radius of hole and x, y vary over R. + + =

\  Volume of the hole = 2∫∫ [z ]0 a


2 2 2
−x −y
dy dx
R

           = 2∫∫ a − x − y dy dx
2 2 2

By changing to polar coordinates, we shall evaluate + =


this double integral.
\ put x = r cos u, y = r sin u,
\ dx dy = r dr du, x2 + y2 = r2
Fig. 3.54
r varies from 0 to b and u varies from 0 to 2p
2p b
\ volume of the hole = 2
∫ 0 0
∫ a 2 − r 2 r dr d u

M03_Eng-Maths (Aditya) CH03.indd 62 7/24/2018 2:03:26 PM


Multiple Integrals and Beta Gamma Functions  n  3.63


2p b

= − ∫  ∫ (a2 − r 2 )1/ 2 ( −2r )dr  d u
0 0 
b
2p
 (a 2 − r 2 ) 3 / 2 
= −∫   du
0  0
3/ 2
2p
2 2
=− ∫ [(a − b 2 )3/ 2 − a3 ] d u = − [(a2 − b 2 )3/ 2 − a3 ][u]02p
2

3 0
3
2 4p 3
= − [(a2 − b 2 )3/ 2 − a3 ] 2p = [a − (a2 − b 2 )3/ 2 ]
3 3
4p 3
We know the volume of the sphere of radius a is a
3
4p 3 4p 3 4p 2
\ volume of the remaining part = a − [a − (a2 − b 2 )3 / 2 ] = (a − b 2 ) 3 / 2
3 3 3

EXERCISE 3.8

+ y2 = 1 and the planes


1. Evaluate
∫ ∫ ∫ dxdydz , where V is the volume enclosed by the cylinder x
2

V
z = 0, z = 2 - x.
x2 y 2 z 2
2. Find the volume of the ellipsoid + + = 1.
a2 b 2 c 2
x2 y 2 z 2
3. Find the volume of the portion of the ellipsoid 2 + 2 + 2 = 1 which lies in the first octant
a b c
using triple integral.

4. Find the volume bounded by xy-plane, the cylinder x2 + y2 = 1 and the plane x + y + z = 3.

5. Find the volume of the paraboloid x2 + y2 = 4z cut off by z = 4.


6. Find the volume of the region D cut off from the solid sphere x2 + y2 + z2 ≤ 1 by the right circular
p
cone with vertex at the origin and semi-vertical angle above the xy-plane.
3
p
[Hint Use spherical polar coordinates; Then 0 ≤ r ≤ 1, 0 ≤ u ≤ 2p, 0 ≤ f ≤ ]
3
7. Find the volume in the positive octant bounded by the plane
x + 2y + 3z = 4 and the coordinate planes.

8. Find the volume of sphere x2 + y2 + z2 = a2 using triple integrals.
9. Find the volume of the region bounded by the paraboloid z = x2 + y2 and the plane z = 4.
10. Find the volume common to the cylinders x2 + y2 = a2 and x2 + z2 = a2.

11. Find the volume cut off from the sphere x2 + y2 + z2 = a2 by the cone x2 + y2 = z2.

M03_Eng-Maths (Aditya) CH03.indd 63 7/24/2018 2:03:28 PM


3.64  n  Engineering Mathematics-III

ANSWERS TO EXERCISE 3.8

4 4pabc pabc 1
1. 2p − 2.  3.  4.  (9p − 4) 5. 32p
3 3 6 3p
p 16 4pa3 a3 2p a3
6. 7.  8.  9.  16 11. 
8p 10.  (2 − 2 )
3 9 3 3 3

3.4 Beta and Gamma functions


3.4.1  Beta function
Definition 3.3  1

The definite integral ∫ x m −1 (1 − x ) n −1 dx , m > 0, n > 0 is defined as the beta function and it is denoted
by b( m, n ). 0

∴ b( m, n ) = ∫ x m −1 (1 − x ) n −1 dx , m > 0, n > 0
0

m, n are called the parameters of beta function.


This integral is also known as the first Eulerian integral.

3.4.2 Symmetric property of beta function


b( m, n ) = b( n , m )
The beta function is symmetric with respect to its parameters.

Proof
1

We have b( m, n ) = ∫ x m −1 (1 − x ) n −1 dx  (1)
0

Put x = 1 - y in (1), then 1 - x = y and dx = −dy


When x = 0, y = 1 and when x = 1, y = 0
0
\ b( m, n ) = ∫ (1 − y ) m −1 y n −1 ( −dy )
1
0 1

= − ∫ y n −1 (1 − y ) m −1 dy = ∫ y n −1 (1 − y ) m −1 dy = b( n , m )  n
1 0

3.4.3 Different forms of beta function


(1)  Beta Function is an Improper Integral

x m −1
That is b( m , n ) = ∫ m +n
dx
0 (1 + x )

M03_Eng-Maths (Aditya) CH03.indd 64 7/24/2018 2:03:32 PM


Multiple Integrals and Beta Gamma Functions  n  3.65

Proof
1

By definition, b( m , n ) = ∫ x m −1 (1 − x ) n −1 dx
0

y (1 + y ) ⋅1 − y ⋅1 1
Put x =   ∴  dx = dy = dy
1+ y (1 + y ) 2
(1 + y ) 2
y x
Now x =   ⇒  (1+ y )x = y   ⇒  x + xy = y   ⇒  y ( x − 1) = − x ⇒ y =
1+ y 1− x
When x = 0, y = 0 and when x = 1, y = ∞
∞ m −1 n −1
 y   y  dy
∴ b( m, n ) = ∫  1 − 1 + y 
0
 1 + y  (1 + y ) 2

∞ ∞
y m −1 (1 + y − y ) n −1 y m −1
=∫ dy = ∫0 (1 + y )m + n dy
0 (1 + y ) m −1+ n −1+ 2


x m −1
∴ b( m, n ) = ∫ m +n
dx n
0 (1 + x )

This integral is an improper integral of the first kind.


(2)  Beta Function Interms of Trignometric Function
p/2

b( m , n ) = 2 ∫ sin 2 m 21 u cos 2 n21 ud u


0

Proof
1

By definition b( m , n ) = ∫ x m −1 (1 − x ) n −1 dx
0

Put x = sin 2 u ∴ dx = 2 sin u cos ud u

p
When x = 0, u = 0 and when x = 1, u =
2
p/2

b( m, n ) = ∫ (sin u) m −1 (1 − sin 2 u) n −1 2 sin u cos ud u


2

0

p/2

= 2 ∫ sin 2 m − 2 u cos 2 n − 2 u sin u cos ud u


0

p/2

⇒ b( m, n ) = 2 ∫ sin 2 m −1 u cos 2 n −1 ud u(1)


0

M03_Eng-Maths (Aditya) CH03.indd 65 7/24/2018 2:03:37 PM


3.66  n  Engineering Mathematics-III

Note  In some practical problems we come across definite integrals involving trignometric functions
which can be evaluated interms of beta functions.
p/2

We have b( m, n ) = 2 ∫ sin 2 m −1 u cos 2 n −1 ud u


0

p/2
1
∫ sin
2 m −1
⇒ u cos 2 n −1 ud u = b( m , n )
0
2

p +1
If p = 2m − 1 ⇒ 2m = p + 1 ⇒ m=
2
q +1
and q = 2n − 1 ⇒ 2n = q + 1 ⇒ n=
2
p/2
1  p + 1 q + 1
∴ ∫ sin u cosq ud u =
p
b ,  n
0
2  2 2 

3.5  The gamma function

Definition 3.4

The integral ∫ e − x x n −1dx ( n > 0) is defined as the gamma function with parameter n and it is denoted
0
by Gn.

∴ Gn = ∫ e − x x n −1 dx , ( n . 0)
0

This integral is also known as Euler’s integral of the second kind.

3.5.1  Properties of gamma function


(1) Prove that Γ1 5 1 .
Proof
∞ ∞ ∞
e −x 
By definition, Γ1 = ∫ e x dx = ∫ e dx = 
−x 1−1 −x
 = −[0 − 1] = 1  n
0 0
 −1  0

(2) Prove that Γ ( n 1 1) 5 nΓn


Proof

By definition Γn = ∫ e − x x n −1dx
0

M03_Eng-Maths (Aditya) CH03.indd 66 7/24/2018 2:03:40 PM


Multiple Integrals and Beta Gamma Functions  n  3.67

∴ Γ( n + 1) = ∫ e − x x n +1−1dx
0

= ∫ e − x x n dx
0

∞ ∞ ∞
 e −x  n −1 e
−x
= x n  − ∫ nx dx = n ∫ e − x x n −1dx = nΓn
 −1  0 0 −1 0

\ Γ ( n + 1) = nΓn

This is true for all positive values of n.

(3) If n is an Integer ≥ 1, then Γn 5 ( n 21)!

We have Γ ( n + 1) = nΓn
Γn = ( n − 1)Γ ( n − 1)
= ( n − 1)( n − 2)Γ ( n − 2)
= ( n − 1)( n − 2) … 3 ⋅ 2 ⋅1⋅ Γ1 
⇒ …
Γn = ( n − 1)( n − 2) 3 ⋅ 2 ⋅1 = ( n − 1)! n

 1
(4) Prove that Γ   5 p
 2
Proof
∞ ∞
 1
By definition, Γ   = ∫ e − x x 1/ 2 −1dx = ∫ e − x x −1/ 2dx
 2 0 0

Put x = y 2 ∴ dx = 2 ydy

When x = 0, y = 0 and when x = ∞, y = ∞


∞ ∞
 1
∴ Γ   = ∫ e − y y −1 ⋅ 2 ydy = 2∫ e − y dy
2 2

 2 0 0


 1

 1
Γ   = 2∫ e − y dy  and Γ   = 2∫ e dx
−x2
Now
2

 2 0
2 0

∞ ∞
 1  1
Γ   ⋅ Γ   = 2∫ e − x dx ⋅ 2∫ e − y dy
2 2

 2  2 0 0

2 ∞∞
  1 
Γ  2   = 4∫ ∫ e
− ( x2 + y2 )
⇒ dxdy [since the limits are constants]
  0 0

M03_Eng-Maths (Aditya) CH03.indd 67 7/24/2018 2:03:44 PM


3.68  n  Engineering Mathematics-III

Put x = r cos u, and   y = r sin u


y
∴ r 2 = x 2 + y 2 and tan u =
x

and dxdy = rdrd u [{ Jacobian value = r ]

p
When x, y varies from 0 to ∞, r varies from 0 to ∞ and u varies from 0 to
2

  1 
2 p/ 2 ∞ p / 2   ∞ − r2 
∴ Γ
  2   = 4 ∫ ∫ e −r2
r dr d u = 4  ∫ d u  ∫ e r dr 
  0 0 0  0 
∞ ∞ ∞
p − r2
= 4[u]p0 / 2 ∫ e − r r dr = 4 ⋅ ∫ e r dr = 2p∫ e − r r dr
2 2

0
20 0

Let r2 = ur 2 = u du
∴ 2rdr = du ⇒ rdr =
2
When r = 0, u = 0 and when r = ∞, u = ∞
2 ∞ ∞
  1  − u du
Γ  2   = 2p∫ e 2 = p∫ e du
∴ −u

  0 0


 e −u  −∞
= p  = −p[e − e ] = −p(0 − 1) = p
0

 −1  0
\ Γ(1/ 2) = p  n

3.5.2 Relation between Beta and Gamma Functions

Γm ⋅ Γn
Prove that b( m , n ) 5 , m > 0, n > 0
Γ( m 1 n)

Proof

By definition, Γm = ∫ e −t t m −1dt
0

Let t = x 2 ∴ dt = 2 xdx
When t = 0, x = 0 and when t = ∞, x = ∞
∞ ∞

Γm = ∫ e − x x 2 m − 2 ⋅ 2xdx = 2∫ e − x x 2 m −1dx (1)


2 2
\
0 0

Similarly, Γn = 2∫ e − y y 2 n −1dy (2)


2

M03_Eng-Maths (Aditya) CH03.indd 68 7/24/2018 2:03:49 PM


Multiple Integrals and Beta Gamma Functions  n  3.69

∞ ∞

Γm ⋅ Γn = 2∫ e − x x 2 m −1dx ⋅ 2∫ e − y y 2 n −1dy
2 2
\ (1) × (2) ⇒
0 0

∞∞

Γm ⋅ Γn = 4 ∫ ∫ e − ( x
2
+y 2 )
⇒ x 2 m −1 y 2 n −1dx dy (3)
0 0

 [since the limits are constants]


Changing to polar coordinates by putting
x = r cos u, y = r sin u

y
we get r 2 = x 2 + y 2 , tan u =  and dxdy = rdrd u
x
p
When x, y varies from 0 to ∞, r varies from 0 to ∞ and u varies from 0 to
2
\ (3) becomes
p/2 ∞

Γm ⋅ Γn = 4 ∫ ∫ e − r ( r cos u) 2 m −1 ( r sin u) 2 n −1 rdrd u


2

0 0
p/ 2 ∞
= 4 ∫ ∫ e − r r 2 m + 2 n − 2 sin 2 n −1 u cos 2 m −1 u r dr d u
2

0 0
p/ 2 ∞

= 4 ∫ sin 2 n −1 u cos 2 m −1 ud u ⋅ ∫ e − r r 2 m + 2 n −1dr


2

0 0

= 2b( n , m ) ⋅ ∫ e − r r 2 m + 2 n −1dr (4)


2

0

I = ∫ e − r r 2 m + 2 n −1dr
2
Let
0

Put t = r2 dt dt
t = r2 ∴ dt = 2rdr ⇒ dr = = 1/ 2
2r 2t
When r = 0, t = 0 and when r = ∞, t = ∞
∞ ∞
dt 1 −t m + n −1/ 2 −1/ 2
I = ∫ e −t (t 1/ 2 )
2 m + 2 n −1

2t 1/ 2 2 ∫0
∴ = e t ⋅ t dt
0

1 −t m + n −1 1
2 ∫0
= e t dt = Γ( m + n ) [ by definiition]
2

∴  (4)  ⇒ Γ(m + n)
Γm ⋅ Γn = 2b( n , m ) ⋅
2
⇒ Γm ⋅ Γn = b( m, n)Γ ( m + n) [{ b( m, n) = b( n, m)]
Γm ⋅ Γn
∴ b( m , n ) =
Γ(m + n)

M03_Eng-Maths (Aditya) CH03.indd 69 7/24/2018 2:03:54 PM


3.70  n  Engineering Mathematics-III

Corollary
 p + 1  q + 1
Γ ⋅Γ
p/2
 2   2 
Prove that
∫ sin u cos ud u =
p q

 p + q + 2
0
2Γ  
 2
p/2
1  p + 1 q + 1
We have ∫ sin u cosq ud u =
p
b , 
0
2  2 2 
 p + 1  q + 1
Γ ⋅Γ
1  2   2 
p/2

⇒ ∫ sin p u cosq ud u =
2  p + 1 q + 1
0
Γ + 
 2 2 
 p + 1  q + 1
Γ ⋅Γ
p/2
 2   2 
⇒ ∫ sin p u cosq ud u =
 p + q + 2
 (5)
0
2Γ  
 2
(1)  In particular, if we put p 5 n, q 5 0 in (5), we get
 n + 1  1   n + 1
Γ ⋅Γ Γ
p/2
 2   2   2  p
∫ sin u d u = = ⋅
n

 n + 2  n + 2 2
2Γ  Γ
 2   2 
0

 n + 1
Γ
p/2
 2  p
Similarly, ∫ cos u d u = ⋅
n

 n + 2 2
Γ
 2 
0

If n = 2, we get,
 2 + 1
Γ
p/2
 2  p
∫ sin u d u = ⋅
2

 2 + 2 2
Γ
 2 
0

 3 1  1
Γ  Γ
 2  p 2  2  p p p p
= ⋅ = ⋅ = =
Γ2 2 1! 2 4 4
p/2
p
∫ cos u du =
2
Similarly,
0
4

(2)  In particular, if we put p 5 0, q 5 0 in (5), then we get


p/2
Γ (1/ 2)Γ (1/ 2)
∫ sin u cos 0 ud u =
0

0
2Γ ( 2/2)

M03_Eng-Maths (Aditya) CH03.indd 70 7/24/2018 2:03:58 PM


Multiple Integrals and Beta Gamma Functions  n  3.71

p/2
(Γ1/ 2) 2
⇒ ∫ du =
0
2Γ1
(Γ1/ 2) 2   ⇒  p (Γ1/ 2)
2
⇒ [u]p0 / 2 = =
2 2 2
2
  1 
⇒ Γ  2   = p ⇒  Γ  1  = p
   2
2
 1 1   1
(3)  b  ,  5  Γ   
2 2   2

Proof
2
 1  1   1 
Γ   Γ   Γ   2
 1 1  2  2   2    1 
b ,  = = = Γ  
 2 2  1 1 Γ1   2 
Γ + 
 2 2

 m 1 1
Γ ( p 1 1) Γ 
1
1  n 
(4)  Prove that ∫ x m (1 2 x n ) p 5
n  m 1 1
0
Γ  p 111 
 n 

Proof 1

Let I = ∫ x m (1 − x n ) p dx
0

Put 1− x = y
n
⇒ x = 1− y
n
⇒ x = (1 − y )1/ n
Differentiating both sides, we get
1 1− n
1 −1 1
dx = (1 − y ) n ( −1) dy = − (1 − y ) n dy
n n
When x = 0, y = 1 and when x = 1, y = 0
0 m
 1 1− n

∴ I = ∫ (1 − y ) n y p  − (1 − y ) n  dy
1  n 
0 m 1− n
1 p +
=− ∫
n1
y (1 − y ) n n
dy

1 p
1 m 1
+ −1 b a

= ∫
n0
y (1 − y ) n n
dy ∫
a
f ( x ) dx = − ∫
b
f ( x )dx 

1 m +1
1 p +1−1 −1
= ∫
n0
y (1 − y ) n dy

M03_Eng-Maths (Aditya) CH03.indd 71 7/24/2018 2:04:03 PM


3.72  n  Engineering Mathematics-III

 m + 1
Γ ( p + 1)Γ 
1  m + 1 1  n 
= b  p + 1, =
n  n  n  m + 1
Γ  p +1+ 
 n 
 m + 1
Γ ( p + 1)Γ 
1
1  n 
∴ ∫0 x (1 − x ) dx = n
m n p

 m + 1
Γ  p +1+ 
 n 

WORKED EXAMPLES
Example 1
1

Find the value of ∫ x 7 (1 2 x ) 6 dx .


0

Solution.
1

We know b( m , n ) = ∫ x m −1 (1 − x ) n −1 dx
0
1 1
Γ8 ⋅ Γ 7 7 !⋅ 6 !
∫x (1 − x )6 dx = ∫ x 8 −1 (1 − x )7 −1 dx = b(8, 7) = =
7
Now
0 0 Γ (8 + 7) 14 !

Example 2
1

Evaluate ∫ x 11 (1 2 x ) 5 dx .
0

Solution.
1
We know b( m , n ) = ∫ x m −1 (1 − x ) n −1 dx
0
1 1
Γ12 ⋅ Γ 6 11!⋅ 5!
∫x (1 − x )5 dx = ∫ x 12 −1 (1 − x )6 −1 dx = b(12, 6) = =
11
Now
0 0
Γ (12 + 16) 27 !

Example 3
p/ 2

∫ sin
8
Evaluate u cos 7 ud u.
0

Solution.
We know
p/2
1  p + 1 q + 1
∫ sin u cosq ud u =
p
b , 
0
2  2 2 
p/2
∴ 1  8 + 1 7 + 1
∫ sin u cos 7 ud u = [ p = 8, q = 7]
8
b , 
0
2  2 2 
 9
Γ   Γ4
1 9  1  2
= b  , 4 =
2 2  2 9 
Γ  + 4
M03_Eng-Maths (Aditya) CH03.indd 72
2  7/24/2018 2:04:08 PM
1 7 + 1Integrals and Beta Gamma Functions    3.73
1  8 +Multiple
p/2 n

0
sin8 u cos 7 ud u = b
2  2
,
2 
 [ p = 8, q = 7]

 9
Γ   Γ4
1 9  1  2
= b  , 4 =
2 2  2 9 
Γ  + 4
2 
 9  9
Γ   × 3! 1⋅ 2 ⋅ 3 ⋅ Γ  
1  2  2 16 16
= = = =
2  17  15 13 11 9  9  15 ⋅13 ⋅11⋅ 3 6435
Γ  2⋅ ⋅ ⋅ ⋅ Γ  
 2 2 2 2 2  2

Example 4
p/ 2

∫ sin
5
Evaluate ud u.
0

Solution.
We know that
p/2
1  p + 1 q + 1
∫ sin u cosq ud u =
p
b , 
0
2  2 2 
p/2 p/2
1  5 + 1 0 + 1
∫ sin 5 ud u = ∫ sin u cos 0 ud u = [ p = 5, q = 0]
5
∴ b , 
0 0
2  2 2 
 1  1  1
Γ3 ⋅ Γ   2 !⋅ Γ   Γ 
1  1 1  2 1  2  2 8
= b  3,  = = = =
2  2 2  6 + 1 2  7 5 3 1  1  15
Γ Γ  ⋅ ⋅ Γ 
 2   2 2 2 2  2
Example 5
p/ 2

Evaluate ∫
0
cos 8 ud u.

Solution.
p/2
1  p + 1 q + 1
∫ sin u cosq ud u =
p
We know that b , 
0
2  2 2 
p/2 p/2

∴ ∫ cos8 ud u = ∫ sin [ p = 0, q = 8]
0
u cos8 ud u
0 0

1  0 + 1 8 + 1 1  1 9
= b ,  = b , 
2  2 2  2 2 2
 1  9
Γ ⋅Γ
1  2   2 
=
2  9 1
Γ + 
 2 2

 1 7 5 3 1  1
Γ ⋅ ⋅ ⋅ Γ
1  2  2 2 2 2  2  1 7 ⋅ 5 ⋅ 3p 35p 35p
= = = 8 =
2 Γ5 25 1 ⋅ 2 ⋅ 3 ⋅ 4 2 256

M03_Eng-Maths (Aditya) CH03.indd 73 7/24/2018 2:04:12 PM


3.74  n  Engineering Mathematics-III

Example 6
p/ 2

Evaluate ∫
0
tan u d u.

Solution.
We know that
p/2
1  p + 1 q + 1
∫ sin u cosq ud u =
p
b , 
0
2  2 2 
p/2 p/2

∫ tan u d u = ∫ sin u ⋅ cos −1/ 2 ud u


1/ 2
Now,
0 0

1 1 
+1 − +1  1 1
1 2
= b , 2 
 { p = 2 , q = − 2 
2  2 2 

1  3 1
=b , 
2  4 4
p  3  1  1  3
Γ ⋅Γ Γ ⋅Γ
2
1  4   4  1  4   4  1  1   3 
∫ tan u d u =
2  3 1
=
2 Γ1
= Γ  Γ  .
2  4  4
0
Γ + 
 4 4

Example 7
p/2
1
Evaluate ∫
0 tan u
d u.

Solution.
We know that
p/2
1  p + 1 q + 1
∫ sin u cosq ud u =
p
b , 
0
2  2 2 
p/2 p/ 2
1
∫ ∫ sin
−1/ 2
∴ du = u cos1/ 2 ud u
0 tan u 0

 1 1 
− +1 +1
1  2 
= b ,2 
2  2 2 
 1  3  1  3
Γ   ⋅Γ  
p
Γ  Γ 
2
1 1  1 3 1  4  4 1  4  4 1  1  3
∫ tan u
du = b  ,  =
2  4 4 2  1 3
=
2 Γ1
= Γ   ⋅Γ   .
2  4  4
0
Γ + 
 4 4

M03_Eng-Maths (Aditya) CH03.indd 74 7/24/2018 2:04:16 PM


Multiple Integrals and Beta Gamma Functions  n  3.75

Example 8
p/ 2 p/ 2
1
Evaluate

0
tan u d u 3 ∫
0 tan u
d u.

Solution.
From Examples 6 and 7, we have
p/2 p/2 2
1 1  1  3 1  1  3 1   1  3 

0
tan u d u × ∫
0 tan u
du = Γ  Γ  × Γ  Γ  = Γ  Γ 
2  4   4  2  4   4  4   4   4  

Example 9
p/ 2 p/ 2
Show that 1

0
sin u d u 3 ∫
0 sin u
d u 5 p.

Solution.
We know that
p/2
1  p + 1 q + 1
∫ sin u cosq ud u =
p
b , 
0
2  2 2 
1 
p/2 p/2 +1
1 2 0 + 1
∴ ∫ sin u d u = ∫ sin u cos ud u = b 
1/ 2 0
, 
0 0
2  2 2 
 3  1  3  1
Γ   ⋅Γ   Γ ⋅Γ
1  3 1 1  4   2  1  4   2 
= b ,  = =
2  4 2 2  3 1 2  5
Γ +  Γ 
 4 2  4
p  3  1  3  1  3  1
Γ   ⋅Γ   Γ ⋅Γ Γ   ⋅Γ  
2
1  4  2 1  4   2   4  2
⇒ ∫ sin u d u = =
2  5   5  2  1  1
=2
 1
0
 − 1 Γ  − 1   Γ   Γ 
 4
4 4 4 4
 1 
p/2 p/ 2 − +1
1 1  1
and ∫ du = ∫ sin −1/ 2 u cos 0 ud u = b  2 , 
0 sin u 0
2  2 2
 1  1  1  1
Γ   ⋅Γ   Γ   ⋅Γ  
1  1 1 1  4   2 = 1  4  2

= b ,  =
2  4 2 2  1 1 2  3
Γ +  Γ 
 4 2  4

 3  1  1  1
Γ   ⋅Γ   Γ   ⋅Γ  
p/2 p/2
1  4  2 1  4  2  1  1
\ ∫ sin u d u × ∫ sin u
du = 2
 
1 2  
3
= Γ  ×Γ  = p × p = p
 2   2
0 0
Γ  Γ 
 4  4

M03_Eng-Maths (Aditya) CH03.indd 75 7/24/2018 2:04:21 PM


3.76  n  Engineering Mathematics-III

Example 10

Γn
Prove that ∫ e 2ax x n 21dx 5 , where a and n are positive.
0 an
Solution.
∞ ∞

We know Γn = ∫ e − x x n −1dx . Let I = ∫ e − ax x n −1dx


0 0

dt
Put t = ax ∴ dt = adx ⇒ dx =
a
When x = 0, t = 0 and when x = ∞, t = ∞
∞ n −1
t  dt 1 ∞ Γn
\ I = ∫ e −t  
a a
= n
a ∫
0
e − t t n −1dt =
an
0


Γn
⇒ ∫e
− ax
x n −1dx =
0 an
Example 11
∞ ∞
1  n 11  p
∫x ∫e
n 2a 2 x 2 2a 2 x 2
Show that e dx 5 Γ  . Deduce that dx 5 . Hence, show that
0 2 a n 11  2  0
2a
∞ ∞
1 p
∫ cos ( x ) dx 5 ∫ sin ( x 2 ) dx 5
2
.
0 0
2 2

Solution.
∞ ∞

Γn = ∫ e − x x n −1dx . Let I = ∫ x n e − a x dx
2 2
We know
0 0

1 dt
Put ax = t . ∴ adx = dt ⇒ dx =
2 t 2a t
When x = 0, t = 0 and when x = ∞, t = ∞

t n / 2 − t dt
I =∫ n
⋅e
0 a 2a t
∞ ∞
n +1
 n + 1
n 1
1 − 1 −1 1
n +1 ∫
= e − t t 2 2 dt = n +1 ∫ e − t t 2 dt = n +1 Γ 
2a 0 2a 0 2a  2 


1  n + 1
∫x
2 2
\ n
e − a x dx = Γ
 2 
n +1
(1)
0 2a
In (1), put n = 0.

1  1 p
∫e
− a2 x 2
\ dx = Γ  = (2)
2a  2  2a
0

1− i (1 − i ) 2 1 − 1 − 2i
In (2), put a = , then a2 = = = −i   ⇒  −a2 = i
2 2 2

M03_Eng-Maths (Aditya) CH03.indd 76 7/24/2018 2:04:27 PM


Multiple Integrals and Beta Gamma Functions  n  3.77


\ (2) ⇒ p
∫e
ix 2
dx =
1− i 
0
2
 2 

(1 + i ) p (1 + i )
∫ (cos (x ) + i sin ( x 2 ) ) dx = p =
2
⇒ ) )
0 2 (1 + i (1 − i 2 2
\ equating real and imaginary parts, we get
∞ ∞
p p
∫ cos (x ) dx = ∫ sin (x ) dx =
2 2
and
0 2 2 0 2 2
∞ ∞
1 p
\ ∫ cos (x ) dx = ∫ sin (x ) dx =
2 2

0 0
2 2

Example 12
1 n 21

Show that Γn 5 ∫  log 


1
dy , n > 0 .
0
 y

Solution.

We have Γn = ∫ e − x x n −1dx , n > 0


0

1 1
Put y = e −x ⇒ ex = ⇒ x = log   \  dx = 1  − 1  dy = − 1 dy
y y 1  y 2  y
y
When x = 0, y = 1 and when x = ∞, y = 0
0 n −1 0 n −1 1 n −1
 1  1   1  1
\ Γn = ∫ y  log   − y dy  = − ∫  log y  dy = ∫  log  dy
1
 y 1 0
 y

Example 13
1 1
x 2 dx dx p
Prove that ∫ 3∫ 5 .
0 12 x 4
0 11 x 4
4 2

Solution.
1 1
x 2dx dx
Let I =∫ ×∫
0 1− x 4
0 1+ x 4
1 1 1
x 2dx −
Let I1 = ∫ = ∫ x 2 (1 − x 4 ) 2
dx .
0 1− x 4 0

M03_Eng-Maths (Aditya) CH03.indd 77 7/24/2018 2:04:32 PM


3.78  n  Engineering Mathematics-III

 m + 1
Γ ( p + 1) Γ 
1
1  n 
Property (4) is ∫ x m (1 − x n ) p dx =
n  m + 1
0
Γ  p +1+ 
 n 

 1   2 + 1
Γ  − + 1 Γ 
1  2   4 
1 1
− 1
\ I 1 = ∫ x 2 (1 − x ) dx =
4 2
[Here m = 2, n = 4, p = − ]
4  1 2 + 1 2
0
Γ  − +1+ 
 2 4  

 1  3  3  3  3
Γ  Γ  Γ  Γ  Γ 
1  2  4 1  4 1  4  4
= = p = p = p
4  
5 4   4
5 1  
1  1
Γ  Γ  Γ  Γ 
 4  4 4  4  4

1
dx
Let I 2 = ∫
0 1+ x 4

sec 2 ud u
Put x 2 = tan u ∴ 2 xdx = sec 2 ud u ⇒ dx =
2 tan u

p
When x = 0, u = 0 and when x = 1, u =
4

p/4 p/4
sec 2 ud u 1 sec 2 ud u
\ I2 = ∫0 (
1 + tan 2 u 2 tan u )
=
2 ∫0 sin u
sec u
cos u
p/4 p/4
1 1 2 du
=
2 ∫0 sin u cos u
du =
2 ∫
0 sin 2u

df
Let 2u = f, ∴ 2d u = d f ⇒ du =
2

p p
When u = 0, f = 0 and when u = ,f=
4 2

p/ 2 p/ 2
\ 2 df 2
∫ sin ∫ sin
−1/ 2 −1/ 2
I2 = ⋅ f = f df
2 0
2 4 0

M03_Eng-Maths (Aditya) CH03.indd 78 7/24/2018 2:04:37 PM


Multiple Integrals and Beta Gamma Functions  n  3.79

2 1  −1/ 2 + 1 0 + 1
= b ,   1 
4 2  2 2   { p = − , q = 0 
 2 
2  1 1
= b , 
8  4 2
 1  1  1  1  1
Γ  Γ  Γ  Γ  Γ 
2  4  2 2  4  2 p  4
= = = ⋅
8  1 1 8  3 4 2 Γ  3
Γ +  Γ   
 4 2  4 4
 3  1
Γ  Γ
p  4 
1 1
x 2 dx dx  4 p
\ I = I1 × I 2 = ∫ ×∫ = p ⋅ =
1 − x4 1+ x4  1  3
0 0
Γ  4 2 Γ  4 2
 4  4
Example 14

xc
Show that ∫ x
dx 5 Γ( c 11) / (log e c ) c 11 .
0 c
Solution.
∞ ∞ ∞
xc xc
Let I =∫ x
dx = ∫0 e x loge c dx = ∫0 e e x dx
− x log c c
[{ e
x log e c
=e
log e c x
= cx ]
0 c

dt
Put x loge c = t ∴ loge c ⋅ dx = dt ⇒ dx =
loge c
When x = 0, t = 0 and when x = ∞, t = ∞
∞ c ∞
 t  dt 1 Γ (c + 1)
\ I = ∫ e −t   log c = (log c )c +1 ∫ e t
− t c +1−1
dt = c +1
0  log e c e e 0 (log e c)

Example 15
 1
Prove that b  m ,  5 2 2 m 21 b( m , m ).
 2
Solution.
p/2

We know that b( m, n ) = 2 ∫ sin 2 m −1 u cos 2 n −1 ud u


0
p/2
\ b( m, m ) = 2 ∫ sin 2 m −1 u cos 2 m −1 ud u
0

p/2

= 2 ∫ (sin u cos u) 2 m −1 d u
0
p/2 2 m −1
 sin 2u 
=2∫ 
 2 
du
0
p/2 p/2
2 1
∫ sin 2 m −1 2u d u = ∫ sin
2 m −1
= 2u d u
22 m −1 0 22 m − 2 0

M03_Eng-Maths (Aditya) CH03.indd 79 7/24/2018 2:04:44 PM


3.80  n  Engineering Mathematics-III

df
Let 2u = f [ 2d u = d f ⇒ d u =
2

p
When u = 0,  f = 0 and when  u = ,f= p
2

sin 2 m −1 f d f
p
1
\ b( m , m ) =
22 m − 2 ∫0 2
p/2
1
∫ sin
2 m −1
= 2 m −1
2⋅ f df [{ sin (p − f) = sin f]
2 0

p/2

⇒ 22 m −1 b( m , m ) = 2 ∫ sin 2 m −1 f d f
0

p/2

= 2 ∫ sin 2 m −1 f cos 0 fd f
0

 2m − 1 + 1 0 + 1
= b , 
 2 2 

⇒  1
22 m −1 b( m , m ) = b  m , 
 2

Example 16

 1 p
Prove that Γm ⋅ Γ  m 1  5 2 m 21 Γ ( 2 m ).
 2 2

Solution.
From Example 15, we have
 1
22 m −1 b( m , m ) = b  m , 
 2
 1
Γm ⋅ Γ    Γm ⋅ Γn 
⇒ Γm Γm  2 
∴ b( m , n ) = Γ ( m + n ) 
2 m −1
2 =
Γ(m + m )  1  
Γm + 
 2

22 m −1 Γm p
=
⇒ Γ ( 2m )  1
Γm + 
 2

⇒  1 p
Γm Γ  m +  = 2 m −1 Γ ( 2m )
 2  2

M03_Eng-Maths (Aditya) CH03.indd 80 7/24/2018 2:04:49 PM


Multiple Integrals and Beta Gamma Functions  n  3.81

Example 17
p ? Γn
Prove that b( n, n ) 5 .
 1
2 2 n 21
Γ  n1 
 2
Solution.
 1
From Example 15, we have   22 m −1 b( m , m ) = b  m , 
 2
 1
If m = n , we get 22 n −1 b( n , n ) = b  n , 
 2

 1
Γn ⋅ Γ  
 2 p ⋅ Γn
⇒ 22 n −1 b( n , n ) =   ⇒  b( n , n ) =
 1  1
Γn +  22 n −1 Γ  n + 
 2  2
Example 18
b( m 11, n ) b( m , n 11) b( m , n )
Prove that 5 5 .
m n m1n
Solution.
Γm Γn
We have b( m , n ) =
Γ(m + n)
Γ ( m + 1)Γn
\ b( m + 1, n ) =
Γ(m + 1 + n)

\ b( m + 1, n ) Γ ( m + 1) ⋅ Γn m Γm ⋅ Γn Γm ⋅ Γn
= = =
m m Γ ( m + n + 1) m Γ ( m + n + 1) ( m + n )Γ ( m + n )
b( m + 1, n ) b( m , n )
⇒ = (1)
m m+n
and b( m , n + 1) Γm ⋅ Γ ( n + 1)
=
n nΓ ( m + n + 1)
Γm ⋅ n Γn Γm ⋅ Γn
= =
n ⋅ ( m + n )Γ ( m + n ) ( m + n )Γ ( m + n )

b( m , n + 1) b( m , n )
⇒ = (2)
n m+n
From (1) and (2), we get

b( m + 1, n ) b( m , n + 1) b( m , n )
= =
m n m+n

M03_Eng-Maths (Aditya) CH03.indd 81 7/24/2018 2:04:54 PM


3.82  n  Engineering Mathematics-III

Example 19
Prove that b( m 11, n ) 1 b( m , n 11) 5 b( m , n ).

Solution.
Γm Γn
We have b( m , n ) =
Γ(m + n)

\ Γ ( m + 1) ⋅ Γn m Γm ⋅ Γn
b( m +1, n ) = =
Γ ( m + n + 1) Γ ( m + n + 1)

m Γm ⋅ Γn m
⇒ b( m + 1, n ) = = ⋅ b( m , n ) (1)
( m + n )Γ ( m + n ) ( m + n )

Γm ⋅ Γ ( n + 1) Γm ⋅ n Γn
and b( m , n + 1) = =
Γ ( m + n + 1) ( m + n )Γ ( m + n )

n Γm Γn
⇒ b( m , n + 1) = ⋅
(m + n) Γ(m + n)
n
⇒ b( m , n + 1) = ⋅ b( m , n )(2)
(m + n)
Adding (1) and (2), we get
m n
⇒ b( m + 1, n ) + b( m , n + 1) = b( m , n ) + b( m , n )
m+n m+n
m+n
= b( m , n )
m+n
⇒ b( m + 1, n ) + b( m , n + 1) = b( m , n )

b( m + 1, n ) b( m , n + 1) b( m , n )
Aliter  By example 18, = =
m n m+n

b( m + 1, n ) + b( m , n + 1)
By rule of ratio and proportion, each ratio =
m+n
b( m + 1, n ) + b( m , n + 1) b( m , n )
∴ =
m+n m+n

⇒ b( m + 1, n ) + b( m , n + 1) = b( m , n )

Example 20
1 1
dx
∫x ∫
m
(1 2x n ) p dx in terms of gamma function and hence, evaluate .
0 0 12 x n

M03_Eng-Maths (Aditya) CH03.indd 82 7/24/2018 2:04:59 PM


Multiple Integrals and Beta Gamma Functions  n  3.83

Solution.
 m + 1
Γ ( p + 1)Γ 
1
1  n 
we have (i)
∫0 x (1 − x ) dx = n 
m n p

 m + 1
Γ  ( p + 1) + 
 n 
1 1 1
dx −
(ii) ∫
0 1− x n
= ∫ (1 − x n )
0
2
dx

1
Here m = 0, p = −
2
 1   0 + 1
Γ  − + 1 Γ
1  2   n 
1
dx
\ ∫ =
1 − x n n Γ  − 1 + 1 + 0 + 1

  2 
0

 n 
 1  1 1
Γ  Γ  Γ  1 
1  2  n

= =
p n  { Γ 2 = p 
n  1 1 n  n + 2
Γ +  Γ
 n 2  2n 
Example 21
1

∫x
5
Evaluate (12 x 3 )10 dx .
0

Solution.
1

Let I = ∫ x 5 (1 − x 3 )10 dx
0

Here m = 5, n = 3, p = 10
 5 + 1
Γ (10 + 1)Γ 
1
1  3 
\ I = ∫ x 5 (1 − x 3 )10 dx = 
3  5 + 1
0
Γ 10 + 1 + 
 3 
1 Γ (11)Γ ( 2) Γ (11) × 1 1
= = = .
3 Γ (13) 3 × 12 × 11 Γ (11) 396
Example 22
1

∫x
7
Express (1 2 x 4 ) 9 dx in terms of gamma function and evaluate.
0

Solution.
1

Let I = ∫ x 7 (1 − x 4 )9 dx
0

M03_Eng-Maths (Aditya) CH03.indd 83 7/24/2018 2:05:04 PM


3.84  n  Engineering Mathematics-III

Here m = 7, n = 4, p = 9
 m + 1
Γ ( p + 1)Γ 
1  n 
\ I= 
n  m + 1
Γ  p +1+ 
 n 

 7 + 1
Γ (9 + 1)Γ 
1  4  1 Γ (10)Γ ( 2) Γ (10) × 1 1
= = = =
4  7 + 1 4 Γ (13) 4 × 12 × 11 × 10Γ(10) 5280
Γ  9 +1+ 
 4 

Example 23
1 1 1 1
dx dx
Evaluate (1) ∫ 4
, (2) ∫ (1 2x 3 ) 5 dx ,  (3) ∫ x 5 (1 2 x 4 ) 7 dx ,  (4) ∫ .
0 12 x 0 0 0 12 x n
Solution.
1
dx
(1) ∫
0 1− x 4
1 1
dx

1− x 4 0

0
= ∫ x 0 (1 − x 4 ) −1/ 2 dx

Here m = 0, n = 4, p = -1/2
 m + 1
Γ ( p + 1)Γ 
\
1
dx 1  n 
∫ = ⋅
1 − x 4 n Γ  p + 1 + m + 1
0
 
n 
 1   0 + 1  1  1  1
Γ  − + 1 Γ   Γ  Γ  Γ  
1  2   4  1  2   
4 = p  4
= ⋅ = ⋅ ⋅
4  1 0 + 1 4  1 1 4  3
Γ  − +1+  Γ +  Γ 
 2 4   2 4  4

∫ (1 − x
3 5
(2) ) dx
0
1 1

∫ (1 − x ) dx = ∫ x (1 − x ) dx
3 5 0 3 5

0 0

Here m = 0, n = 3, p = 5
 m + 1
Γ ( p + 1)Γ 
1
1  n 
\ ∫0 (1 − x ) dx = n ⋅ 
3 5

 m + 1
Γ  p +1+ 
 n 

M03_Eng-Maths (Aditya) CH03.indd 84 7/24/2018 2:05:09 PM


Multiple Integrals and Beta Gamma Functions  n  3.85

 0 + 1
Γ (5 + 1)Γ 
1  3 
= ⋅
3  0 + 1
Γ 5 +1+ 
 3 
 1  1
Γ6 ⋅ Γ   5!⋅ Γ  
1  3 1  3
= ⋅ = ⋅
3  1 3  19 
Γ 6 +  Γ 
 3  3
 1
Γ 
120  3 36 729
= . = =
3 16 13 10 7 4 1 1 16 ⋅13 ⋅ 7 1456
. . . . . Γ
3 3 3 3 3 3 3
1


(3) x 5 (1 − x 4 )7 dx
0

Here m = 5, n = 4, p = 7
 m + 1
Γ ( p + 1)Γ 
We know
1
1  n   [Using Property 4]
∫0 x (1 − x ) dx = n ⋅
5 4 7

 m + 1
Γ  p +1+ 
 n 
 5 + 1
Γ (7 + 1)Γ 
1  4 
= ⋅
4  5 + 1
Γ  7 +1+ 
 4 
 3 1  1
Γ8 ⋅ Γ   7! ⋅ Γ  
1  2 1 2  2  = 7! p Γ  19 
= ⋅ =  
4  3  4  19  8 2
Γ 8 +  Γ 
 2  2
1
dx
(4) ∫
0 1− x n
1
Here m = 0, n = p = −

2
 1   0 + 1
Γ  − + 1 Γ 
1
dx
1
1  2   n 
We know ∫ = ∫ x 0 (1 − x n ) −1/ 2 dx =
1 − xn 0 n  1 0 + 1
[m = 0, n, p = -1/2]
0
Γ  − +1+ 
 2 n 
 1  1  1
Γ  Γ  Γ 
1  n   2 p  n
= =
n  1 1 n  1 1
Γ +  Γ + 
 n 2  n 2

M03_Eng-Maths (Aditya) CH03.indd 85 7/24/2018 2:05:14 PM


3.86  n  Engineering Mathematics-III

EXERCISE 3.9
p/2
b( m + 1, n ) m 1  1  3
1. Prove that = .
b( m, n + 1) n
2. Prove that ∫0
cot ud u = Γ   Γ   .
2  4  4

1 n −1
p/2 p/2
dx  1
3. Prove that ∫ cos x dx × ∫ = p. 4. Prove that ∫ log  dx = Γn.
0 0 cos x 0
 x 

∞ p/2

5. Evaluate ∫ xe − x dx given that Γ   = 0.902.


5
∫ sin
3
3
6. Find the value of x cos5 / 2 x dx .
0
 3 0

p/2 1
xdx 1  2 1
7. Find the value of ∫ sin11 u cos12 u d u. 8. Prove that ∫ = b , .
5  5 2
0 0 1− x 5

9. Prove that ∫ ( x − a) m (b − x ) n dx = (b − a) m + n +1 b( m + 1, n + 1).


a

1
( −1) n n !
10. Prove that ∫ x m (log x ) n dx = , where n is positive m > -1.
0 ( m + n ) n +1
1 p −1
 1 Γp
11. Show that ∫ x q −1  log  dx = where p > 0, q > 0.
 x qn
0

1 1

∫ x (1 − x ) dx (ii) 
∫ x (1 − x ) dx
7 9 13 12
12. Evaluate (i) 
0 0

1 1 1

∫x ∫ (1 − x ) dx (iii) 
(1 − x 2 )7 dx (ii) x ∫ x (1 − x ) dx
7 3 5 8 2 3 4
13. Evaluate (i) 
0 0 0

14. Express ∫ x p −1e − kx dx ( k > 0) in terms of gamma function.


0

1 1
x 2dx dx p
15. Show that ∫ 4 1/ 2 ∫
⋅ = .
0 (1 − x ) 0 (1 − x )
4 1/ 2
4

ANSWERS to exercise 3.9


8 12 13/ 2
5. 0.451 7.  6. 
77 25/ 2
   
4 4
Γ  Γ 
 7 ! 9 !  13! 12 ! 1    5 2 Γp
12. (i)    (ii)     13. (i)  (ii)  5  (iii)  14. 
 17 !   24 !  490  24  15 kp
2Γ  
 5

M03_Eng-Maths (Aditya) CH03.indd 86 7/24/2018 2:05:21 PM


Vector Differentiation 4
4.0 Introduction
In Science and Engineering we often deal with the analysis of forces and velocities and other
quantities which are vectors. These vectors are not constants but vary with position and time. Hence,
they are functions of one or more variables.
Vector Calculus extends the concepts of differential calculus and integral calculus of real functions
in an interval to vector functions and thus enabling us to analyse problems over curves and surfaces in
three dimension. Vector Calculus finds applications in a wide variety of fields such as fluid flow, heat
flow, solid mechanics, electrostatics etc.
In Vector Calculus we deal mainly with two kinds of functions, scalar point functions and vector
point functions and their fields.

4.1  Scalar and Vector Point Functions


Definition 4.1  If to each point P( r ) (the point P with position vector r ) of a region R in space there
is a unique scalar or real number denoted by f( r ), then f is called a scalar point function in R.
The region R is called a scalar field.

Definition 4.2  If to each point P( r ) of a region R in space there is a unique vector denoted by F( r ),
then F is called a vector point function in R. The region R is called a vector field.

Note
1. In applications, the domain of definition of point functions may be points in a region of space,
points on a surface or points on a curve.
2. If we introduce cartesian coordinate system, then r = xi + y j + zk or
r = ( x, y, z ) and instead of F( r ) and f( r ) we can write
F( x, y, z ) = ( F1 ( x, y, z ), F2 ( x, y, z ), F3 ( x, y, z )) or
 F( x, y, z ) = F1 ( x, y, z )i + F2 ( x, y, z ) j + F3 ( x, y, z )k
and f( r ) as f( x, y, z )
3. A vector or scalar field that has a geometrical or physical meaning should depend only on
the points P where it is defined but not on the particular choice of the cartesian coordinates.
In otherwords, the scalar and vector fields have the property of invariance under a transformation
of space coordinates.

Examples of scalar field


1. Temperature T within a body is scalar field, namely temperature field.
2. When an iron bar is heated at one end, the temperature at various points will attain a steady state
and the temperature will depend only on the position.

M03_Eng-Maths (Aditya) CH04.indd 1 7/23/2018 4:46:30 PM


4.2  n  Engineering Mathematics-III

3. The pressure of air in earth’s atmosphere is a scalar field called pressure field.
4. f( x, y, z ) = x 3 + y 3 + z 3 − 3 xyz defines a scalar field.

Examples of vector field


1. The velocity of a moving fluid at any instant is a vector point function and defines a vector field.
2. Earth’s magnetic field is a vector field.
3. Gravitational force on a particle in space defines a vector field.
4. F ( x, y, z ) = x 2 i − y 2 j + zk defines a vector field.

Note  Vector and scalar functions may also depend on time or on other parameters.

Definition 4.3  Derivative of a vector function


f ( t 1 ∆t ) 2 f ( t )
A vector function f (t ) is said to be differentiable at a point t, if lim exists.
∆t → 0 ∆t
df
Then it is denoted by or f ′ and is called the derivative of the vector function f at t.
dt
Note
1. If f (t ) = f1 (t )i + f 2 (t ) j + f 3 (t )k then f (t ) is differentiable at t if and only if its components f1(t),
d f (t )
f2(t), f3(t) are differentiable at t and = f1′(t )i + f 2′ (t ) j + f 3′(t )k
dt
df d2 f
2. If the derivative of w.r.to t exists, it is denoted by 2 . Similarly, we denote higher derivatives.
dt dt
dc
3. If c is a constant vector, then = 0.
dt
dc
For c = c1 i + c2 j + c3 k and = 0i + 0 j + 0 k = 0.
dt

4.1.1  Geometrical Meaning of Derivative


Let r (t ) be the position vector of a point P with respect to the origin O.
As t varies continuously over a time interval P traces the curve C. Thus, the vector function r (t )
represents a curve C in space.
Q
Let r and r + ∆r be the position vectors of neighbouring
points P and Q on the curve C.

Then PQ = OQ − OP
= r + ∆r − r
= ∆r
∆r
\  is along the chord PQ.
∆t
P
∆r dr dr
If lim exists, it is denoted by and is in the
∆t → 0 ∆t o
dt dt C
directing of the tangent at P to the curve.
Fig. 4.1

M03_Eng-Maths (Aditya) CH04.indd 2 7/23/2018 4:46:37 PM


Vector Differentiation  n  4.3

dr dr
If ≠ 0, then or r ′(t ) is called a tangent vector to the curve C at P.
dt dt
r ′ (t )
The unit tangent vector at P is = = u(t ).
r ′ (t )
Both r ′(t ) and u(t ) are in the direction of increasing t. Hence, their sense depends on the orientation
of the curve C.

4.2  Differentiation Formulae


If f and g are differentiable vector functions of t and f is a scalar function of t then
d d f dg d d f df
1. ( f ± g) = ± 2. (ff ) = f + f
dt dt dt dt dt dt
d dg d f d dg d f
3. ( f ⋅ g ) = f ⋅ + ⋅ g 4. ( f × g) = f × + ×g
dt dt dt dt dt dt
d df dg dh
5. ( f ⋅ g × h ) = ⋅g×h + f ⋅ ×h + f ⋅g× .
dt dt dt dt

d f d f ds
Note  If f is a continuous function of a scalar s and s is a continuous function of t, then = .
dt ds dt
6. Let f (t ) be a vector function. f (t ) changes if its magnitude is changed or its direction is changed
or both magnitude and direction are changed. We shall find conditions under which a vector
function will remain constant in magnitude or in direction.
(i) Let f (t ) be a vector of constant length k.
2
Then f ⋅ f = f = k2
Differentiating w.r.to t, we get

df df df df
⋅f+f⋅ = 0  ⇒  2 f ⋅ = 0  ⇒  f ⋅ =0
dt dt dt dt
df df
\ = 0 or  = is ⊥ to f ⋅
dt dt
(ii) Let f (t ) be a vector function with constant direction and let a be the unit vector in that
direction
Then f (t ) = fa, where    f = f
df df da
\ = a+f .
dt dt dt
da
But a is a constant vector, since its direction is fixed and magnitude is 1. \  =0
dt
df df
\ = a
dt dt
df df df
Now f × = fa × a=f a × a = 0 ( a × a = 0)
dt dt dt
df df
\ = 0 or  is parallel to f .
dt dt

M03_Eng-Maths (Aditya) CH04.indd 3 7/23/2018 4:46:48 PM


4.4  n  Engineering Mathematics-III

4.3 Level Surfaces
Let f be a continuous scalar point function defined in a region R in space. Then the set of all points
satisfying the equation f (x, y, z) = C, where C is a constant, determines a surface which is called a
level surface of f. At every point on a level surface the function f takes the same value C. If C is an
arbitrary constant, the for different values of C, we get different level surfaces of f.
No two level surfaces intersect. For, if f = C1 and f = C2 be two level surfaces of f intersecting at a point P.
Then f(P) = C1 and f(P) = C2 and so f has two values at P which contradicts the uniqueness of value of the
function f. So, f  =  C1 and f = C2 do not intersect.
Thus, only one level surface of f passes through a given point
For example, if f (x, y, z) represents the temperature of (x, y, z) in a region R of space, then the
level surfaces of equal temperature are called isothermal surfaces.

4.4 Gradient of a Scalar Point Function oR Gradient of a Scalar Field


4.4.1  Vector Differential Operator
The symbolic vector is called Hamiltonian operator or vector differential
operator and is denoted by ∇ (read as del or nabla).

\ .

It is also known as del operator. This operator can be applied on a scalar point function f (x, y, z)
or a vector point function F( x, y, z ) which are differentiable functions. This gives rise to three field
quantities namely gradient of a scalar, divergence of a vector and curl of a vector function.

Definition 4.4  Gradient


If f (x, y, z) is a scalar point function continuously differentiable in a given region R of space, then the
∂f ∂f ∂f
gradient of f is defined by ∇f = i +j +k .
∂x ∂y ∂z
It is abbreviated as grad f. Thus, grad f = ∇f.

Note  Since ∇f is a vector, the gradient of a scalar point function is always a vector point function.
Thus, ∇f defines a vector field.
Gradient is of great practical importance because some of the vector fields in applications can be
obtained from scalar fields and scalar fields are easy to handle.

4.4.2  Geometrical Meaning of =f


Let f (x, y, z) be a scalar point function. Let f (x, y, z) = C be a level surface of f. Let P be a point
on this surface with position vector r = xi + yj + zk .
Then the differential d r = dx i + dy j + dz k is tangent to the surface at P.
 ∂f ∂f ∂f 
Now ∇f ⋅ d r =  i +j +k ⋅ ( dx i + dy j + dz k )
 ∂x ∂y ∂z 

M03_Eng-Maths (Aditya) CH04.indd 4 7/23/2018 4:46:50 PM


Vector Differentiation  n  4.5

∂f ∂f ∂f
= dx + dy + dz = df = 0  [{ f = C ]
∂x ∂y ∂z
\  ∇f is normal to the surface f (x, y, z) = C at P.
∇f
So, a unit normal to the surface at P is n =
∇f
∇f
There is another unit normal in the opposite direction = − .
∇f

4.4.3  Directional Derivative


The directional derivative of a scalar point function f in a given direction a is the rate of change of f
in that direction. It is given by the component of ∇f in the direction of a
a
\  the directional derivative = ∇f ⋅ .
a
a ∇f a
Since ∇f ⋅ = cos u, where u is the angle between ∇f and a.
a a
= ∇f cos u
So, the directional derivative at a given point is maximum if cos u is maximum.
i.e., cos u = 1⇒  u = 0.
\  the maximum directional derivative at a point is in the direction of ∇f and the maximum
directional derivative is ∇f .
Note
1. The directional derivative is minimum when cos u = -1 ⇒ u = p
\  the minimum directional derivative is − ∇f
2. In fact, the vector ∇f is in the direction in which f increases rapidly.
i.e., outward normal and -∇f points in the direction in which f decreases rapidly.

4.4.4 Equation of Tangent Plane and Normal to the Surface


(i)
Equation of tangent plane
Let A be a given point on the surface f(x, y, z) = C.
Let r0 = x0 i + y0 j + z0 k be the position vector of A.
Let P be any point on the tangent plane to the surface at the point A and let r = xi + yj + zk be the
position vector of P.
Then ∇f at A is normal to the surface and r − r0 lies on the tangent plane at A.
\  the equation of the tangent plane at the point A is ( r − r0 ). ∇f = 0

Note  The cartesian equation of the plane at the point A (x0, y0, z0) is
∂f ∂f ∂f
( x − x0 ) + ( y − y0 ) + ( z − z0 ) =0
∂x ∂y ∂z
where the partial derivatives are evaluated at the point (x0, y0, z0).

M03_Eng-Maths (Aditya) CH04.indd 5 7/23/2018 4:46:54 PM


4.6  n  Engineering Mathematics-III

(ii) Equation of the normal at the point A


Let A be a given point on the surface f (x, y, z) = C and let r0 = x0 i + y0 j + z0 k be the position
vector of A.
Let r be the position vector of any point P on the normal at the point A. Then r − r0 is parallel to
the normal at the point A.
\  the equation of the normal at the point A is ( r − r0 ) × ∇f = 0.
The cartesian equation of the normal at the point A is
x − x0 y − y0 z − z0
= = ,
∂f ∂f ∂f
∂x ∂y ∂z
where the partial derivatives are evaluated at (x0, y0, z0).

4.4.5 Angle between Two Surfaces at a Common Point


We know that the angle between two planes is the angle between their normals.
We define angle between two surfaces at a point of intersection P is the angle between their tangent
planes at P and hence, the angle between their normals at P.
The angle between two surfaces f(x, y, z) = C1 and g(x, y, z) = C2 at a common point P is the angle
between their normals at the point P.
The normal at P to the surface f(x, y, z) = C1 is ∇f.
The normal at P to the surface g(x, y, z) = C2 is ∇g.
∇f ⋅ ∇g
If u is the angle between the normals at the point P, then cos u =
∇f ∇g
p ∇f ⋅ ∇g
(i) If u = , then the normals are perpendicular and cos u = 0  ⇒  = 0   ⇒  ∇f ⋅ ∇g = 0
2 ∇f ∇g

\ if two surfaces are orthogonal at the point P then ∇f ⋅ ∇g = 0


p
Conversely, if ∇f ⋅ ∇g = 0, then u =       That is they are orthogonal.
2
(ii) If u = 0, the normals at the common point coincide.
\ the two tangent planes coincide and the surfaces touch at the common point.

4.4.6  Properties of gradients


If f and g are scalar point functions which are differentiable, then
1. ∇C = 0, where C is constant. 2. ∇(Cf   ) = C∇f, where C is a constant.
3. ∇(f ± g) = ∇f ± ∇g 4. ∇(fg) = f  ∇g + g∇f
 f  g ∇f − f ∇g
5. ∇   = if g ≠ 0
 g g2

1. ∇C = 0,  C is constant.
∂f ∂f ∂f
Proof  We know ∇f = i +j +k (1)
∂x ∂y ∂z

M03_Eng-Maths (Aditya) CH04.indd 6 7/23/2018 4:46:58 PM


Vector Differentiation  n  4.7

∂f
= ∑i
∂x
∂C  ∂C ∂C ∂C 
\ ∇C = ∑ i = 0  C is a constant ∂x = 0, ∂y = 0, ∂z = 0  n
∂x  
2. ∇Cf = C∇f
∂ ∂f
Proof  We have ∇Cf = ∑ i (Cf) = C∑ i = C∇f [ using (1)] n
∂x ∂x

3. ∇(f ± g) = ∇f ± ∇g

Proof  We have ∇( f ± g ) = ∑ i ( f ± g ) [ using (1)]
∂x
 ∂f ∂g  ∂f ∂g
= ∑ i ±i = ∑i ± ∑i = ∇f ± ∇g
 ∂x ∂x  ∂x ∂x
\ ∇( f ± g ) = ∇f ± ∇g  n

4.  ∇(fg) = f ∇g + g∇f



Proof  We have ∇( fg ) = ∑ i ( fg )
∂x
 ∂g ∂f 
= ∑i  f +g 
 ∂x ∂x 

 ∂g   ∂f 
= ∑i  f
 ∂x  ∑  ∂x 
+ i g

∂g ∂f
= f ∑i + g∑ i = f ∇g + g ∇f
∂x ∂x
\ ∇( fg ) = f ∇g + g ∇f  n

 f  g ∇f − f ∇ g
5. ∇   =
 g g2
 f ∂  f
Proof  We have ∇  = ∑i
 g ∂x  g 
 ∂f ∂g 
 g ∂x − f ∂x 
= ∑i  
 g2 
1  ∂f ∂g  g ∇f − f ∇g
= 2 g∑ i − f ∑i =
g  ∂x ∂x  g2

\  f  g ∇f − f ∇ g 
∇  = n
 g g2

M03_Eng-Maths (Aditya) CH04.indd 7 7/23/2018 4:47:02 PM


4.8  n  Engineering Mathematics-III

WORKED EXAMPLES
Example 1
Find grad f for the following functions.
(i)  f(x, y, z) 5 3x2y 2 y3z2 at the point (1, 22, 1)
(ii)  f(x, y, z) 5 log (x2 1 y2 1 z2) at the point (1, 2, 1).

Solution.
(i) Given f(x, y, z) = 3x2y - y3z2
∂f ∂f ∂f
We know grad f = ∇f = i +j +k
∂x ∂y ∂z
Differentiating f partially w.r. to x, y, z respectively, we get
∂f ∂f ∂f
= 6 xy, = 3x 2 − 3 y 2 z 2 , = −2 y 3 z
∂x ∂y ∂z

∂f
At the point (1, 22, 1), = 6 ⋅1( −2) = −12
∂x
∂f
= 3 ⋅12 − 3 ⋅ ( −2) 2 12 = 3 − 12 = −9
∂y
∂f
= −2 ⋅ ( −2)3 ⋅1 = 16
∂z
\  at the point (1, 22, 1),   ∇f = −12i − 9 j + 16 k .

(ii) Given f(x, y, z) = log (x2 + y2 + z2)


∂f ∂f ∂f
We know grad f = ∇f = i +j +k
∂x ∂y ∂z
Differentiating f partially w.r.to x, y, z respectively, we get,

∂f 1 ∂f 1 ∂f 1
= 2 ⋅ 2x , = 2 ⋅ 2y , = 2 ⋅ 2z
∂x x + y 2 + z 2 ∂y x + y 2 + z 2 ∂z x + y 2 + z 2

∂f 2 ⋅1 2 1
At the point (1, 2, 1), = 2 = =
∂x 1 + 2 + 1
2 2
6 3

∂f 2⋅ 2 4 2
= 2 = =
∂y 1 + 2 + 1
2 2
6 3
∂f 2 ⋅1 2 1
= 2 = =
∂z 1 + 2 + 1
2 2
6 3
1 2 1 1
\  at the point (1, 2, 1),     grad f = i + j + k = [i + 2 j + k ].
3 3 3 3

M03_Eng-Maths (Aditya) CH04.indd 8 7/23/2018 4:47:06 PM


Vector Differentiation  n  4.9

Example 2
Find the directional derivative of f (x, y, z) 5 x2yz 1 4xz2 at the point (1, 22, 21) in the direction
of the vector 2 i 2 j 2 2 k .

Solution.
Given f (x, y, z) = x2yz + 4xz2
∂f ∂f ∂f
We know grad f = ∇f = i +j +k
∂x ∂y ∂z
Differentiating f partially w.r.to x, y, z respectively, we get
∂f ∂f ∂f
= 2xyz + 4 z 2 , = x 2z , = x 2 y + 8xz
∂x ∂y ∂z
∂f
At the point (1, 22, 21), = 2 ⋅1( −2)( −1) + 4( −1) 2 = 8
∂x
∂f 2
= 1 ⋅ ( −1) = −1
∂y
∂f 2
= 1 ( −2) + 8 ⋅1( −1) = −2 − 8 = −10
∂z
\  at the point (1, 22, 21), ∇f = 8i − j − 10 k

Given direction is a = 2i − j − 2k
\  the directional derivative of f at the point (1, -2, -1) in the direction of a is

a (2i − j − 2k ) 16 + 1 + 20 37
= ∇f ⋅ = (8i − j − 10 k ) ⋅ = =
a 4 +1+ 4 9 3

Example 3
r
If r 1 xi 1 yj 1 zk and r 5 r prove that (i) ∇r 5 , (ii) ∇r n 5 nr n22 r , 
r
 1 r r
(iii) ∇   52 3  (iv) ∇(log r ) 5 2 .
r r r
Solution.
Given r = xi + yj + zk   and  r = r = x 2 + y 2 + z 2   ⇒  r 2 = x 2 + y 2 + z 2 (1)
r
(i) ∇r =
r
∂r ∂r ∂r
We know ∇r = i +j +k
∂x ∂y ∂z
Differentiating (1) partially w.r.to x, we get
∂r ∂r x
2r = 2 x   ⇒  =
∂x ∂x r

M03_Eng-Maths (Aditya) CH04.indd 9 7/23/2018 4:47:11 PM


4.10  n  Engineering Mathematics-III

∂r y ∂r z
Similarly, = and =
∂y r ∂z r
x y z 1 r
\ ∇r = i + j + k = [ xi + yj + zk ] =
r r r r r
(ii) ∇r n = nr n − 2 r

We know ∂ n ∂ ∂
∇r n = i (r ) + j (r n ) + k (r n )
∂x ∂y ∂z
 ∂r   ∂r   ∂r 
= i  nr n −1  + j  nr n −1  + k  nr n −1 
 ∂x   ∂y   ∂z 
x y z  n r n −1
= n r n −1  i + j + k  = [ xi + yj + zk ] = nr n − 2 r
r r r  r

(iii)  ∇   = − 3
1 r
 r r
 1 ∂  1 ∂  1 ∂  1
We know, ∇  = i   + j   + k  
 r ∂x r ∂y r ∂z r
 1 ∂r   1 ∂r   1 ∂r 
= i − 2  + j − 2  + k − 2 
 r ∂x   r ∂y   r ∂z 
1 x y z  1 r
=− i + j + k  = − 3 ( xi + yj + zk ) = − 3
r 2  r r r  r r

r
(iv) ∇(log r ) =
r2
∂ ∂ ∂
We know, ∇ (log r) = i (log r ) + j (log r ) + k (log r )
∂x ∂y ∂z
 1 ∂ r   1 ∂ r   1 ∂ r  1x y z  r
=i  + j  +k  =  i + j + k = 2
 r ∂x   r ∂y   r ∂z  r  r r r  r

Example 4
Find the directional derivative of the function 2yz 1 z2 in the direction of the vector i 1 2 j 1 2 k
at the point (1, -1, 3).

Solution.
Given f = 2yz + z2
∂f ∂f ∂f
We know ∇f = i +j +k
∂x ∂y ∂z
Differentiating f partially w.r.to x, y, z respectively, we get

∂f ∂f ∂f
= 0,   = 2 z,   = 2 y + 2z
∂x ∂y ∂z

M03_Eng-Maths (Aditya) CH04.indd 10 7/23/2018 4:47:16 PM


Vector Differentiation  n  4.11

∂f ∂f ∂f
At the point (1, 21, 3), = 0,   = 2(3) = 6 ,  = 2( −1) + 2 ⋅ 3 = 4
∂x ∂y ∂z
\  at the point (1, 21, 3), ∇f = 6 j + 4 k

Given direction is a = i + 2 j + 2k
\  the directional derivative of f at the point (1, -1, 3) in the direction of a is
a (i + 2 j + 2k ) 12 + 8 20
= ∇f ⋅ = (6 j + 4 k ) ⋅ = =
a 1+ 4 + 4 9 3

Example 5
Find the directional derivative of x3 1 y3 1 z3 at the point (1, 21, 2) in the direction of i 1 2 j 1 k .

Solution.
Given f (x, y, z) = x3 + y3 + z3
∂f ∂f ∂f
We know ∇f = i +j +k
∂x ∂y ∂z
Now differentiating f partially w.r.to x, y, z respectively, we get

∂f ∂f ∂f
= 3x 2 ,  = 3y2 ,  = 3z 2
∂x ∂y ∂z

∂f ∂f ∂f
At the point (1, -1, 2), = 3 ⋅12 = 3,  = 3( −1) 2 = 3, = 3 ⋅ 22 = 12
∂x ∂y ∂z

\  at the point (1, 21, 2), ∇f = 3i + 3 j + 12k


Given direction is a = i +2j +k
\  the directional derivative of f at the point (1, -1, 2) in the direction of a is
a (i + 2 j + k ) 3 + 6 + 12 21 6 7 6
= ∇f ⋅ = (3i + 3 j + 12k ) ⋅ = = = 21 =
a 1+ 4 +1 6 6 6 2
Example 6
Find a unit normal vector to the surface x3 1 y3 1 3xyz 5 3 at the point (1, 2, -1).

Solution.
The given surface is x3 + y3 + 3xyz = 3, which is taken as f = C
\ f = x3 + y3 + 3xyz
We know that ∇f is normal to the surface.
∇f
So, unit normal to the surface is n=
∇f

M03_Eng-Maths (Aditya) CH04.indd 11 7/23/2018 4:47:21 PM


4.12  n  Engineering Mathematics-III

∂f ∂f ∂f
Now ∇f = i +j +k
∂x ∂y ∂z
Differentiating f partially w.r.to x, y, z respectively,
∂f ∂f ∂f
we get, = 3 x 2 + 3 yz , = 3 y 2 + 3 xz , = 3 xy
∂x ∂y ∂z
∂f
At the point (1, 2, 21), = 3 ⋅12 + 3 ⋅ 2( −1) = −3
∂x
∂f ∂f
= 3 ⋅ 22 + 3 ⋅1( −1) = 9 and = 3 ⋅1⋅ 2 = 6
∂y ∂z
\  at the point (1, 2, 21),         ∇f = −3i + 9 j + 6 k
\  unit normal to the given surface at the point (1, 2, -1) is
−3i + 9 j + 6 k −3i + 9 j + 6 k
n= =
9 + 81 + 36 126
Note  If the surface equation is written as x + y + 3xyz - 3 = 0, then we take
3 3

f (x, y, z) = x3 + y3 + 3xyz - 3. Here C = 0.

Example 7
Find a unit normal to the surface x2y 1 2xz2 5 8 at the point (1, 0, 2).
Solution.
Given f (x, y, z) = x2y + 2xz2
∂f ∂f ∂f
We know, ∇f = i +j +k
∂x ∂y ∂z
Differentiating f partially w.r.to x, y, z respectively, we get
∂f ∂f ∂f
= 2 xy + 2 z 2 , = x2 , = 4 xz
∂x ∂y ∂z
∂f ∂f 2 ∂f
At the point (1, 0, 2),  = 2 ⋅1⋅ 0 + 2 ⋅ 22 = 8, = 1 = 1,  = 4 ⋅1⋅ 2 = 8
∂x ∂y ∂z
\  at the point (1, 0, 2),          ∇f = 8i + j + 8k
\  unit normal vector to the given surface at the point (1, 0, 2) is

∇f 8i + j + 8k 8i + j + 8k
n= = =
∇f 64 + 1 + 64 129
Example 8
Find the maximum value of the directional derivative of f 5 x3yz at the point (1, 4, 1).
Solution.
Given f = x3yz
∂f ∂f ∂f
We know, ∇f = i +j +k
∂x ∂y ∂z

M03_Eng-Maths (Aditya) CH04.indd 12 7/23/2018 4:47:24 PM


Vector Differentiation  n  4.13

The directional derivative is maximum in the direction of ∇f and the maximum value = ∇f
Differentiating f partially w.r.to x, y, z respectively, we get

∂f ∂f ∂f
= 3 x 2 yz , = x 3 z, = x3 y
∂x ∂y ∂z

∂f ∂f 3 ∂f 3
At the point (1, 4, 1), = 3 ⋅1⋅ 4 ⋅1 = 12, = 1 ⋅1 = 1 and = 1 ⋅4 = 4
∂x ∂y ∂z

\  at the point (1, 4, 1), ∇f = 12i + j + 4 k

Maximum value of the directional derivative = ∇f = 12i + j + 4 k = 144 + 1 + 16 = 161

Example 9
In what direction from the point (1, 1, 22), is the directional derivative of f 5 x2 2 2y2 1 4z2
maximum? Also find the maximum directional derivative.

Solution.
Given f = x2 - 2y2 + 4z2
We know that the directional derivative is maximum in the direction of ∇f. The maximum value
= ∇f
∂f ∂f ∂f
We have ∇f = i +j +k
∂x ∂y ∂z
Differentiating f partially w.r.to x, y, z respectively, we get
∂f ∂f ∂f
= 2 x, = −4 y, = 8z
∂x ∂y ∂z

∂f ∂f ∂f
At the point (1, 1, 22), = 2 ⋅1 = 2,  = −4 ⋅1 = −4, = 8( −2) = −16
∂x ∂y ∂z

\  at the point (1, 1, 22),       ∇f = 2i − 4 j − 16 k = 2[i − 2 j − 8k ]


\  the directional derivative is maximum in the direction of 2(i − 2 j − 8k )

Maximum value = ∇f = 2(i − 2 j − 8k ) = 2 1 + 4 + 64 = 2 69

Example 10
Find the angle between the surfaces x2 1 y2 1 z2 5 9 and x2 1 y2 2 z 5 3 at the point (2, 21, 2).

Solution.
The given surfaces are
x2 + y2 + z2 = 9       (1)      and       x2 + y2 - z = 3 (2)
P(2, -1, 2) is a common point of (1) and (2)
Let f = x2 + y2 + z2 and g = x2 + y2 - z

M03_Eng-Maths (Aditya) CH04.indd 13 7/23/2018 4:47:28 PM


4.14  n  Engineering Mathematics-III

∂f ∂f ∂f
Now, ∇f = i +j +k
∂x ∂y ∂z
Differentiating f partially w.r.to x, y, z respectively we get,
∂f ∂f ∂f
= 2 x, = 2 y, = 2z
∂x ∂y ∂z
∂f ∂f ∂f
At the point (2, 21, 2), = 2 ⋅ 2 = 4, = 2( −1) = −2, = 2( +2) = +4
∂x ∂y ∂z
\  at the point (2, 21, 2), ∇f = 4 i − 2 j + 4 k
∂g ∂g ∂g
Now ∇g = i +j +k
∂x ∂y ∂z
Differentiating g partially w.r.to x, y, z respectively, we get
∂g ∂g ∂g
= 2 x, = 2 y, = −1
∂x ∂y ∂z
∂g ∂g ∂g
at the point (2, 21, 2), = 2 ⋅ 2 = 4, = 2( −1) = −2, = −1
∂x ∂y ∂z
\  at the point (2, 21, 2), ∇g = 4 i − 2 j − k
If u is the angle between the surfaces (1) and (2) at (2, -1, 2), then
∇f ⋅ ∇g (4i − 2 j + 4 k ) (4i − 2 j − k ) 16 + 4 − 4 16 8
cos u = = ⋅ = = =
∇f ∇g 16 + 4 + 16 16 + 4 + 1 36 21 6 21 3 21
 8 
\ u = cos −1 
 3 21 

Example 11
Show that the surfaces 5x2 2 2yz 2 9x 5 0 and 4x2y 1 z3 2 4 5 0 are orthogonal at the point
(1, 21, 2).

Solution.
The given surfaces are
5x2 - 2yz - 9x = 0    (1)  and 4x2y + z3 - 4 = 0 (2)
Let f = 5x2 - 2yz - 9x   and   g = 4x2y + z3 - 4
To prove (1) and (2) cut orthogonally at the point (1, -1, 2),
i.e., to prove ∇f⋅∇g = 0
∂f ∂f ∂f
Now ∇f = i +j +k
∂x ∂y ∂z
∂f ∂f ∂f
= 10 x − 9, = −2 z and = −2 y
∂x ∂y ∂z
\ ∇f = (10 x − 9)i − 2 zj − 2 yk

M03_Eng-Maths (Aditya) CH04.indd 14 7/23/2018 4:47:31 PM


Vector Differentiation  n  4.15

∂g ∂g ∂g
and ∇g = i +j +k
∂x ∂y ∂z
∂g ∂g ∂g
= 8 xy, = 4 x 2 and = 3z 2
∂x ∂y ∂z
\ ∇g = 8 xyi + 4 x 2 j + 3 z 2 k
At the point (1, 21, 2), ∇f = (10 − 9)i − 2 ⋅ 2 j − 2( −1)∇kf = i − 4 j + 2k
and ∇g = 8 ⋅1⋅ ( −1)i + 4 ⋅12 j + 3 ⋅ 2∇ 2
kg = −8i + 4 j + 12k
\ ∇ f ⋅ ∇ g = ( i − 4 j + 2 k ) ⋅ ( −8i + 4 j + 12k ) = −8 − 16 + 24 = 0
Hence, the two surfaces cut orthogonally at the point (1, -1, 2).

Example 12
Find a and b if the surfaces ax2 2 byz 5 (a 1 2)x and 4x2y 1 z3 5 4 cut orthogonally at the point
(1, 21, 2).

Solution.
The given surfaces are
ax2 - byz - (a + 2)x = 0     (1)   and  4x2y + z3 - 4 = 0 (2)
Let f = ax - byz - (a + 2)x  and 
2
   g = 4x2y + z3 - 4
Given the surfaces (1) and (2) cut orthogonally at the point (1, -1, 2).
\ ∇f ⋅ ∇g = 0 (3)

Now ∂f ∂f ∂f
∇f = i +j +k
∂x ∂y ∂z
∂f ∂f ∂f
= 2ax − a − 2, = − bz and = −by
∂x ∂y ∂z
\ ∇f = (2ax − a − 2)i − bzj − byk
and ∂g ∂g ∂g
∇g = i +j +k
∂x ∂y ∂z
∂g ∂g ∂g
= 8 xy, = 4 x 2 and = 3z 2
∂x ∂y ∂z
\ ∇g = 8 xyi + 4 x 2 j + 3 z 2 k
At the point (1, 21, 2), ∇f = (2a − a − 2)i − b ⋅ 2 j − b( −1)k
⇒ ∇f = ( a − 2)i − 2bj + bk
and ∇g = −8i + 4 j + 12k
\ ∇f ⋅ ∇g = (( a − 2)i − 2bj + bk ) ⋅ ( −8i + 4 j + 12k )
= −8(a − 2) − 8b + 12b = −8a + 4b + 16
From (3), ∇f⋅∇g = 0  ⇒  -8a + 4b + 16 = 0  ⇒ 2a - b = 4 (4)
Since (1, -1, 2) is a point on the surface f = 0, we get
a + 2b − ( a + 2) = 0  ⇒  2b = 2   ⇒  b = 1

M03_Eng-Maths (Aditya) CH04.indd 15 7/23/2018 4:47:37 PM


4.16  n  Engineering Mathematics-III

5
\  (4) ⇒ 2a = 4 + b = 4 + 1 = 5   ⇒  a =
2
5
\ a= , b =1
2
Example 13 
Find the angle between the normals to the surface xy = z2 at the points (1, 4, 2) and (-3, -3, 3).

Solution.
The given surface is xy - z2 = 0
\ f = xy - z2
We know ∇f is normal to the surface at the point (x, y, z)
Let n1, n 2, be the normals to the surface at the points (1, 4, 2) and (-3, -3, 3) respectively.
\ n1 = ∇f at the point (1, 4, 2)
and n 2 = ∇f at the point ( −3, − 3, 3)
∂f ∂f ∂f
Now ∇f = i +j +k
∂x ∂y ∂z
∂f ∂f ∂f
= y, = x and = −2z
∂x ∂y ∂z
\ ∇f = yi + xj − 2 zk
At the point (1, 4, 2),        ∇f = 4i + j − 4 k \ n1 = 4i + j − 4 k
At the point (23, 23, 3), ∇f = −3i − 3 j − 6 k \ n2 = −3i − 3 j − 6 k
If u is the angle between the normals, then
n ⋅n (4i + j − 4k ) ⋅ ( −3i − 3 j − 6 k )
cos u = 1 2 =
n1 n2 16 + 1 + 16 9 + 9 + 36
−12 − 3 + 24 9 1
= = =
33 54 33 54 22
 1 
\ u = cos −1 
 22 

Example 14
Find the directional derivative of the function f 5 xy2 1 yz3 at the point (2, 21, 1) in the direction
of the normal to the surface xlog z 2 y2 1 4 5 0 at the point (21, 2, 1).

Solution.
Given f = xy2 + yz3
∂f ∂f ∂f
\ ∇f = i +j +k = y 2 i + (2 xy + z 3 ) j + 3 yz 2 k
∂x ∂y ∂z
At the point (2, 21, 1), ∇f = ( −1) 2 i + ( −4 + 1) j + 3( −1)1∇2 f
k = i − 3 j − 3k
The directional derivative of f in the direction of the normal to the surface xlogz - y2 + 4 = 0 at the
point (-1, 2, 1) is required.

M03_Eng-Maths (Aditya) CH04.indd 16 7/23/2018 4:47:42 PM


Vector Differentiation  n  4.17

Let f = xlogz - y2 + 4
∂f ∂f ∂f x
\ ∇f = i +j +k = log zi − 2 yj + k
∂x ∂y ∂z z
 −1
At the point (21, 2, 1), ∇f = log1i − 4 j +   k = 0i − 4 j − k = −4 j − k
 1
\ a = −4 j − k
a
Required directional derivative is = ∇f ⋅
a
( −4 j − k ) 12 + 3 15
= (i − 3 j − 3k ) ⋅ = =
16 + 1 17 17

Example 15
If ∇f 5 2 xyz 3 i 1 x 2 z 3 j 1 3x 2 yz 2 k , then find f if f (1, 22, 2) 5 4.

Solution.
Given ∇f = 2 xyz 3 i + x 2 z 3 j + 3 x 2 yz 2 k (1)
∂f ∂f ∂f
But ∇f = i +j +k (2)
∂x ∂y ∂z
Equating the coefficients of i , j , k , from (1) and (2), we get
∂f ∂f ∂f
= 2 xyz 3   (3) = x 2 z 3   (4) = 3 x 2 yz 2 (5)
∂x ∂y ∂z
Integrating (3) partially w.r.to x, we get
f = x 2 yz 3 + f1 ( y, z )(6)
Integrating (4) partially w.r.to y, we get,
f = x 2 z 3 y + f 2 ( x, z )(7)
Integrating (5) partially w.r.to z, we get,
f = x 2 yz 3 + f 3 ( x, y ) (8)
From (6), (7), (8), f is obtained by adding all the terms and an arbitrary constant C, but omitting
f1(y, z), f2(x, z), f3(x, y) and choosing only one of the repeated terms.
Thus, f = x2yz3 + C
Given f (1, -2, 2) = 4
\ 1 × (-2) × 8 + C = 4  ⇒  C = 4 + 16 = 20
\ f = x2yz3 + 20

Example 16
Find the equation of the tangent plane and the equation of the normal to the surface
x2 2 4y2 1 3z2 1 4 5 0 at the point (3, 2, 1).

Solution.
The given surface is x2 - 4y2 + 3z2 + 4 = 0

M03_Eng-Maths (Aditya) CH04.indd 17 7/23/2018 4:47:47 PM


4.18  n  Engineering Mathematics-III

Let f = x2 - 4y2 + 3z2 + 4


∂f ∂f ∂f
\ ∇f = i +j +k = 2 xi − 8 yj + 6 zk
∂x ∂y ∂z
At the point (3, 2, 1), ∇f = 6i − 16 j + 6 k
We know that the equation of the tangent plane at the point (x0, y0, z0) is

∂f ∂f ∂f
( x − x0 ) + ( y − y0 ) + ( z − z0 ) =0
∂x ∂y ∂z

∂f ∂f ∂f
Now = 2 x, = −8 y and = 6z
∂x ∂y ∂z

∂f ∂f ∂f
Here (x0, y0, z0) = (3, 2, 1)  \  = 6, = −16 and =6
∂x ∂y ∂z
\  the equation of the tangent plane at the point (3, 2, 1) is
( x − 3)6 + ( y − 2)( −16) + ( z − 1)6 = 0
⇒ 3( x − 3) − 8( y − 2) + 3( z − 1) = 0  [dividing by 2]
⇒ 3 x − 8 y + 3 z − 9 + 16 − 3 = 0
⇒ 3x − 8 y + 3z + 4 = 0
The equation of the normal at the point (x0, y0, z0) is
x − x0 y − y0 z − z0
= =
∂f ∂f ∂f
∂x ∂y ∂z
The equation of the normal at the point (3, 2, 1) is
x − 3 y − 2 z −1 x − 3 y − 2 z −1
= =   ⇒  = = .
6 −16 6 3 −8 3
Example 17
If the directional derivative of
f(x, y, z) 5 a(x 1 y) 1 b(y 1 z) 1 c(z 1 x) has maximum value 12 at the point (1, 2, 1) in the
x 21 y 2 2 z 21
direction parallel to the line 5 5 , find the value of a, b, c.
1 2 3
Solution.
Given f = a(x + y) + b(y + z) + c(z + x)

\ ∂f ∂f ∂f
∇f = i +j +k
∂x ∂y ∂z
⇒ ∇f = ( a + c )i + ( a + b ) j + ( b + c ) k

M03_Eng-Maths (Aditya) CH04.indd 18 7/23/2018 4:47:51 PM


Vector Differentiation  n  4.19

We know that the directional derivative is maximum in the direction of ∇f.


x −1 y − 2 z −1
But given it is maximum in the direction parallel to the line = = .
1 2 3
a+c a+b b+c
\ = = =K
1 2 3
⇒ a + c = K   (1) a + b = 2 K   (2) b + c = 3K (3)
Adding we get,
a + c + a + b + b + c = K + 2K + 3K

⇒ 2( a + b + c) = 6 K ⇒  a + b + c = 3K (4)

Using (3), (4)  ⇒ a + 3K = 3K ⇒ a=0

From (1), 0 + c = K ⇒ c=K

From (2), 0 + b = 2K ⇒ b = 2K

Given the maximum value of directional derivative = 12

⇒ ∇f = 12

⇒ ( a + c) 2 + ( a + b) 2 + (b + c) 2 = 12

⇒ ( a + c) 2 + ( a + b) 2 + (b + c) 2 = 144

⇒ K 2 + 4 K 2 + 9 K 2 = 144

144 12
⇒ 14 K 2 = 144  ⇒  K 2 =   ⇒  K = ±
14 14
24 12
\ a = 0, b = ± ,c=±
14 14
Example 18
If u 5 x 1 y 1 z , v 5 x 2 1 y 2 1 z 2 , w 5 xy 1 yz 1 zx , then show that the vectors ∇u, ∇v, ∇w
are coplanar.

Solution.
Given u = x + y + z,  v = x2 + y2 + z2,  w = xy + yz + zx
∂u ∂u ∂u
Now, ∇u = i +j + k ∇u = i + j + k
∂x ∂y ∂z
∂v ∂v ∂∇vv = 2 xi + 2 yj + 2 zk
∇v = i +j +k
∂x ∂y ∂z
∂w ∂w ∂w
∇w = i +j + k ∇w = ( y + z ) i + ( z + x ) j + ( x + y ) k
∂x ∂y ∂z

M03_Eng-Maths (Aditya) CH04.indd 19 7/23/2018 4:47:56 PM


4.20  n  Engineering Mathematics-III

We know that three vectors a , b , c are coplanar, if their scalar triple product a ⋅ b × c = 0.
\  ∇u, ∇v, ∇w are coplanar, if ∇u ⋅ ∇v × ∇w = 0
1 1 1
Now  ∇u ⋅ ∇v × ∇w = 2x 2y 2z
y+z z+x x+ y
1 1 1
=2 x y z
y +z z +x x+y
1 1 1
= 2 x +y +z x +y +z x +y +z R2 → R2 + R3
y +z z +x x+y
1 1 1
= 2( x + y + z ) 1 1 1 =0  [since R1 = R2 ]
y +z z +x x+y

\  the vectors ∇u, ∇v, ∇w are coplanar.

EXERCISE 4.1
1. If f(x, y, z) = 3xz2y - y3z2, find ∇f at the point (1, -2, -1).
2. If f = 2xz - y2 find grad f at the point (1, 3, 2).
3. Find the directional derivative of f = 3x2 + 2y - 3z at the point (1, 1, 1) in the direction of
2i + 2 j − k .
4. Find the directional derivative of xyz - xy2z2 at the point (1, 2, -1) in the direction of the vector
i − j − 3k .
5. Find the directional derivative of the function f = x2 - y2 + 2z2 at the point P (1, 2, 3) in the
direction of the line PQ where Q = (5, 0, 4).
6. Find the unit normal vector to the surface
(i)  x2 + 2y2 + z2 = 7 at the point (1, -1, 2). (ii) x2 + y2 - z2 = 1 at the point (1, 1, 1).
(iii) x + y - z = 1 at the point (1, 1, 1).
2 2
(iv) x2 + y2 = z at the point (1, 2, 5).
7. Find the angle between the surfaces x2 + y + z = 2 and xlogz = y2 - 1 at the point (1, 1, 1).
8. Find the angle between the surfaces 2yz + z2 = 3 and x2 + y2 + z2 = 3 at the point (1, 1, 1).
9. Find the angle between the surfaces xyz = 4 and x2 + y2 + z2 = 9 at the point i + 2 j + 2k .
10. Find the equation of the tangent plane and normal line to the surface xz2 + x2y - z + 1 = 0 at the
point (1, -3, 2).
11. Find the equation of the tangent plane and normal line to the surface 2xz2 - 3xy - 4x = 7 at the
point (1, -1, 2).
12. Find the equation of the tangent plane and normal line to the surface 2z - x2 = 0 at the point P(2, 0, 2).
13. Find f if
(i)  ∇f = ( y 2 − 2 xyz 3 )i + (3 + 2 xy − x 2 z 3 ) j + (8 z 3 − 3 x 2 yz 2 )k

M03_Eng-Maths (Aditya) CH04.indd 20 7/23/2018 4:47:59 PM


Vector Differentiation  n  4.21

(ii)  ∇f = 2 xyz 3 i + x 2 z 3 j + 3 x 2 yz 2 k if f(1, − 2, 2) = 4


(iii) ∇f = (6 xy + z 3 )i + (3 x 2 − z ) j + (3 xz 2 − y )k
(iv) ∇f = (2 xyz + x )i + x 2 zj + x 2 yk
(v)  ∇f = ( y + sin z )i + xj + x cos zk .
14. Find the angle between the normals to the intersecting surfaces xy - z2 - 1 = 0 and y2 - 3z - 1 = 0
at the point (1, 1, 0).
15. Find the angle between the normals to the surface x2 = yz at the points (1, 1, 1) and (2, 4, 1).
16. Find the values of a and b so that the surfaces ax3 - by2z = (a + 3)x2 and 4x2y - z3 = 11 may cut
orthogonally at the point (2, -1, -3).
17. The temperature at any point in space is given by T = xy + yz + zx. Find the direction in which the
temperature changes most rapidly from the point (1, 1, 1) and determine the maximum rate of
change.
18. In what direction is the directional derivative of the function f = x2 - 2y2 + 4z2 from the point
(1, 1, -1) is maximum and what is its value?
19. Find the maximum value of the directional derivative of the function f = 2x2 + 3y2 + 5z2 at the
point (1, 1, -4).
20. Find ∇f at the point (1, 1, 1) if f(x, y, z) = x2y + y2x + z2.
21. Find the directional derivative of f(x, y, z) = x2 - 2y2 + 4z2 at the point (1, 1, -1) in the direction
2i − j − k .
22. Find the directional derivative of the function f = xy + yz + zx in the direction of the vector
2i + 3 j + 6 k at the point (3, 1, 2).
23. Find the directional derivative of f = x2yz + 4xz2+ xyz at (1, 2, 3) in the direction of 2i + j − k .
24. Find the directional derivative of f = x2yz + 4xz2 at the point P(1, -2, -1) in the direction of PQ,
where Q is (3, -3, -3).
25. Find a unit normal to the surface xy3z2 = 4 at the point (-1, -1, 2).
26. In what direction from (3, 1, -2) is the directional derivative of f = x2y2z4 maximum? Find also
the magnitude of this maximum.
27. What is the greatest rate of increase of f = xyz2 at the point (1, 0, 3)?
28. Find the angle between the spheres x2 + y2 + z2 = 29 and
x2 + y2 + z2 + 4x - 6y - 8z - 47 = 0 at the point (4, -3, 2).
29. Find f if ∇f = (6 xy + z 3 )i + (3 x 2 − z ) j + (3 xz 2 − y )k .

ANSWERS TO EXERCISE 4.1


19 29 28
1. −6i − 9 j − 4 k 2. 4i − 6 j + 2k 3.  4.  5. 
3 11 21
i − 2 j + 2k 2i + 2 j + k
6. (i)  (ii) i + j − k  (iii)   (iv) 2i + 4 j − 5k
3 3 3 3 5
 1  3 2
7. cos −1  8. cos −1 cos −1
9. 
 30  5 3

M03_Eng-Maths (Aditya) CH04.indd 21 7/23/2018 4:48:04 PM


4.22  n  Engineering Mathematics-III

x −1 y + 3 z − 2
10. 2x - y - 3z + 1 = 0, = =
−2 1 3
x −1 y +1 z − 2 x−2 y z−2
11. 7x - 3y + 8z - 26 = 0, = = 12. 2x - z = 2; = =
7 −3 3 −2 0 1
13.   (i)  f = xy2 - x2yz3 + 3y + 2z4 + c (ii) f = x2yz3 + 20
x2
(iii) f = 3x2y + xz3 - yz + c     (iv)  f = x 2 yz + + c   (v)  f = xy + x sin z + c
2
 2  13 7 64
14. cos −1   cos −1
15.  16.  a = − ,b =
 26  3 22 ∇f3 9
n=
17. i + j + k , 2 3 18. 
2i − 4 j − 8k , 2 21 ∇f
19. 1652 20. ∇f = 3i + 3 j + 2k

45 86 −4(i + 3 j − k ) (i + 3 j − k )
21.  16 22.  23.  =37 25. 
24.  =−
6 7 6 3 4 1+ 9 +1 11

 19 
26.  96 19 27. 9 28.  u = cos −1  29.  f = 3x2y + xz3 - yz + c
 29 

4.5 Divergence of a Vector Point Function or Divergence of a Vector Field


Definition 4.5  If F( x, y, z ) be a vector point function continuously differentiable in a region R of
space, then the divergence of F is defined by
∂F ∂F ∂F
∇⋅F = i ⋅ + j ⋅ + k ⋅
∂x ∂y ∂z
It is abbreviated as div F and thus, div F = ∇⋅ F
∂F ∂F ∂F
If F = F1i + F2 j + F3 k , then ∇⋅F = 1 + 2 + 3
∂x ∂y ∂z
If is a constant vector, then =?F 5 0 and conversely if = ? F 5 0, then F is a constant vector.
F

Note  (i)  From the definition it is clear that div F is a scalar point function. So, the divergence of a
vector field is a scalar point function. The notation ∇ ⋅ F is not a scalar product in the usual sense, since
∂ ∂ ∂
∇ ⋅ F ≠ F ⋅ ∇. In fact F ⋅ ∇ = F1 + F2 + F3 is a scalar operator.
∂x ∂y ∂z

4.5.1  Physical Interpretation of divergence


Physical interpretation of divergence applied to a vector field is that it gives approximately the ‘loss’
of the physical quantity at a given point per unit volume per unit time.
(i) If v ( x, y, z ) is the moving fluid at a point (x, y, z), then the ‘loss’ of the fluid per unit volume per
unit time at the point is given by div v . Thus, divergence gives a measure of the outward flux per
unit volume of the flow at (x, y, z).
If there is no ‘loss’ of fluid anywhere, then div v = 0 and the fluid is said to be incompressible.

M03_Eng-Maths (Aditya) CH04.indd 22 7/23/2018 4:48:13 PM


Vector Differentiation  n  4.23

(ii) If v represents an electric flux, div v is the amount of electric flux which diverges per unit volume
in unit time.
(iii) If v represents the heat flux, div v is the rate at which heat is issuing from a point per unit volume.
Definition 4.6  Solenoidal vector 
If div F = 0 everywhere in a region R, then F is called a solenoidal vector point function and R is
called a solenoidal field.

4.6 
Curl of a Vector Point Function or Curl of a Vector Field
Definition 4.7  If F( x, y, z ) be a vector point function continuously differentiable in a region R, then
the curl of F is defined by

It is abbreviated as curl F
Thus, curl
If F = F1i + F2 j + F3 k , then
curl F = ∇ × F
 ∂ ∂ ∂
= i +j + k  × (F1i + F2 j + F3 k )
 ∂x ∂y ∂z 
 ∂F ∂F   ∂F ∂F   ∂F ∂F 
= i  3 − 2+ j  1 − 3+k  2 − 1
 ∂y ∂z   ∂z ∂x   ∂x ∂y 
i j k
∂ ∂ ∂
This is symbolically written as ∇ × F =
∂x ∂y ∂z
F1 F2 F3
If F is a constant vector, then curl  F = 0
4.6.1  Physical Meaning of Curl F
If F represents the linear velocity of the point P of a rigid body that rotates about a fixed axis (e.g., top)
with constant angular velocity v, then curl F at P is equal to 2v.
If the body is not rotating, then v = 0    \ Curl F = 0
Definition 4.8  Irrotational vector field
Let F( x, y, z ) be a vector point function. If curl F = 0 at all points in a region R, then F is said to be an
irrotational vector in R. The vector field R is called an irrotational vector field.

Definition 4.9  Conservative vector field


A vector field F is said to be conservative if there exists a scalar function f such that F = ∇f

Note
1. In a conservative vector field F = ∇f
∴ ∇ × F = ∇ × ∇f = 0 ⇒ F is irrotational.
2. This scalar function f is called the scalar potential of F.
Only irrotational vectors will have scalar potential f.

M03_Eng-Maths (Aditya) CH04.indd 23 7/23/2018 4:48:21 PM


4.24  n  Engineering Mathematics-III

WORKED EXAMPLES
Example 1
Prove that ∇ 3 ∇f 5 0, where f is a scalar point function.

Solution.
∂ ∂ ∂ ∂f ∂f ∂f
We have ∇=i + j + k ,  ∇f = i +j +k
∂x ∂y ∂z ∂x ∂y ∂z
i j k
∂ ∂ ∂
\ ∇ × ∇f =
∂x ∂y ∂z
∂f ∂f ∂f
∂x ∂y ∂z
 ∂ 2f ∂ 2f   ∂ 2f ∂ 2f   ∂ 2f ∂ 2f 
= i − − j −  + k  − 
 ∂y ∂z ∂z ∂y   ∂x ∂z ∂z ∂x   ∂x ∂y ∂y ∂x 
=0  ∂ 2f ∂ 2f ∂ 2f ∂ 2f ∂ 2f ∂ 2f 
= 0  Assuming = , = , = 
 ∂y ∂z ∂z ∂y ∂z ∂x ∂x ∂z ∂x ∂y ∂y ∂x 
\  ∇f is always an irrotational vector.

Example 2
Find the divergence and curl of the vector v 5 xyz i 1 3x 2 y j 1 ( xz 2 2 y 2 z ) k at the point
(2, -1, 1).

Solution.
Given v = xyzi + 3 x 2 y j + ( xz 2 − y 2 z )k
∂ ∂ ∂
\ div v = ∇ ⋅ v = ( xyz ) + (3 x 2 y ) + ( xz 2 − y 2 z )
∂x ∂y ∂z
= yz + 3 x 2 + 2 xz − y 2
At the point (2, 21, 1), ∇ ⋅ v = ( −1) ⋅1 + 3 ⋅ 4 + 2 ⋅ 2 ⋅1 − ( −1) 2 = −1 + 12 + 4 − 1 = 14

i j k
∂ ∂ ∂
and Curl v = ∇ × v =
∂x ∂y ∂z
xyz 3 x 2 y xz 2 − y 2 z
∂ ∂  ∂ ∂
= i  (xz 2 − y 2z ) − (3x 2 y )  − j  (xz 2 − y 2z ) − (xyz ) 
 ∂y ∂z   ∂x ∂z 
 ∂ ∂ 
+ k  (3 x 2 y ) − ( xyz ) 
 ∂ x ∂ y 
= i[0 − 2 yz − 0] − j[ z 2 − 0 − xy ] + k [6 xy − xz ]
= −2 yzi − ( z 2 − xy ) j + (6 xy − xz )k

M03_Eng-Maths (Aditya) CH04.indd 24 7/23/2018 4:48:25 PM


Vector Differentiation  n  4.25

At the point (2, 21, 1),

∇ × v = −2( −1) ⋅1i − (12 − 2( −1)) j + [6 ⋅ 2( −1) − 1∇⋅ 2]


× kv = 2i − 3 j − 14k

Example 3

Show that the vector F 5 (6 xy 1 z 3 )i 1 (3 x 2 2 z ) j 1 (3 xz 2 2 y )k is irrotational.

Solution.
Given F = (6 xy + z 3 )i + (3 x 2 − z ) j + (3 xz 2 − y )k

F is irrotational if curl F = 0

i j k
∂ ∂ ∂
Now curl F = ∇ × F =
∂x ∂y ∂z
6 xy + z 3 (3 x 2 − z ) (3 xz 2 − y )
∂ ∂  ∂ ∂ 
= i  (3 xz 2 − y ) − (3 x 2 − z )  − j  (3 xz 2 − y ) − (6 xy + z 3 ) 
 ∂ y ∂ z   ∂x ∂z 
∂ ∂ 
+ k  (3 x 2 − z ) − (6 xy + z 3 ) 
 ∂x ∂y 
= i[ −1 + 1] − j[3 z 2 − 3 z 2 ] + k [6 x − 6 x ] = 0.
\  F is irrotational vector.

Example 4
Prove that (i) div r 5 3, (ii) curl r 5 0 where r is the position vector of a point (x, y, z) in space.
Solution.
Given r is the position vector of a point (x, y, z) in space.
\ r = xi + y j + zk
∂ ∂ ∂
(i) div r = ∇ ⋅ r = ( x) + ( y) + ( z) = 1 + 1 + 1 = 3
∂x ∂y ∂z
i j k
∂ ∂ ∂
(ii) Curl r =
∂x ∂y ∂z
x y z
∂ ∂  ∂ ∂  ∂ ∂ 
= i  ( z ) − ( y) − j  ( z ) − ( x) − k  ( y) − ( x)
 ∂y ∂z   ∂x ∂z   ∂x ∂y 
= i [0 − 0] − j[0 − 0] + k [0 − 0] = 0
\  r is an irrotational vector.

M03_Eng-Maths (Aditya) CH04.indd 25 7/23/2018 4:48:30 PM


4.26  n  Engineering Mathematics-III

Example 5
Find the value of a if the vector
F 5 (2 x 2 y 1 yz ) i 1 ( xy 2 2 xz 2 ) j 1 ( axyz 2 2 x 2 y 2 ) k is solenoidal.

Solution.
Given F = (2 x 2 y + yz )i + ( xy 2 − xz 2 ) j + ( axyz − 2 x 2 y 2 )k
is solenoidal.
∂ ∂ ∂
\ ∇ ⋅ F = 0  ⇒  ( 2 x 2 y + yz ) + ( xy 2 − xz 2 ) + ( axyz − 2 x 2 y 2 ) = 0
∂x ∂y ∂z
⇒ 4 xy + 2 xy + axy = 0
⇒ 6 xy + axy = 0
⇒ xy(6 + a) = 0  ⇒  (6 + a) = 0   ⇒  a = −6 [{ x ≠ 0, y ≠ 0]

Example 6 
Show that F 5 ( y 2 2 z 2 1 3 yz 2 2 x ) i 1 (3 xz 1 2 xy ) j 1 (3 xy 2 2 xz 1 2 z ) k is irrotational
and solenoidal.

Solution.
Given F = ( y 2 − z 2 + 3 yz − 2 x )i + (3xz + 2 xy ) j + (3xy − 2 xz + 2 z )k .
We have to prove F is irrotational and solenoidal.
i.e., to prove ∇ × F = 0 and ∇ ⋅ F = 0
i kj
∂ ∂

∇×F =
∂x ∂y
∂z
y 2 − z 2 + 3 yz − 2 x 3xz + 2 xy 3xy − 2 xz + 2 z

= i(3 x − 3 x ) − j[3 y − 2 z − ( −2 z + 3 y )] + k [3z + 2 y − (2 y + 3z )] = 0


\  F is irrotational.
∂ 2 ∂ ∂
∇⋅F = ( y − z 2 + 3 yz − 2 x ) + (3 xz + 2 xy ) + (3 xy − 2 xz + 2 z )
∂x ∂y ∂z
= −2 + 2 x + ( −2 x + 2) = 0

\  F is solenoidal.

Example 7
If r 5 xi 1 y j 1 z k and r 5 r , prove that r n r is solenoidal if n 5 23 and irrotational for all
values of n.

Solution.
Given r = xi + y j + zk  \  r = r = x 2 + y 2 + z 2   ⇒  r 2 = x 2 + y 2 + z 2  (1)
r r = r ( xi + y j + zk ) = r xi + r y j + r zk
n n n n n

∂ n ∂ ∂
\ div ( r n r ) = ∇ ⋅ ( r n xi + r n y j + r n zk ) = ( r x ) + ( r n y ) + ( r n z ) (2)
∂x ∂y ∂z

M03_Eng-Maths (Aditya) CH04.indd 26 7/23/2018 4:48:37 PM


Vector Differentiation  n  4.27

∂ n ∂r ∂ n ∂r ∂ n ∂r
But ( r x ) = r n + x ⋅ nr n −1 , ( r y ) = r n + y ⋅ nr n −1 , ( r z ) = r n + z ⋅ nr n −1
∂x ∂x ∂y ∂y ∂z ∂z
∂ n ∂r ∂ n ∂r
nr n −1
( r y ) = r n + y ⋅and , ( r z ) = r n + z ⋅ nr n −1
y y ∂ ∂z ∂z
∂r x ∂r y ∂r z
We have, r2 = x2 + y2 + z2 , = , = , =
∂x r ∂y r ∂z r
∂ n x
\ ( r x ) = r n + nxr n −1 ⋅ = r n + nx 2 r n − 2
∂x r
∂ n y
( r y ) = r n + nyr n −1 ⋅ = r n + ny 2 r n − 2
∂y r
∂ n z
and ( r z ) = r n + nzr n −1 ⋅ = r n + nz 2 r n − 2
∂z r
Substitute in (2).
\ div ( r n r ) = r n + nx 2 r n − 2 + r n + ny 2 r n −2 + r n + nz 2 r n − 2
= 3r n + nr n − 2 ( x 2 + y 2 + z 2 ) = 3r n + nr n − 2 ⋅ r 2 = 3r n + nr n = ( n + 3)r n

If n = -3, then div ( r n r ) = 0  \  r n r is solenoidal if n = -3


i j k
∂ ∂ ∂
Now ∇ × r n r =
∂x ∂y ∂z
rn x rn y rn z
∂ ∂  ∂ ∂  ∂ ∂ 
= i  (r n z ) − (r n y) − j  (r n z ) − (r n x) + k  (r n y) − (r n x)
 ∂y ∂z   ∂x ∂z   ∂ x ∂ y 

 ∂r ∂r   ∂r ∂r   n −1 ∂r ∂r 
= i  nzr n −1 − nyr n −1  − j  nzr n −1 − nxr n −1  + k  nyr − nxr n −1 
 ∂y ∂z   ∂x ∂z   ∂x ∂y 
 y z  n −1 x z  n −1 x n −1 y 
= i  nzr n −1 − nyr n −1  − j  nzr ⋅ − nxr
n −1
 + k  nyr ⋅ r − nxr ⋅ r 
 r r r r
= i( nr n − 2 yz − nr n − 2 yz ) − j ( nr n − 2 xz − nr n − 2 xz ) + k ( nr n − 2 xy − nr n − 2 xy ) = 0

\ ∇ × ( r n r ) = 0 for all values of n.


Hence, r n r is irrotational for all values of n.

Example 8
Prove that F 5 ( y 2 cos x 1 z 3 ) i 1 ( 2 y sin x 2 4 ) j 1 3 xz 2 k is irrotational and find its scalar
potential.

Solution.
Given F = ( y 2 cos x + z 3 )i + (2 y sin x − 4) j + 3 xz 2 k

M03_Eng-Maths (Aditya) CH04.indd 27 7/23/2018 4:48:43 PM


4.28  n  Engineering Mathematics-III

i j k
∂ ∂ ∂
Now ∇×F =
∂x ∂y ∂z
y 2 cos x + z 3 2 y sin x − 4 3 xz 2
= i(0 − 0) − j (3 z 2 − 3 z 2 ) + k (2 y cos x − 2 y cos x ) = 0
\  F is irrotational.
Hence, there exist a scalar function f such that F = ∇f
∂f ∂f ∂f
⇒ ( y 2 cos x + z 3 )i + (2 y sin x − 4) j + 3 xz 2 k = i +j +k
∂x ∂y ∂z
∂f ∂f ∂f
\ = y 2 cos x + z 3   (1) = 2 y sin x − 4  (2) and  = 3 xz 2 (3)
∂x ∂y ∂z
Integrating (1) w.r.to x, f = y 2 sin x + z 3 x + f1 ( y, z )(4)
Integrating (2) w.r.to y, f = y 2 sin x − 4 y + f 2 ( x, z )(5)
Integrating (3) w.r.to z, f = xz 3 + f 3 ( x, y ) (6)

= y 2 sin x + xz 3 − 4 y + c is the scalar potential, where c is an arbitrary constant.


From (4), (5), (6), f

Example 9
(i) Find a such that (3 x 2 2 y 1 z ) i 1 (4 x 1 ay 2 z ) j 1 ( x 2 y 1 2 z ) k is solenoidal.
(ii) Find a, b, c if ( x 1 y 1 az ) i 1 (bx 1 2 y 2 z ) j 1 ( 2 x 1 cy 1 2 z ) k is irrotational.

Solution.
(i) Let F = (3 x − 2 y + z )i + (4 x + ay − z ) j + ( x − y + 2 z )k
Given F is solenoidal.
\ ∇⋅F = 0
∂ ∂ ∂
⇒ (3 x − 2 y + z ) + ( 4 x + ay − z ) + ( x − y + 2 z ) = 0
∂x ∂y ∂z
⇒ 3 + a + 2 = 0 ⇒ a = −5

(ii) Let F = ( x + y + az )i + (bx + 2 y − z ) j + ( − x + cy + 2 z )k


Given F is irrotational.
i j k
\ ∇ × F = 0  ⇒  ∂ ∂ ∂
=0
∂x ∂y ∂z
x + y + az bx + 2 y − z − x + cy + 2 z

⇒ i  ∂ ( − x + cy + 2 z ) − ∂ (bx + 2 y − z )  − j  ∂ ( − x + cy + 2 z ) − ∂ ( x + y + az ) 
 ∂y ∂z   ∂x ∂z 
 
∂ ∂ 
+ k  (bx + 2 y − z ) − ( x + y + az )  = 0
 ∂x ∂y 

M03_Eng-Maths (Aditya) CH04.indd 28 7/23/2018 4:48:50 PM


Vector Differentiation  n  4.29

⇒ i (c + 1) − j ( −1 − a) + k (b − 1) = 0
⇒ (c + 1)i + (1 + a) j + (b − 1)k = 0
\ c + 1 = 0, 1 + a = 0, b − 1 = 0
\ a = −1, b = 1 and c = −1

Example 10
Determine f(r) so that the vector f(r) r is both solenoidal and irrotational.

Solution.
If r is not specified, it will always represent the position vector of any point (x, y, z).

\ r = xi + y j + zk  and r = r = x 2 + y 2 + z 2   \  r 2 = x 2 + y 2 + z 2 (1)
\ f ( r )r = f ( r )( xi + y j + zk ) = f ( r ) xi + f ( r ) y j + f ( r ) zk
Given f(r) r is solenoidal.
∂ ∂ ∂
\ ∇ ⋅ (f ( r )r ) = 0  ⇒  ( f ( r ) x ) + ( f ( r ) y ) + ( f ( r ) z ) = 0 (2)
∂x ∂y ∂z
∂ ∂r
But ( f ( r ) x ) = f ( r ) + xf ′( r )
∂x ∂x
∂ ∂r
( f ( r ) y ) = f ( r ) + yf ′( r )
∂y ∂y

and ∂ ∂r
( f ( r ) z ) = f ( r ) + zf ′( r )
∂z ∂z
∂r x ∂r y ∂r z
Differentiating (1) we get, = , = , =
∂x r ∂y r ∂z r
∂ x x2
\ ( f ( r ) x ) = f ( r ) + xf ′( r ) ⋅ = f ( r ) + f ′( r )
∂x r r
∂ y2
Similarly, ( f (r) y) = f (r) + f ′( r )
∂y r
∂ z2
and ( f (r) z) = f (r) + f ′( r )
∂z r
x2 y2 z2
\ (2)  ⇒  f ( r ) + f ′( r ) + f ( r ) + f ′( r ) + f ( r ) + f ′( r ) = 0
r r r
f ′( r ) 2
⇒ 3 f (r) + ( x + y2 + z2 ) = 0
r
f ′( r ) 2
⇒ 3 f (r) + ⋅r = 0
r
f ′( r ) 3
⇒ =−  3 f ( r ) + rf ′( r ) = 0   ⇒ 
f (r ) r
[here r is real variable.]

M03_Eng-Maths (Aditya) CH04.indd 29 7/23/2018 4:48:58 PM


4.30  n  Engineering Mathematics-III

f ′( r ) 1
Integrating w.r.to ‘r’, we get ∫ dr = −3∫ dr
f (r) r
⇒ log e f ( r ) = −3 log e r + log c
c c
⇒ log e f ( r ) = − log e r 3 + log e c = log e 3
  ⇒  f ( r ) = 3
r r
where c is the constant of integration.
i j k
∂ ∂ ∂
Now ∇ × (f ( r )r ) =
∂x ∂y ∂z
f ( r )x f ( r ) y f ( r )z
∂ ∂  ∂ ∂ 
= i  ( f (r ) z ) − ( f (r) y) − j  ( f (r) z) − ( f (r) x)
 ∂ y ∂ z   ∂ x ∂ z 
∂ ∂ 
+ k  ( f (r ) y) − ( f (r ) x)
 ∂x ∂y 
 ∂r ∂r 
= ∑ i  zf ′( r ) ⋅ − y ⋅ f ′( r ) ⋅ 
 ∂ y ∂ z
 y z  yz yz 
= ∑ i  zf ′( r ) ⋅ − y ⋅ f ′( r ) ⋅  = ∑ i f ′( r )  −  = 0
 r r r r 
c
\  f ( r )r is irrotational for all f(r) and it is solenoidal for f ( r ) =, where c is arbitrary constant.
r3
c
Hence, the required function is f ( r ) = 3 , for which f ( r )r is both solenoidal and irrotational.
r

EXERCISE 4.2
1. If F = xy 2 + 2 x 2 yz j − 3 yz 2 k, then find div F and curl F at (1, 1, -1).
2. If F = x 2 y i + y 2 z j + z 2 x k then find curl curl F.
3. Find div F and curl F at (1, 1, 1)
if F = ( x 2 − y 2 + 2 xz )i + ( xz − xy + yz ) j + ( z 2 + x 2 )k.
4. Show that the following vectors are solenoidal.
(i)  F = (2 + 3 y )i + ( x − 2 z ) j + xk
(ii)  F = ( y 2 − z 2 + 3 yz − 2 x )i + (3 xz + 2 xy ) j + (3 xy − 2 xz + 2 z )k
(iii) F = 3 x 2 yi − 4 xy 2 j + 2 xyzk
5. Find the value of a if F = ay 4 z 2 i + 4 x 3 z 2 j + 5 x 2 y 2 k is solenoidal.
6. If the vector 3 xi + ( x + y ) j − azk is solenoidal, then find a.
7. Show that the following vectors are irrotational.
(i)  F = ( y 2 + 2 xz 2 )i + (2 xy − z ) j + (2 x 2 z − y + 2 z )k
(ii)  F = (sin y + z )i + ( x cos y − z ) j + ( x − y )k
(iii) F = (4 xy − z 2 )i + 2 x 2 j − 3 xz 2 k

M03_Eng-Maths (Aditya) CH04.indd 30 7/23/2018 4:49:05 PM


Vector Differentiation  n  4.31

8. Find the value of a if F = ( axy − z 2 )i + ( x 2 + 2 yz ) j + ( y 2 − axz )k is irrotational.

9. If F = ( ax 2 + 2 y 2 + 1)i + (4 xy + by 2 z − 3) j + (c − y 3 )k is irrotational, then find the values of


a, b, c.
10. Show that F = (2x + 3y + z 2 )i + (3x + 2 y + z ) j + ( y + 2zx )k is irrotational and hence, find its
scalar potential.
11. Prove that F = ( y 2 cos x + z 3 )i + (2 y sin x − 4) j + 3 xz 2 k is irrotational and find its scalar potential.
12. Show that F = (6 xy + z 3 )i + (3 x 2 − z ) j + (3 xz 2 − y )k is irrotational, find its scalar potential.
13. Find the div F and curl F, where F = grad ( x 3 + y 3 + z 3 − 3 xyz ).
1
14. If v = w × r , prove that w = curl v, where w is a constant vector and is the position vector of
the point (x, y, z). 2
15. If r is the position vector of a point (x, y, z) in space and A is a constant vector, prove that A r is
solenoidal.
16. Prove that the vector F = ( x + 3 y )i + ( y − 3 z ) j + ( x − 2 z )k is solenoidal.
17. Show that v = xyz 2 u is solenoidal, where
u = (2 x 2 + 8 xy 2 z )i + (3 x 3 y − 3 xy ) j − (4 y 2 z 2 + 2 x 3 z )k .

ANSWERS TO EXERCISE 4.2


1. 5; − 5i − 6 k  2. 2[ zi + x j + yk ] 3.  6; − 2i + 2k
5.  a can be any real number   6.  4 8.  2
9.  a = 3, b = -3, c = 2 10.  f = x2 + y2 + 3xy + yz + z2x + c
11. f = y2 sinx + xz3 - 4y + c 12.  f = 3x2y + xz3 - yz + c
13. div F = b( x + y + z ) Curl F= O

4.7  Vector Identities


We shall list the vector identities into two categories.
(i) ∇ operator applied once to point functions.
(ii) ∇ operator applied twice to point functions.

TYPE 1.
If f and g are scalar point functions we have already proved the following results.
1. ∇c = 0, where c is a constant. 2.
∇(cf) = c∇f, where c is constant.
3. ∇(f ± g) = ∇f ± ∇g 4. ∇(fg) = f∇g + g∇f
f  g ∇f − f ∇ g
5. ∇  =
 g  f2

M03_Eng-Maths (Aditya) CH04.indd 31 7/23/2018 4:49:12 PM


4.32  n  Engineering Mathematics-III

6. If F and G are vector point functions, then ∇ ⋅ ( F 1 G ) 5 ∇ ⋅ F 1 ∇ ⋅ G .

Proof
 ∂ ∂ ∂
∇ ⋅ (F + G) =  i + j + k  ⋅ (F + G)
 ∂x ∂y ∂z 
 ∂F ∂G   ∂F ∂G   ∂F ∂G 
= i ⋅ + + j ⋅ + + k ⋅ +
 ∂x ∂x  
 ∂y ∂y   ∂z ∂z 
 ∂F ∂F ∂F   ∂G ∂G ∂G 
= i ⋅ + j⋅ + k ⋅  + i ⋅ + j⋅ +k⋅
 ∂x ∂y ∂z   ∂x ∂y ∂z 
= ∇ ⋅ F + ∇ ⋅G
Similarly, ∇ ⋅ ( F − G ) = ∇ ⋅ F − ∇ ⋅ G n

7. If f is a scalar point function and G is a vector point function, then ∇ ? ( fG ) 5 ∇f ? G 1 f ( ∇?G )

Proof  Let G = G1 i + G2 j + G3 k, then fG = fG1 i + fG2 j + fG3 k

∂ ∂ ∂
\ ∇ ⋅ ( fG ) = ( fG1 ) + ( fG2 ) + ( fG3 )
∂x ∂y ∂z
∂G1 ∂f ∂G2 ∂f ∂G3 ∂f
= f + G1 + f + G2 + f + G3
∂x ∂x ∂y ∂y ∂z ∂z
∂f ∂f ∂f  ∂G ∂G ∂G 
= G1 + G2 + G3 + f  1 + 2 + 3 
∂x ∂y ∂z  ∂x ∂y ∂z 

\ ∇ ⋅ ( fG ) = ∇f ⋅ G + f (∇ ⋅ G ) n

8. If f is a scalar point function and G is a vector point function, then



∇33 ((fG 5∇
fG )) 5 ∇ff 3
3G 1 ff ((∇
G1 ∇3 G ))
3G
Proof  Let G = G1 i + G2 j + G3 k  \ fG = fG1 i + fG2 j + fG3 k
i j k
∂ ∂ ∂
Now ∇ × ( fG ) =
∂x ∂y ∂z
fG1 fG2 fG3
∂ ∂  ∂ ∂  ∂ ∂ 
= i  ( fG3 ) − ( fG2 )  − j  ( fG3 ) − ( fG1 )  + k  ( fG2 ) − ( fG1 ) 
 ∂y ∂z   ∂x ∂z   ∂x ∂y 
 ∂G3 ∂f ∂G2 ∂f   ∂G3 ∂f ∂G1 ∂f 
= i f + G3 −f − G2  − j  f + G3 −f − G1 
 ∂y ∂y ∂z ∂z   ∂x ∂x ∂z ∂z 
 ∂G2 ∂f ∂G1 ∂f 
+kf + G2 −f − G1 
 ∂x ∂x ∂y ∂y 
  ∂G ∂G   ∂G ∂G   ∂G ∂G  
= f  3 − 2  i −  3 − 1  j +  2 − 1  k 
 ∂ y ∂ z   ∂ x ∂ z   ∂x ∂y  

 ∂f ∂f   ∂f ∂f   ∂f ∂f 
+  G3 − G2  i −  G3 − G1  j +  G2 − G1  k
M03_Eng-Maths (Aditya) CH04.indd 32  ∂y ∂z   ∂x ∂z   ∂x ∂y  7/23/2018 4:49:21 PM
 ∂G3 ∂f ∂G2 ∂f   ∂G3 ∂f ∂G1 ∂f 
= i f + G3 −f − G2  − j  f + G3 −f − G1 
 ∂y ∂y ∂z ∂z   ∂x ∂x ∂z ∂z 
Vector Differentiation  n  4.33
 ∂G2 ∂f ∂G1 ∂f 
+kf + G2 −f − G1 
 ∂x ∂x ∂y ∂y 
  ∂G ∂G   ∂G ∂G   ∂G ∂G  
= f  3 − 2  i −  3 − 1  j +  2 − 1  k 
 
  ∂y ∂z  ∂x ∂z  ∂x ∂y  
 ∂f ∂f   ∂f ∂f   ∂f ∂f 
+  G3 − G2  i −  G3 − G1  j +  G2 − G1  k
 ∂y ∂z   ∂x ∂z   ∂x ∂y 
i j k i j k
∂ ∂ ∂ ∂f ∂f ∂f
= f +
∂x ∂y ∂z ∂x ∂y ∂z
G1 G2 G3 G1 G2 G3
\ ∇ × ( fG ) = f (∇ × G ) + (∇f ) × G. n

9. If F and G are vector point functions, then


∇( F ⋅ G ) 5 ( F ? ∇)G 1 (G ? ∇) F 1 F 3 ( ∇ 3 G ) 1 G 3 ( ∇ 3 F )

∂f ∂f ∂f ∂f
Proof  We know that ∇f = i + j +k = ∑i
∂x ∂y ∂z ∂x

\ ∇( F ⋅ G ) = ∑ i (F ⋅ G)
∂x
 ∂G ∂F 
= ∑ i F ⋅ +G⋅ 
 ∂x ∂x 
 ∂G   ∂F 
= ∑F ⋅  i + ∑  G ⋅  i (1)
 ∂x  ∂x
We know that a × ( b × c ) = ( a ⋅ c )b − ( a ⋅ b)c
\ ( a ⋅ b )c = ( a ⋅ c )b − a × ( b × c )
 ∂G   ∂G   ∂G 
\ F⋅  i = ( F ⋅ i )   − F ×  × i
 ∂x  ∂x ∂x 
 ∂G   ∂G 
= ( F ⋅ i)  + F × i ×
 ∂x   
∂x 
 ∂G   ∂  ∂G 
\ ∑  F ⋅ ∂x  i =  F ⋅ ∑ i ∂x  G + F × ∑  i × ∂x 
= ( F ⋅ ∇)G + F × (∇ × G )(2)
Interchanging F and G, we get
 ∂F 
∑  G ⋅ ∂x  i = (G ⋅ ∇) F + G × (∇ × F )(3)
Substituting (2) and (3) in (1) we get
∇( F ⋅ G ) = ( F ⋅ ∇)G + F × (∇ × G ) + (G ⋅ ∇) F + G × (∇ × F )
\ ∇( F ⋅ G ) = ( F ⋅ ∇)G + (G ⋅ ∇) F + F × (∇ × G ) + G × (∇ × F ) n

M03_Eng-Maths (Aditya) CH04.indd 33 7/23/2018 4:49:29 PM


4.34  n  Engineering Mathematics-III

10. If F and G are vector point functions then

∇ ? ( F 3 G ) 5 G ? ( ∇ 3 F ) 2 F ?( ∇ 3 G )
i .e., div F 3 G 5 G ? Curl F 2 F ? Curl G .

Proof

∇ ⋅ (F × G) = ∑ i ⋅ (F × G)
∂x
 ∂F ∂G 
= ∑i⋅ ×G + F × 
 ∂x ∂x 
 ∂F   ∂G 
= ∑i⋅ × G + ∑ i ⋅  F × 
 ∂x   ∂x 
In a scalar triple product ⋅ and × can be interchanged.
 ∂F   ∂G 
\  we get ∇ ⋅ (F × G) = ∑  i ×  ⋅G − ∑ i ×  ⋅F
 ∂x   ∂x 
⇒ ∇ ⋅ ( F × G ) = (∇ × F ) ⋅ G − (∇ × G ) ⋅ F n
11. If F and G are vector product functions, then
∇ × ( F × G ) = F (∇ ⋅ G ) − G (∇ ⋅ F ) + (G ⋅ ∇) F − ( F ⋅ ∇)G
Proof

∇ × (F × G) = ∑ i × (F × G)
∂x
 ∂F ∂G 
= ∑i × ×G + F × 
 ∂x ∂x 
 ∂F   ∂G 
⇒ ∇ × (F × G ) = ∑ i ×  × G + ∑ i ×  F ×  (1)
 ∂x   ∂x 

We know a × ( b × c ) = ( a ⋅ c )b − ( a ⋅ b)c
 ∂F   ∂F  ∂F  
\ ∑ i ×  ∂x × G  = ∑ (i ⋅ G ) ∂x −  i ⋅ ∂x  G 
∂F  ∂F 
= ∑ (G ⋅ i ) − ∑ i ⋅  G
∂x  ∂x   n
 ∂  ∂F 
= G ⋅∑i  F − ∑i⋅  G
 ∂x   ∂x 
 ∂F 
⇒ ∑ i ×  ∂x × G  = (G ⋅ ∇)F − (∇ ⋅ F )G (2)

 ∂G    ∂G  ∂G 
Similarly,  ∑ i ×  F ×  = ∑  i ⋅  F − ∑ (i ⋅ F ) 
 ∂x   ∂ x  ∂x 

M03_Eng-Maths (Aditya) CH04.indd 34 7/23/2018 4:49:36 PM


Vector Differentiation  n  4.35

 ∂G  ∂G
= ∑ i⋅ F − ∑ (i ⋅ F )
 ∂x  ∂x
 ∂G   ∂
= ∑i⋅  F − ∑  F ⋅ i  G
 ∂x  ∂x
 ∂
= (∇ ⋅ G ) F − F ⋅  ∑ i  G
 ∂x 
 ∂G 

∑ i ×  F × ∂x  = (∇ ⋅ G )F − (F ⋅ ∇)G (3)
Substituting (2) and (3) in (1), we get
∇ × ( F × G ) = (G ⋅ ∇) F − (∇ ⋅ F )G + (∇ ⋅ G ) F − ( F ⋅ ∇)G
\ Curl F × G = F (div G ) − G (div F ) + (G ⋅ ∇) F − ( F ⋅ ∇)G n

TYPE II – Identities – ∇ Applied Twice


∂2 f ∂2 f ∂2 f
1. If f is scalar point function, then div grad f 5 ∇ 2 f 5 1 1
∂x 2 ∂y 2 ∂z 2
∂f ∂f ∂f
Proof  We know, grad f = ∇f = i +j +k
∂x ∂y ∂z
div (grad f ) = ∇ ⋅ ∇f
 ∂ ∂ ∂   ∂f ∂f ∂f 
= i + j + k  ⋅i + j + k 
 ∂x ∂y ∂z   ∂x ∂y ∂z 
∂  ∂f  ∂  ∂f  ∂  ∂f  ∂ 2 f ∂ 2 f ∂ 2 f
=  + +  = + +
∂x  ∂x  ∂y  ∂y  ∂z  ∂z  ∂x 2 ∂y 2 ∂z 2
∂2 f ∂2 f ∂2 f
\ div (grad f ) = ∇ 2 f = + +  n
∂x 2 ∂y 2 ∂z 2

∂2 ∂2 ∂2
Note  ∇ ⋅ ∇ = ∇ 2 = + + is a scalar operator called the Laplacian operator.
∂x 2 ∂y 2 ∂z 2

2. If F is a vector point function, then div curl F 5 0.

Proof  Let F = F1 i + F2 j + F3 k, where F1, F2, F3 are scalar functions of x, y, z.

i j k
∂ ∂ ∂
Curl F = ∇ × F =
∂x ∂y ∂z
F1 F2 F3
 ∂F ∂F   ∂F ∂F   ∂F ∂F 
= i 3 − 2 − j 3 − 1+k 2 − 1
 ∂y ∂z   ∂x ∂z   ∂x ∂y 

M03_Eng-Maths (Aditya) CH04.indd 35 7/23/2018 4:49:42 PM


4.36  n  Engineering Mathematics-III

\ ∂  ∂F3 ∂F2  ∂  ∂F1 ∂F3  ∂  ∂F2 ∂F1 


div Curl F = ∇ ⋅ ∇ × F =  ∂y − ∂z  + ∂y  ∂z − ∂x  + ∂z  ∂x − ∂y 
∂x
∂ 2 F3 ∂ 2 F2 ∂ 2 F1 ∂ 2 F3 ∂ 2 F2 ∂ 2 F1
= − + − + −
∂x ∂y ∂x ∂z ∂y ∂z ∂y ∂x ∂z ∂x ∂z ∂y
 ∂ 2 F3 ∂ 2 F3 ∂ 2 F2 ∂ 2 F2 ∂ 2 F1 ∂ 2 F1 
⇒ div Curl F = 0∇ ⋅ ∇ × F Since = , = , =  n
  ∂ x ∂ y ∂y ∂x ∂x ∂z ∂z ∂x ∂y ∂z ∂z ∂y 
3. If F is a vector point function, then
curl (Curl F ) 5 ∇ 3 ( ∇ 3 F ) 5 ∇( ∇ ? F ) 2 ∇ 2 F.

Proof  Let F = F1 i + F2 j + F3 k, where F1, F2, F3 are scalar functions.

i j k
∂ ∂ ∂
Then Curl F =
∂x ∂y ∂z
F1 F2 F3
 ∂F ∂F   ∂F ∂F   ∂F ∂F 
= i 3 − 2  − j 3 − 1 + k  2 − 1
 ∂y ∂z   ∂x ∂z   ∂x ∂y 
i j k
∂ ∂ ∂
\ Curl curl F =
∂x ∂y ∂z
∂F3 ∂F2 ∂F1 ∂F3 ∂F2 ∂F1
− − −
∂y ∂z ∂z ∂x ∂x ∂y
 ∂  ∂F ∂F  ∂  ∂F ∂F  
= ∑i   2 − 1 −  1 − 3 
 ∂y  ∂x ∂y  ∂z  ∂z ∂x  
 ∂ F2 ∂ F1 ∂ F1 ∂ F3 
2 2 2 2
= ∑i  − 2 − 2 + 
 ∂y ∂x ∂y ∂z ∂z ∂x 
 ∂ 2 F2 ∂ 2 F3  ∂ 2 F1 ∂ 2 F1  
= ∑i + − 2 + 2 
 ∂y∂x ∂z∂x  ∂y ∂z  
 ∂ 2 F ∂ 2 F2 ∂ 2 F3  ∂ 2 F1 ∂ 2 F1 ∂ 2 F1  
= ∑ i  21 + + − + 2 + 2 
 ∂x ∂x∂y ∂x∂z  ∂x 2 ∂y ∂z  

 ∂  ∂F ∂F ∂F   ∂ F ∂ F ∂ F  
2 2 2
= ∑ i   1 + 2 + 3  −  21 + 21 + 21  
 ∂x  ∂x ∂y ∂z   ∂x ∂y ∂z  
 ∂  ∂2 ∂2 ∂ 2  
= ∑ i  (∇ ⋅ F ) −  2 + 2 + 2  F1 
 ∂x  ∂x ∂y ∂z  

= ∑i {

∂x
(∇ ⋅ F ) − ∇ 2 F1 }
∂

=  ∑ i  (∇ ⋅ F ) − ∇ 2 ∑ iF1
 ∂x 
( )
M03_Eng-Maths (Aditya) CH04.indd 36 7/23/2018 4:49:49 PM
 ∂  ∂F ∂F ∂F   ∂ F ∂ F ∂ F  
2 2 2
= ∑ i   1 + 2 + 3  −  21 + 21 + 21  
 ∂x  ∂x ∂y ∂z   ∂x ∂y ∂z  
Vector Differentiation  n  4.37
 ∂  ∂2 ∂2 ∂ 2  
= ∑ i  (∇ ⋅ F ) −  2 + 2 + 2  F1 
 ∂x  ∂x ∂y ∂z  

= ∑i {

∂x
(∇ ⋅ F ) − ∇ 2 F1 }
∂

=  ∑ i  (∇ ⋅ F ) − ∇2
 ∂x 
(∑ iF )1

\ ∇ × (∇ × F ) = ∇(∇ ⋅ F ) − ∇ 2 F n

WORKED EXAMPLES
Example 1
 1 n
Prove that ∇  n  52 n12 r .
r  r
Solution.
We have r = xi + y j + zk and r2 = x2 + y2 + z2

∂r x ∂r y ∂r z
\ = , = , =
∂x r ∂y r ∂z r
 1 ∂  1 ∂  1 ∂  1
\ ∇ n  = i  n  + j  n  + k  n 
r  
∂x r  ∂y r  ∂z  r 
 − n ∂r   − n ∂r   − n ∂r 
= i  n +1  + j  n +1  + k  n +1 
 r ∂x   r ∂y   r ∂z 
n  x y z n n
=− i r + j r + k r  = − r n + 2 ( xi + y j + zk ) = − r n + 2 r
r n +1
 1 n
\ ∇  n  = − n+ 2 r
r  r
 1 2
∇ 2  = − 4 r
 1 n r  r
Note  We have ∇  n  = − n+ 2 r
r  r  1 2  1 3
∇ 2  = − 4 r ∇ 3  = − 5 r
If n = 1, 2, 3, 4, … r  r r  r
 1 1  1 2  1 3  1 4
Then ∇   = − 3 r ,  ∇  2  = − 4 r ,  ∇  3  = − 5 r ,  ∇  4  = − 6 r and so on..
 r r r  r r  r r  r
 1 3  1 4
Example 2 ∇ 3  = − 5 r ∇  4  = − 6 r and so on..
 r  r  r  r
Prove that ∇ 2 ( r n ) 5 n( n 11) r n22 .
 1 4
∇  4  = − 6 r and so on..
Solution. r  r
We have r = xi + y j + zk and r2 = x2 + y2 + z2

∂r x ∂r y ∂r z
\ = , = , =  
∂x r ∂y r ∂z r

M03_Eng-Maths (Aditya) CH04.indd 37 7/23/2018 4:49:56 PM


4.38  n  Engineering Mathematics-III

∂ n ∂r
\  ∇( r n ) = ∑ i ( r ) = ∑ i nr n −1
∂x ∂x
n −1 x
n−2   (1)
= ∑ i nr = nr n − 2 ∑ xi = nr n − 2 ( xi + y j + zk ) = nr r if n ≥ 3
r
∇ 2 ( r n ) = ∇ ⋅ (∇r n ) = ∇ ⋅ ( nr n − 2 r )
= n[∇r n − 2 ⋅ r + r n − 2 (∇ ⋅ r )]
= n[( n − 2)r n − 4 r ⋅ r + r n − 2 3] [ using (1)]

= n[( n − 2)r n − 4 r 2 + 3r n − 2 ] = nr n − 2 [n − 2 1+ 3] = n ( n + 1)r n − 2


∇( r ) = r
1 r
Note  We have ∇( r n ) ∇ r )n=− 2 r r
= (nr ∇( r ) = 2 ⋅ r 2 − 2 r = 2r
2

If n = 1, 2, 3, 4, … 1 r
∇( r ) = r
r ∇( r 2 ) = 2 ⋅ r 2 − 2 r∇=( r23r) = 3rr
1 n −2 n−4
∇( r ) = r ,  ∇( r 2 ) = 2 ⋅ r 2 − 2 r = 2r ,  ∇( r ) = 3rr ,  ∇( r ) = 4 r r … ∇( r ) = ( n − 2)r r , etc.
3 4 2

r
∇( r 3 ) = 3rr ∇( r 4 ) = 4r 2 r … ∇( r n − 2 ) = ( n − 2)r n − 4 r , etc.
∇( r 2 ) = 2 ⋅ r 2 − 2 r = 2r
Example 3 ∇( r 4 ) = 4 r 2 r … ∇( r n − 2 ) = ( n − 2)r n − 4 r , etc.
∇( r 3 ) = 3rr
  1  3
Prove = 4 r 2∇
∇( r 4 ) that r ?… ∇r∇( rn −r23)= (5n −r 42.)r n − 4 r , etc.

Solution.
 1 3  1  −4
We have ∇  3  = − 5 r ,  ∇  r 4  = r 6 r and ∇ ⋅ r = 3
r  r
  1   −3   −3 
\ ∇ ⋅  r∇  3   = ∇ ⋅  r 5 r  = ∇ ⋅  4 r 
  r   r  r 
  1 1 
= −3 ∇  4  ⋅ r + 4 ∇ ⋅ r 
  r  r 
 4 3   4 3  −4 3  3
= −3  − 6 ( r ⋅ r ) + 4  = −3  − 6 r2 + 4  = −3  4 + 4  = 4
 r r   r r  r r  r

Example 4
If f and c satisfy Laplace equation, prove that the vector (f ∇ c - c ∇ f) is solenoidal.

Solution.
Given f and c satisfy Laplace equation.

∂ 2f ∂ 2f ∂ 2f ∂2c ∂2c ∂2c


\ + + = 0   (1) and  + + = 0 (2)
∂x 2 ∂y 2 ∂z 2 ∂x 2 ∂y 2 ∂z 2
To prove (f ∇ c - c ∇ f) is solenoidal, we have to prove div (f ∇ c - c ∇ f) = 0
Now div (f ∇ c - c ∇ f) = ∇⋅(f ∇ c - c ∇ f)

M03_Eng-Maths (Aditya) CH04.indd 38 7/23/2018 4:50:04 PM


Vector Differentiation  n  4.39

= ∇ ⋅ (f∇c) − ∇ ⋅ (c∇f)
= ∇f ⋅ ∇c + f(∇ ⋅ ∇c) − [∇c ⋅ ∇f + c(∇ ⋅ ∇f)]
= f∇ 2 c − c∇ 2f [{ ∇f ⋅ ∇c = ∇c ⋅ ∇f]
= fx 0 − c x 0 = 0  [from (1) and ( 2)]

\ (f ∇ c - c ∇ f) is solenoidal.

Example 5
d 2f 2 df
Show that ∇2 f ( r ) 5 2
1 .
dr r dr
Solution.
r
We have, ∇f ( r ) = f ′ ( r )
r
r  f ′ ( r)  f ′( r )
\ ∇ 2 f ( r ) = ∇ ⋅ ∇f ( r ) = ∇ ⋅ f ′ ( r ) =  ∇  ⋅ r + (∇ ⋅ r )
r  r r
 f ′( r )  3f ′( r )
= ∇  ⋅r +  [{ ∇ ⋅ r = 3]
 r  r
 r ∇f ′ ( r ) − f ′ ( r ) ∇r  3 f ′( r )    f  g ∇f − f ∇g 
=
  ⋅ r + { ∇  g  = g2 
r2 r  
 r r  r
 rf ′′( r ) − f ′( r )  ⋅ r 3 f ′( r ) { ∇f ′( r ) = f ′′( r ) r
= r r + 
r2 r r
and ∇r = 
[rf ′′( r ) − f ′( r )]r ⋅ r 3 f ′( r ) r
= +
r3 r
[rf ′′( r ) − f ′( r )] 2 3 f ′( r )
= r +  [{ r ⋅ r = r 2 ]
r3 r
rf ′′( r ) − f ′( r ) 3f ′( r ) 2f ′( r ) d 2 f 2 df
= + = f ′′( r ) + = 2 +
r r r dr r dr

M03_Eng-Maths (Aditya) CH04.indd 39 7/23/2018 4:50:11 PM


This page is intentionally left blank

M03_Eng-Maths (Aditya) CH04.indd 40 7/23/2018 4:50:11 PM


Vector Integration 5
5.1  Integration of Vector FunCtions
d
Let f (t ) and F (t ) be two vector functions of a scalar variable t such that F (t ) = f (t ). Then F (t ) is
dt
called an indefinite integral of f (t ) with respect to t and is written as ∫ f (t )dt = F (t ) + c , where c is an
arbitrary constant vector independent of t and is called the constant of integration.
The definite integral of f (t ) between the limits t = t1 and t = t2 is given by
t2
t2
∫ f (t ) dt =  F (t ) 
t1
t1
.= F (t 2 ) − F (t1 ).

As in the case of differentiation of vectors, in order to integrate a vector function, we integrate its
components.
If f (t ) = f1 (t ) i + f 2 (t ) j + f 3 (t ) k, then

∫ f (t ) dt = i∫ f (t ) dt + j ∫ f
1 2 (t ) dt + k ∫ f 3 (t ) dt

5.1.1  Line integral


An integral evaluated over a curve C is called a line integral. We call C as the path of integration. We
assume every path of integration of a line integral to be piecewise smooth consisting of finitely many
smooth curves.

Definition 5.1  A line integral of a vector point function F ( r ) over a curve C, where r is the position
vector of any point on C, is defined by ∫ F ⋅ dr
C
If F = F1 i + F2 j + F3 k and     r = x i + y j + zk , then
dr = dx i + dy j + dz k and  ∫ F ⋅ dr = ∫ F1 dx + F2 dy + F3 dz
C C
Here F1, F2, F3 are functions of x, y, z, where x, y, z depend on a parameter t ∈ [a, b], since r (t ) is the
equation of the curve C.
b
 dx dy dz 
Then we can write ∫ F ⋅ dr = ∫  F1 + F2 + F3  dt .
C a
 dt dt dt 
If the path of integration C is a closed curve, we write ∫ instead of ∫
C
.
C
Note
dr
1. Since is a tangent vector to the curve C the line integral ∫ F ⋅ dr is also called the tangential
dt C
line integral of F over C and line integral is a scalar.
2. Two other types of line integrals are also considered. ∫ F × dr and ∫ f dr are vectors.
C C

M03_Eng-Maths (Aditya) CH05.indd 1 7/23/2018 4:53:43 PM


5.2  n  Engineering Mathematics-III

WORKED EXAMPLES
Example 1

If F 5 3 xy i 2 y 2 j , evaluate ∫ F ? dr where C is the arc of the parabola y 5 2x


C
2
from (0, 0) to
(1, 2).

Solution.
Given F = 3 xy i − y 2 j
r = x i + y j , where r is the position vector of any point (x, y) on y = 2x2.
\ dr = dx i + dy j
and F ⋅ dr = (3 xy i − y 2 j ) ⋅ ( dxi + dy j ) =
3xydx - y dy
2

\ ∫ F ⋅ dr = ∫ (3xy dx − y dy)
2

C C

Equation of C is y = 2x2  \  dy = 4x dx.


1 A (1, 2)
\ ∫ F ⋅ dr = ∫ (3x ⋅ 2 x dx − 4 x ⋅ 4 x dx )
2 4

C 0
1

= ∫ (6 x 3 − 16 x 5 ) dx
0 O
1 (0, 0)
 x4 x6  3 8 9 − 16 7 x=1
= 6 − 16  = − = =−
 4 6 0 2 3 6 6
Fig. 5.1

Example 2
If F 5 (3 x 2 1 6 y ) i 2 14 yz j 1 20 xz 2 k , evaluate ∫ F ? dr from (0, 0, 0) to (1, 1, 1) along the curve
C
C given by x 5 t, y 5 t2, z 5 t3.
Solution.
Given F = (3 x 2 + 6 y )i − 14 yz j + 20 xz 2 k
and r = xi + y j + zk  \  dr = dx i + dy j + dz k
and F ⋅ dr = (3 x 2 + 6 y )i − 14 yz j + 20 xz 2 k  ⋅  dxi + dy j + dzk 
= (3 x 2 + 6 y )dx − 14 yzdy + 20 xz 2 dz
\ ∫ F ⋅ dr = C∫ (3x + 6 y ) dx − 14 yz dy + 20 xz 2 dz
2

C
Given x = t,    y = t2,   z = t3 is the curve.
\  dx = dt,   dy = 2t dt,  dz = 3t2 dt
When x = 0, t = 0 and x = 1, t = 1.   Limits for t are t = 0, t = 1
1
\ ∫ F ⋅ dr = ∫ (3 ⋅ t + 6 ⋅ t )dt − 14 ⋅ t ⋅ 2t dt + 20t ⋅ 3t dt
2 2 5 7 2

   
C 0
1
1  t3 t7 t 10 
= ∫ (9t − 28t + 60t ) dt = 9 − 28 + 60  = 3 − 4 + 6 = 5.
2 6 9
0  3 7 10  0

M03_Eng-Maths (Aditya) CH05.indd 2 7/23/2018 4:53:49 PM


Vector Integration  n  5.3

Example 3

∫ (y
2
Evaluate the line integral dx 2 x 2 dy ) around the triangle whose vertices are (1, 0), (0, 1),
C
(-1, 0) in the positive sense.

Solution.
Given the path C consists of the sides of the ∆ ABC, where A(-1, 0), B(1, 0) and C(0, 1).
Equation of AB is y = 0
y − 0 x −1 y
Equation of BC is = ⇒ y = −x + 1
0 −1 1− 0
C (0, 1)
y −1 x − 0
Equation of CA is = ⇒ y = x +1
1− 0 0 +1

∫ F ⋅ dr = AB∫ ( y dx − x dy)
2 2

+ ∫ ( y dx − x dy) O
2 2
(−1, 0)A B (1, 0)
BC
Fig. 5.2
+ ∫ ( y dx − x dy)
2 2

CA

On AB, y = 0,     \  dy = 0 and x varies from -1 to 1


1

\ ∫
AB
( y 2dx − x 2dy ) = ∫ 0 dx = 0
−1

On BC, y = -x + 1    \  dy = -dx and From B to C, x varies from 1 to 0.


0 0
\
BC
∫ ( y 2 dx − x 2 dy ) = ∫ ( − x + 1) 2 dx − x 2 ( − dx ) = ∫ ( x 2 − 2x + 1 + x 2 )dx
1 1
0

= ∫ ( 2x 2 − 2x + 1)dx
1
0
 x3 x2  2  2
=  2 − 2 + x  = 0 −  − 1 + 1 = −
 3 2 1  3  3

On CA, y = x + 1  \  dy = dx  and From C to A, x varies from 0 to -1


−1 −1

∫ ( y dx − x dy ) = ∫ ( x + 1) dx − x dx = ∫ ( x + 2x + 1 − x )dx
2 2
\ 2 2 2 2

CA 0 0
−1

= ∫ ( 2x + 1)dx = [ x 2 + x ]0 = 1 − 1 − 0 = 0
−1

 2 2
\ ∫ F ⋅ dr = 0 +  − 3  + 0 = − 3
C

M03_Eng-Maths (Aditya) CH05.indd 3 7/23/2018 4:53:51 PM


5.4  n  Engineering Mathematics-III

Example 4 

If F 5 (3 x 2 1 6 y ) i 2 14 yz j 1 20 xz 2 k , evaluate ∫ F ? dr , where C is the straight line joining


C
(0, 0, 0) to (1, 1, 1).
Solution.
Given F = (3 x 2 + 6 y )i − 14 yz j + 20 xz 2 k
r = x i + y j + zk ∴ dr = dx i + dy j + dzk
\ F ⋅ dr = (3 x + 6 y )i − 14 yz j + 20 xz 2 k  ⋅  dxi + dy j + dzk 
2

= (3 x 2 + 6 y )dx − 14 yz dy + 20 xz 2 dz

\ ∫ F ⋅ dr = C∫ (3x + 6 y )dx − 14 yz dy + 20 xz 2 dz
2

Equation of the line joining (0, 0, 0) to (1, 1, 1) is


x−0 y−0 z−0
= =   ⇒  x = y = z = t , say
1− 0 1− 0 1− 0
\ dx = dt,  dy = dt,  dz = dt
At the point (0, 0, 0), t = 0 and at the point (1, 1, 1), t = 1
1

\ ∫ F ⋅ dr = ∫ (3t + 6t )dt − 14t dt + 20t dt


2 2 3

C 0
1

= ∫ (3t 2 + 6t − 14t 2 + 20t 3 )dt


0
1
1
 t4 t3 t2  11 13
= ∫ ( 20t 3 − 11t 2 + 6t )dt =  20 − 11 + 6  = 5 − + 3 =
0  4 3 2  0 3 3

Definition 5.2  Work done by a force B


If F ( x, y, z ) is a force acting on a particle which is moved along arc AB then ∫ F ⋅ dr gives the total
work done by the force F in displacing the particle from A to B. A

Conservative force field


A line integral ∫ F ⋅ dr is independent of path in domain D if and only if F = ∇f for some scalar
C
function f defined in D. Such a force field is called a conservative field.
In the conservative field the total work done by F from A to B is B

∫ F ⋅ dr = C∫ ∇f ⋅ dr
C
 ∂f ∂f ∂f 
= ∫ dx + dy + dz
C  ∂ x ∂ y ∂z 
B
= ∫ df = ∫ df
C A A
Fig. 5.3
\ ∫ F ⋅ dr = [f] = f( B) − f( A)
B
A
C

M03_Eng-Maths (Aditya) CH05.indd 4 7/23/2018 4:53:56 PM


Vector Integration  n  5.5

So, in a conservative field the work done depends on the value of f at the end points A and B of the
path, but not on the path.
Note
1. f is scalar potential.
2. If F is conservative, then F = ∇f  ⇒  ∇ × F = ∇ × ∇f = 0
\
F is irrotational.
3. If C is a simple closed curve and F is conservative, then ∫ F ⋅ dr = 0 .
C

WORKED EXAMPLES
Example 5
Show that F 5 ( e x z 2 2 xy ) i 2 ( x 2 2 1) j 1 ( e x 1 z ) k is a conservative field. Hence, evaluate

C
∫ F ? dr where the end points of C are (0, 1, -1) and (2, 3, 0).
Solution.
To prove that F is conservative, we have to prove ∇ × F = 0
i j k
∂ ∂ ∂
Now ∇× F =
∂x ∂y ∂z
e x z − 2 xy 1 − x 2 ex + z
= i[0] − j (e x − e x ) + k ( −2 x + 2 x ) = 0
Hence, F is conservative. \  F = ∇f
∂f ∂f ∂f
⇒ (e x z − 2 xy )i + (1 − x 2 ) j + (e x + z )k = i +j +k
∂x ∂y ∂z
∂f ∂f ∂f
\ = e x z − 2 xy   (1) = 1 − x 2   (2) = e x + z (3)
∂x ∂y ∂z
Integrating (1) w. r. to x,  f = zex - x2y + f1(y, z)
Integrating (2) w. r. to y,  f = (1 - x2)y + f2(x, z)
z2
f = ex z +
Integrating (3) w. r. to z,    + f3 ( x, y )
2
z2
\ f = ze x − x 2 y + y + +C
2
∫ F ⋅ dr = [ f ]
(2, 3, 0)
\ (0, 1, −1)
C

z2
= [ze x − x 2 y + y + + c ](( 02,,13,,−01))
2
  1  1 19
= 0 − 22 ⋅ 3 + 3 + C −  −1 − 0 + 1 + + C   = −12 + 3 − = − .
  2   2 2

M03_Eng-Maths (Aditya) CH05.indd 5 7/23/2018 4:54:01 PM


5.6  n  Engineering Mathematics-III

Example 6
If F 5 (4 xy 2 3 x 2 z 2 ) i 1 2 x 2 j 2 2 x 3 z k , then check whether the integral ∫ F ? d r is independent
of the path C. C

Solution.
Given F = (4 xy − 3x 2 z 2 ) i + 2x 2 j − 2x 3 zk

i j k
∂ ∂ ∂
Now ∇× F =
∂x ∂y ∂z
4 xy − 3 x 2 z 2 2x2 −2 x 3 z

∂ ∂ 
= i  ( −2 x 3 z ) − (2 x 2 ) − j
 ∂y ∂z 

∂x { ∂
}
( −2 x 3 z ) − (4 xy − 3 x 2 z 2 )
∂z
∂ ∂ 
+ k  (2 x 2 ) − (4 xy − 3 x 2 z 2 )
 ∂ x ∂ y 
= i{0 − 0} − j{−6 x 2 z + 6 x 2 z} + k{4 x − 4 x} = 0

\  F is conservative.
Hence, ∫ F ⋅ dr is independent of the path C.
C

Example 7
Show that F 5 (2 xy 1 z 3 ) i 1 x 2 j 1 3xz 2 k is a conservative field. Find the scalar potential and
work done in moving an object in this field from (1, -2, 1) to (3, 1, 4).

Solution.
Given F = (2 xy + z 3 )i + x 2 j + 3 xz 2 k

i j k
∂ ∂ ∂
Now ∇× F =
∂x ∂y ∂z
2 xy + z 3 x2 3 xz 2

∂ ∂  ∂ ∂ 
= i  (3 xz 2 ) − ( x 2 )  − j  (3 xz 2 ) − (2 xy + z 3 ) 
 ∂y ∂z   ∂x ∂z 
∂ ∂ 
+ k  ( x 2 ) − (2 xy + z 3 ) 
 ∂ x ∂ y 
= i[0 − 0] − j[3 z 2 − 3 z 2 ] + k [2 x − 2 x ] = 0

\  F is conservative.
So, there exists a scalar function f such that F = ∇f.

M03_Eng-Maths (Aditya) CH05.indd 6 7/23/2018 4:54:04 PM


Vector Integration  n  5.7

∂f ∂f ∂f
⇒ (2 xy + z 3 )i + x 2 j + 3 xz 2 k = i +j +k
∂x ∂y ∂z
∂f ∂f ∂f
\ = 2 xy + z 3  (1) = x 2  (2) = 3 xz 2  (3)
∂x ∂y ∂z
Integrating (1) partially w.r.to x,  f = x2y + z3x + f1(y, z)
Integrating (2) partially w.r.to y,  f = x2y + f2(x, z)
Integrating (3) partially w.r.to z,   f = xz3 + f3(x, y)
\   f = x2y + xz3 + C
Since F is conservative, work done by the force F from (1, -2, 1) to (3, 1, 4) is equal to

= [f](1, − 2, 1) =  x 2 y + xz 3 + C  (1, − 2, 1)
( 3, 1, 4 ) ( 3, 1, 4 )


= 32 ⋅1 + 3 ⋅ 43 + C − (12 ( −2) + 1⋅13 ) + C  = 9 + 192 + 1 = 202 units.

EXERCISE 5.1
1. Prove that if F = f∇c, then F ⋅ (∇ × F ) = 0.
2. Prove that Curl (f grad f) = 0.
3. Show that ∇ ⋅(f∇c − c∇f) = f∇ 2 c − c∇ 2f.
4. Prove that ∇ × (f∇c) = ∇f × ∇c .

5. Prove that ∇ × [ f ( r )r ] = 0.

∫ F ⋅ dr, where F = (2 xy + z )i + x j + 3 xz 2 k, along the straight line joining the points


3 2
6. Evaluate
C
(1, -2, 1) and (3, 2, 4).

7. Find
C
∫ F ⋅ dr where F = (2 y + 3)i + xz j + ( yz − x)k along the line joining the points (0, 0, 0) to
(2, 1, 1).
8. Find the work done in moving a particle in the force field F = 3 x 2 i + (2 xz − y ) j − zk from t = 0 to
t = 1 along the curve x = 2t2, y = t, z = 4t3.
9. Show that F = (2 xy + z 3 )i + x 2 j + 3 xz 2 k is conservative. Find its scalar potential and find the
work done in moving a particle from (1, -2, 1) to (3, 1, 2).
10. Find the work done by the force F = − xyi + y 2 j + zk in moving a particle over a circular path
x 2 + y 2 = 4, z = 0 from (2, 0, 0) to (0, 2, 0).
11. Find the work done when a force F = ( x 2 − y 2 + x )i − (2 xy + y ) j moves a particle in the xy plane
from (0, 0) to (1, 1) along the curve y2 = x. If the path is y = x, whether the work done is different
or same. If it is same, state the reason.
12. Find the total work done in moving a particle in a force field given by F = 3 xyi − 5 z j + 10 xk
along the curve x = t2 + 1, y = 2t2, z = t3 from t = 1 to t = 2.

M03_Eng-Maths (Aditya) CH05.indd 7 7/23/2018 4:54:10 PM


5.8  n  Engineering Mathematics-III

13. For the vector function F = 2 xyi + ( x 2 + 2 yz ) j + ( y 2 + 1)k determine ∫ F ⋅ dr


C
around the unit
circle with centre at the origin in the xy plane.
14. Evaluate ∫ F ⋅ dr, where F = ( x − 3 y)i + ( x − 2 y) j and C is the closed curve in the xy plane.
C
x = 2 cos t, y = 2 sin t and t = 0 to t = 2p.

15. Prove that ∇ 2   = 0.


1
 r
16. Prove that ∇ × (∇r n ) = 0.

17. If F = 5 xyi + 2 y j , then evaluate ∫ F ⋅ dr, where C is the part of the curve y = x2 between x = 1 and
C
x = 2.
18. Show that the vector field F, where
F = ( y + y 2 + z 2 )i + ( x + z + 2 xy ) j + ( y + 2 xz )k, is conservative and find its scalar potential.

ANSWERS TO EXERCISE 5.1


13
6. 211 [Hint: F is conservative] 7.  5 8.  9. 34
6
16 −2 −2
10. 11.  , , F is conservative. 12.  303
3 3 3
135
13. 0 14. 24p 17. 
4
18.  f = xy + xy2 + yz + xz2 + c.

5.2  Green’s Theorem in a Plane


Green’s theorem gives a relation between a double integral over a region R in the xy plane and the line
integral over a closed curve C enclosing the region R. It helps to evaluate line integral easily.
Statement of Green’s theorem
If P(x, y) and Q(x, y) are continuous functions with continuous partial derivatives in a region R in the
xy plane and on its boundary C which is a simple closed curve then
 ∂Q ∂P 
∫C (P dx 1Q dy ) 5R∫∫  ∂x 2 ∂ y  dxdy
where C is described in the anticlockwise sense (which is the positive sense).

Green’s theorem in a plane


Proof  Let R be the region in the xy-plane bounded by the simple closed curve C traced in the
anticlockwise sense, which is the positive sense. We assume any line parallel to the axes meet the
curve in not more than two points. The curve C consists of two arcs APB and BQA as in figure.
Let y = f1(x) and y = f2(x) be the equations of these arcs.

M03_Eng-Maths (Aditya) CH05.indd 8 7/23/2018 4:54:15 PM


Vector Integration  n  5.9

Clearly, f1(x) ≤ f2(x) in [a, b]


y = f2(x)
b  f2 ( x )
∂P ∂P  Q
Now, ∫∫R ∂y dx dy = ∫a  f ∫( x ) ∂y dy  dx
1 
A R B
b f2 ( x )

= ∫ [ P ( x, y )] dx
a f1 ( x )
P
C
b y = f1(x)
= ∫  P (( x, f 2 ( x ) ) − P (( x, f1 ( x ) ) dx
a
b b
x=a x=b
= ∫ P (( x, f 2 ( x ) ) dx − ∫ P (( x, f1 ( x ) ) dx O
a a
Fig. 5.4
b

However, ∫ P ( x, f
a
2 ( x ) ) dx is numerically equal to the line integral
∫ P ( x, y ) dx taken along the
AQB

curve AQB.

But the positive sense is BQA (anticlockwise)


b
\
∫ P (x , f
a
2 ( x ) ) dx = − ∫
BQA
P( x , y ) dx

Similarly, ∫ P ( x, f ( x)) dx = ∫
a
1
APQ
P ( x, y ) dx

\ ∂P
∫∫ ∂y dy = − ∫
R APB
P ( x, y )dx − ∫
BQA
P ( x, y )dx

 
= −  ∫ P( x , y )dx + ∫ P( x , y )dx  = − ∫ P( x , y ) dx
 A PB BQA  C

∂P
⇒ ∫ P ( x, y)dx = −∫∫ ∂y dx dy
C R
(1)

Now, we regard the curve C as constituted of the arcs QAP and PBQ.
Let their equations be x = f1(y) and x = f2(y)

Then f1(y) ≤ f2(y) in [c, d]

∂Q
y=d
 x = f2 ( y ) ∂Q 
∫∫R ∂x dx dy = ∫  x =f∫ ( y ) ∂x dx  dy
y=c 
 1 

M03_Eng-Maths (Aditya) CH05.indd 9 7/23/2018 4:54:17 PM


5.10  n  Engineering Mathematics-III

d Q
= ∫ [ Q( x, y ) ]x = f2 ( y ) dy y=d
x =f ( y )

c
1
x = φ1(y) x = φ2(y)
A R B
d
y=c
= ∫ [Q(f2 ( y ), y ) − f (f1 ( y ), y ) ] dy P
c C

d d

= ∫ Q(f2 ( y ), y ) dy − ∫ Q (f2 ( y ), y ) dy
c c
O

d
Fig. 5.5
But, ∫ Q (f2 ( y ), y ) dy is the line integral ∫ Q( x, y ) dy
c PBQ
d

and ∫ Q (f ( y), y) dy
c
2 is the line integral ∫ Q( x, y ) dy
PAQ

However, the positive sense of arc is QAP.


d
\ ∫ Q(f ( y), y) dy = − ∫
c
2
QAP
Q( x, y ) dy

∂Q
\ ∫∫ ∂x dx dy = ∫
R PBQ
Q( x, y )dy + ∫
QAP
Q( x, y )dy = ∫ Q( x, y )dy
C

∂Q
\ ∫C Q( x, y)dy = ∫∫R ∂x dx dy  (2)

Adding the equations (1) and (2), we get

∂P ∂Q
∫ P ( x, y)dx + ∫ Q( x, y)dy = −∫∫ ∂y dx dy + ∫∫ ∂x dx dy
C C R R

⇒  ∂Q ∂P 
∫ P dx + Q dy = ∫∫  ∂x − ∂y  dx dy
C R

Note  We have proved the theorem by taking a simple closed region. The theorem is also valid in a
region which can be divided into regions enclosed by simple closed curves.

1
Corollary  Area of the region R bounded by C is = ∫∫ dxdy = 2 ∫ (xdy − ydx )
R C

∂P ∂Q
Proof  In Green’s theorem, take P = -y  and   Q = x. \  = −1 and =1
∂y ∂x
Then
∫ ( − ydx + xdy ) = R∫∫ (1 + 1)dxdy = 2R∫∫ dxdy
C

\ 1
2 C∫
( xdy − ydx ) = ∫∫ dxdy n
R

M03_Eng-Maths (Aditya) CH05.indd 10 7/23/2018 4:54:21 PM


Vector Integration  n  5.11

5.2.1  Vector Form of Green’s Theorem


Let F = Pi + Q j   and   r = xi + y j

\ d r = dx i + dy j and  F.d r = P dx + Q dx

i j k
∂ ∂ ∂  ∂Q   ∂P   ∂Q ∂P 
Now, ∇× F = = i 0 − − j 0 − +k  − 
∂x ∂y ∂z  ∂z  

∂z   ∂x ∂y 
P Q 0
 ∂Q ∂P   ∂Q ∂P   ∂Q ∂P 
= i (0) − j (0) + k  − k
= −
 ∂x ∂y   ∂x ∂y  { ∂z = 0; ∂z = 0 
 

\ ∂Q ∂P
∇× F ⋅k = −
∂x ∂y

\ Green’s theorem becomes ∫ F ⋅ dr = ∫∫ ∇ × F ⋅ k d R,   where d R = dx dy


C R

WORKED EXAMPLES
Example 1

∫ [( x
2
Using Green’s theorem evaluate 2 y 2 )dx 1 2 xydy ], where C is the closed curve of the
C
region bounded by y2 5 x and x2 5 y.

Solution.
 ∂Q ∂P  y
Green’s theorem is ∫ ( Pdx + Qdy) = R∫∫  ∂x − ∂y  dxdy.
C
y = x2

The given line integral is ∫ [( x 2 − y 2 )dx + 2 xydy ] y2 = x


A
C
Q
Here P = x − y and
2 2
Q = 2 xy
P
o x=1 x
∂P ∂Q
\ = −2 y and = 2y
∂y ∂x

∂Q ∂P
\ − = 2y + 2y = 4y
∂x ∂y
Fig. 5.6

M03_Eng-Maths (Aditya) CH05.indd 11 7/23/2018 4:54:26 PM


5.12  n  Engineering Mathematics-III

\ ∫ ( x 2 − y 2 ) dx + 2 xydy = ∫∫ 4 ydxdy
C R
1 x

=∫ ∫ 4 ydydx
0 x2
x 1
1
 y2  1
 x 2 x5  1 1 3
= 4∫   dx = 2∫ ( x − x 4 )dx = 2  −  = 2  −  =
0
 2  x2 0  2 5  0  2 5 5

Example 2
p  p 
Evaluate ∫ [(sin x 2 y )dx 2 cos xdy ],
C
where C is the triangle with vertices ( 0 , 0 ),  , 0  and  , 1
2  2 
p  p 
( 0 , 0 ),  , 0  and  , 1 .
2  2 

Solution.
Green’s theorem is ∫ ( Pdx + Qdy ) =  ∂Q ∂P 
C
∫∫  − ∂y  dxdy
R  ∂x

Given line integral is ∫ [(sin x − y )dx − cos xdy ]


C
Here P = sin x - y and   
Q = -cos x
∂P ∂Q
\ = −1    and  = sin x
∂y ∂x
∂Q ∂P
\ − = sin x + 1
∂x ∂y
∫ [(sin x − y)dx − cos xdy] = R∫∫ (sin x + 1)dxdy
C

Equation of OB is y − 0 = x − 0   ⇒  y =
2x
0 −1 p p
0− y y=
2x
2 y=1 π
p π 
B  , 1
Equation of AB is x= 2 
2
py p π
x=
In this region R, x varies from to
2 2 P Q
and y varies from 0 to 1.

1 p/2

\ ∫ [(sin x − y )dx − cos xdy ] = ∫  ∫ (sin x + 1)dx  dy o
π
x
(0, 0)
C 0 
 py / 2 
1

= ∫ [ − cos x + x ]py / 2 dy
p/2
Fig. 5.7
0
1
 p p  py py  
= ∫  − cos +  −  − cos +  dy
0 
 2 2   2 2  
1
p py py 
= ∫  + cos −  dy
2 2 2 
0
M03_Eng-Maths (Aditya) CH05.indd 12 1 7/23/2018 4:54:29 PM
1 Vector Integration  n  5.13
= ∫ [ − cos x + x ]py / 2 dy
p/2

0
1
 p p  py py  
= ∫  − cos +  −  − cos +  dy

0
2 2  2 2  
1
p py py 
= ∫  + cos −  dy
2 2 2 
0
1
 py 
p sin
2 p y2  p 2 p p p 2 p 2 p
=  y+ −  = + sin − = + − = +
 2 p 2 2  2 p 2 4 2 p 4 p 4
 2 0

Example 3

∫e
2x
Evaluate by Green’s theorem (sin ydx 1 cos ydy ), C being the rectangle with vertices
C
 p  p
(0, 0), (p, 0),  p,  and  0 ,  .
 2  2

Solution.
Green’s theorem is ∫ ( Pdx + Qdy ) =  ∂Q ∂P 
C
∫∫  − ∂y  dxdy
R  ∂x

The given line integral is ∫ e − x (sin ydx + cos ydy )


C

Here P = e-x sin y   and  Q = e-x cos y

∂P ∂Q
\ = e − x cos y  and  = −e − x cos y
∂y ∂x

∂Q ∂P
\ − = −e − x cos y − e − x cos y = −2e − x cos y
∂x ∂y
y

\ e − x (sin ydx + cos ydy ) =


∫ ∫∫ −2e
−x
cos ydxdy  π C  π
C R 0,  B π, 
 2
2
p
2 p x=0 x=π
= −2 ∫ ∫ e − x cos y dxdy
0 0 A
O (π, 0) x
 p
 p
  (0, 0)
2

= −2  ∫ cos y dy   ∫ e − x dx  Fig. 5.8
0  0 
p
p/2  e 
−x
 p  −p
= −2 [sin y ]0  −p
 = 2  sin  (e − e ) = 2(e − 1)
0

 −1  0 2

M03_Eng-Maths (Aditya) CH05.indd 13 7/23/2018 4:54:32 PM


5.14  n  Engineering Mathematics-III

Example 4
Find the area bounded between the curves y2 5 4x and x2 5 4y using Green’s theorem.

Solution.
We know, by Green’s theorem the area bounded by a simple closed curve C is
1
2 C∫
( xdy − ydx )

Here C consists of the curves C1 and C2.


1 1
\  area =  ∫ xdy − ydx + ∫ xdy − ydx  = [ I1 + I 2 ]
2 C1 C2 
 2
On C1:  x2 = 4y
1 y
\ 2xdx = 4dy  ⇒  dy = xdx
2
and x varies from 0 to 4. x 2 = 4y
y 2 = 4x

\ I1 = ∫ xdy − ydx
C1
y=4 P
(4, 4)
C2
4 2
1 x
= ∫ x⋅ xdx − dx x=4
0
2 4 C1
4
 x2 x2 
4 4 2
x 1  x3  o
= ∫  −  dx = ∫ dx =   x
 2 4 4 4  3 0 (0, 0)
0 0

64 16
= =
4⋅3 3
1
On C2: y2 = 4x  \ 2ydy = 4dx  ⇒  dx = ydy
2 Fig. 5.9
and y varies from 4 to 0.

\ I2 = ∫ xdy − ydx
C2

0
y2 1
=∫ dy − y ⋅ y dy
4
4 2
4
 y2 y2 
0 0
y2 1
4
1  y3  16
= ∫  −  d y = ∫ − dy = ∫ y 2 dy =   =
 4 2  4 40 4  3 0 3
4 4

1 16 16  16
\ area = + =
2  3 3  3

Example 5

∫ (3x
2
Verify Green’s theorem in the plane for 2 8 y 2 )dx 1 ( 4 y 2 6 xy )dy , where C is the
C
boundary of the region bounded by x 5 0, y 5 0, x 1 y 5 1.

M03_Eng-Maths (Aditya) CH05.indd 14 7/23/2018 4:54:35 PM


Vector Integration  n  5.15

Solution.
Green’s theorem is ∫ ( Pdx + Qdy ) =  ∂Q ∂P 
C
∫∫  − ∂y  dxdy
R  ∂x
y
The given integral is ∫ (3 x 2 − 8 y 2 )dx + ( 4 y − 6 xy )dy
C (0, 1)
B
Here P = 3x - 8y  and  Q = 4y - 6xy
2 2
x+y=1
∂P ∂Q
\ = −16 y and = −6 y
∂y ∂x
∂Q ∂P
\ − = −6 y + 16 y = 10 y (1, 0)
∂x ∂y o x
A
1 1− x
 ∂Q ∂P 
\ ∫∫  − ∂y  dxdy = ∫ ∫ 10 y dy dx
R  ∂x 0 0
Fig. 5.10
1− x 1
 y2 
1
 (1 − x )3 
1
−5 5
= 10∫   dx = 5∫ (1 − x ) 2 dx = 5   = [0 − 1] =
0
 2  0 0  −3  0 3 3

⇒  ∂Q ∂P  5
∫∫  − ∂y  dxdy = 3
R  ∂x
(1)


We shall now compute the line integral ∫ Pdx + Qdy


C

∫ Pdx + Qdy = C∫ (3x − 8 y 2 )dx + ( 4 y − 6 xy )dy


2
Now
C

= ∫ (3 x − 8 y 2 )dx + ( 4 y − 6 xy )dy + ∫ (3 x − 8 y 2 )dx + ( 4 y − 6 xy )dy


2 2

OA AB

+ ∫ (3 x − 8 y 2 )dx + ( 4 y − 6 xy )dy = I1 + I 2 + I 3
2

BO

On OA: y = 0  \  dy = 0 and x varies from 0 to 1.


1
1
 x3 
\ I1 = ∫ 3 x dx = 3   = 1 2

0  3 0

On AB: x + y = 1  ⇒  y = 1 - x  \  dy = -dx and x varies 1 to 0.


0

\ I 2 = ∫ (3 x 2 − 8(1 − x ) 2 )dx + [4(1 − x ) − 6 x(1 − x )]( − dx )


1

= ∫ [3 x 2 − 8(1 − x ) 2 − 4(1 − x ) + 6( x − x 2 )]dx


1
0
 8(1 − x )3 (1 − x ) 2  x 2 x3  
=  x3 − −4 + 6 − 
 −3 −2  2 3  1
 8   1 1  8 8
= 0 + + 2 + 0 − 1 + 6  −    = + 2 − 1 − 1 =
 3   
2 3  3 3
M03_Eng-Maths (Aditya) CH05.indd 15 7/23/2018 4:54:38 PM
5.16  n  Engineering 0Mathematics-III
= ∫ [3 x 2 − 8(1 − x ) 2 − 4(1 − x ) + 6( x − x 2 )]dx
1
0
 8(1 − x )3 (1 − x ) 2  x 2 x3  
=  x3 − −4 + 6 − 
 −3 −2  2 3  1
 8   1 1  8 8
= 0 + + 2 + 0 − 1 + 6  −    = + 2 − 1 − 1 =
 3   2 3   3 3

On BO: x = 0  \  dx = 0 and y varies from 1 to 0
0
0
\ I 3 = ∫ 4 ydy = 2  y 2 1 = −2
1

8 5
\ ∫ Pdx + Qdy = C∫ (3x − 8 y 2 )dx + ( 4 y − 6 xy )dy = 1 + − 2 = 
2
(2)
3 3

(1) and (2) give the same value. Hence, Green’s theorem is verified.

Example 6

∫ ( xy 1 y
2
Verify Green’s theorem for ) dx 1 x 2 dy , where C is the boundary, of the area between
C
y 5 x2 and y 5 x.

Solution. y
Green’s theorem is
y = x2
 ∂Q ∂P 
∫ ( Pdx + Qdy) =
C
∫∫ 
R  ∂x
−  dxdy
∂y  y=x

The given line integral is

∫ ( xy + y ) dx + x 2 dy A (1,1)
2

C
Q
Here P = xy + y2  and   Q = x2 x=1
∂P ∂Q
\ = x + 2 y  and  = 2x P
∂y ∂x
O (0,0) (1,0) x
∂Q ∂P
\ − = 2x − x − 2 y = x − 2 y
∂x ∂y
Fig. 5.11
x
 ∂Q ∂P   2 y2 
1 x 1
\ ∫∫  −  dxdy = ∫ ∫ ( x − 2 y ) dy dx = ∫  xy −  dx
R  ∂x ∂y  0 x2 0
 2  x2
1
1 1
 x5 x 4  1 1 1
= ∫ [ x 2 − x 2 − ( x 3 − x 4 )] dx = ∫ [( x 4 − x 3 )] dx =  −  = − = −
0 0  5 4  0 5 4 20
 ∂Q ∂P  1
∫∫  − ∂y  dxdy = − 20
R  ∂x 
(1)

M03_Eng-Maths (Aditya) CH05.indd 16 7/23/2018 4:54:40 PM


Vector Integration  n  5.17

We shall now compute the line integral ∫ Pdx + Qdy


C

∫ Pdx + Qdy = C∫ ( xy + y ) dx + x 2 dy
2
Now
C
= ∫ ( xy + y ) dx + x 2 dy + ∫ ( xy + y ) dx + x 2 dy = I1 + I 2
2 2

C 1 C2

On C1: y = x2, \  dy = 2x dx and x varies from 0 to 1.


1

I1 = ∫ ( x ⋅ x 2 + x 4 ) dx + x 2 ⋅ 2 x dx
0

1
= ∫ ( x 3 + x 4 + 2 x 3 ) dx
0
1
1
 x 4 x5  3 1 19
= ∫ (3 x 3 + x 4 ) dx = 3 +  = + =
0  4 5  0 4 5 200
On C2: y = x, \  dy = dx and x varies from 1 to 0.
0
0
 x3 
1

\ I 2 = ∫ ( x ⋅ x + x ) dx + x dx = ∫ 3 x dx = 3   = −1
2 2 2

1 0  3 1

19 1
\ ∫ Pdx + Qdy = 20 − 1 = − 20 (2)
C

(1) and (2) give the same value.


Hence, Green’s theorem is verified.

5.3  Surface Integrals


Suppose a surface is bounded by a simple closed curve C, then we can regard the surface as having
two sides separated by C. One of which is arbitrarily chosen as the positive side and the other is the
negative side. If the surface is a closed surface, then the outerside is taken as the positive side and the
inner side is the negative side. A unit normal at any point of the positive side of the surface is denoted
by n and is called the outward drawn normal and its direction is considered positive.
Any integral which is evaluated over a surface is called a
surface integral.

F
Definition 5.3  Surface Integral n
Let S be a surface of finite area which is smooth or piece- P
wise smooth (e.g. a sphere is a smooth surface and a cube is a ds
piecewise smooth surface). Let F ( x, y, z ) be a vector point
function defined at each point of S. Let P be any point on the S
surface and let n be the outward unit normal at P. Then the
surface integral of F over S is defined as ∫∫ F ⋅ n dS
S Fig. 5.12

M03_Eng-Maths (Aditya) CH05.indd 17 7/23/2018 4:54:43 PM


5.18  n  Engineering Mathematics-III

If we associate a vector dS (called vector area) with the differential of surface area dS such that
dS = dS and direction of dS is n, then
dS = n dS
\  ∫∫ F ⋅ n dS can also be written as ∫∫ F ⋅ dS
S S

Note
1. In physical application the integral ∫∫ F ⋅ dS is called the normal flux of F through the surface S,
S
because this integral is a measure of the volume emerging from S per unit time.

5.3.1 Evaluation of surface integral


To evaluate a surface integral over a surface it is usually expressed as a double integral over the
orthogonal projection of S on one of the coordinate planes. This is possible if any line perpendicular
to the coordinate plane chosen meets the surface S in not more than one point.
Let R be the orthogonal projection of S on the xy plane.
Then the element surface dS is projected to an element area dx dy in the xy plane as in fig.
\  dx dy = dS cos u, where u is the angle between the planes of dS and xy-plane.
Let n be the unit normal to dS and k is the unit normal to the xy-plane.
Since angle between the planes is equal to the angle between the normals,
u is the angle between the normals n and k .
n⋅k
\ cos u =
n k k
z n
= n ⋅ k [Since n = 1, k = 1]
ds
We take the acute angle between the normals and P
So, we take n ⋅ k
dx dy
\ dx dy = dS n ⋅ k   ⇒  dS =
n⋅k
dx dy o
Hence, ∫∫ F ⋅ n dS = ∫∫ F ⋅ n y
S n⋅k
R
Similarly, taking the projection on the yz and zx
planes, we get dx

dy dz dy
∫∫
S
F ⋅ n dS = ∫∫ F ⋅ n
R n⋅i
x

Fig. 5.13
dz dx
and ∫∫ F ⋅ n dS =
S
∫∫ F ⋅ n
R n⋅ j
Corollary
dx dy dy dz dz dx
The surface area ∫∫ dS = R∫∫
S n⋅k
= ∫∫
R n⋅i
= ∫∫
R n⋅ j
1 2

M03_Eng-Maths (Aditya) CH05.indd 18 7/23/2018 4:54:50 PM


Vector Integration  n  5.19

5.4  Volume Integral


Any integral which is evaluated over a volume bounded by a surface is called a volume integral.
If V is the volume bounded by a surface S, then

∫∫∫
V
f( x, y, z )dV and ∫∫∫ F dV are called volume integrals.
V
If we divide V into rectangular blocks by drawing planes parallel to the coordinate planes, then
dV = dx dy dz.

\ ∫∫∫
V
fdV = ∫∫∫ f( x, y, z ) dxdydz
V

If F = F1 i + F2 j + F3 k

then ∫∫∫
V
FdV = i ∫∫∫ F dxdydz + j ∫∫∫ F dxdydz + k ∫∫∫ F dxdydz
V
1
V
2
V
3

WORKED EXAMPLES
Example 1
Evaluate ∫∫ F ⋅ n dS if F 54 yi 118 z j 2 xk and S is the surface of the plane 3x 1 2y 1 6z 5 6
S
contained in the first octant.

Solution.
Given F = 4 yi + 18 z j − xk and the surface 3x + 2y + 6z = 6. z
C
Let f = 3x + 2y + 6z

Let R be the projection of S in the xy plane. n


S
\  R is the ∆ AOB
\ dx dy B
∫∫ F ⋅ n dS = R∫∫ F ⋅ n
S n⋅k
o
R y

3x + 3y = 6
where n is unit normal to S and k is the unit normal to xy-plane. A
x
Fig. 5.14
Normal to the surface is ∇f = i ∂f + j ∂f + k ∂f = 3i + 2 j + 6 k
∂x ∂y ∂z

∇f 3i + 2 j + 6 k 1
\ unit normal is   n = = = (3i + 2 j + 6 k )
∇f 9 + 4 + 36 7
1
\ F ⋅ n = (4 yi + 18 z j − xk ) ⋅ (3i + 2 j + 6k )
7
1 6
= (12 y + 36 z − 6 x ) = ( 2 y + 6 z − x )
7 7

M03_Eng-Maths (Aditya) CH05.indd 19 7/23/2018 4:54:54 PM


5.20  n  Engineering Mathematics-III

1 6
n ⋅ k = (3i + 2 j + 6 k ) ⋅ k =
7 7
6 dx dy
\ ∫∫ F ⋅ n dS = R∫∫ 7 (2 y + 6 z − x) 6
= ∫∫ (2 y + 6 z − x)dx dy
R
S
7
y
We have 3x + 2y + 6z = 6
B
⇒ 6z = 6 - 3x - 2y (0, 3)

\ 2y + 6z - x = 2y + 6 - 3x - 2y - x = 6 - 4x
\ 3x + 2y = 6
∫∫ F ⋅ n dS = R∫∫ (6 − 4 x) dx dy
S

The plane 3x + 2y + 6z = 6 meets the xy-plane z = 0 in


line AB.
\  the equation of AB is 3x + 2y = 6 O A (2, 0) x
R
\  the point A is (2, 0) and the point B is (0, 3)
Fig. 5.15
6 − 3x
Now 3x + 2y = 6  ⇒ y=
2

6 − 3x
\ In R, x varies from 0 to 2 and y varies from 0 to
2
6 −3 x 6 −3x
2 2 2 2
\ (6 − 4 x ) dy dx = 2∫
∫∫ F ⋅ n dS = ∫ ∫ ∫ (3 − 2 x ) dy dx
S 0 0 0 0
6 −3x
2 2
= 2 ∫ [ (3 − 2 x ) y ] dx
0 0
2
(6 − 3 x )
= 2∫ (3 − 2 x ) dx
0
2
2

= 3∫ (3 − 2 x )( 2 − x ) dx
0

= 3∫ (6 − 7 x + 2 x 2 ) dx
0
2
 7x2 x3 
= 3 6 x − +2 
 2 3 0
 4 8  16 
= 3 6 × 2 − 7 × + 2 ×  = 3 12 − 14 +  = −6 + 16 = 10
 2 3  3

M03_Eng-Maths (Aditya) CH05.indd 20 7/23/2018 4:54:57 PM


Vector Integration  n  5.21

Example 2

Evaluate ∫∫ F ⋅ n dS
S
if F 5 yz i 1 zx j 1 xyk and S is part of the surface x2 1 y2 1 z2 5 1,

which lies in the first octant.

Solution. z

Given F = yz i + zx j + xyk and the surface is x2 + y2 + z2 = 1

Let f = x2 + y2 + z2
∂f ∂f ∂f
The normal to the surface is ∇f = i +j +k
∂x ∂y ∂z o
y
= 2 xi + 2 y j + 2 zk R
x2 + y2 = 1
\  unit normal is ∇f 2 xi + 2 y j + 2 zk
n= = x
∇f 4x2 + 4 y2 + 4z2 Fig. 5.16
2( xi + y j + zk ) [{ x2 + y2 + z2 = 1]
= = xi + y j + zk
2 x +y +z 2 2 2

\ F ⋅ n = ( yzi + zx j + xyk ) ⋅ ( xi + y j + zk )
= xyz + xyz + xyz = 3 xyz

The projection of the surface of the sphere in the first octant into the xy plane is R, which is the
quadrant of the circle x2 + y2 = 1, z = 0, x ≥ 0, y ≥ 0 and k is the unit normal to R.
dx dy y
\ ∫∫ F ⋅ n dS =
S
∫∫ F ⋅ n
R n⋅k
dx dy x2 + y 2 = 1
= ∫∫ 3xyz
R n⋅k
But n ⋅ k = ( xi + y j + zk ) ⋅ k = z
\ 1
∫∫ F ⋅ n dS = R∫∫ 3xyz
S z
dx dy
O (1, 0) x
R
= ∫∫ 3xy dx dy
R
1 1− x 2 Fig. 5.17
=∫ ∫ 3xy dx dy
0 0
1− x 2
y 2 
1
3
1

= 3∫  
2 ∫0
xdx = x (1 − x 2 )dx
0
 2 0
1
3 x2 x4  3  1 1 3 1 3
=  −  =  −  = ⋅ =
2 2 4 0 2 2 4 2 4 8

M03_Eng-Maths (Aditya) CH05.indd 21 7/23/2018 4:55:02 PM


5.22  n  Engineering Mathematics-III

Example 3

Evaluate ∫∫ F ⋅ n dS, where F 5 4 xz i 2 y 2 j 1 yzk and S is the surface of the cube bounded by the
planes x 5 0, x 5 1, y 5 0, y 5 1, z 5 0, z 5 1.

Solution.
Given F = 4xz i − y 2 j + yzk
S is the surface of the cube, which is piecewise smooth surface consisting of six smooth surfaces.

∫∫ F ⋅ n dS = ABEF
S
∫∫ F ⋅ n dS + OCDG
∫∫ F ⋅ n dS z

+ ∫∫
BCDE
F ⋅ n dS + ∫∫
OAFG
F ⋅ n dS G D

F
+ ∫∫ F ⋅ n dS + DEFG
OABC
∫∫ F ⋅ n dS k
E

j
o
On the face ABEF: x = 1, n=i C y
i
\ F ⋅ n = (4 xzi − y 2 j + yzk ) ⋅ i = 4 xz = 4 z A B
and dy dz dy dz x
dS = = = dy dz
n ⋅i i ⋅i Fig. 5.18
1 1 1
 z2  1
\ ∫∫ F ⋅ n dS = ∫ ∫ 4 z dz dy = 4 ⋅ [ y ]
1
0   = 4 ⋅1⋅ = 2
ABEF 0 0  2 0 2

On the face OCDG: x = 0, n = −i


F ⋅ n = (4 xzi − y 2 j + yzk ) ⋅ ( −i ) = −4 xz = 0
\ ∫∫
OCDG
F ⋅ n dS = 0

On the face BCDE: y = 1, n= j


dx dz dx dz
dS = = = dx dz
\ n⋅ j j⋅ j
and F ⋅ n = (4 xzi − y 2 j + yzk ) ⋅ j = − y 2 = −1
1 1

∫∫ F ⋅ n dS = ∫ ∫ ( −1)dx dz = − [ x ]0 [ z ]0 = −1
1 1
\
BCDE 0 0

On the face OAFG: y = 0, n=−j


\ F ⋅ n = (4 xzi − y 2 j + yzk ) ⋅ ( − j ) = y 2 = 0
\ ∫∫
OAFG
F ⋅ n dS = 0

On the face DEFG: z = 1, n=k

M03_Eng-Maths (Aditya) CH05.indd 22 7/23/2018 4:55:08 PM


Vector Integration  n  5.23

\ F ⋅ n = (4 xzi − y 2 j + yzk ) ⋅ k = yz = y
dx dy dx dy
and dS = = = dx dy
n⋅k k ⋅k
1
1 1
 y2  1 1
\ ∫∫ F ⋅ n dS = ∫ ∫ y dx dy = [ x ]0 
1
 = 1× =
 2 0 2 2
DEFG 0 0

On the face OABC: z = 0, n = −k


F ⋅ n = (4 xz i − y 2 j + yzk ) ⋅ ( − k ) = − yz = 0
\ ∫∫
OABC
F ⋅ n dS = 0

\ 1 3
∫∫
S
F ⋅ n dS = 2 + ( −1) + = .
2 2

5.5 Gauss Divergence Theorem


The divergence theorem enables us to convert a surface integral of a vector function on a closed
surface into volume integral.

Statement of Gauss divergence theorem


Let V be the volume bounded by a closed surface S. If a vector function F is continuous and has
continuous partial derivatives inside and on S, then the surface integral of F over S is equal to the
volume integral of divergence of F taken throughout V.

i.e.,
S
∫∫ F ? d S 5 ∫∫∫
V
=? FdV

If n is the outward normal to the surface dS = n dS


z

\
∫∫ F ⋅ n dS = ∫∫∫
S V
∇ ⋅FdV n2
Q S2 z = f2(x, y)
C′
Proof  Let F = F1i + F2 j + F3 k P z = f1(x, y)
S1

\ F ⋅ n = F1 (i ⋅ n ) + F2 ( j ⋅ n ) + F3 ( k ⋅ n ) n1

O
and  F ⋅ n dS = F1 (i ⋅ n )dS + F2 ( j ⋅ n )dS + F3 ( k ⋅ n )dS y

= F1 dy dz + F2 dz dx + F3 dx dy R
C
∂F1 ∂F2 ∂ F3
But ∇⋅F = + + x
∂x ∂y ∂z
Fig. 5.19

M03_Eng-Maths (Aditya) CH05.indd 23 7/23/2018 4:55:14 PM


5.24  n  Engineering Mathematics-III

Hence, Gauss theorem in Cartesian form is


 ∂F ∂F ∂F 
∫∫ ( F dy dz + F
S
1 2 dz dx + F3 dx dy ) ≡ ∫∫∫  1 + 2 + 3  dx dy dz
V
 ∂x ∂y ∂z 
We shall assume that S is a closed surface such that any line drawn parallel to coordinate axes cuts S
in almost two points. The lines drawn parallel to Z-axis touching the surface S determine the curve C ′
on it and intersect the xy-plane along the curve C. Now, the curve C ′ divides the surface S into two
parts S1 and S2.
S1 and S2 are called the lower and upper surfaces.
Let z = f1(x, y) and z = f2(x, y) be the equations of S1 and S2, respectively.
The projection of S on the xy-plane is the region R bounded by C.
∂F
Now consider the triple integral ∫∫∫ 3 dx dy dz over the volume V enclosed by S.
V
∂z

∂F3  z = f2 ( x , y ) ∂F3 

∫∫∫ ∂z
dx dy dz = ∫∫R  z = f ∫( x , y ) ∂z  dx dy
V  1 

= ∫∫ [ F3 ( x, y, z ) ]
z = f2 ( x , y )
dx dy
z = f1 ( x , y )
R

= ∫∫  F3 ( x, y, f 2 ( x, y ) ) − F3 ( x, y, f1 ( x, y ) ) dx dy
R

∂F3

∫∫∫ dx dy dz = ∫∫ F3 ( x, y, f 2 ( x, y ) ) dx dy − ∫∫ F3 ( x, y, f1 ( x, y ) ) dx dy (1)
V
∂ z R R

Let a line parallel to the z-axis meet S1 at the point P and S2 at the point Q. Let dS1 and dS2 be element
surface at P and Q, respectively and their projections in the xy-plane be dx dy.
Let n1 be the outward unit normal at P to S1 and n 2 be the outward unit normal at Q to S2.
Let the angle between n 2 and kbe g 2 and g 2 is acute, since k is unit vector in the direction of
the positive z-axis.
Then dx dy = cosg 2 dS2 = k⋅n 2 d S2
Let the angle between n1and kbe g1and it is obtuse. [{ k is upward and n1 is downward]
\ dx dy = -cosg1 dS1 = - k⋅n1 d S1

Hence, ∫∫ F ( x , y f
R
3 2 ( x , y ) ) dx dy = ∫∫ F3 k ⋅ n 2 dS 2
S2

and ∫∫ F ( x, y f ( x, y)) dx dy = − ∫∫ F k ⋅ n
R
3 1
S1
3 1 dS1

Substituting in (1), we get

∂F3
∫∫∫
V
∂z
dx dy dz = ∫∫ F3 k ⋅ n2 dS2 + ∫∫ F3 k ⋅ n1 dS1
S2 S1

M03_Eng-Maths (Aditya) CH05.indd 24 7/23/2018 4:55:22 PM


Vector Integration  n  5.25

⇒ ∂F3
∫∫∫ dx dy dz = ∫∫ F3 k ⋅ n dS (2)
V
∂z S

Similarly, projecting S on the yz- and zx-planes, we get


∂F2
∫∫∫ ∂y dx dy dz = ∫∫ F2 j ⋅ n dS (3)
V S

∂F1
and ∫∫∫
V
∂x
dx dy dz = ∫∫ F1 i ⋅ n dS (4)
S

Adding equations (2), (3) and (4), we get

 ∂F1 ∂F2 ∂F3 


∫∫∫  ∂x
V
+
∂y
+
∂z 
dx dy dz = ∫∫ ( F1i + F2 j + F3 k ) ⋅ n dS
S

⇒ ∫∫∫ ∇ ⋅ F dV = ∫∫ F ⋅ n dS
V S

5.5.1 Results Derived from Gauss Divergence Theorem


The following results are immediate consequence of Gauss divergence theorem:
(1) ∫∫S f n dS = ∫∫∫
V
∫∫S F × n dS = −∫∫∫
∇ f dV (2)
V
∇ × F dV

where f is the scalar point function defined in the region V enclosed by the closed surface S.

Solution.
(1) ∫∫ f n dS = ∫∫∫ ∇ f dV .
S V

Gauss divergence theorem is



∫∫∫ ∇ ⋅ F dV = ∫∫ F ⋅ n dS(1)
V S

Let F = f a, where a is an arbitrary constant vector.

\ (1) becomes
∫∫∫ (∇ ⋅ f
V
)
a dS = ∫∫ f a ⋅ n dS (2)
S
Now, ∇ ⋅ f a = ∇f ⋅ a + f(∇ ⋅ a) = ∇f ⋅ a [{ ∇ ⋅ a = 0]

\ ∫∫∫ (∇ ⋅ f a) dV = ∫∫∫ (∇f ⋅ a) dV (3)


V V

and ∫∫ F ⋅ n dS = ∫∫ f a ⋅ n dS = ∫∫ f n dS ⋅ a(4)
S S S

\ Using (3) and (4) in (2), we get


∫∫∫ ∇f ⋅ a dV = ∫∫ f n dS ⋅ a
V S

M03_Eng-Maths (Aditya) CH05.indd 25 7/23/2018 4:55:27 PM


5.26  n  Engineering Mathematics-III

⇒ a ⋅ ∫∫∫ ∇f dV = a ⋅ ∫∫ f n dS
V S

⇒ ∫∫∫∫∫∫∇f∇f
V V
dVdV= a=⋅a∫∫⋅ ∫∫ n dS
f fn dS
s

s
[{ a is arbitrary ]

2. ∫∫ F × n dS = − ∫∫∫ ∇ × F dV
S V

Gauss divergence theorem is ∫∫∫ ∇ ⋅ F dV = ∫∫ F ⋅ n dS(1)


V S

∫∫∫ ∇ ⋅ F dV
Let
V
= a =× ∫∫ ⋅ n dS a is an arbitrary constant vector.
F , Fwhere
S
 \ ∇ ⋅ F = ∇ ⋅ ( a × F ) = F ⋅ ( ∇ × a) − a ⋅ ( ∇ × F ) = − a ⋅ ( ∇ × F ) [{ ∇ × a = 0]
and F ⋅ n = a × F ⋅ n = a ⋅ (F × n)
\ (1) becomes − ∫∫∫ a ⋅ (∇ × F ) dV = ∫∫ (a ⋅ F × n ) dS

V S

⇒ −a ⋅ ∫∫∫ ∇ × F dV = a ⋅ ∫∫ F × n dS
V S

⇒ − ∫∫∫ ∇ × F dV = ∫∫ F × n dS
V S


∫∫ F × n dS = −∫∫∫ ∇ × F dV .
S V

If S is closed surface, then prove that Eq.


(1) ∫∫ dS 5 ∫∫∫ ∇? n dV (2)
S V
∫∫ r 3 n dS 5 0
∫∫ dS 5 0 (3) S S
r
∫∫S r n dS 5 4∫∫∫
∫∫S r 3 ? n dS 5 0 (6)
(4) ∫∫∫ ( ∇ 3 n )dV 5 0 (5) 4
r 4 r dV
V V

(7) ∫∫ f ( r ) r 3 n dS 5 0 (8)
S
∫∫ ( = r
2
? n) dS 5 6V (9) ∫∫ ( = 3 r ) ? n dS 5 0
S S

Solution.
(i) To prove ∫∫ dS 5 ∫∫∫ = ? n dV .
S V

Gauss divergence theorem is ∫∫ F ⋅ n dS = ∫∫∫ ∇ ⋅ F dV (1)


V∇⋅F = ∇⋅n
S
∇⋅F = ∇⋅n
Let F = n  \  ∇ ⋅ F = ∇ ⋅ n  and  F ⋅ n = n ⋅ n = 1
F⋅n = n⋅n = 1
\ (1) becomes
F⋅n = n⋅n = 1 ∫∫
∫∫SS dS =∫∫∫
dS = ∇ ⋅ n dV
∫∫∫ ∇ ⋅ n dV .
∫∫ dS = ∫∫∫ ∇ ⋅ n dV
V
V

∫∫ dS 5 0.
S V
(2) To prove
S

We have ∫∫∫ ∇f dV = ∫∫ f n dS (1)


V S

M03_Eng-Maths (Aditya) CH05.indd 26 7/23/2018 4:55:35 PM


Vector Integration  n  5.27

Let f = 1.
 ∂ ∂ ∂
\ ∇f =  i + j + k  (1) = 0   \ 
 ∂x ∂y ∂z  ∫∫∫ ∇f dV = 0
V

\ ∫∫
S
n dS = 0  ⇒ 
∫∫ dS = 0  [using (1)]
S

(3) To prove ∫∫ r 3 n dS 5 0.
S

We have ∫∫ F × n dS ≡ −∫∫∫ ∇ × F dV
S V

Let F = r and r = xi + yj + zk

i j k
\ ∂ ∂ ∂ = i (0 - 0) + j(0 - 0) + k(0 − 0) = 0
∇× F =
∂x ∂y ∂z
x y z
\ ∫∫∫
∫∫∫
∫∫∫∇×
∇×
∇×
VV V
FFFdv ===00 0∴
dvdv ∴∴∫∫∫∫∫∫FF×F×n×ndS
ndS
SS S
===000⇒∫∫∫∫∫∫
dS rr×r×n×ndS
ndS
SS S
===000
dS [using (1)]

(4) To prove ∫∫ ∇ 3 n dV 5 0.
V

We have ∫∫ F × n dS = −∫∫∫ ∇ × F dV (1)


S V

Let F = n \  F × n = n × n = 0

\ ∫∫ F × n dS = 0  \  ∫∫∫ ∇ × F dV = 0  ⇒  ∫∫∫ ∇ × n dV
V
= 0 [using (1)]
S V

r
(5) To prove ∫∫ ? n dS 5 0.
S r3
Gauss divergence theorem is ∫∫ F . n dS = ∫∫∫ ∆ ⋅ F dS (1)
S V

 r 1
  \  ∇ ⋅ F = ∇ ⋅  3  = 3 (∇ ⋅ r ) + ∇  3  ⋅ r
r 1
Let F =
r 3
r  r  r 
∂ ∂ ∂
\ ∇⋅r = ( x) + ( y) + ( z) = 1 + 1 + 1 = 3
∂x ∂y ∂z
∂r ∂r x
Now r 2 = x 2 + y 2 + z 2 ⇒ 2r = 2x ⇒ =
∂x ∂x r
∂r y ∂r z
Similarly, = and =
∂y r ∂z r

M03_Eng-Maths (Aditya) CH05.indd 27 7/23/2018 4:55:43 PM


5.28  n  Engineering Mathematics-III

 1 ∂ 3 ∂ ∂
\ ∇  3  = ∇( r −3 ) = i ( r ) + j (r 3 ) + k (r 3 )
r  ∂x ∂y ∂z
∂r ∂r ∂r
= i ( −3) r 4 + j ( −3)r 4 + k ( −3)r 4
∂x ∂y ∂z
33  x y z  3 3
=
=− + ] =k − =3−r 5 [ xi + yj + zk ] = − 4 r
− 45 [xi i++yj +j zk
rr  r r r  r r 5 r
r 3 3 3 3
\ ∇ ⋅ F = ∇ ⋅  3  = 3 − 5 (r ⋅ r ) = 3 − 3 = 0
r  r r r r

\ ∫∫∫ ∇ ⋅ F dV = 0
V
⇒ ∫∫ F ⋅ n dS = 0
S
[using (1)]

r  r
\ ∫∫ r
S
3
⋅ n dS = ∫∫∫  ∇ ⋅ 3  dS = 0.
V  r 

(6) To prove
∫∫ r n dS 5 4∫∫∫ r 2 r dV .
4

S V

We have ∫∫∫ ∇f dV = ∫∫ f n dS
V S

∂ 4 ∂ ∂
Let f = r 4   \  ∇f = i
    (r ) + j (r 4 ) + k (r 4 )
∂x ∂y ∂z
x y z
= 4 r 3 ⋅ i + 4 r 3 ⋅ j + 4 r 3 ⋅ k = 4 r 2 [ xi + yj + zk ] = 4 r 2 r
r r r

\ (1) becomes, ∫∫∫ 4r r dV = ∫∫ r 4 n dS  ⇒  4 r 2 r dV = r 4 n dS


∫∫∫ ∫∫
2

V S V S

(7) To prove: ∫∫ f ( r ) r 3 ndS 5 0.


S


We have ∫∫ F ×in dS = −j∫∫∫ ∇ × Fk dV (1)
S V
∂ ∂ ∂
Let ∇ × F = f(r)r,  r = xi + yj + zk
∂x ∂y ∂z
i j k
f ( x) x f ( x) y f ( x) z
∂ ∂ ∂ = i  ∂ f (r) z − ∂ f (r) y 
\   ∇ × F = ∑  ∂y ∂z 
∂x ∂y ∂z  
f ( x) x f ( x) y f ( x) z
∂ ∂   ∂z ∂r   ∂y ∂r 
Now  f ( r ) z − f ( r ) y  =  f ( r ) + zf 9( r )  −  f ( r ) + y f ′( r ) 
 ∂y ∂z   ∂y ∂y   ∂z ∂z 
y z f ′( r )
= 0 + z f ′( r ) − 0 − yf ′( r ) ⋅ = [ yz − yz ] = 0
r r r

M03_Eng-Maths (Aditya) CH05.indd 28 7/23/2018 4:55:50 PM


Vector Integration  n  5.29

 ∂ ∂ 
\ i  ( f ( r ) z ) − ( f ( r ) y ) = 0
 ∂y ∂z 
∂ ∂  ∂ ∂
Similarly, j  (f ( r )z ) − (f ( r ) y ) = 0 and k  ( f ( r ) y ) − ( f ( r ) x ) = 0
 ∂ x ∂ z   ∂x ∂z 

\ ∇× F = 0 ∴ ∫∫∫ ∇ × F dV = 0
V

\ (1) becomes ∫∫ F × n dS = 0  ⇒  ∫∫ f (r ) r × n dS = 0
S
S

(8) To prove ∫∫ ( = r 2 ⋅ n) dS 5 6 V .
S

Gauss divergence theorem is ∫∫ F ⋅ n dS = ∫∫∫ ∇ ⋅ F dV (1)


S V

Let F = ∇r and r = xi + yj + zk   ⇒  r = x 2 + y 2 + z 2
2 2

\ ∇ ⋅ F = ∇ ⋅ ∇r 2
∂ 2 ∂ ∂
Now, ∇r 2 = i ( x + y2 + z2 ) + j ( x2 + y2 + z2 ) + k ( x2 + y2 + z2 )
∂x ∂y ∂z
= 2 xi + 2 yj + 2 zk = 2[ xi + yj + zk ] = 2r
\ ∇ ⋅ ∇r = ∇ ⋅ 2r = 2∇ ⋅ r
2

But  ∂ ∂ ∂
∇⋅r = i +j + k  ⋅ ( xi + yj + zk ) = 1 + 1 + 1 = 3
 ∂x ∂y ∂z 

\ ∇ ⋅ ∇r 2 = 2 ⋅ 3 = 6  \  ∫∫∫ ∇ ⋅ F dV = ∫∫∫ 6 dV
V V
= 6V

\ (1) becomes ⇒ 
∫∫ F ⋅ n dS = 6V  
S
∫∫ ∇r
2
⋅ n dS = 6V
S

WORKED EXAMPLES
EXAMPLE 1
Let V be the region bounded by a closed surface S. Let f and g be scalar point functions that
together with their derivatives in any directions are uniformly continuous within the region V.
Then
∫∫∫ ( f ∇ g 2 g ∇ f ) dV 5 ∫∫ ( f ∇ g 2 g ∇f ) ? n dS.
2 2

V S
Solution.
Gauss divergence theorem is
∫∫∫ ∇ ⋅ F dV = ∫∫ F ⋅ n dS
V S

M03_Eng-Maths (Aditya) CH05.indd 29 7/23/2018 4:55:57 PM


5.30  n  Engineering Mathematics-III

Put F = f ∇g  \  ∇ ⋅ F = ∇ ⋅ (f ∇g ) = f (∇ ⋅ ∇g ) + ∇f ⋅ ∇g = f ∇ 2 g + ∇f ⋅ ∇g
and F ⋅ n = ( f ∇g ) ⋅ n
\ by divergence theorem becomes
∫∫∫ (f ∇ g + ∇f ⋅ ∇g ) dV = ∫∫ (f ∇g ⋅ n ) dS(1)
2

V S

Interchanging f and g, we get


∫∫∫ (g ∇ f + ∇g ⋅ ∇f ) dV = ∫∫ (g ∇f ⋅ n ) dS(2)
2

V S

(1) - (2) ⇒ ∫∫∫ (f ∇ g − g ∇ f ) dV = ∫∫ (f ∇g − g ∇f ) ⋅ n dS(3)


2 2

V S

Note  This result is known as Green’s theorem.


Equation (1) is called Green’s first identity and equation (3) is called Green’s second identity.

EXAMPLE 2
1 r
Prove that ∫∫∫ dV 5 ∫∫ 2 ? n dS.
V r2 S r
Solution.
Gauss divergence theorem is
∫∫∫ ∇ ⋅ F dV = ∫∫ F ⋅ n dS(1)
V S

r
Put = r −2 r. Then  ∇ ⋅ F = ∇ ⋅ ( r −2 r ) = (∇ ⋅ r )r −2 + ∇r −2 ⋅ r
F=
r2
∂ ∂ ∂
If r = xi + yj + zk , then ∇ ⋅ r = (x ) + (y ) + (z ) = 1 + 1 + 1 = 3
∂x ∂y ∂z
and r2 = x2 + y2 + z2
∂r ∂r x , ∂r y ∂r z
\ 2r = 2x ⇒ = = and =
∂x ∂x r ∂y r ∂z r

∂ −2 ∂ ∂
∇ r −2 = i ( r ) + j ( r −2 ) + k ( r −2 )
∂x ∂y ∂z

∂r ∂r ∂r
= i ( −2)r −3 + j ( −2)r −3 + k ( −2)r −3
∂x ∂y ∂z
x y z −2 2r
= −2r −3 i − 2r −3 j − 2r −3 k = 4 ( xi + yj + zk ) = − 4
r r r r r

\  −2  3 2 3 2 1
∇ ⋅ F = 3r −2 +  4 r ⋅ r  = 2 − 4 × r 2 = 2 − 2 = 2
r  r r r r r

\ (1) becomes 1 r
∫∫∫ r
V
2
dV = ∫∫
S r2
⋅ n dS

M03_Eng-Maths (Aditya) CH05.indd 30 7/23/2018 4:56:04 PM


Vector Integration  n  5.31

EXAMPLE 3
Using divergence theorem, evaluate ∫∫ F ? n dS, where F 5 4 x z i 2 y 2 j 1 y z k and S is the surface
S
of the cube bounded by the planes x 5 0, x 5 2, y 5 0, y 5 2, z 5 0, z 5 2.

Solution.
Gauss divergence theorem is ∫∫ F ⋅ n dS = ∫∫∫ ∇ ⋅ F dV
S V

Given F = 4 xzi − y j + yzk


2

\ ∂ ∂ ∂
∇⋅F = (4 xz ) + (−y 2 ) + ( yz ) = 4 z − 2 y + y = 4 z − y
∂x ∂y ∂z
2 2 2
\
∫∫ F ⋅ n dS = ∫ ∫ ∫ (4z − y ) dx dy dz
0 0 0

2 2
= ∫ ∫ ( 4 z − y ) [ x ]0 dy dz
2

0 0

2 2
= ∫ ∫ ( 4 z − y )2 dy dz
0 0

2
 y2
2

= 2∫  4 zy −  dz
0 
2 0
2 2
 4
= 2∫  4 z ⋅ 2 −  dz = 2 ⋅ ∫ (8z − 2) dz
0
 2 0
2
 8z 2   4 
= 2 − 2z  = 2 8 ⋅ − 2 ⋅ 2 = 2[16 − 4] = 2 × 12 = 24.
 2 0  2 

Example 4

∫∫ F ⋅ n dS where F 5 x i 1 y
3 3
Using Gauss divergence theorem, evaluate j 1 z 3 k and S is the
S
sphere x2 1 y2 1 z2 5 a2.

Solution.
Gauss divergence theorem is ∫∫ F ⋅ n dS = ∫∫∫ ∇ ⋅FdV
S V

Given F=x i +y j+z k


3 3 3

\ ∂ 3 ∂ ∂
( x ) + ( y 3 ) + ( z 3 ) = 3 x + 3 y + 3 z = 3( x + y + z )
2 2 2 2 2 2
∇⋅ F =
∂x ∂y ∂z
\
∫∫ F ⋅ n dS = ∫∫∫ + y 2 + z 2 ) dx dy dz
2
3( x
S V

M03_Eng-Maths (Aditya) CH05.indd 31 7/23/2018 4:56:08 PM


5.32  n  Engineering Mathematics-III

We shall evaluate this triple integral by using spherical polar coordinates.


x = r sin u cos f,  y = r sin u sin f,  z = r cos u
∂( x , y , z )
dr d u df = r sin u dr d u df
2
then dx dy dz =
∂( r , u, f)
and x2 + y2 + z2 = r2
Here r varies from 0 to a, u varies from 0 to p and f varies from 0 to 2p.
2p p a z
\
∫∫ F ⋅ n dS = ∫ ∫ ∫ 3r
4
sin udrd udf
S 0 0 0

2p p a P
= 3 ∫ df∫ sin ud u∫ r 4 dr
0 0 0 θ
r
a
r 
5
= 3 [ f ]0 [ − cos u]0
2p p
o
 
 5 0 y
5 φ
a
= 3 ⋅ 2p(−
− cos p + cos 0) ⋅
5
M
a5 12p 5 x
= 6p ⋅ 2 ⋅ = a Fig. 5.20
5 5

Note  We have div F = 3( x 2 + y 2 + z 2 ). Since the equation of the surface is x 2 + y 2 + z 2 = a 2 , we


cannot take div F = 3a 2 because F is defined in the volume inside and on S.
But x 2 + y 2 + z 2 = a 2 is true only for points on S.

Example 5
Verify Gauss divergence theorem for F 5 4 xzi 2 y 2 j 1 yzk over the cube bounded by x 5 0,
x 5 1, y 5 0, y 5 1, z 5 0, z 5 1.

Solution.
Gauss divergence theorem is
S
∫∫ F ⋅ n dS = ∫∫∫
V
∇ ⋅FdV

Given F = 4 xzi − y 2 j + yzk

\ ∂ ∂ ∂
∇⋅ F = (4 xz ) + ( − y 2 ) + ( yz ) = 4 z − 2 y + y = 4 z − y
∂x ∂y ∂z
1 1 1
\ ∫∫∫ ∇ ⋅FdV = ∫ ∫ ∫ (4 z − y) dx dy dz
V 0 0 0
[{ dV = dx dy dz ]

1 1 1 1

= ∫ ∫ ( 4 z − y ) [ x ]0 dy dz = ∫ ∫ [4 z − y ] dy dz
1

0 0 0 0

M03_Eng-Maths (Aditya) CH05.indd 32 7/23/2018 4:56:14 PM


Vector Integration  n  5.33

1 1
1
 y2  1
 1  z2 1  1 3
= ∫  4 zy −  dz = ∫  4 z −  dz =  4 − z  = 2 − =
0
 2  0 0
 2   2 2  0 2 2
3 (1)
⇒ ∫∫∫
V
∇ ⋅ F dv =
2
z
We shall now evaluate ∫∫ F ⋅ n dS
S D
G
Here the surface S consists of the six faces of the cube.
F
E
\
∫∫ F ⋅ n dS = ∫∫ F ⋅ n dS + ∫∫ F ⋅ n dS
S S1 S2
O
+ ∫∫ F ⋅ n dS + ∫∫ F ⋅ n dS C y
S3 S4
A B
+ ∫∫ F ⋅ n dS + ∫∫ F ⋅ n dS x
S5 S6
Fig. 5.21
We shall simplify the computation and put it in the form of a table.

Face Equation Outward normal n F⋅n dS


S1 = ABEF x=1 i 4xz = 4z dy dz
S2 = OCDG x=0 −i -4xz = 0 dy dz
S3 = BCDE y=1 j -y2 = -1 dx dz
S4 = OAFG y=0 −j y2 = 0 dx dz
S5 = DEFG z=1 k yz = y dx dy
S6 = OABC z=0 −k -yz = 0 dx dy
1
1 1
 z2  1
\
∫∫ F ⋅ n dS = ∫ ∫ 4 z dy dz = 4 [ y ] 1
0  
 2 0
= 4 ⋅1⋅ = 2
2
S1 0 0

∫∫ F ⋅ n dS = S∫∫ 0 dy dz = 0
S2 2

1 1

∫∫ F ⋅ n dS = ∫ ∫ −1 dx dz = − [ x ] [ z ]
1 1
0 0
= −1
S 3 0 0

∫∫ F ⋅ n dS = S∫∫ 0 dx dz = 0
S4 4

1
1 y 
1 1 2
1
∫∫ F ⋅ n dS = ∫ ∫ y dx dy = [ x ]0   =
 2 0 2
S5 0 0

and ∫∫ F ⋅ n dS = S∫∫ 0 dx dy = 0
S6 6

M03_Eng-Maths (Aditya) CH05.indd 33 7/23/2018 4:56:20 PM


5.34  n  Engineering Mathematics-III

\ 1 3
∫∫
S
F ⋅ n dS = 2 + 0 + ( −1) + 0 + + 0 = (2)
2 2
From (1) and (2), ∫∫ F ⋅ n dS = ∫∫∫
S V
∇ ⋅FdV

Hence, Gauss’s divergence theorem is verified.


Example 6
Verify divergence theorem for F 5 x 2 i 1 zj 1 yzk over the cube formed by the planes x 5 61,
y 5 61, z 5 61.

Solution.
Gauss divergence theorem is ∫∫ F ⋅ n dS = ∫∫∫ ∇ ⋅FdV
S V
Given F = x 2i + zj + yzk
\ ∂ ∂ ∂
∇ ⋅ F = ( x 2 ) + ( z ) + ( yz ) = 2 x + 0 + y = 2 x + y
∂x ∂y ∂z
1 1 1
\  ∫∫∫ ∇ ⋅FdV =
V
∫ ∫ ∫ (2 x + y) dx dy dz
−1 −1 −1

1 1 1 1

∫ ∫ [1 + y − (1 − y ) ]dy dz = ∫ ∫
1 1 1
= ∫ ∫  x + yx  −1 dy dz = 2 y dy dz = 0
2
−1 −1
−1 −1 −1 −1

a 
  ∫ f ( x )dx = 0 if f ( x ) is odd function, Here y is odd function 
 −a 
∫∫ F ⋅ n dS⇒= ∫∫∫ ∇ ⋅FdV = 0 (1)
S V 
H E
\ shall now compute ∫∫ F ⋅ n dS
We
S G
S is the surface consisting of the six faces of the cube. F
z
∫∫ F ⋅ n dS = ∫∫ F ⋅ n dS + ∫∫ F ⋅ n dS
S S1 S2 y
o
x
+ ∫∫ F ⋅ n dS + ∫∫ F ⋅ n dS A
D
S3 S4

+ ∫∫ F ⋅ n dS + ∫∫ F ⋅ n dS B C
S5 S6 Fig. 5.22
We shall simplify the computations and put it in the form of a table.
Faces Equation Outward normal n F?n dS
S1 = BCFG x=1 i x2 = 1 dy dz
S2 = ADEH x = -1 −i -x2 = -1 dy dz
S3 = CDEF y=1 j z dz dx
S4 = ABGH y = -1 −j -z dz dx
S5 = EFGH z=1 k yz = y dx dy
S6 = ABCD z = -1 −k -yz = y dx dy

M03_Eng-Maths (Aditya) CH05.indd 34 7/23/2018 4:56:26 PM


Vector Integration  n  5.35

1 1
\
∫∫ F ⋅ n dS = ∫ ∫ dy dz = [ y ] [ z ]
1 1
−1 −1
= (1 + 1) (1 + 1) = 4
S1 −1 −1
1 1

∫∫ F ⋅ n dS = ∫ ∫ −1dy dz = − [ y ] [ z ]
1 1
= −[1 + 1] [1 + 1] = −4
−1 −1
S2 −1 −1
1 1

∫∫ F ⋅ n dS =
S3
∫ ∫ zdz dx = 0
−1 −1
[{ z is odd function ]

1 1 1 1

∫∫ F ⋅ n dS =
S4
∫ ∫ − zdz dx = − ∫ ∫ zdz dx = 0
−1 −1 −1 −1
1 1

∫∫
S
F ⋅ n dS = ∫ ∫ y dx dy = 0
−1 −1
[{ y is odd function ]
5
1 1

∫∫ F ⋅ n dS =
S6
∫ ∫ y dx dy = 0
−1 −1

\
∫∫ F ⋅ n dS = 4 − 4 + 0 + 0 + 0 + 0 = 0(2)
S

From (1) and (2), ∫∫ F ⋅ n dS = ∫∫∫ ∇ ⋅FdV


S V

Hence, Gauss’s divergence theorem is verified.

Example 7
Verify divergence theorem for the function F 5 4 xi 2 2 y 2 j 1 z 2 k taken over the surface of the
region, bounded by the cylinder x2 1 y2 5 4 and z 5 0, z 5 3.

Solution.
Gauss divergence theorem is
S
∫∫ F ⋅ n dS = ∫∫∫
V
div FdV

Given F = 4 xi − 2 y 2 j + z 2 k   \  ∇ ⋅ F = ∂ (4 x ) + ∂ ( −2 y 2 ) + ∂ ( z 2 )
∂x ∂y ∂z y
= 4 − 4 y + 2z

and z varies from 0 to 3,


Also given x 2 + y 2 = 4

⇒ y 2 = 4 − x 2   ⇒  y = ± 4 − x 2 x

and y = 0  ⇒ x 2 = 4  ⇒  x = ±2

2 4− x2 3
\
∫∫∫ ∇ ⋅FdV =
V
∫ ∫ ∫ (4 − 4 y + 2 z ) dz dy dx
−2 − 4 − x 2 z = 0 Fig. 5.23

M03_Eng-Maths (Aditya) CH05.indd 35 7/23/2018 4:56:33 PM


5.36  n  Engineering Mathematics-III

4 − x2 3
2
 z2 
= ∫ ∫
−2 − 4 − x 2


( 4 − 4 y ) z + 2  dy dx
2 0
2 4 − x2

= ∫ ∫
−2 − 4 − x 2
[( 4 − 4 y ) ⋅ 3 + 9] dy dx

2 4 − x2

= ∫ ∫
−2 − 4 − x 2
[ 21 − 12 y ] dy dx

4 − x2

2
y2 
= ∫  21 y − 12  dx
−2
 2 − 4 − x2

( )
2

= ∫ [ 21 4 − x 2 + 4 − x 2 − 6( 4 − x 2 − ( 4 − x 2 )] dx
−2

= ∫ 42
−2
4 − x 2 dx

2 { 4 − x 2 is even function 
= 84∫ 4 − x dx 2  
0

2
x 4 x p
= 84  4 − x 2 + sin −1  = 84 [0 + 2 sin −1 1 − 0] = 84 ⋅ 2 = 84p
2 2 2 0 2
2 4− x2 3

∫∫∫ ∇ ⋅FdV = 84
V
∫ ∫ −2 − 4 − x 2 z = 0
∫ (4 − 4 y + 2 z ) dz dy dx (1)

We shall now compute the surface integral


S
∫∫ F ? n dS.
S consists of the bottom surface S1, top surface S2 and the curved surface S3 of the cylinder.
z
On S1: Equation is z = 0, n = − k S2

\ F ⋅ n = − z 2 = 0  ⇒ 
∫∫
S
F ⋅ n dS = 0
1

On S2: Equation is z = 3, n = k S3

\ dx dy dx dy
F ⋅ n = z 2 = 9, dS = = = dx dy
n⋅k k ⋅k O
y
\
∫∫ F ⋅ n dS = ∫∫ 9 dx dy = 9∫∫ dx dy
S2 S2 S2
S1
x
= 9 (area of the circle S2) = 9 p 22 = 36p.
Fig. 5.24

M03_Eng-Maths (Aditya) CH05.indd 36 7/23/2018 4:56:38 PM


Vector Integration  n  5.37

On S3:  Equation of the cylinder is x2 + y2 = 4


Let f = x2 + y2
\ ∂ ∂ ∂f
∇f = i f+ j f+ k = i 2 x + 2 yj + 0 k = 2( xi + yj )
∂x ∂y ∂z
∇f 2( xi + yj ) 2( xi + yj ) 1
\  the normal n = = = = ( xi + yj )
∇f 2 x 2 + y 2 2 4 2
1
\ F ⋅ n = (4 xi − 2 y 2 j + z 2 k ) ⋅ ( xi + yj ) = 2 x 2 − y 3
2
Since S3 is the surface of a cylinder x + y2 = 4, we use cylindrical polar coordinates to evaluate
2

∫∫ F ⋅ n dS
S3
z

\  x = 2 cos u,  y = 2 sin u,  z = z  \  dS = 2 du dz


u varies from 0 to 2p and z varies from 0 to 3
3 2p
2d θ
\
∫∫ F ⋅ n dS = ∫ ∫ (2 ⋅ 4 cos u − 8sin 3 u) 2d u dz
2
ds dz
S3 0 0 S3
3 2p

= 16∫ ∫ (cos
2
u − sin 3 u) d u dz O
0 0 y
3 2p
1 + cos 2u 1  2d θ
= 16∫ ∫  − (3 sin u − sin 3u)  d u dz
0 0
2 4  x
3 2p Fig. 5.25
1  sin 2u  1  cos 3u  
= 16∫   u +  −  −3 cos u +  dz
0 2
 2  4 3   0

coss 0   
3
1  sin 4p  1 cos 6p 
= 16∫   2p + − 0  −  −3 cos 2p + −  −3 cos 0 +   dz
0 
2 2  4 3  3   
3
 3 1 3 1
= 16∫  p + − − +  dz
 4 12 4 12 
0
3

= 16p∫ dz = 16p [ z ]0 = 16p × 3 = 48p


3


∫∫
S
F ⋅ n dS = 36p + 48p = 84p(2)

From (1) and (2), ∫∫ F ⋅ n dS = ∫∫∫ ∇ ⋅FdV


S V
Hence, Gauss’s divergence theorem is verified.
Example 8
Verify Gauss divergence theorem for F 5 a( x 1 y )i 1 a( y 2 x ) j 1 z 2 k over the region bounded
by the upper hemisphere x2 1 y2 1 z2 5 a2 and the plane z 5 0.

M03_Eng-Maths (Aditya) CH05.indd 37 7/23/2018 4:56:43 PM


5.38  n  Engineering Mathematics-III

Solution.
Gauss divergence theorem is

∫∫
F ⋅ n dS =
S
∇ ⋅FdV∫∫∫
V
Given F = a( x + y )i + a( y − x ) j + z 2 k
∂ ∂ ∂ 2
\ ∇⋅F = (a( x + y )) + (a( y − x )) + ( z ) = a + a + 2 z = 2( a + z )
∂x ∂y ∂z
\  ∫∫∫
V
∇ ⋅FdV = ∫∫∫ 2(a + z ) dV
V

= 2a∫∫∫ dV + 2∫∫∫ z dV
V V
a a − x2
2 a2 − x 2 − y 2

= 2aV + 2 ∫ ∫ ∫ z dz dy dx
− a − a2 − x 2 0

a2 − x 2 a2 − x 2 − y 2
a
 z2 
= 2aV + 2 ∫ ∫   dy dx
− a − a2 − x 2
 2 0
a a2 − x 2
2p 3  2 3
= 2a a +∫ ∫ ( a 2 − x 2 − y 2 ) dy dx  { V = 3 pa 
3 −a − a2 − x 2

a a2 − x 2
4pa 4
= +∫2 ∫ ( a 2 − x 2 − y 2 ) dy dx  [{ a2 − x 2 − y 2 is even in y ]
3 −a 0

a2 − x 2
4pa 4
a
 y3 
= + 2 ∫ ( a 2 − x 2 ) y −  dx
−a 
3 3 0

4pa 4
a
 ( a 2 − x 2 )3 / 2 
= + 2 ∫ ( a 2 − x 2 ) a 2 − x 2 −  dx
−a  
3 3

4pa 4
a
 ( a 2 − x 2 )3 / 2 
= + 2 ∫ ( a 2 − x 2 )3 / 2 −  dx
−a  
3 3
a
4pa 4 2
= + 2 ⋅ ∫ ( a 2 − x 2 )3 / 2 dx
3 3 −a
a
4pa 4 4 4pa 4 8
= + × 2∫ ( a 2 − x 2 )3 / 2 dx = + I [{ (a2 − x 2 )3/ 2 is even]
3 3 0 3 3
a
where I = ∫ ( a 2 − x 2 )3 / 2 dx
0

Put x = a sin u \  dx = a cos udu


p
When  x = 0,  sin u = 0 ⇒ u = 0 and when x = a, sin u = 1 ⇒ u =
2

M03_Eng-Maths (Aditya) CH05.indd 38 7/23/2018 4:56:49 PM


Vector Integration  n  5.39

p
2
\ I = ∫ ( a 2 − a 2 sin 2 u)3 / 2 a cos ud u
0
p
2

= ∫ a3 cos3 u ⋅ a cos ud u
0
p/2
4 −1 4 − 3 p 3 1 p 3pa 4
= a4 ∫
0
cos 4 u d u = a 4 ⋅ ⋅
4 4−2 2
⋅ = a4 ⋅ ⋅ =
4 2 2 16
4pa 4 8 3pa 4 (8 + 3) 4 11 4
∫∫∫ ∇ ⋅F =
V 3
+ ⋅
3 16
=
6
pa = pa (1)
6
Now we shall compute the double integral ∫∫ F ⋅ n dS z
S
S consists of S1 and S2
S2
\
∫∫ F ⋅ n dS = ∫∫
S S
F ⋅ n dS + ∫∫ F ⋅ n dS
S
1 2
1 2

On S1: z = 0, n = − k
\ F ⋅ n = ( a( x + y )i + a( y − x ) j + z 2 k ) ⋅ ( − k ) = − z 2 = 0
O y
\
∫∫ F ⋅ n dS = 0
S1 S1
On S2: x + y + z = a
2 2 2 2
x
Let f = x2 + y2 + z2 Fig. 5.26
\ ∇f = 2 x i + 2 y j + 2 z k
= 2( x i + y j + z k )
∇f 2( xi + yj + zk ) xi + yj + zk xi + yj + zk z
\ n= = = and n ⋅ k = ⋅k =
∇f 2 x + y + z 2 2 2 a a a
xi + yj + zk
F ⋅ n = [ a( x + y ) i + a ( y − x ) j + z 2 k ] ⋅
a
z3 z3
= ( x + y) x + ( y − x) y + = x 2 + y 2 +
a a
dx dy
∫∫S F ⋅ n dS = R∫∫ F ⋅ n n ⋅ k , where R is the projection of S2 on the xy-plane.
2

\  z 3  dx dy
∫∫S F ⋅ n dS = R∫∫  x + y2 +
2

2
a z
a
 a( x 2 + y 2 ) 
= ∫∫ 
R z
+ z 2  dx dy


 a( x 2 + y 2 ) 
=
R
∫∫ 
z
+ [a 2 − x 2 − y 2 ] dx dy

M03_Eng-Maths (Aditya) CH05.indd 39 7/23/2018 4:56:55 PM


5.40  n  Engineering Mathematics-III

Changing to polar coordinate, we have

x = r cos u,  y = r sin u,  r2 = x2 + y2 and dx dy = r dr du


a 2p
 ar 2 2 
\ ∫∫ F ⋅ n dS = ∫ ∫  2 2 + ( a − r ) r dr d u
2

S2 0 0  a −r 
a 2p
 − a( a 2 − r 2 ) + a3 
=∫ ∫ + ( a 2 − r 2 ) r dr d u
0 0  a −r
2 2

a 2p
 a3 
=∫ ∫  − a a 2
− r 2
+ + ( a 2 − r 2 ) r dr d u
0 0  a −r
2 2

2p a
 a 3

= ∫ d u∫  − a a 2 − r 2 + + ( a 2 − r 2 ) r dr
0 0 a −r
2 2

a

= [u]0 ∫ ( −a
2p
a2 − r 2 )r + a3 (a2 − r 2 ) −1/ 2 r + (a2 − r 2 ) r ] dr
0

 a a a3
a a

= 2p ∫ + (a2 − r 2 )( −2r )dr − ∫ (a2 − r 2 ) −1/ 2 ( −2r )dr + ∫ (a2 r − r 3 )dr 
 0 2 2 0 0 
 a  (a2 − r 2 )3/ 2  a a3  (a2 − r 2 )1/ 2  a  r 2 r 4  a 
= 2p    −   + a
2
−  
2 3 2 1  2 4 0 
      
 2 0  2 0 
a a4 a4 
= 2p  (0 − a3 ) − a3 (0 − a) + − 
3 2 4
 a 4
a 
4
11a4 11pa4
= 2p  − + a4 +  = 2p × =
 3 4 12 6

\ 11pa 4 11pa 4 (2)


∫∫ F ⋅ ndS = 0 +
S 6
=
6

From (1) and (2), 


S
∫∫ F ⋅ n dS = ∫∫∫
V
∇ ⋅FdV

Hence, Gauss’s divergence theorem is verified.

Example 9
Evaluate ∫∫ x dy dz 1 x 2 y dz dx 1 x 2 z dx dy over the surface z 5 0, z 5 h, x2 1 y2 5 a2.
3

S
Solution.
We know Gauss divergence theorem in cartesian form is

 ∂F ∂F ∂F 
∫∫ F dy dz + F
S
1 2 dz dx + F3 dx dy = ∫∫∫  1 + 2 + 3  dx dy dz
V  ∂x ∂y ∂z 

M03_Eng-Maths (Aditya) CH05.indd 40 7/23/2018 4:56:59 PM


Vector Integration  n  5.41

z
∫∫ x dy dz + x 2 y dz dx + x 2 z dx dy
3
Given surface integral is z=h
S

Here F1 = x 3 , F2 = x 2 y, F3 = x 2 z
∂F1 ∂F2 ∂F3
\ = 3x 2 , = x2 , = x2
∂x ∂y ∂z
∂F1 ∂F2 ∂F3
\ + + = 3x 2 + x 2 + x 2 = 5 x 2 O
∂x ∂y ∂z y

∫∫ F dy dz + F
S
1 2 dz dx + F3 dx dy = ∫∫∫ 5 x 2 dx dy dz
V x
z=0

h a a2 − y 2
Fig. 5.27
=5∫ ∫ ∫
2
x dx dy dz
z = 0 y = − a x = − a2 − y 2

h  a a2 − y 2 
= 5∫ ∫  2 ∫ x 2 dx  dy dz  [{ x 2 is even ]

0 y=−a  0

a2 − y 2
 x3 
h a

= 10 ∫ ∫   dydz
z=0 y=−a 
3 0

h a
10
3 z∫= 0 y =∫− a
= ( a 2 − y 2 )3/ 2 dydz

10
h
 a 2 
= ∫  2∫ ( a − y ) dy 
2 3/ 2
dz
3 0  0 
a a
20 20 20 h
=
3
[ z ]0h ∫ (a2 − y 2 )3/ 2 dy = 3 h∫ ( a2 − y 2 )3/ 2 dy = 3 × I
0 0

where I = ∫ ( a 2 − y 2 )3 / 2 dy
0

Put y = a sin u  \dy = a cos u du


p
When  y = 0,  sin u = 0  ⇒  u = 0 and when y = a, sin u = 1  ⇒  u =
2
p/2 p/2

\I= ∫ ( a 2 − a 2 sin 2 u)3/ 2 a cos ud u = a 4 ∫ cos


3
u cos ud u
0 0
p/2
3 1 p 3pa 4
= a4 ∫0
cos 4 ud u = a 4 ⋅ ⋅ ⋅ =
4 2 2 16
20 3pa 4 5 4
\ ∫∫
S
F1 dy dz + F2 dz dx + F3 dx dy =
3

16
= pa h
4

M03_Eng-Maths (Aditya) CH05.indd 41 7/23/2018 4:57:04 PM


5.42  n  Engineering Mathematics-III

5.6  Stoke’s Theorem


Stoke’s theorem gives a relation between line integral and surface integral.

Theorem 5.1  If S is an open surface bounded by a simple closed curve C and if F is continuous having
continuous partial derivatives in S and on C, then ∫ F? dr 5 S∫∫ curl F? n dS,
C
where C is traversed in the positive direction.

Proof  Let F = F1i + F2 j + F3 k and r be the position vector of any point P on S.


\ r = xi + yj + zk ⇒ dr = dxi + dyj + dzk
\ ( )
F ⋅ dr = F1i + F2 j + F3 k ⋅ ( dxi + dyj + dzk ) = F1dx + F2 dy + F3 dz
z
\ ∫ F ⋅ d r = ∫ (F dx + F dy + F dz )
C C
1 2 3
C
n

Let z = f(x, y) be the equation of the surface S enclosed


by the curve C. C
Any line parallel to Z-axis intersects the surface in at
most one point. The positive direction of the normal n
is that it makes an acute angle with the positive Z-axis O
(or k ). y
The projection of S on the xy-plane is a region R
enclosed by C′. R
C′
Now, x
∫ F dx = ∫ F ( x, y, z )dx
1 1
C C Fig. 5.28
= ∫ F ((x , y , f (x , y )) dx = ∫ P(x , y )dx
C′
1
C′

where P ( x, y ) = F1 ( x, y f ( x, y ) )

By Green’s theorem,
∂P

∫ P(x , y )dx = ∫∫ − ∂y dx dy[{ Q = 0 here]
C′ R

But P ( x, y ) = F1 ( x, y f ( x, y ) )

∂P ∂F1 ∂F1 ∂f
\ = + ⋅        [{P(x, y) = F1(x, y, z) and z = f(x, y)]   (1)
∂y ∂y ∂z ∂y
 ∂F1 ∂F1 ∂f 
\ ∫ P(x , y )dx = −∫∫  ∂y
C′ R
+ ⋅
∂z ∂y 
dx dy (2)

Now ∫∫ ∇ × F ⋅ n dS = ∫∫ ∇ × ( F i + F
S S
1 2 j + F3 k ) ⋅ n dS

M03_Eng-Maths (Aditya) CH05.indd 42 7/23/2018 4:57:11 PM


Vector Integration  n  5.43

Consider ∫∫ ( = × F1 i ) ⋅ n dS
S

i j k
∂ ∂ ∂  ∂F1   ∂F1  ∂F1 ∂F1
But ∇ × F1i =
∂x ∂y ∂z = i (0) − j  0 − ∂z  + k  0 − ∂y  = ∂z j − ∂y k
F1 0 0

\ ∂F1 ∂F
(∇ × F1i ) ⋅ n = j ⋅ n − 1 k ⋅ n(3)
∂z ∂y
We have r = xi + yj + zk = xi + yj + f ( x, y )k [since z = f(x, y)]

\ ∂r ∂f
= j+ k
∂y ∂y

But ∂ r is a tangent vector to S at P, and hence, ∂ r is ⊥ to n.  \  ∂ r ⋅ n = 0


∂y ∂y ∂y
∂f ∂f
Substituting in (4), we get j ⋅ n + k ⋅n = 0 ⇒ j ⋅n = − k ⋅n
∂y ∂y
∂F1  ∂f  ∂F  ∂F ∂f ∂F1 
\ (3) ⇒ ∇ × F1i ⋅ n = − k ⋅ n − 1 k ⋅ n = −  1 + k ⋅n
∂z  ∂y  ∂y  ∂z ∂y ∂y 

\  ∂F1 ∂f ∂F1 
∫∫ (∇ × F i ) ⋅ n dS = −∫∫  ∂z
S
1
S
+
∂y ∂y 
(k ⋅ n ) dS

⇒  ∂F1 ∂f ∂F1 
∫∫ (∇ × F i ) ⋅ n dS = −∫∫  ∂z + dx dy(5)
∂y ∂y 
1
S R

From (2) and (5), we get


∫ F dx = ∫∫ ∇ × F i ⋅ n dS
C′
1
S
1

Similarly, ∫ F dy = ∫∫ (∇ × F
C′
2
S
2 j ) ⋅ n dS(6)

and ∫ F dz = ∫∫ (∇ × F k ) ⋅ n dS(7)
C′
3
S
3

Adding (5), (6), and (7), we get


∫ F dx + F dy + F dz = ∫∫ ∇ × ( F i + F
C′
1 2 3
S
1 2 )
j + F3 k ⋅ n dS

⇒ ∫ F ⋅ d r = ∫∫ ∇ × F ⋅n dS
C S

M03_Eng-Maths (Aditya) CH05.indd 43 7/23/2018 4:57:19 PM


5.44  n  Engineering Mathematics-III

Note
If S is the region R in the xy-plane, bounded by the simple closed curve C, then n = k is the outward
unit normal.
\  Stoke’s theorem in the plane is ∫ F ⋅ dr = R∫∫ Curl F ⋅ k dR,
C
which is Green’s theorem.

Cartesian form of Stoke’s theorem


If F = F1i + F2 j + F3 k ,then
  then
i j k
∂ ∂ ∂  ∂F3 ∂F2   ∂F3 ∂F1   ∂F2 ∂F1 
Curl F =
∂x ∂y ∂z = i  ∂y − ∂z  − j  ∂x − ∂z  + k  ∂x − ∂y 
F1 F2 F3
and F ⋅ dr = F1 dx + F2 dy + F3 dz
\  the cartesian form of Stoke’s theorem is ∫ (F dx + F dy + F dz )
C
1 2 3

 ∂F3 ∂F2   ∂F ∂F   ∂F ∂F  
= ∫∫ 
S   ∂y
− 
∂z 
dydz +  1 − 3  dzdx +  2 − 1  dxdy 
 ∂z ∂x   ∂x ∂y  
Note
If F = Pi + Qj and r = xi + yj , then d r = dx i + dy j and F ⋅ dr = P dx + Q dy

i j k
∂ ∂ ∂  ∂Q ∂P   ∂Q ∂P 
Curl F = = i (0 − 0) − j (0 − 0) + k  − = − k
∂x ∂y ∂z  ∂x ∂y   ∂x ∂y 
P Q 0
\ ∂Q ∂P
Curl F ⋅ k = −
∂x ∂y
 ∂Q ∂P 
\  Stokes theorem in the plane is ∫ ( P dx + Q dy ) = ∫∫  − dxdy
C R  ∂x ∂y 
  which is Green’s thorem.

WORKED EXAMPLES
Example 1
Prove that r ⋅ dr 5 0, where C is the simple closed curve.
C
Solution.
Let r be the position vector of any point P(x, y, z) on C.  \  r = xi + yj + zk .

M03_Eng-Maths (Aditya) CH05.indd 44 7/23/2018 4:57:26 PM


Vector Integration  n  5.45

Stokes theorem is ∫ F ⋅ d r = S∫∫ curl F ⋅ n dS


C
Here F = r .
i j k
∂ ∂ ∂
\ Curl F = Curl r = = i (0 − 0) − j (0 − 0) + k (0 − 0) + (0 − 0) = 0
∂x ∂y ∂z
x y z
\ ∫ r ⋅ dr = 0
C

EXAMPLE 2
If A is solenoidal, then prove that ∫∫ ∇2 A ? ndS
ds 52 ∫ Curl A ? dr .
S C

Solution.
Given A is solenoidal.   \  ∇ ⋅ A = 0
We know ∇ × (∇ × A ) = ∇(∇ ⋅ A ) − ∇ 2 A = −∇ 2 A

Stoke’s theorem is ∫∫ ∇ × F ⋅ n dS = ∫ F ⋅ d r
S C

Putting F = ∇ × A, we get ∇ × F = −∇ 2 A

\ ∫∫ −∇ A ⋅ n dS = ∫ ∇ × A ⋅ d r
2

S C


∫∫ ∇ A ⋅ n dS = − ∫ Curl A ⋅ d r
2

S C

EXAMPLE 3
Prove that ∫ f d r 52∫∫ ∇f3 n dS.
C S

Solution.
Stoke’s theorem is ∫ F ⋅ d r = ∫∫ Curl F ⋅ n dS = ∫∫ ∇ × F ⋅ n dS
C S S

Put F = f a, where a an arbitrary constant vector.


\
∫ (f a) ⋅ d r = ∫∫ ∇ × f a ⋅ n dS
C S

We know curl f a = ∇ × fa = ∇f × a + f∇ × a = ∇f × a [{ ∇ × a = 0]
\ ∫ (f a) ⋅ d r = ∫∫ (∇f × a) ⋅ n dS
C S

M03_Eng-Maths (Aditya) CH05.indd 45 7/23/2018 4:57:35 PM


5.46  n  Engineering Mathematics-III


∫ f a ⋅ d r = −∫∫ (a × ∇f) ⋅ n dS
C S

⇒ a ⋅  ∫ f dr  = − ∫∫ a ⋅ ( ∇f × n ) dS  [Interchanging dot and cross]


C 
S


a ⋅  ∫ f d r  = −a ⋅ ∫∫ ∇f × n dS = a ⋅  − ∫∫ ∇f × n dS 
C  S  S 

\ ∫ fd r
C
= − ∫∫ ∇f × n dS [{ a is arbitrary]
S

Example 4
If S is the surface of the sphere x2 1 y2 1 z2 5 a2, then show that
S
∫∫ Curl F ⋅ n dS 5 0.
Solution.
Suppose the sphere is cut by a plane into two parts S1 and S2 and let C be the curve binding these two
parts.
n

Then ∫∫ Curl F ⋅ n dS = ∫∫
S S
Curl F ⋅ n dS + ∫∫ Curl F ⋅ n dS
S
1 2 S1

By Stoke’s theorem, ∫∫ Curl F ⋅ n dS = ∫ F ⋅ dr


S1 C

and ∫∫
S
Curl F ⋅ n dS = − ∫ F ⋅ dr , because for S
C
2 C S2
2

the positive sense of the curve C is the opposite direction of C in S1 n


Fig. 5.29
\
∫∫ Curl F ⋅ n dS = C∫ F ⋅ dr − C∫ F ⋅ dr = 0
S

Example 5

∫ ( xydx 1 xy
2
Evaluate dy ) by Stoke’s theorem, where C is the square in the xy-plane with
C
vertices (1, 0), (21, 0), (0, 1), (0, 21).

Solution.
Stoke’s theorem is ∫ F ⋅ dr = S∫∫ curl F ⋅ n dS
C

Given  ∫ ( xy dx + xy 2 dy )  and r = xi + yj    \  dr = dxi + dyj .


C

Here F ⋅ dr = xy dx + xy 2 dy  \  F = xy i + xy 2 j

M03_Eng-Maths (Aditya) CH05.indd 46 7/23/2018 4:57:42 PM


Vector Integration  n  5.47

i j k
\ ∂ ∂ ∂
Curl F = ∇ × F = = i ( 0 − 0) − j ( 0 − 0) + k ( y 2 − x )
∂x ∂y ∂z
xy xy 2 0
⇒ Curl F = ( y 2 − x )k
Also given C is the square in the xy plane with vertices (1, 0), (-1, 0), (0, 1), (0, -1).
y
\ n = k and dS = dx dy
\ Curl F ⋅ n = ( y 2 − x )k ⋅ k = y 2 − x B (0, 1)
y=x+1 y = −(x − 1)
\
∫∫ Curl F ⋅ n dS = R∫∫ ( y − x ) dx dy
2

S (−1, 0)
(1, 0)
x′ x
where R is the region inside the square. C O A

∫ xy dx + xy dy = R∫∫ ( y − x ) dx dy y=x−1
2 2
That is
C y = −(x + 1) D (0, −1)

We shall now evaluate this double integral.


y′
Fig. 5.30
Equation of AB in intercept form is
x y
+ = 1  ⇒  x + y = 1  ⇒  y = -x + 1  ⇒  y = -(x - 1)
1 1
x y
Equation of BC is  + = 1  ⇒  y - x = 1  ⇒  y = x + 1
−1 1
x y
Equation of CD is  + = 1  ⇒  x + y = -1  ⇒  y = -(x + 1)
−1 −1
x y
Equation of AD is  + = 1  ⇒  y - x = -1  ⇒  y = x - 1
1 −1
0 x +1 1 − ( x −1)

\  ∫ ( xy dx + xy 2 dy ) = ∫ ∫ ( y 2 − x ) dydx + ∫ ∫ ( y 2 − x ) dydx
C −1 − ( x +1) 0 x −1

x +1 − ( x −1)
 y3
0
 1
 y3 
= ∫  − xy  dx + ∫  − xy  dx
−1   − ( x +1) 0   x −1
3 3
0

∫ 3 { ( x + 1) }
1
= 3
− ( −( x + 1))3  − x [ x + 1 − ( −( x + 1)) ] dx
−1

1
1
+ ∫  ( −( x − 1))3 − ( x − 1)3  − x[ − ( x − 1) − ( x − 1)]} dx
0
3

{ }
0
1
= ∫
−1
3
[( x + 1)3 + ( x + 1)3 ] − x[( x + 1) + ( x + 1)] dx

∫{ }
1
1
+ − [( x − 1)3 + ( x − 1)3 ] + x[ x − 1 + x − 1] dx
0
3
M03_Eng-Maths (Aditya) CH05.indd 47 7/23/2018 4:57:48 PM
x +1 − ( x −1)
0
 y3  1
 y3 
= ∫−1  3 
− xy dx + ∫0  3 − xy  dx
− ( x +1) x −1
0

∫ 3 { ( x + 1) }
1
= 3
− ( −( x + 1))3  − x [ x + 1 − ( −( x + 1)) ] dx
−1
5.48  n  Engineering1 Mathematics-III
1
+ ∫  ( −( x − 1))3 − ( x − 1)3  − x[ − ( x − 1) − ( x − 1)]} dx
0
3

{ }
0
1
= ∫
−1
3
[( x + 1)3 + ( x + 1)3 ] − x[( x + 1) + ( x + 1)] dx

∫{ }
1
1
+ − [( x − 1)3 + ( x − 1)3 ] + x[ x − 1 + x − 1] dx
0
3
0 1
2   2 
= ∫−1  3 ( x + 1) − 2 x( x + 1) dx + ∫0  − 3 ( x − 1) + 2 x( x − 1) dx
3 3

0 1
 2 ( x + 1) 4  x3 x 2    2 ( x − 1) 4  x3 x 2  
= − 2  +   + − + 2 −  
3 4  3 2   −1  3 4  3 2  0
2  1  1 ( −1) 2   2   1   1 1 
=   − 2 0 −  ( −1) +
3
  − 0 −    + 2  − 
3 4  3 2  3  4  3 2
1 2 1 2 2 1
= − +1+ + −1 = =
6 3 6 3 6 3

Example 6

Evaluate ∫ [( x 1 y )dx 1 ( 2 x 2 z )dy 1 ( y 1 z )dz ]


C
where C is the boundary of the triangle

with the vertices (2, 0, 0), (0, 3, 0) and (0, 0, 6), using Stoke’s theorem.

Solution. z
Stoke’s theorem is C (0, 0, 6)


F ⋅ dr =
C
∫∫
Curl F ⋅ n dS ,
S
where S is the surface of the triangle ABC bounded
by the curve C, consisting of the sides of the triangle o 3 B
in the figure. 2 (0, 3, 0) y
Given F ⋅ d r = ( x + y )dx + (2x − z )dy + ( y + z )dz A
(2, 0, 0) x y
Here F = ( x + y )i + (2 x − z ) j + ( y + z )k x + = 1
2 3
i j k Fig. 5.31
∂ ∂ ∂
\ Curl F =
∂x ∂y ∂z
x + y 2x − z y+z
∂ ∂  ∂ ∂  ∂ ∂ 
= i  ( y + z ) − (2 x − z )  − j  ( y + z ) − ( x + y )  + k  (2 x − z ) − ( x + y ) 
 ∂y ∂z   ∂x ∂z   ∂x ∂y 
= i[1 − ( −1)] − j[0 − 0] + k (2 − 1)] = 2i + k

x y z
Equation of the plane ABC is + + = 1 [intercept form]
2 3 6

M03_Eng-Maths (Aditya) CH05.indd 48 7/23/2018 4:57:54 PM


Vector Integration  n  5.49

x y z ∂f 1 ∂f 1 ∂f 1
\ f= + + , = , = , =
2 3 6 ∂x 2 ∂y 3 ∂z 6
∂f ∂f ∂f = 1 i + 1 j + 1 k = 1 (3i + 2 j + k )
\ ∇f = i +j +k
∂x ∂y ∂z 2 3 6 6
1
(3i + 2 j + k )
∇f 1
\ n= = 6 = (3i + 2 j + k )
∇f 1 14
9 + 4 +1
6

\ 1 1 7
Curl F ⋅ n = (2i + k ) ⋅ (3i + 2 j + k ) = (6 + 1) =
14 14 14
\ 7 7 dx dy
∫∫ Curl F ⋅ n dS = S∫∫
S 14
dS =
14
∫∫
R n⋅k
where R is the orthogonal projection of S on the xy-plane.

But 1 1
n⋅k = (3i + 2 j + k ) ⋅ k =
14 14

\ 7 dxdy
∫∫ Curl F ⋅ n dS =
S
∫∫
14 R 1
14
1
= 7 ∫∫ dx dy = 7 × Area of ∆ OA B = 7 ⋅ ⋅ 2 ⋅ 3 = 21
R 2
\ ∫ [(x + y )dx + (2x − z )dy + ( y + z )dz ] = 21.
C

Example 7
Using Stoke’s theorem, evaluate ∫ F ⋅ dr , where F 5 y 2 i 1 x 2 j 2 ( x 1 z ) k and C is the boundary
C

of the triangle with vertices at (0, 0, 0), (1, 0, 0), (1, 1, 0).

Solution.
Given F = y 2 i + x 2 j − ( x + z )k
Stoke’s theorem is

∫ F ⋅ dr = ∫∫ curl F ⋅ n dS
C S


i j k
∂ ∂ ∂
Now curl F = ∇ × F =
∂x ∂y ∂z
y2 x2 −x − z
∂ ∂  ∂ ∂ 
=  ( − x − z ) − ( x 2 ) i −  ( − x − z ) − ( y 2 ) j
 ∂ y ∂ z   ∂ x ∂ z 
∂ ∂ 
+  ( x 2 ) − ( y 2 ) k
M03_Eng-Maths (Aditya) CH05.indd 49  ∂ x ∂ y  7/23/2018 4:58:01 PM
i j k
n 
∂ ∂ ∂
5.50  curl F = ∇Mathematics-III
Engineering ×F =
∂x ∂y ∂z
y2 x2 −x − z

∂ ∂  ∂ ∂ 
=  ( − x − z ) − ( x 2 ) i −  ( − x − z ) − ( y 2 ) j
 ∂ y ∂ z   ∂ x ∂ z 
∂ ∂ 
+  ( x 2 ) − ( y 2 ) k
 ∂ x ∂ y 
= (0) i − [ −1] j + [2 x − 2 y ] k = j + 2 ( x − y ) k .

Given C is the boundary of the triangle formed by the points (0, 0, 0), (1, 0, 0) and (1, 1, 0) which lie
in the xy-plane. ∴ n=k
B(1, 1)
\ curl F ⋅ n = 2 ( x − y ) y=x
\ ∫ F ⋅ dr = ∫∫ 2 ( x − y) dx dy
C S

Equation of OB is y = x
O A(1,0)
1 x
Fig. 5.32
\ ∫ F ⋅ dr = 2∫ ∫ ( x − y) dy dx
C 0 0

x
 y2
1
= 2∫  xy −  dx
0 
2 0
1
1
 x2  1
x2 x3  1
= 2∫  x 2 − − 0  dx = 2∫ dx =   = .
0 
2  0
2  3 0 3

Example 8
Verify Stoke’s theorem for F 5 ( y 2 z 1 2) i 1 ( yz 1 4) j 2 xzk , where S is the surface of the
cube x 5 0, x 5 2, y 5 0, y 5 2, z 5 0 and z 5 2 above the xy-plane.

Solution. z

Given F = ( y − z + 2)i + ( yz + 4) j − xzk .


G D
Stoke’s theorem is F
E


C
F ⋅ dr = ∫∫ curl F ⋅ n dS
S
O
C y
i j k
A B
∂ ∂ ∂ x
Now curl F =
∂x ∂y ∂z Fig. 5.33
y−z+2 yz + 4 − xz

M03_Eng-Maths (Aditya) CH05.indd 50 7/23/2018 4:58:06 PM


Vector Integration  n  5.51

∂ ∂  ∂ ∂ 
= i  ( − xz ) − ( yz + 4)  − j  ( − xz ) − ( y − z + 2) 
 ∂ y ∂ z   ∂ x ∂ z 
∂ ∂ 
+ k  ( yz + 4) − ( y − z + 2) 
 ∂ x ∂ y 
= i [(0 − y )] − j [ − z − ( −1)] + k (0 − 1) = − yi + ( z − 1) j − k

We shall compute ∫∫ Curl F ⋅n dS .


S

Given S is the open surface consisting of 5 faces of the cube except the face OABC.

∫∫ Curl F ⋅n dS = ∫∫
S S
Curl F ⋅n dS + ∫∫ Curl F ⋅n dS + ∫∫ Curl F ⋅n dS
S S
1 2 3

+ ∫∫ Curl F ⋅n dS + ∫∫ Curl F ⋅n dS
S4 S5

Face Equation Outward normal n F?n dS

S1 = ABEF x=2 i -y dy dz
S2 = OCDG x=0 −i y dy dz

S3 = BCDE y=2 j z-1 dx dz


S4 = OAFG y=0 −j -(z - 1) dx dz
S5 = DEFG z=2 k -1 dx dy

2 2 2 2 2
 −y 2 
\
∫∫ Curl F ⋅n dS = ∫ ∫ − y dy dz = ∫ dz ⋅ ∫ ( − y ) dy = [ z ]0 
2
 = 2( −2) = −4
S1 0 0 0 0  2 0
2
2 y 
2 2 2 2 2

∫∫ Curl F ⋅n dS = ∫0 ∫0 y dy dz = ∫0 ∫0
dz y dy = [ z ]0   = 2⋅⋅ 2 = 4
 2 0
S2
2 2 2 2 2
2  ( z − 1) 
∫∫ Curl F ⋅n dS = ∫ ∫ ( z − 1) dz dx = ∫ dx ⋅ ∫ ( z − 1) dz = [ x ]0 ⋅ 

S3 0 0 0 0
 2  0

= 2⋅
1
2
{ }
( 2 − 1) 2 − ( −1) 2 = 1 − 1 = 0

2 2

∫∫
S
Curl F ⋅n dS = ∫ ∫ −( z − 1) dz dx = 0
0 0
[as above]
4

2 2
and
∫∫ Curl F ⋅n dS = ∫ ∫ −1 dx dy = − [ x ]0 [ y ]0 = −4
2 2

S5 0 0

\ ∫∫S Curl F ⋅n dS = −4 + 4 + 0 + 0 − 4 = −4(1)

M03_Eng-Maths (Aditya) CH05.indd 51 7/23/2018 4:58:15 PM


5.52  n  Engineering Mathematics-III

We shall now compute the line integral over the simple closed curve C bounding the surface consisting
of the edges OA, AB, BC and CO in z = 0 plane

\
C
∫ F ⋅ dr = OA∫ F ⋅ dr + AB∫ F ⋅ dr + BC∫ F ⋅ dr + CO∫ F ⋅ dr
Now F ⋅ dr = ( y − z + 2)i + ( yz + 4) j − xzk  ⋅  dxi + dy j + dzk 
= ( y − z + 2)dx + ( yz + 4)dy − xzdz
⇒ F ⋅ dr = ( y + 2)dx + 4 dy [{ z = 0]

On OA:  y = 0   \  dy = 0 and F ⋅ dr = 2dx and x varies from 0 to 2


2

∫ F ⋅ dr = ∫ 2dx = 2 [ x ]0 = 4
2
\
OA 0

On AB:  x = 2   \  dx = 0 and F ⋅ dr = 4 dy and y varies from 0 to 2


2

∫ F ⋅ d r = ∫ 4dy = 4 [ y ]0 = 8
2
\
AB 0

On BC:  y = 2   \  dy = 0 and F ⋅ dr = 4 dx and x varies from 2 to 0


0

∫ F ⋅ dr = ∫ 4dx = 4 [ x ]2 = 4( −2) = −8
0
\
BC 2

On CO:  x = 0  \  dx = 0, F ⋅ dr = 4 dy and y varies from 2 to 0


0

∫ F ⋅ dr = ∫ 4dy = 4 [ y ]2 = −8
0
\
CO 2

\ ∫ F ⋅ dr = 4 + 8 − 8 − 8 = −4(2)
C

From (1) and (2), ∫ F ⋅ dr = S∫∫ Curl F ⋅n dS


C

Hence, Stoke’s theorem is verified.

Example 9
Verify Stoke’s theorem for F 5 ( x 2 2 y 2 ) i 1 2 xy j in the rectangular region in the xy plane
bounded by the lines x 5 0, x 5 a, y 5 0, y 5 b.

Solution.
Given F = ( x 2 − y 2 )i + 2 xy j

Stoke’s theorem is

∫ F ⋅ dr =
C S
∫∫ Curl F ⋅ n dS

M03_Eng-Maths (Aditya) CH05.indd 52 7/23/2018 4:58:23 PM


Vector Integration  n  5.53

y
i j k
y=b
∂ ∂ ∂ B
Curl F = C
∂x ∂y ∂z
x=0 x=a
x2 − y2 2 xy 0
= i( 0 − 0) − j ( 0 − 0) + k ( 2 y + 2 y ) = 4 y k A
O y=0 (a, 0) x
Since the surface is a rectangle in the xy-plane, normal n = k
Fig. 5.34
\ Curl F ⋅ n = 4 yk ⋅ k = 4 y
a b
∫∫ Curl F ⋅n dS = ∫ ∫ 4 y dx dy
S 0 0
b
aa b b
 y2 
⇒ ∫∫S Curl F ⋅n dS =
= ∫0∫∫0 ∫
4
dx y dx
4 y dy = [ x ]a
0
4   = 2ab (1)
 2 
2

0 0 0

We shall now compute the line integral.

\
C
∫ F ⋅ dr = OA∫ F ⋅ dr + AB∫ F ⋅ dr + BC∫ F ⋅ dr + CO∫ F ⋅ dr
Now F ⋅ dr = ( x 2 − y 2 )dx + 2 xydy
On OA:  y = 0    \  dy = 0 and F ⋅ dr = x 2 dx and x varies from 0 to a
a
a
 x3  a3
\ ∫ ⋅ = ∫0 = =
2
F dr x dx  
OA  3 0 3

On AB:  x = a    \  dx = 0 and F ⋅ dr = 2aydy and y varies from 0 to b


b
b
 y2 
\ ∫AB ⋅ = ∫0 2aydy = 2a  2  0 = ab
2
F dr

On BC:  y = b    \  dy = 0 and F ⋅ dr = ( x 2 − b 2 )dx and x varies from a to 0


0
0
 x3 2   a3 2  a3
\ ∫ ⋅ = ∫a − = − = − − = −
2 2 2
F dr ( x b ) dx  b x  0  b a ab
BC 3 a  3  3
On CO:  x = 0    \  dx = 0 and F ⋅ dr = 0
\ ∫
CO
F ⋅ dr = 0

a3 a3
\ ∫ F ⋅ dr =
C 3
+ ab 2 + ab 2 −
3
= 2ab 2(2)

From (1) and (2), ∫ F ⋅ dr = S∫∫ Curl F ⋅n dS


C
Hence, Stoke’s theorem is verified.

Note  Stoke’s theorem in the plane is Green’s theorem. This is indeed Green’s theorem verification.

M03_Eng-Maths (Aditya) CH05.indd 53 7/23/2018 4:58:31 PM


5.54  n  Engineering Mathematics-III

Example 10
Verify Stoke’s theorem for the vector field F 5 (2 x 2 y ) i 2 yz 2 j 2 y 2 z k over the upper half
surface x2 1 y2 1 z2 5 1, bounded by its projections on the xy-plane.

Solution. z
Stoke’s theorem is
∫ F ⋅ dr = ∫∫S curl F ⋅ n dS
C
S

Given F = (2 x − y )i − yz 2 j − y 2 zk
O y
i j k C
∂ ∂ ∂
\ Curl F = x
∂x ∂y ∂z
Fig. 5.35
2 x − y − yz 2 − y 2 z
∂ ∂ 
= i  ( − y 2 z ) − ( − yz 2 ) 
 ∂ y ∂ z 
∂ ∂  ∂ ∂ 
− j  ( − y 2 z ) − (2 x − y )  + k  ( − yz 2 ) − (2 x − y ) 
 ∂x ∂z   ∂ x ∂ y 
= i[ − 2 yz + 2 yz ] − j[0 − 0] + k [0 − ( −1)] = k
\ Curl F ⋅ n = k ⋅ n
The surface is the upper hemisphere x2 + y2 + z2 = 1
dxdy
∫∫ Curl F ⋅n dS = S∫∫ k ⋅ n dS = R∫∫ k ⋅ n
S k ⋅n
,

where R is the projection of S on the xy-plane.


\  R is the circle x2 + y2 = 1 in the xy-plane.
\ ∫∫ Curl F ⋅n dS = R∫∫ dx dy
S
⇒ ∫∫ Curl F ⋅n dS = Rarea
S
∫∫ dxofdythe circle = p ⋅ 1 2
= p(1)

Now C is the circle x2 + y2 = 1 in the z = 0 plane.


Parametric equations are     x = cos u, y = sin u, 0 ≤ u ≤ 2p
\ ∫ F ⋅ dr = C∫ [(2 x − y)dx − yz dy − y zdz ] = ∫ (2x − y )dx[{ z = 0]
2 2

C C
Now x = cos u  ⇒  dx = -sin udu
2p
\ ∫ F ⋅ dr = ∫ (2 cos u − sin u)( − sin u) du
C 0
2p

= ∫ ( −2 sin u cos u + sin


2
u) d u
0
2p
 1 − cos 2u 
= ∫  − sin 2u +
0
2  d u
2p
 cos 2u 1  sin 2u  
= + u − 
 2 2  2   0
M03_Eng-Maths (Aditya) CH05.indd 54 7/23/2018 4:58:38 PM
1 sin 4p 1
Vector Integration  n  5.55

2p

= ∫ ( −2 sin u cos u + sin


2
u) d u
0
2p
 1 − cos 2u 
= ∫  − sin 2u +
0
2  d u
2p
 cos 2u 1  sin 2u  
= + u − 
 2 2  2   0

⇒ ∫ F ⋅ dr == S∫∫12 Curl
C
(cos 4Fp⋅n−dS
cos 0) + 2p −
sin 4p
2
 1
− 0  = [1 − 1 + 2p] = p (2)
 2
From (1) and (2), ∫ F ⋅ dr = S∫∫ Curl F ⋅n dS
C
Hence, Stoke’s theorem is verified.

Example 11
Verify Stoke’s theorem for F 5 ( x 2 1 y 2 ) i 2 2 xy j taken around the rectangle bounded by the
lines x 5 6a, y 5 0 and y 5 b.

Solution. y=b
C B
Stoke’s theorem is

∫ F ⋅ dr = S∫∫ Curl F ⋅ n dS
C
x = −a x=a

Given F = ( x 2 + y 2 )i − 2 xy j D O A x
(−a, 0) (a, 0)
i j k Fig. 5.36
∂ ∂ ∂
\ Curl F =
∂x ∂y ∂z
x2 + y2 −2 xy 0
= i[0 − 0] − j (0 − 0) + k ( −2 y − 2 y ) = −4 y k
Since S is the rectangular surface, n = k

∫∫ Curl F ⋅n dS = S∫∫ −4 y k ⋅ k dx dy
S
b
b a
 y2 
= −4∫ ∫ y dx dy = −4   [ x ]− a = −2b 2 ⋅ 2a = −4 ab 2
a

0 −a  2 0
2(1)
∫∫ Curl F ⋅n dS = ∫∫
S
−4 y k ⋅ k dx dy
- 4ab
S

b
b a
 y. 2 
We shall now compute the y dx dy ∫= F−4⋅ dr
4∫ ∫integral
= −line   [ x ]− a
a

0 −a C  2  0

2 + dy j ] = ( x + y ) dx − 2 xy
2 2
Now F ⋅ dr = [( x 2
+ y 2
)i − 2 xy 2j ] ⋅ [ dxi
Now F ⋅ dr = [( x + y )i − 2 xy j ] ⋅ [dxi + dy j ] = ( x +dyy )dx − 2 xy dy
2 2

M03_Eng-Maths (Aditya) CH05.indd 55 7/23/2018 4:58:45 PM


5.56  n  Engineering Mathematics-III

\ ∫ F ⋅ dr = AB∫ F ⋅ dr + BC∫ F ⋅ dr + CD∫ F ⋅ dr + DA


C
∫ F ⋅ dr
On AB: x = a  \  dx = 0 and F ⋅ dr = −2ay dy and y varies from 0 to b
b
b
 y2 
\ ∫ ⋅ = ∫0 − = −  = − ab
2
F dr ( 2 a ) ydy 2 a 
AB  2 0

On BC: y = b  \  dy = 0 and F ⋅ dr = ( x 2 + b 2 )dx and x varies from a to -a


−a −a
 x3 2 
\ ∫ F ⋅ dr = ∫a ( x + b ) dx =  3 + b x  a
2 2

BC
1 −2 3
= ( − a 3 − a 3 ) + b 2 ( − a − a) = a − 2ab 2
3 3
On CD: x = -a  \  dx = 0 and F ⋅ dr = 2aydy and y varies from b to 0
0
0
 y2 
\ ∫CD ⋅ = ∫b =   = a(0 − b ) = − ab
2 2
F dr 2 ay dy 2 a
 2 b

On DA: y = 0  \  dy = 0 and F ⋅ dr = x 2 dx and x varies from -a to a


a
a a
 x3  2 3
\ ∫ F ⋅ dr = ∫− a = ∫0 =   = a
2 2
x dx 2 x dx 2
DA  3 0 3
2 2
\ ∫ F ⋅ dr = − ab − a3 − 2ab 2 − ab 2 + a3 = −4ab 2
2

C 3 3
From (1) and (2),  ∫ F ⋅ dr = ∫∫ Curl F ⋅n dS
C S
Hence, Stoke’s theorem is verified.

Example 12
Verify stokes theorem for F 5 y 2 zi 1 z 2 xj 1 x 2 yk , where S is the open surface of the cube
formed by the planes x 5 2a, x 5 a, y 5 2a, y 5 a, z 5 2a, z 5 a in which z 5 2a is cut open.

Solution. H E
Stoke’s theorem is ∫ F . dr = ∫∫ curl F .n ds G
S F
z
Given F = y zi + z x j + x yk
2 2 2
y
o
x
l j k D
A

\ Curl F = ∂ ∂ ∂
B C
∂x ∂y ∂z
Fig. 5.37
y z z2 x
2
x2 y

M03_Eng-Maths (Aditya) CH05.indd 56 7/23/2018 4:58:53 PM


Vector Integration  n  5.57

 ∂ ∂  ∂ ∂  ∂ ∂ 2 
= i  ( x 2 y ) − ( z2 x )  − j  (x2 y ) − ( y 2 z )  + k  ( z 2 x ) − ( y z)
 ∂y ∂z   ∂x ∂z   ∂x ∂y 
= ( x 2− 2zx) i + ( y2− 2xy ) j + (z 2 − 2yz) k
We shall now compute ∫∫ curl F. n dS
S
Given S is the open surface consisting of the five faces of the cube except face ABCD
∴ ∫∫ curl F . ndS = ∫∫ curl F. ndS + ∫∫ curl F. ndS + ∫∫ curl F. ndS + ∫∫ curl F. ndS + ∫∫ curl F. ndS
S S1 S2 S3 S4 S5

Face Equation Normal n Curl F.n dS


S1 = BCFG x=a i a - 2az
2
dy dz
S2 = ADEH x = -a −i -(a + 2az)
2
dy dz
S3 = CDEF y=a j a - 2ax
2
dz dx
S4 = ABGH y = -a −j -(a + 2ax)
2
dz dx
S5 = EFGH z=a k a2 - 2ay dx dy
a a

∫∫ curl F . ndS = ∫ ∫ (a − 2 az)dydz


2

S1 −a −a
a
 a  a  a  z2 
=  ∫ dy   ∫ a 2 − 2az dydz  = [ y ]− a a 2 z − 2 a  = [ a + a ] a 2 ( a + a ) −a (a 2 − a 2) = 4 a4
−a  −a   2 −a
a a
 a  a 
∫∫ curl F. ndS = ∫ ∫ − (a + 2az ) dydz = −  ∫ dy   ∫ a 2 + 2az dz
2

S2 −a −a
a
−a  −a 
 z2 
= − [ y ]− a a 2 z + 2 a  = − [ a + a ] a 2 ( a + a ) + a (a 2 − a 2) = − 4 a4
a

 2 −a
a a
Similarly, ∫∫ curl F. ndS = ∫ ∫ (a
2
− 2ax )dzdx = 4a 4
S3 −a −a

a a

∫∫ curl F . n dS = ∫ ∫ −( a
2
+ 2 ax) dzdx = −4a4
S4 −a −a

a a
and ∫∫ curl F. ndS = ∫ ∫ (a − 2ay ) dxdy = 4a
2 4

S5 −a −a

\ ∴ ∫∫ curl F. n dS = 4a4 − 4a4 + 4 a4 − 4 a4 + 4 a4 = 4a4(1)


S

We shall now compute the line integral over the simple closed curve C consisting of the edges AB, BC,
CD, DA. Here z = -a, dz = 0
∴ F. dr = y 2 zdx + z 2 xdy ∴
+ xF2.ydz
dr == y−2ay + z+2axdy
zdx2dx 2
xdy+ x 2 ydz = −ay 2dx + a 2 xdy

M03_Eng-Maths (Aditya) CH05.indd 57 7/23/2018 4:59:04 PM


5.58  n  Engineering Mathematics-III

D
On AB:  y = -a ∴dy = 0 A
(−a, a)
(−a, −a)
F. dr = − a dx and x varies from –a to a.
3

∫ ∫ − a dx = −a [ x ]
a
\ ∴ F. dr = 3 3
−a
= −a3 .2a = −2a4
AB −a
B C
On BC:  x = a ∴dx = 0 , F. dr = a3dy and y varies from -a to a. (a, −a) (a, a)
a
Fig. 5.38
∫ ∫ a dy = a [ r]
a
\ ∴ F. d r = 3 3
−a
= a3 .2 a = 2 a4
BC −a

On CD: y = a ∴ dy = 0, F. dr = − a3dx and x varies from a to -a


−a

∫ ∫ − a dx = −a [ x ]
−a
\ ∴ F. dr = 3 3
a
= −a3 ( −2a) = 2a4
CD a

On DA: x = -a ∴dx = 0, F. dr = − a3dy and y varies from a to -a.


−a

∫ ∫ − a dy = −a3 [ y ]a = −a2 ( −2a) = 2a4 (2)


−a
\ ∴ F. dr = 3

DA a

\ ∴ ∫ F ⋅ d r = −2 a4 + 2 a4 + 2 a4 + 2 a4 = 4 a4(3)
c

From (1) and (2), we get ∫∫ curl F. nds = ∫ F. dr


S C
Hence, Stoke’s theorem is verified.

EXERCISE 5.2

∫∫ F ⋅ n dS, where F = 12 x yi − 3 yz j + 2 zk and S is the portion of the plane x + y + z = 1


2
1. Evaluate
S
included in the first octant.

∫∫ F ⋅ n dS, where F = (2 x − 3z )i + 2 y j − 4 xzk , where S is the surface of the solid


2
2. Evaluate
S
bounded by the planes x = 0, y = 0, z = 0 and 2x + 2y + z = 4.
3. Evaluate ∫∫ F ⋅ n dS , where F = zi + x j − y 2 zk and S is the curved surface of the cylinder x2 + y2 = 1
S
included in the first octant between the planes z = 0 and z = 2.
4. If F = xy 2 i − yz 2 j + zx 2 k , find ∫∫ F ⋅ n dS over the sphere x
S
2
+ y2 + z2 = 1.

∫∫ F ⋅ n dS, where F = 4 xzi − y j + yzk and S is the surface of the cube bounded by
2
5. Evaluate
S
x = 0, x = 1, y = 0, y = 1, z = 0 and z = 1.

M03_Eng-Maths (Aditya) CH05.indd 58 7/23/2018 4:59:13 PM


Vector Integration  n  5.59

∫ (x + xy )dx + ( x 2 + y 2 ) dy, where C is the square formed by the lines y = ±1, x = ±1,
2
6. Evaluate
C
by Green’s theorem.
7. Using Green’s theorem evaluate ∫ ( x 2 + y )dx − xy 2 dy taken around the square whose vertices are
C
(0, 0), (1, 0), (1, 1) (0, 1)
8. Using Green’s theorem find the value of ∫ ( xy − x 2 )dx + x 2 y dy along the closed curve C formed
C
by y = 0, x = 1 and y = x.

∫ (15 x − 4 y 2 )dx + ( 2 y − 3x ) dy , where C is the curve enclosing the


2
9. Verify Green theorem for
C
area bounded by y = x2, x = y2
10. Verify Green theorem in the plane for ∫ (3 x 2 − 8 y 3 )dx + ( 4 y − 6 xy ) dy, where C is the boundary
C
of the region defined by x = 0, y = 0, x + y = 1.
11. Using Green’s theorem find the area of x 2 / 3 + y 2 / 3 = a 2 / 3 .
1
[Hint: Area = ∫ ( xdy − ydx ), C is the boundary of the curve]
2C
12. Using Green’s theorem in xy plane find the area of the region in the xy plane bounded by
y3 = x2 and y = x.
∫ (2 x − y 2 )dx + ( x 2 + y 2 ) dy , where C is the boundary of the
2
13. Using Green’s theorem evaluate
C
area in the xy plane bounded by x-axis and the semi circle x2 + y2 =1 in the upper half of the plane.
14. Verify Gauss divergence theorem for F = x 2 i + zi + yzk taken over the cube bounded by x = 0,
x = 1, y = 0, y = 1, z = 0, z = 1.
15. Verify Gauss divergence theorem for F = ( x 3 − yz )i − 2 x 2 y j + 2k over the parallelopiped bounded
by the planes x = 0, x = 1, y = 0, y = 2, y = 2, z = 0, z = 3.
16. Verify Gauss divergence theorem for F = x 2 i + z j + yzk over a unit cube.
17. Verify Gauss divergence theorem for F = ( x 3 − yz )i − zx 2 y j + 2k over the cube x = 0, x = a, y = 0,
y = a, z = 0, z = a.
18. Verify the divergence theorem for F = 2 xyi + yz 2 j + xzk , where S is the rectangular
parallelopiped bounded by x = 0, y = 0, z = 0, x = 2, y = 1, z = 3.
19. Using divergence theorem show that
1

∫∫
S
x 2dy + y 2dzdy + 2z ( xy − x − y )dxdy = , where S is the surface of the cube
2
x = y = z = 0, y = z =1.

∫∫ (2 xyi + yz j + xzk ).dS , where S is the surface of the


2
20. Use divergence theorem to evaluate
S
region bounded by x = y = z = 0, y = 3, x + 2z = 6.
21. Prove that ∫∫ [x ( y − z )i + y ( z − x ) j + z ( x − y )k ] ⋅ dS = 0, where S is any closed surface.
S

M03_Eng-Maths (Aditya) CH05.indd 59 7/23/2018 4:59:20 PM


5.60  n  Engineering Mathematics-III

22. Verify Stoke’s theorem for F = 2 zi + x j + y 2 k , where S is the surface of the paraboloid z = 4 - x2 - y2
and C is the simple closed curve in the xy plane.
23. Verify Stoke’s theorem for F = yi + z j + xk , where S is the upper half surface of the sphere
x2 + y2 + z2 = 1 and C its boundary.
24. Verify Stoke’s theorem for F = ( x 2 − y 2 )i + 2 xy j + xyzk over the surface of the box bounded by
the planes x = 0, y = 0, x = a, y = b, z = c above the xy plane.
25. Verify Stoke’s theorem for F = ( x 2 − y 2 )i + 2 xy j in the rectangular region in the xy plane bounded
by x = 0, x = a, y = 0, y = b.
26. Verify Stoke’s theorem for F = − y 3 i + x 3 j and the closed curve C is the boundary of the ellipse
x2 y2
+ = 1.
a2 b2
27. If f is scalar point function, use Stoke’s theorem to prove curl (grad f) = 0.
28. Evaluate ∫∫ ∇ × F ⋅ n dS,
S
where S is the surface x2 + y2 + z2 = a2 above the xy-plane and

F = yi + ( x − 2 xz ) j − xyk .
29. Evaluate ∫ yzdx + zx dy + xy dz , where C is the curve x
C
2
+ y2 = 1, z = y2.

30. Evaluate ∫∫ ∇ × F ⋅ n dS for F = (2 x − y + z )i + ( x + y − z 2 ) j + (3x − 2 y + 4 z )k over the surface of


the cylinder x2 + y2 = 4, bounded by the plane z = 9 and open at the end z = 0.
31. Find the area of a circle of radius a using Green’s theorem.

∫ [(2xy − x )dx + ( x 2 + y 2 )dy ] where C is the closed curve of


2
32. Using Green’s theorem evaluate
C
the region bounded by y = x2 and y2 = x
33. Verify Green’s theorem in a plane for the integral
C
∫ ( x − 2 y)dx + xdy taken around the circle

x2 + y2 = 4.

∫ [(x − xy 3 )dx + ( y 2 − 2xy ) dy ] where C is the square


2
34. Verify Green’s theorem in the plane for
C
with vertices (0, 0) (2, 0), (2, 2), (2, 0).
35. Evaluate ∫∫∫ ∇ ⋅ F dV if F = x i + y j + z 2 k and V is the volume of the region enclosed by the
2 2

V
cube x = 0, x = 1, y = 0, y = 1, z = 0, z = 1.
36. If S is any closed surface enclosing volume V and F = axi + byj + czk prove that
∫∫ F ⋅ n dS = (a + b + c) V
S
37. Verify Gauss divergence theorem for F = ( x 2 − yz )i + ( y 2 − zx ) j + ( z 2 − xy )k taken over the
rectangular parallelopiped bounded by 0 ≤ x ≤ a, 0 ≤ y ≤ b, 0 ≤ z ≤ c.
38. Verify Stoke’s theorem for F = y 2 zi + z 2 x j + x 2 yk where S is the open surface of the cube formed
by the planes x = -a, x = a, y = -a, y = a, z = -a, z = a in which z = -a is cut open.

M03_Eng-Maths (Aditya) CH05.indd 60 7/23/2018 4:59:28 PM


Vector Integration  n  5.61

39. Evaluate ∫∫ Curl F ⋅ n dS, where F = ( y − z )i + yz j − xz k and S is the open surface bounded by
S
the planes x = 0, x = 1, y = 0, y = 1, z = 0, z = 1 above the xy plane.

ANSWERS TO EXERCISE 5.2


49 16 4 3 4
1.   2.   3. 3  4.  p  5.   6. 0  7.  −
120 3 3 2 3
1 3 1 4 351
8.  − 11. pa3 12.  13.  20.  28. 0 29. 0
12 8 10 3 2
30. 8p 31. pa2 32. 0 35. 3 36. (a + b + c) V 39.  -1

M03_Eng-Maths (Aditya) CH05.indd 61 7/23/2018 4:59:31 PM


This page is intentionally left blank

M03_Eng-Maths (Aditya) CH01.indd 42 7/24/2018 1:07:09 PM


A
Appendix

Curve Tracing
A.1  Curve Tracing
In dealing with the problems of finding the area of curves, length of arc, volume of solids of
revolution, surface area of revolution etc, it is necessary to know the shape of the curve represented
by the equation.
It is not always possible to draw the curve by plotting few of the points. We can only draw the curve
with the knowledge of the important characteristics of the curve like increasing, decreasing nature,
maxima and minima, special points on the curve, concavity and convexity, asymptotes of the curve
etc.
We shall now give the general procedure for tracing the graph of y = f ( x ). The equation may be
given in cartesian form, parametric form or polar form.

A.1.1  Procedure for Tracing the Curve Given by the Cartesian Equation f(x, y) 5 0
1. Symmetry
The curve is symmetrical
(i) about the x-axis if the equation is even degree in y .
(ii) about the y-axis if the equation is even degree in x .
( iii) about the origin O , when ( x , y ) is replaced by ( − x , − y ) , the equation is unaltered
(iv) about the line y = x if the equation is unaltered when x and y are interchanged. i.e., ( x , y ) is
replaced by ( y , x )
(v) about the line y = − x if the equation is unaltered when( x , y ) is replaced by ( − y , − x ).

2. Special points on the curve


Intersection with the axes and the origin, points of inflection etc.

3. Tangents at the origin


It is obtained by equating the lowest degree terms to zero, if it is a polynomial equation in x and y
passing through the origin.

4. Asymptotes
Find the vertical, horizontal and oblique asymptotes.

5. Region
Identify the domain or region of the plane in which the graph exists.

M03_Eng-Maths (Aditya)_App2.indd 1 7/25/2018 11:09:48 AM


A.2  n  Engineering Mathematics-III

dy
6. Sign of
dx
Determine the intervals of increasing, decreasing, Critical points etc.
d 2y
7. Sign of
dx 2
Intervals of concavity upwards and downwards and point of inflexion.
8. Loop
If the curve intersects the line of symmetry at two points A and B , then there is a loop
between A and B .

Note
However, the order of the steps can be interchanged depending on the nature of the equation of the curve.

WORKED EXAMPLES
Example 1
Trace the curve y 2 5 x 3.   [It is called the semi-cubical parabola]

Solution.
The given curve is y 2 = x 3 .
1. Symmetry
The given equation is even degree in y, so the curve is symmetrical about the x-axis.
2. Origin: it is a point on the curve.
3. Tangent at the origin:
The tangent at the origin is got by equating the lowest degree terms to zero.
That is y 2 = 0 ⇒ y = 0
∴ the x-axis is the tangent at the origin.
4. Region
y 2 ≥ 0 ⇒ x 3 ≥ 0 ⇒ x ≥ 0.  ∴  the curve lies on the right side of y-axis.
dy
5. Sign of
dx
dy dy 3x 2
y 2 = x 3 ⇒ 2y = 3x 2   ⇒   =
dx dx 2y
dy
∴ if  y > 0, > 0  That is, the curve is increasing for all x ≥ 0 and y > 0.
dx
So, the curve is increasing in the first quadrant.
dy
Also if  y < 0, < 0 i.e., the curve is decreasing for all x ≥ 0 and y < 0.
dx
So, the curve is decreasing in the 4 th quadrant.

M03_Eng-Maths (Aditya)_App2.indd 2 7/24/2018 2:25:04 PM


Curve Tracing  n  A.3

d 2y
6. Sign of
dx 2
 dy   2 dy 
y ⋅ 2x − x 2 ⋅  3  2xy − x ⋅ dx
d 2y 3  dx 
=  =  
dx 2 2  y2  2 y2 
   
3 x 3 x 3 x4 3 x4 3 x4 1 3 x
=  4 y 2 − 3x 3  = ⋅ 3  4 x 3 − 3x 3  =
3  3
= 2
= ⋅ = .
2y 4 y 4y 4 yy 4 y x3 4 y

d 2y d 2y
∴ > 0 if y > 0 (as already x > 0) and < 0 if y < 0.
dx 2 dx 2
∴ the curve is concave up if y > 0 and concave y
down if y < 0.
So, the curve is concave up in the first quadrant and the
curve is concave down in the fourth quadrant.
o x
7. Asymptotes
It has no asymptotes.
With these information we shall draw the curve.
The curve is as shown in Fig. A.1.
Fig. A.1
Example 2
Trace the curve y 2 ( 2 a 2 x ) 5 x 3.  [This curve is called the Cissoid of Diocles]

Solution.
x3
The given equation of the curve is y 2 = . (1)
2a − x
1. Symmetry
The equation is even degree in y, so the curve is symmetrical about the x-axis.
2. Origin: It is a point on the curve.
The tangent at the origin is given by y 2 = 0 ⇒ y = 0 That is the x-axis is the tangent at the origin.
3. Region:
x3 x3 x3
y2 ≥ 0 ⇒ ≥0 ⇒ ≤ 0  ⇒   ≤ 0 ⇒ 0 ≤ x < 2a .
2a − x x − 2a x − 2a
∴ the curve lies between the lines x = 0 and x = 2a .
4. Asymptote
When x → 2a , y → ∞  ∴ x = 2a is a vertical asymptote.
dy
5. Sign of
dx
Differentiating (1) with respect to x, we get

M03_Eng-Maths (Aditya)_App2.indd 3 7/24/2018 2:25:08 PM


A.4  n  Engineering Mathematics-III

y
dy ( 2a − x ) ⋅ 3x 2 − x 3 ( −1) 6ax 2 − 2x 3
2y = =
dx ( 2a − x ) 2 ( 2a − x ) 2

dy 2x 2 (3a − x ) x 2 (3a − x )
⇒ = =
dx 2 y ( 2a − x ) 2 y ( 2a − x ) 2
o x
Since 0 ≤ x < 2a, 3a − x > 0 and ( 2a − x ) 2 > 0. x = 2a
dy
∴ > 0 if y > 0
dx
dy
and < 0 if y < 0
dx Fig. A.2
∴ the curve is increasing in the first quadrant and the curve is decreasing in the 4 quadrant th

Also it touches x = 2a at infinity


With these informations, we can draw the curve.
The curve is as shown in Fig. A.2.

Example 3
x 2 (a2 2 x 2 )
Trace the curve y 2 5 . [This curve is called Lemniscate of Bernoulli]
a2 1 x 2

Solution.
x 2 (a 2 − x 2 )
The given equation of the curve is y 2 =  (1)
a2 + x 2
1. Symmetry
The equation is even degree in x and y .
So, the curve is symmetrical about the x-axis as well as the y-axis.
When ( x , y ) is replaced by ( − x , − y ) equation is unaltered.
∴ the curve is symmetrical about the origin.
2. Origin
Origin is a point on the curve.
3. Tangent at the origin
We have

x 2 (a 2 − x 2 )
y2 =   ⇒   y 2 (a 2 + x 2 ) = x 2 a 2 − x 4   ⇒   a 2 ( x 2 − y 2 ) = x 2 y 2 + x 4
a2 + x 2
Tangent at the origin is got by equating the lowest degree terms to zero.

i.e., x 2 − y 2 = 0  ⇒ y = ± x

∴ y = x and y = − x are the tangents at the origin.

M03_Eng-Maths (Aditya)_App2.indd 4 7/24/2018 2:25:13 PM


Curve Tracing  n  A.5

4. Special points
To find the point of intersection with the x-axis, put y = 0 in (1)

∴ x 2 (a2 − x 2 ) = 0   ⇒   x 2 = 0  or a2 − x 2 = 0   ⇒   x = 0, 0  or x = ±a

So, the curve passes through the origin twice and the points ( −a, 0), (a, 0) .
To find the intersection with the y-axis, put x = 0 in (1).
∴ y =0
So, it meets the y-axis only at the origin.
y
5. Region
x (a − x )
2 2 2 y = −x
y2 = y=x
a2 + x 2
y 2 ≥ 0 ⇒ x 2 (a2 − x 2 ) ≥ 0
⇒ a2 − x 2 ≥ 0
B A
(−a, 0) (a, 0) x
⇒ x 2 − a2 ≤ 0 ⇒ −a≤ x ≤ a. O

\ the curve lies between x = -a and x = a.


6. Loop
Since the curve meets the line of symmetry, the x-axis,
at O(0, 0), A( a, 0), B( − a, 0) there is a loop between
O and A and a loop between O and B. Fig. A.3
With these informations, we shall draw the curve.
The curve is as shown in Fig. A.3.

Example 4
Trace the curve x 3 1 y 3 5 3axy , a > 0.  [This curve is called the Folium of Descartes]

Solution.
The given equation of the curve is x 3 + y 3 = 3axy , a > 0 (1)
1. Symmetry
The equation is unaltered if x and y are interchanged.
So, the curve is symmetric about the line y = x .
2. Origin
Origin lies on the curve.
3. Tangent at the origin
Tangents at the origin are got by equating the lowest degree terms to zero

∴ xy = 0  ⇒  x = 0, y = 0

So, the y-axis and the x-axis are the tangents at the origin.

M03_Eng-Maths (Aditya)_App2.indd 5 7/24/2018 2:25:16 PM


A.6  n  Engineering Mathematics-III

4. Special points
To find the point of intersection with y = x, put y = x in (1)

∴ x 3 + x 3 = 3ax 2

⇒ 2x 3 − 3ax 2 = 0
3a
⇒ x 2 ( 2x − 3a) = 0   ⇒  x 2 = 0 or 2x − 3a = 0   ⇒  x = 0, 0 or x =
2
When x = 0, y = 0

3a 3a  3a 3a 
When x = ,y =   ∴ the point of intersections are O(0, 0) and A  , 
2 2  2 2

The curve meets the axes only at the origin, twice.


5. Loop y
Since the curve intersects the line of symmetry
y = x at O and A, there is a loop between O and A. x
y=
6. Asymptotes 3a , 3a
x=0 A
The coefficients of x3 and y3 are constants and so 2 2
there is no vertical or horizontal asymptotes.
To find the oblique asymptotes of
y=0 x
x 3 − y 3 − 3axy = 0 O

Put x = 1, y = m in the highest degree terms x 3 + y 3

∴  f3 ( m ) = 1 + m 3 , f′3 ( m ) = 3m 2 x+y+a=0

Now put x = 1, y = m in −3axy


∴ f2 ( m ) = −3am . Fig. A.4
Solve, f3 ( m ) = 0     ⇒  1 + m = 0  ⇒  m = −1
3

−f2 ( m ) −(3am ) a
Now c= =− =
f3′( m ) 3m 2 m

a
When m = −1,   c = = −a   ∴ asymptote is y = −x − a  ⇒   x + y + a = 0
−1
With these informations, we can draw the curve. The curve is as shown in Fig. A.4.

Example 5
Trace the curve y 2 5( x 21)( x 2 2 )( x 23).

Solution.
The equation of the given curve is y 2 = ( x − 1)( x − 2)( x − 3)  (1)
1. Symmetry
The equation is even degree in y and so the curve is symmetrical about the x-axis.

M03_Eng-Maths (Aditya)_App2.indd 6 7/24/2018 2:25:20 PM


Curve Tracing  n  A.7

2. Special points
To find the point of intersection with the x-axis, put y = 0

∴ ( x − 1)( x − 2)( x − 3) = 0  ⇒  
x = 1, 2, 3

It does not intersect the y-axis, because when x = 0, y 2 = −6 < 0. So, y is imaginary
3. Region
−∞ ∞
If x < 1, then x − 1 < 0, x − 2 < 0, x − 3 < 0 1 2 3

∴ ( x − 1)( x − 2)( x − 3) < 0 ⇒ y 2 < 0   ∴ y is imaginary

So, the curve does not exist if x < 1


But if 1 ≤ x ≤ 2 and x ≥ 3, y 2 ≥ 0
So, the curve lies in between x = 1 and x = 2 and x ≥ 3.
4. Loop
The curve lies between the points A (1, 0)and B( 2, 0) and symmetric about the x-axis and so there is
a loop between A and B
dy
5. Sign of
dx
y 2 = ( x − 1)( x − 2)( x − 3) = x 3 − 6 x 2 + 11x − 6
dy   dy 3x 2 − 12x + 11
2y = 3x − 12x + 11
2 ⇒   =
dx dx 2y
dy dy
If x > 3, > 0, when y > 0 and < 0, when y < 0
dx dx
So, for all x ≥ 3, the curve is strictly increasing above the x-axis and strictly decreasing below the
x-axis.
With these information, we shall draw the graph of the curve.
The curve is as shown in Fig. A.5.
y

O (1, 0) (2, 0) (3, 0) x

x=1 x=2 x=3

Fig. A.5
Example 6
x2 11
Trace the curve whose equation is y 5 .
x 2 21

M03_Eng-Maths (Aditya)_App2.indd 7 7/24/2018 2:25:24 PM


A.8  n  Engineering Mathematics-III

Solution.
x 2 +1
The equation of the given curve is y = (1)
x 2 −1
1. Symmetry
Since the equation is even degree in x, the curve is symmetrical about the y-axis.
2. Asymptotes
When x = −1 and x = 1, y → ∞
∴ x = −1 and x = 1 are vertical asymptotes.

1
1+
x 2 +1 x2 =1
lim y = lim 2 = lim
x →∞ x →∞ x − 1 x →∞ 1
1− 2
x

∴ y = 1 is the horizontal asymptote.


3. Region
x 2 +1 y +1
y = y ( x 2 − 1) = x 2 + 1  ⇒   x 2 ( y − 1) = y + 1  ⇒   x 2 =
  ⇒  
x −1
2
y −1
y +1
Series x 2 ≥ 0 ⇒ ≥ 0 ⇒ y ≤ −1 or y ≥ 1
y −1
The curve lies in the part of y ≤ −1 and y > 1
i.e., the curve lies above the line y = 1 and below the line y = −1 for all x ≠ ±1
dy
4. Sign of
dx
x2 + 1
y=
x2 −1
dy ( x 2 − 1) ⋅ 2 x − ( x 2 + 1)( 2 x ) 2 x[ x 2 − 1 − x 2 − 1] 4x
∴ = = =− 2
dx ( x − 1)
2 2
( x − 1)
2 2
( x − 1) 2

dy dy
∴ < 0 if x > 0   and  > 0 if x < 0
dx dx
So, the curve is increasing if x < 0 and is decreasing if x > 0
When x = 0, y = −1. In the interval (-1, 1) the curve increases upto the point (0, -1) and then
decreases.
If −1 < x < 1, then x 2 − 1 < 0. ∴ y < 0
So, in this part, the curve lies below the x-axis.
dy
If x > 1, then < 0 and so, the curve is decreasing.
dx
dy
If x < −1,then > 0 and so, the curve is increasing.
dx

M03_Eng-Maths (Aditya)_App2.indd 8 7/24/2018 2:25:29 PM


Curve Tracing  n  A.9

d 2y
5. Sign of
dx 2
dy 4x
We have =− 2
dx ( x − 1) 2

d 2 y −4 ( x − 1) ⋅1 − x ⋅ 2( x − 1)( 2 x ) 
2 2 2

=
dx 2 ( x 2 − 1) 4

( x 2 − 1)[ x 2 − 1 − 4 x 2 ] 4(1 + 3 x 2 )
= −4 =
( x 2 − 1) 4 ( x 2 − 1)3

d 2y
If x > 1, > 0   ∴ the curve is concave up [∴ x2 - 1 > 0]
dx 2
d 2y
If x < -1, > 0   ∴ the curve is concave up
dx 2
6. Assymptotes
x = -1, x = 1 are the vertical asymptotes. y = 1 is the horizontal asymptote.
The curve lies in the region y < -1 and y > 1 and decreasing if x > 1 and concave up.
Increasing if x < -1 and concave up
We draw the curve.
The curve is as shown in Fig. A.6.
y

(0, 1) y=1

O x

(0, −1)
y = −1
x = −1 x=1

Fig. A.6

M03_Eng-Maths (Aditya)_App2.indd 9 7/24/2018 2:25:31 PM


A.10  n  Engineering Mathematics-III

A.1.2 Procedure for tracing of curve given by parametric equations x 5 f(t), y 5 g(t)


If the current coordinates (x, y) of the curve are expressed interms of another variable t, then t is called
the parameter and the equations x = f(t), y = g(t) are called the parametric equations of the curve.
1. Symmetry
(i) If f(−t) = f(t) and g(−t) = −g(t), then the curve is symmetrical about the x-axis.
(ii) If f(−t) = −f(t) and g(−t) = g(t), then the curve is symmetrical about the y-axis.
(iii) If f(−t) = f(t) and g(−t) = g(t), then the curve is symmetrical in the opposite quadrants.
2. Special points
To find the points of intersection with x-axis, put y = 0 ⇒ g(t) = 0
To find the points of intersection with the y-axis, put x = 0 ⇒ f(t) = 0.
3. Region
Determine the limits of x and y and hence the limt of t
dy
4. Sign of derivative
dx
dx dy
Find and and the values of t for which x and y are increasing or decreasing.
dt dx
Find the tangent parallel to the axes.
dy
i.e., = 0 or ∞.
dx
d 2y
Also check for concavity. i.e., > 0 or < 0
5. Period dx 2
If x and y are periodic functions of t with a common period, study the curve in this period.

Note  If it is possible to eliminate the parameter t and get the Cartesian form, then we can trace the
curve by the first method.

WORKED EXAMPLES
Example 7
a t
Trace the curve x 5 a cos t 1 log e tan 2 ; y 5 a sin t .  [This curve is called the tractrix]
2 2
Solution
a t
Given x = a cos t + log e tan 2 and y = a sin t .
2 2
Let x = f (t)                 and          y = g(t).

1. Symmetry
a  t a t
f ( −t ) = a cos( −t ) + loge tan 2  −  = a cos t + loge tan 2 = f (t )
2  2 2 2
and g ( −t ) = a sin( −t ) = −a sin t = − g (t ).
∴ the curve is symmetrical about the x-axis.
2. Intersection with the axes
To find the intersection with the x-axis, put y = 0     ∴  a sin t = 0 ⇒ t = 0, p, 2p, ...

M03_Eng-Maths (Aditya)_App2.indd 10 7/24/2018 2:25:32 PM


Curve Tracing  n  A.11

When t = 0, x → ∞ and y = 0
∴ x-axis is an asymptote to the curve
p p
When t = t =, x =, 0x = and y = ya = a
0 and
2 2
∴ the curve intersects the y-axis at (0, a).
3. Sign of derivative
dx a 1 t t 1
= −a sin t + ⋅ ⋅ 2 tan ⋅ sec 2 ⋅
dt 2 t 2 2 2
tan 2
2
a 1 1
= −a sin t + ⋅ ⋅
2 t t
tan cos 2
2 2
y
a a(1 − sin 2 t ) a cos 2 t π
= −a sin t + = = t= (0, a)
sin t sin t sin t 2

dy
and = a cos t .
dt
t=0 t=π
dy
O x
dy a cos t sin t
∴ = dt = = = tan t
dx dx 2
a cos t cos t Fig. A.7
dt sin t

p dy y
When t = , = ∞ and the point is (0, a)
2 dx (0, a)
∴y-axis is tangent at (0, a)
p dx dy
If 0 < t < , then > 0, >0
2 dt dt
∴ x increases from −∞ to 0 and y increase x′ O x
from 0 to a
i.e., (x, y) varies from (−∞, 0) to (0, a)
p dx dy
If < t < p, then > 0, <0
2 dt dt (0, −a)
∴ x increases from 0 to ∞ y′
and y decreases from a to 0 Fig. A.8
i.e., (x, y) varies from (0, a) to (∞, 0).
∴ the graph above the x-axis is as in Fig. A.7
Since the curve is symmetric about the x-axis, taking reflection about the x-axis, we get the graph of
the given equation as in Fig. A.8.
a  a − a2 − y 2 
Note  The cartesian equation of the tractix is x = a2 − y 2 + log  
2 a+ a −y 
2 2

M03_Eng-Maths (Aditya)_App2.indd 11 7/24/2018 2:25:34 PM


A.12  n  Engineering Mathematics-III

Example 8
Trace the curve x 5 a ( u 2 sin u), y 5 a (1 2 cos u).  [This curve is called a cycloid]
Cycloid is a curve traced out by a fixed point on the circumference of a circle when it rolls on a
fixed straight line without slipping.
This fixed line is called the base of the curve.

Solution.
Given parametric equations are x = a(u − sin u) and y = a(1 − cos u)
Let x = f (u) and y = g (u)
1. Symmetry
f ( −u) = a( −u − sin( −u)) = a( −u + sin u) = −a(u − sin u) = −f (u)

g ( −u) = a(1 − cos( −u)) = a(1 − cos u) = g (u)


∴ the curve is symmetric about the y-axis.
We shall consider the graph for u ≥ 0.
2. To find the point of intersection with the x-axis

Put y = 0 ⇒ a(1 − cos u) = 0 ⇒ cos u = 1   ⇒ u = 0, 2p, 4p, …

When u = 0, x = 0 and y = 0

∴ the origin corresponds to u = 0 and the origin lies on the curve.


Since −1 ≤ cos u ≤ 1, 0 ≤ y ≤ 2a.
When u = 2p , x = a( 2p − sin 2p) = 2pa and y = a(1 − 1) = 0 .
∴ the points of intersections with x-axis are (0, 0) and ( 2pa, 0) .
When u = p, x = a(p − sin p) = ap and y = a(1 − cos p) = 2a , which is the maximum value of y.
3. Sign of the derivative

dx u dy
= a(1 − cos u) = 2a sin 2 and = a sin u
du 2 du
dx dy
If 0 < u < p, then > 0 and >0
du du
∴ x increases from 0 to 2ap and y increases from
0 to 2a.
We shall draw one arch of the curve between (aπ, 2a) y = 2a
u = 0 and u = 2p .
i.e., between the points (0, 0) and ( 2ap, 0) on the
curve.
We see that the curve increases from (0, 0)
O (2aπ, 0) x
to ( ap, 2a) and decreases from ( ap, 2a) to
( 2ap, 0) as in fig. A.9.
Fig. A.9

M03_Eng-Maths (Aditya)_App2.indd 12 7/24/2018 2:25:41 PM


Curve Tracing  n  A.13

y = 2a

−6aπ −4aπ −2aπ O 2aπ 4aπ 6aπ x

Fig. A.10

Similarly another arch is from u = 2p and u = 4p .


Because of the symmetry about the y-axis, we can reflect about the y-axis and get the full graph as
in fig. A.10.
The x-axis is the base line on which the circle rolls. Then the points 0, 2ap, 4ap, 6ap, are the points
where the fixed point of the circle touches the base line.

Note
There are different forms of cycloids with base line the x-axis, y = 0 or the line y = 2a .
1. The parametric equations are x = a(u + sin u), y = a(1 + cos u)
When u = 0, x = 0 and y = 2a
So, the point corresponding to u = 0 is (0, 2a)
The curve meets the x-axis, y = 0
y

1 + cos u = 0   ⇒ cos u = −1

u = p, 3p, …, − p, …
(0, 2a)
So, one arch of the curve is between
u = −p and p
−3aπ −aπ O aπ 3aπ x
When u = −p , x = −ap and y = 0.
When u = p ,    x = ap and    y = 0
Fig. A.11
So, the graph is as shown in the Fig. A.11.
2. The parametric equations are x 5 a(u1sinu), y 5 a(12cosu)
When u = 0, x = 0, y = 0.
The curve meets the x-axis, y = 0  ⇒   a(1 − cos u) = 0
  ⇒ cos u = 1   ⇒   u = 0, 2p, 4p, …, −2p, − 4p, …
When u = 2p ,   x = 2ap,  y = 0 and when u = 4p,  x = 4ap,  y = 0
When u = −2p, x = −2ap, y = 0 and when  u = −4p,  x = −4ap, y = 0

M03_Eng-Maths (Aditya)_App2.indd 13 7/24/2018 2:25:48 PM


A.14  n  Engineering Mathematics-III

dx dy
= a(1+ cosu), = a sinu
du du
u u
2a sin .cos
dy a sin u 2 2 = tan u
= =
dx a(1+ cosu) u 2
2a cos 2
2
dy
=0 at u = 0, 2p, 4p, ......
dx
dy
and =∞ at u = p, 3p, 5p, ......
dx
When u varies from 0 to p, x increases from 0 to ap and y increases from 0 to 2a
When u varies from p to 2p, x increases from ap to 2ap and y decreases from 2a to 0.
So, the graph is as shown in Fig. A.12.

y
(aπ, 2a) y = 2a
(0, 2a)

2a

(−3aπ, 0) (−aπ , 0) O (0, 0) (aπ , 0) (3aπ, 0) x

Fig. A.12

A.1.3 Procedure for Tracing of Curve given by Equation in Polar Coordinates f(r, u) 5 0


Let the polar equation of the curve be r = f (u).
Relation between Cartesian and polar is x = r cos u and y = r sin u
1. Symmetry
(i) When u is replaced by −u and if the equation is unaltered, then the curve is symmetrical
about the initial line u = 0 (i.e., the x-axis).
(ii) When u is replaced by p − u and if the equation y
is unaltered, then the curve is symmetrical about π
p θ=
2
the line u = (i.e., the y-axis).
2 P (r, θ )
(iii) When u is replaced by p + u and if the equation
is unaltered, then the curve is symmetrical about r
the pole 0.
p
(iv) When u is replaced by − u and if the equation
2
is unaltered, then the curve is symmetrical about θ
p x
0
the line u = (i.e., the line y = x).
4

M03_Eng-Maths (Aditya)_App2.indd 14 7/24/2018 2:25:52 PM


Curve Tracing  n  A.15

(v) When u is replaced by p and r is replaced by –r and if the equation is unaltered, then the
curve is symmetrical about the pole.
2. Pole
If r = 0 for u = a, then the curve passes through the pole and the line u = a is tangent at the
pole.
3. Region
If r is imaginary for values of u lying between u = u1 , and u = u 2 , then the curve does not lie
between the lines u = u1 , and u = u 2 .

4. Points of intersection
p p 3p
Determine the points where the curve meets the lines u = 0, u = , u = , u = p, u =
4 2 2
5. Tangent line
du
Find the values of f with the formula tan f = r where f is the angle between the tangent at
dr
the point p ( r , u) and the radius vector op.
6. Loop
If a curve meets a line u = a at A and B and the curve is symmetrical about the line, then a loop
of the curve exists between A and B.

worked examples
Example 1
Trace the curve r 5 a (1 1 cos u). [This curve is called a cardioid]

Solution.
The given curve is r = a(1 + cos u)
1. Symmetry
If u is replaced by −u, then r = a(1 + cos( −u)) = a(1+ cos u)
Therefore, the equation is unaltered. Hence, the curve is symmetrical about the initial line u = 0.
2. r = 0 ⇒ 1 + cos u = 0 ⇒ cos u = −1  ⇒ u = p
∴ the tangent at the pole is the line u = p
3. When u = 0, r = 2a, which is the maximum value of r.
When u varies from 0 to p, r decreases from 2a to 0.
du
4. We know that tan f = r .
dr
dr du 1
Therefore, = a( − sin u)  ⇒   =−
du dr a sin u
u
 −1  2 cos 2
2 u  p u
∴ tan f = a(1 + cos u)  =− = − cot = tan  + 
 a sin u  u u 2  2 2
2 sin cos
2 2
p u
f= +
2 2

M03_Eng-Maths (Aditya)_App2.indd 15 7/24/2018 2:25:58 PM


A.16  n  Engineering Mathematics-III

p
When u = 0, f =
2
\ the tangent at the point (2a, 0) is perpendicular to the initial line u = 0
p p 2p
u: 0 p
3 2 3
3a a
r: 2a a 0
2 2
points: A B C D O
The curve is as in figure A to O
By symmetry about u = 0, by reflecting the point ABCDO about u = 0,
we get the full curve as in Fig. A.13.
y

θ = 2π
3 B π
θ=
C 3

D
a 3a
a 2
2
θ=π 2a A θ=0 x
O (2a, 0)

Fig. A.13
Example 2
Trace the curve r 5 a sin 3u. [This curve is called 3 leaved rose]
Solution.
The given curve is r = a sin 3u.
1. Symmetry
When u is replaced by p − u, r = a sin 3(p − u) = a sin(3p − 3u) = a sin 3u
So, the equation is unaltered.
p
Hence, the curve is symmetrical about the line u = (i.e., y-axis)
2
2. The maximum value of r is a, when sin 3u is maximum,
That is, when sin 3u = 1.
p 5p 9p p 5p 9p
⇒ 3u = , , ,.....   ⇒  u = , , , ……   ⇒  u = 30°, 150°, 270°, ...
2 2 2 6 6 6

M03_Eng-Maths (Aditya)_App2.indd 16 7/24/2018 2:26:01 PM


Curve Tracing  n  A.17

So, the curve lies within circle r = a, and r varies from –a to 0.


We get the third loop OBO.
p 2p 4p 5p
3. r = 0 ⇒ sin 3u = 0   ⇒  3u = 0, p, 2p, 3p, 4p, 5p   ⇒  u = 0, , , p, ,
3 3 3 3
p 2p 4p 5p
Therefore, u = 0, u =,u= , u = p, u = ,u= are tangents at the ori-
3 3 3 3
gin and the curve passes through the pole.

4. Loop
p
As u varies from 0 to , r varies from 0 to a. In other words, the curve is from O to A.
6
p p
As u varies from to , r varies from a to 0. In other words, the curve is from A to O.
6 3
Therefore, a loop OAO is formed.

p p
By the symmetry about u = , reflecting about u = , we get the second loop in the second
2 2
quadrant. As u varies from
4p 3p y
to , r varies from 0 to –a and
3 2 2π
3p 5p θ= π
as u varies from to , r varies 5π 3 θ=
θ= 3
2 3 6
from -a to O. we get the third loop θ=
π
OBO. C A 6

Note
1. More generally, the curve is of the
form r = a sin nu or r = a cos nu.
When n is odd, it is called n-leaved θ=π O x
θ=0
rose
2. When n is even, it is a 2n-leaved
rose.
For example, r = a sin 2u will have
4 leaves and the curve lies within
the circle r = a. B
θ=

4π 3
θ=
Limacon of Pascal 3

The polar curve r = a + b cos u, where θ=
2
a, b > 0 is called Limacon of Pascal.
Fig. A.14
When a = b, it becomes the cardioid
r = a(1 + cos u), which is discussed in
a
worked example 1. When < 1, that is, a < b, it is called a Limacon of Pascal with an inner loop.
b
a a
When 1 < < 2, it is called a dimpled Limacon and when ≥ 2 , it is called a Convex Limacon.
b b

M03_Eng-Maths (Aditya)_App2.indd 17 7/24/2018 2:26:05 PM


A.18  n  Engineering Mathematics-III

Example 3
Trace the curve r 5 1 1 2 cos u.
Solution.
The given curve is r = 1 + 2 cos u. Here a = 1, b = 2. ∴ a<b
Therefore, it is a Limacon with an inner loop.
1. Symmetry
When u is replaced by −u, we get r = 1 + 2 cos( −u) = 1 + 2 cos u
Therefore, the equation is unaltered. The curve is symmetrical about the initial line u = 0
2. Pole
1 2p 4p
r = 0 ⇒ cos u = − ⇒ u = ,
2 3 3 2p 4p
Inotherwords, the curve passes through the pole and u = ,u= are the tangents at the pole.
3 3
3. When u = 0, r 5 3 is the farthest point (3, 0).
p
When u = , r = 1. There is no asymptote, since r is finite for every value of u.
2
Since the curve is symmetric about u = 0, we shall find the variation of r as u varies from 0 to p.

p p p 2p 5p
u= 0 p
6 3 2 3 6
and r= 3 1+ 3 2 1 0 1− 3 −1
The points are A B C D O E F
The curve is ABCDO and OEF.
By symmetry about u = 0, we get the full curve as in Fig. A.15.
y

π
2π θ=
θ= 3
3
C
D
5π B π
θ= θ=
6 6

(3, 0) x
θ=π O F A θ=0
E

Fig. A.15

M03_Eng-Maths (Aditya)_App2.indd 18 7/24/2018 2:26:08 PM


Curve Tracing  n  A.19

Exercise A.1
Trace the following curves
x3 x 2 (a − x)
1. y 2 =  2. a 2 y 2 = x 2 ( a 2 − x 2 )  3.  y 2 =
2a − x a+ x
2x − 1 2 2 2
4. y 2 =  5.  x 3
+ y 3
= a 3
  6. y 2 x 2 = x 2 − a2
x2 −1
x3
7. y 2 =  8.  x 2 ( x 2 + y 2 ) = a 2 ( x 2 − y 2 )  9. r = a sin 2u
x −1
3at 3at 2
10.  r = a(1 − cos u) 11. 
r = 1 − 2sin u 12.  x= ,y = ,a>0
1+ t 3
1+ t 3
[Hint: Folium of Descartes]
2 2
xy 2 = a2 (a − x ), a > 0
13. y (a − x ) = x (a + x ), a > 0. 14. 

15.  x = a cos3 u, y = a sin 3u.

Answers to exercise A.1


y y
1. 2. 

y = −x y=x

x 20 x
O (−a, 0) (a, 0)
O

x = 2a

3. y 4.  y

y=x (0, 1)

x x
(−1, 0) ( 1 , 0) (1, 0)
2

y = −x (0, −1)
x=1
x = −1
1
x = −a x=
2

M03_Eng-Maths (Aditya)_App2.indd 19 7/24/2018 2:26:12 PM


A.20  n  Engineering Mathematics-III

5. y 6.  y
y=1
(0, a)

x
x
(−a, 0) (a, 0)

(0, −a) y = −1
x=a

7. y 8.  y

y=x
y = x −1
2

x=1
x
(−a, 0) (a, 0)
x O
(− 1 , 0)
2

y = −x

y = x +1
2

9. y 10.  y
π π
θ= θ=
2 2

θ=
4
π a
θ=
4
2a x
θ=π O θ=0
x
θ=π θ=0

M03_Eng-Maths (Aditya)_App2.indd 20 7/24/2018 2:26:14 PM


Curve Tracing  n  A.21

11. y 13.  y
π
θ=
2 y = −x

x
θ=0
B O A
(−a, 0) (a, 0) x
x=a

y=x

θ=
2

y
14. y 15. 
x=a (0, a)

O (a, 0) x (−a, 0) O (a, 0) x

x=0

(0, −a)

M03_Eng-Maths (Aditya)_App2.indd 21 7/24/2018 2:26:15 PM


This page is intentionally left blank

M03_Eng-Maths (Aditya)_App2.indd 22 7/24/2018 2:26:15 PM


Index
A Double integration, 3.1
Area, 5.14 Dummy variable, 1.22
Area as double integral, 3.27
Asymptotes, A.1
E
Elementary functions, 1.2
B Ellipse, 3.7
Beta function, 3.64 Exponential, 1.1
Beta gamma functions, 3.1
Bromwhich’s integral, 2.21
F
Final value theorem, 1.32
C First order linear differential, 2.25
First shifting property, 2.1
Cardiod, 3.36
Folium of descartes, A.5
Cartesian coordinates, 3.1
Change of order of integration, 3.8
Change of variables, 3.26
G
Cissoid, A.3 Gauss divergence theorem, 5.25
Common point, 4.6 Geometrical meaning, 4.2
Concave, A.9 Gradients, 4.6
Conservative vector field, 4.23 Green’s theorem, 5.8
Constant, 4.31
Constant coefficients, 2.25 H
Contour integral, 2.21 Half-sine wave rectifier function, 1.30
Convolution, 2.15 Heaviside function, 1.35
Convolution theorem, 2.16 Hyperbola, 3.6
Coordinate planes, 3.54
Curl, 4.23 I
Curved surface, 3.38 Improper integrals, 1.25
Curve tracing, A.1 Integral–differential equation, 2.43
Cycloid, A.12 Integrals, 1.21
Cylinder, 3.42 Integration, 3.7
Interval, 1.1
D Inverse Laplace transforms, 2.1
Delta function, 1.37 Irrotational, 4.25
Derivatives, 1.21 Irrotational vector field, 4.23
Differentiation and integration, 1.11
Differentiation formulae, 4.3 L
Diocles, A.3 Laplace transforms, 1.1
Dirac-delta function, 1.37 Leminiscate, 3.18
Direct and shifting methods, 2.2 Lemniscate of Bernoulli, A.4
Directional derivative, 4.5, 4.13 Linearity property, 1.4, 2.1
Divergence, 4.24 Linear operator, 1.4

Z01_Eng-Maths (Aditya)_Index.indd 1 7/24/2018 7:14:41 PM


I.2  n  Index

Line integral, 5.1 Scalar point, 4.24


Logarithmic, 2.10 Scale property, 1.7
Loop, A.2 Semi-circle, 3.25
Shifting property, 1.7
M Simultaneous differential
Meoander function, 1.31 equations, 2.37
Monge’s form, 3.38 Simultaneous equations, 2.45
Solenoidal, 4.26
O Space, 4.25
Octant, 3.45 Special points, A.1
Operational calculus, 1.1 Sphere, 3.43
Ordinary second order differential, 2.32 S-shifting, 1.7
Orthogonally, 4.14 Stoke’s theorem, 5.42
Surface area, 3.38
P Surface integrals, 5.17
Parabola, 3.37 Symmetry, A.1
Paraboloid, 3.63
Parallelogram, 3.30 T
Parametric, 3.41 Tangent, 4.17, A.1
Parametric equations, A.10 Tangent plane, 4.5
Partial fraction method, 2.3 The gamma function, 3.66
Periodic functions, 1.27 Triangle, 5.12
Pierre Simon de, 1.1 Trignometric function, 2.10, 3.65
Plane, 5.8 Triple integral, 3.49, 3.56
Point function, 4.32
Polar coordinates, 3.16 U
Unit impulse function, 1.37
Q Unit step function, 1.35, 1.36
Quadrant, 3.26
Quadrant bounded, 3.7 V
Variable coefficients, 2.32
R Variables, 3.19
Radius, 3.62 Vector, 4.1
Real function, 1.27 Vector differential
Real number, 4.1 operator, 4.4
Rectangle, 5.13 Vector differentiation, 4.1
Region, A.1 Vector form, 5.11
Residues, 2.21 Vector functions, 5.1
Vector integration, 5.1
S Vector point function, 4.1
Scalar, 4.1 Vertices, 5.12
Scalar field, 4.4 Volume integral, 5.19

Z01_Eng-Maths (Aditya)_Index.indd 2 7/24/2018 7:14:41 PM

You might also like